Download as pdf or txt
Download as pdf or txt
You are on page 1of 352

IOV-Registered Valuers Foundation

LAND & BUILDING

IOV Registered Valuers Foundation

2|Page
IOV-Registered Valuers Foundation

This material has been prepared in accordance to the syllabus prescribed by IBBI and for the
sole purpose of being the referral study material to the 50 hours Mandatory Education
Programme.

The views and opinions expressed in these materials are those of the authors and not of IOV-
RVF, New Delhi. Though prepared and edited with due care, IOV-RVF, New Delhi do not
undertake any liability for the views expressed herein.

No part of this publication may be reproduced, transmitted or utilized or stored in any form
or by any means now known or hereinafter invented, electronic, digital or mechanical,
including photocopying, scanning, recording or by any information storage or retrieval
system, without prior permission of IOV-RVF.

IOV Registered Valuers Foundation


Plot. No.3, 2nd Floor, Aggarwal Square,
LSC JN, 80Ft Rd, Parwana Road,
Pitampura, Delhi-110034.
ACKNOWLEDGEMENT
This book is based on studies & research in the field of valuation by prominent research
organisations, writers, editors & other contributors.

We wish to express our gratitude to eminent valuer members of the Institution of Valuers
who played a pivotal role in getting this course material constructed.

We would like to thank the Education, Training & Research Committee of IOV-RVF, Chaired
by Dr. Goutam Sengupta (Vice Chancellor, Techno India University, Kolkata) for the
motivation and support provided during the course of making this book.

Inspiration to prepare this course material has been derived by the works of International
Valuation Standards Council, Dr. Ashok Nain, Prof. Syamales Datta, Late Dr. P.C. Gupta, Late
Dr. Roshan H Namavati, C.H. Gopinatha Rao, Late D. N. Banerjee, V. Shanmugavel, B.
Kanagasabapathy, Ram Mohan Bhave and Gopala Krishna Raju.

This book would not be complete without the editing efforts of Nitin Lele, Sandip Deb, R.K.
Patel and Gopala Krishna Raju.

We sincerely appreciate the contributions of S. Pichaiya, Pramod Athalye, Devendra Patekar,


Vikas Londhe, Abhay Kumar, Munish Aggarwal, Ashok Kumar Gupta, Neeraj Kapoor, Anand
Raj, B.V. Ramanna and G.S. Raju.

And last but not the least, a whole hearted thanks to Centre for Research & Technical
Education (CRTE) for their unending support during the whole process of shaping up the
book.

Vinay Goel Tanuj Kumar Bhatnagar

Managing Director & CEO Director

4|Page
IOV-Registered Valuers Foundation

(This page is blank)


PREFACE
IOV-RVF initiated the training programme for Registered Valuers in India. It brought out a
collection of study material last year for the student members of IOV-RVF. The study
material predominantly included the IBBI indicated syllabus and was provided to about
3000 members or more who have undergone the IOV-RVF training. The study material was
appreciated by faculty members & students alike.

It is pertinent to mention here that the contents of the entire study material were the result
of many years of facilitating, researching and contributions of eminent & learned members
of the Institution of Valuers (IOV), which has a legacy of 50 golden years of enriching its
members by way of monthly journals, seminars, workshops and lectures etc.

Our government through Ministry of Corporate Affairs (MCA) and the regulating authority
(IBBI) is continuously making efforts to upgrade the valuation profession by way of
amending the rules, and otherwise in respect of registration, valuation examination
including syllabus/course curriculum, eligibility requirements and functions of the valuers
etc., in consultation with the Registered Valuer Organisations (RVOs) in their monthly
meetings held regularly with IBBI.

Due to ever changing hues of valuation arena, developments under IBC, implementation of
Companies (Registered Valuers & Valuation) Rules, 2017 and change in examination
syllabus for the valuation examination conducted by IBBI, it has become important for IOV-
RVF to improvise the content of its study material to upgrade it as per changed
environment. It is now the appropriate time to recreate the existing material in a book form
for the benefit of aspiring valuers. IOV-RVF believes that a book form can at best be
considered as a window to never-ending expanse of knowledge in valuation world.

Valuation involves understanding of vide range of subjects including law, economics,


technology, business management etc. that a single person is often found inadequate in
producing a comprehensive book on subject. IOV-RVF thought it appropriate to include
more number of domain experts to contribute to the book so as to overcome this
limitation. The result is a vantage point view of valuation practice in India, cumulating the
first hand experience of practicing valuers and domain experts. It is pertinent to understand
that valuation is based on scientific methodology encompassing interdisciplinary fields such
as engineering, finance, economics, law, social science statistics etc.

The prime motive of valuation method is to understand ways to arrive at conclusive figures
to accommodate all the benefits and liabilities that can accrue to owner of property.
Accordingly, in order to understand what it means to carry on the profession of valuation,
students must develop both a foundation of valuation as a subject and an understanding of
*the critical issues of the valuation profession. This book provides a framework for learning

6|Page
IOV-Registered Valuers Foundation

these necessary topics in a way that emphasizes the uniqueness of each topic and each
chapter as per the syllabus. The text also lays emphasis on the necessary skills in building
and maintaining the Valuation profession in order to make valuations as per the valuation
standards and conducting the valuation assignments in the professional and ethical manner,
which is the essence of the government’s efforts in institutionalizing and regulating the
valuation profession.

This book is primarily aimed at training the aspiring valuers for qualifying IBBI exam by
providing them ample opportunities to understand the concepts and absorb first hand
experiences which have been illustrated by way of Case Studies and examples.

A structured approach has been followed so that the reader is be exposed to General
Valuation Practices in India, its role in corporate valuation, IBC-2016 and other statutory
requirements under Companies Act-2013, Valuation methods, Valuation Standards,
professional and ethical standards, valuation related laws, economics, understanding of case
studies, case laws in valuation, environmental issues concerning valuation and report
writing. The book primarily focuses on the indicative syllabus as prescribed by IBBI for the
purpose of valuation examination.

This book places a clear emphasis on Valuers’ understanding for need of ethics, its role
towards fellow professionals and the society at large. Valuers’ sensitivity to environmental
issues that may create or destroy value of asset has been detailed and amply highlighted.
Entire chapter has been contributed to report writing in line with Valuation Rules.

Valuers’ task of identifying various interests in property, methods of calculating the


monetary value of such interests, marketing propositions of such valued interests under
various situations of business cycle have been adequately addressed in this book.

Lastly, since the subject is ever-evolving it shall be our endeavor to incorporate the updated
developments on issues related to valuation in our forthcoming editions.
LAW REAL ESTATE I 01
1. Land Acquisition
2. General Building Rules and Regulations
3. Rent Control Laws
4. Right of Way and Section 52
5. Salient Features of the Real Estate

Page | 1
IOV-Registered Valuers Foundation

Page | 2 (This page is blank)


L&B/Chapter-1/Law-Real Estate

LAW REAL ESTATE I 01

1. LAND ACQUISITION
The Right to Fair Compensation and Transparency in the Land Acquisition, Rehabilitation
and Resettlement Act, 2013. The Right to Fair Compensation and Transparency in Land
Acquisition, Rehabilitation and Resettlement Act, 2013:

1) Application of Act (S.2);


2) Definition (S.3);
3) Notification and Acquisition (Chapter IV);

4) Rehabilitation and Resettlement Award (Chapter V);

5) Acquisition of part of house or building (S.94);

6) Exemption from Income Tax, stamp duty and fees (S.96).

Eminent Domain is the right asserted by the Sovereign State to do anything in public
interest and it is extended to the Sovereign State's right to acquire private property for
public purpose. The Constitutional Amendment in 1978 took away the fundamental right
to property when it repealed Art 19(1)(f) (i.e. the right to acquire, hold and dispose
property) and Art 31 (which earlier regulated compulsory acquisition of property and
restricted acquisition only to public purpose). An Article300A was inserted in 1978 about
acquisition of property only under the authority of law - thus right to property became a
constitutional right.

Art 31A deals with exempting laws that provide for acquiring estates from being deemed
void on grounds of violating Art 14 or Art 19. Art 31A proviso dealt with land which is
part of an estate and is within personal cultivation of and within ceiling limit applicable
to the person along with building / structures on it cannot be acquired except on
payment of compensation not be less than the market value.

Page | 3
IOV-Registered Valuers Foundation

Sri Radhy Shyam (Dead) Through L.Rs. vs. State of U.P. & others CA 3261 of 2011
(Arising out of Special Leave Petition (C) No.601 of 2009) In this a large chunk of land
measuring 4.3840 hectares was not acquired apparently because the same belong to an
ex-member of the legislative assembly while land of poorer farmers were acquired and
justified as land of neighboring village was also acquired under provisions of Land
Acquisition Act, 1894: "while examining the land owner's challenge to the acquisition of
land in a petition filed under Article 226 of the Constitution, the High Court should not
adopt a pedantic approach, and decide the matter keeping in view the constitutional
goals of social and economic justice and the fact that even though the right to property
is no longer a fundamental right, the same continues to be an important constitutional
right and in terms of Article 300-A, no person can be deprived of his property except by
authority of law ...the High Court should insist upon filing of reply affidavit by the
respondents and production of the relevant records and carefully scrutinize the same
before pronouncing upon legality of the impugned notification/action because a
negative result without examining the relevant records to find out whether the
competent authority had formed a bona fide opinion on the issue of invoking the
urgency provision and excluding the application of Section 5-A is likely to make the land
owner a landless poor and force him to migrate to the nearby city only to live in a slum.

If the acquisition is intended to benefit private person(s) and the provisions contained in
Section 17(1) and/or 17(4) are invoked, then scrutiny of the justification put forward by
the State should be more rigorous in cases involving the challenge to the acquisition of
land, the pleadings should be liberally construed and relief should not be denied to the
petitioner by applying the technical rules of procedure embodied in the Code of Civil
Procedure and other procedural laws. The Court ordered payment of 5 lakh
compensation towards litigation cost and required reconsideration of the case.

The earlier Land Acquisition Act 1894 did not have any concept of consultation, all
acquisition was forced. It had provisions of an urgency clause but it was not defined. The
concept of public purpose was also not defined. There was little scope for appeal or a
hearing. Resorting to the writ jurisdiction of the High Court was the only option. There
was no designated process for computing compensation. There was nothing included in
rehabilitation in that Act.

The Right to Fair Compensation and Transparency in Land Acquisition, Rehabilitation and
Resettlement Act (RFCTLARR), 2013 was notified in the gazette on 27th Sept. 2013. Prior
to this date the earlier Land Acquisition Act of 1894 passed by British Parliament applied
on the matter of land acquisition. However, on the Rehabilitation & Resettlement of
families affected and displaced as a result of land acquisition this is the first pan-India.

Page | 4
L&B/Chapter-1/Law-Real Estate

This Act came into force on 01st January, 2014 by repealing the Land Acquisition Act,
1894. This Act allows States to enact laws to increase or add to the entitlements
enumerated under this Act. The Department of Land Resources (DoLR), Ministry of Rural
Development, Government of India is administering the Act. MORD notified the Right to
Fair Compensation and Transparency in Land Acquisition, Rehabilitation and
Resettlement (Compensation, Rehabilitation and Resettlement and Development Plan)
Rules, 2015 on 18th December,2015.

The Amendment Bill was introduced in the Lok Sabha by the Minister for Rural
Development on February 24, 2015. The Bill amends the Right to Fair Compensation and
Transparency in Land Acquisition, Rehabilitation and Resettlement Act, 2013 (LARR Act,
2013). The Bill replaces the Right to Fair Compensation and Transparency in Land
Acquisition, Rehabilitation, and Resettlement (Amendment) Ordinance, 2014.

This bill not only provides for land acquisition but also for rehabilitation and resettlement
(R&R). The provisions of this bill shall be applicable where the government acquires land
either for its own use or for the use of any private company for public purposes. The
earlier Act only allowed land acquisition by the government for government-owned
companies as well as schemes run by societies/authorities/co-operative societies while
the new Bill allows acquisition for a public purpose by privet companies and Public
Private Partnership (PPPs) apart from government-controlled organizations.

According to this bill, the private companies are entitled to provide rehabilitation and
resettlement if they acquire land through private negotiations. R&R shall be equal to or
more than 100 acres in rural areas and 50 acres in urban areas. As per this bill, the term
“public purpose” includes:
1) National security and strategic defense purposes
2) Roads, railways, and ports built by government and PSEs
3) Project affected people
4) Planned development or improvement of villages
5) Residential purposes for the poor
6) Government projects benefiting public.

This Bill creates five special categories of land use which are exempted from certain
provisions. This includes:
1) Defense
2) Rural infrastructure
3) Affordable housing
4) Industrial corridors
5) Infrastructure projects including PPP projects.

Page | 5
IOV-Registered Valuers Foundation

However, under the LARR Act, 2013 land could be acquired for all these purposes only
when 80% of the project affected people give their consent. But this bill exempts the
above-mentioned land uses from this consent clause. This Bill stipulates the appointment
of an R&R committee to review R&R progress in the case where land acquired is 100
acres or more for the public purpose.

APPLICATION OF ACT

As per Sec 2, this act will apply when:

a) Government acquires land for its own use, or to hold and control.

b) Where Government acquires land for private companies or for specified public
purpose - require prior consent of 80 per cent of the affected families.

c) Government acquires land for Public Private Partnership Projects - consent of 70


percent of the affected families is obtained, MORD Notification dated 9 th
February 2016, states "the limit of extent of land referred to in sub-section (1) of
section 46 of the said Act shall be 20 (twenty) hectares in urban areas and 40
(forty) hectares in rural areas"

The rules state that rehabilitation and resettlement under the Act would apply where a
private company purchases land equal to or more than 20 hectares (50 acres) in urban
areas and forty hectares (100 acres) in rural areas.

As per the 2013 Act, in the Scheduled Areas (tribal areas to which the fifth schedule of
the constitution applies, no land acquisition can happen in contravention of any law
relating to land transfer, prevailing in such Scheduled Areas.

In fact any order or judgment of a High Court which has become final would also need
to be adhered to. As per this provision in scheduled areas no land acquisition under even
Land Acquisition Act 2013 can happen in contravention of Panchayat (Extension to the
Scheduled Areas) Act, 1996 (PESA).

PESA was enacted to extend the Panchayat raj provisions contained in the Part IX of the
Constitution to the Scheduled Areas with exceptions and modification. PESA states in,
"Sec 4(i) "the Gram Sabha or the Panchayat at the appropriate level shall be consulted
before making the acquisition of land in the Scheduled Area as for development projects
and before re-settling or rehabilitating persons affected by such projects in the
Scheduled Areas; the actual planning and implementation of the projects in the
Scheduled Areas shall be coordinated at the State level.

Sec 4 (k) the recommendations of the Gram Sabha or the Panchayat at the appropriate
level shall be made mandatory prior to grant of prospecting license or mining lease for

Page | 6
L&B/Chapter-1/Law-Real Estate

minor minerals in the Scheduled Areas; Sec 4 (I) the prior recommendation of the Gram
Sabha or the Panchayat at the appropriate level shall be made mandatory for grant of
concession for the exploitation of minor minerals by auction"

Sec 4 (m) (iii) the power to prevent alienation of land in the Scheduled Areas and to take
appropriate action to restore any unlawfully alienated land of a Scheduled Tribe;The Scheduled
Areas have been designated by the President of India and are located in Andhra Pradesh,
Chhattisgarh, Gujarat, Himachal Pradesh, Jharkhand, Madhya Pradesh, Maharashtra, Odisha,
Rajasthan and Telangana.

State Areas

Andhra Pradesh Visakhapatnam, East and West Godavari, Adilabad, Srikakulam,


Vizianagaram, Mahboobnagar, Prakasam (only some mandals)Dumka,
(including Telangana)
Godda, Deogarh, Sahebgunj, Pakur, Ranchi,Singhbhum (East &West),
Jharkhand Gumla, Simdega, Lohardaga,parts of Palamu, Garwa)

Chhattisgarh Sarguja, Bastar, Raigad, Raipur, Rajnandgaon, Durg, Bilaspur,Shahdol,


Chindwada, Kanker

Himachal Pradesh Lahaul and Spitidistricts, Chamba district (Kinnaur,


PangitehsilandBharmour sub-tehsil)

Madhya Pradesh Jhabua, Mandla, Dhar, Khargone, Betul, Seoni, Balaghat,Morena, East
Nimar (khandwa), Sailana tehsil in Ratlam district,

Gujarat Parts of the districts of Surat, Bharauch, Dangs, Valsad,

Panchmahl, Sadodara, Sabarkanta

Maharashtra Parts of the districts of Thane, Nasik, Dhule, Ahmednagar,Pune,


Nanded, Amravati, Yavatmal, Gadchiroli, Chandrapur

Odisha Mayurbhanj, Sundargarh, Koraput (fully scheduled area in

these threedistricts), Raigada, Keonjhar, Sambalpur,

Boudhkondmals, Ganjam, Kalahandi, Bolangir, Balasor (parts ofthese


districts only)

Rajasthan Banswara, Dungarpur (which are fully tribal districts),


Udaipur,Chittaurgarh, Siroi (partly tribal areas)

Page | 7
IOV-Registered Valuers Foundation

DEFINITIONS

Some definitions from section 3 of the Act are enumerated below:

• "Displaced family" means any family, who on account of acquisition of land has
to be relocated and resettled from the affected area to the resettlement area;
• "Family" includes a person, his or her spouse, minor children, minor brothers and
minor sisters dependent on him. An adult of either gender with or without spouse
or children or dependents shall be considered as a separate family. Widows,
divorcees and women deserted by families shall be considered separate families;
• "Market value" means the value of land determined in accordance with section
26;

• "Agricultural land" means land used for raising nursery, cultivation of crops,
trees, horticulture, grass, garden produce, dairying, poultry farming, pisciculture,
sericulture, seed farming, breeding of livestock, medicinal herbs and also open
land used for the grazing of cattle

• "Cost of acquisition" includes - solatium (payment to compensate injured


feelings or emotional pain), any enhanced compensation ordered by the Land
Acquisition Authority or the Court and interest, demurrage payable for damages
caused in the process of acquisition to the land and standing crops. It also
includes the cost of acquisition of such land and building for the settlement' of
displaced or adversely affected families and the cost of development of
infrastructure and amenities at such resettlement areas;

There are 13 other Act which deal with acquisition and are listed in the fourth schedule
and vide a departmental notification the provision the RFCTLARR Act, 2013 will become
applicable to acquisition under those Acts also. These are:

1) The Ancient Monuments and Archaeological Sites and Remains Act, 1958 (24 of
1958).
2) The Atomic Energy Act, 1962 (33 of 1962).

3) The Damodar Valley Corporation Act, 1948 (14 of 1948).

4) The Indian Tramways Act, 1886 (11 of 1886).

5) The Land Acquisition (Mines) Act, 1885 (18 of 1885).

6) The Metro Railways (Construction of Works) Act, 1978 (33 of 1978).

7) The National Highways Act, 1956 (48 of 1956).

8) The Petroleum and Minerals Pipelines (Acquisition of Right of User in Land) Act, 1962
(50 of 1962).

9) The Requisitioning and Acquisition of Immovable Property Act, 1952 (30 of 1952).
10) The Resettlement of Displaced Persons (Land Acquisition) Act, 1948 (60 of 1948).

Page | 8
L&B/Chapter-1/Law-Real Estate

11) The Coal Bearing Areas Acquisition and Development Act, 1957 (20 of 1957).

12) The Electricity Act, 2003 (36 of 2003).

13) The Railways Act, 1989 (24 of 1989).

• "Affected family" includes

1) A family whose land or other immovable property has been acquired;


2) A family which does not own any land but whose primary source of livelihood stand
affected by the acquisition of land:

a. Are agricultural laborers,


b.any kind of tenants
c. holding of usufruct right The right to enjoy the use and advantages of another's
d.share-croppers property short of destruction. (Temporarily)

e. traditional artisans

3) People who may be working in the affected area for three years prior to the
acquisition of the land,

4) Family of gatherers of forest produce, hunters, fisher folk and boatmen - whose
primary source of livelihood for three years prior to the acquisition of the land is
dependent on forests or water bodies and is affected due to acquisition of land
5) Holders of Community forest rights recognized under the Scheduled Tribes and
Other Traditional Forest Dwellers (Recognition of Forest Rights) Act, 2006 due to
acquisition of land

6) A member of the family who has been assigned land by the State or the Central
Government under any scheme and such land is under acquisition;
7) A family residing on any land in the urban areas for preceding three years or more
prior to the acquisition of the land or whose primary source of livelihood for three
years prior to the acquisition of the land is affected by the acquisition of such land;

NOTIFICATION AND ACQUISITIONINCLUDING COMPUTATION OF


COMPENSATION AWARD (CH. IV)

Chapter IV (sections 11 to 31) of the Act deals with notification for acquisition,
declaration and also computation of Compensation Award. A preliminary notification of
the proposed acquisition under section 11(1) of the 2013 Act must be published in the
following manner, namely,

1) In the Official Gazette;

2) In two daily newspapers circulating in the locality of such area of which one shall be
in the regional language;

Page | 9
IOV-Registered Valuers Foundation

3) In the local language in the Panchayat, Municipality or Municipal Corporation, as the


case may be and in the offices of the District Collector, the Sub-divisional Magistrate
and the Tehsil;

4) Uploaded on the website of the appropriate Government;


5) In the affected areas, in such manner as may be prescribed (rules provide for
announcement by beating of drums)

The date of this notification shall be the date for determination of market value. The
notification must be issued in the form II annexed to the rules. It includes the following
points:

• "Social Impact Assessment Study was carried out by Social Impact Assessment
(SIA) Unit and a report submitted / preliminary investigation was conducted by a
team constituted by Collector as laid down under rule 4. The summary of the
Social Impact Assessment Report/ preliminary investigation is as follows...". A
copy of the report must also be attached to this preliminary notification.

• The notification puts a bar on subsequent transfer or create encumbrance


through pledge and mortgage: "Under section 11(4) of the Act, no person shall
make any transaction or cause any transaction of land i.e. sale/purchase, etc., or
create any encumbrances on such land from the date of publication of such
notification without prior approval of the Collector."
• It must call for objections thus "Objections to the acquisition, if any, may be filed
by the person interested within 60 (sixty days) from the date of publication of this
notification as provided under section 15 of the Act before Collector."
• The concerned gram sabha(s) at the village level, municipalities and the
Autonomous Councils in Sixth Schedule areas shall be informed of the contents of
the notification in a special meeting.

• No person shall make or cause any transaction or create any encumbrances on


such land from the date of publication of such notification till the completion of
the process.

• Collector shall ensure updating of land records as prescribed within a period of


two months

• As per sec 12 of the Act, on publication of notice, in the presence of the property
owner it shall be lawful for designated officials to enter upon and survey and take
levels of any land in that area; to dig or bore into the sub-soil, set out boundaries
of the land proposed to be taken; placing marks and cutting trenches and where
otherwise the survey cannot be completed and the levels taken and the
boundaries and line marked, to cut down and clear away any part of any standing
crop, fence or jungle. Entry in absence of owner can be done only after noticing
him 60 days prior to entry. A seven days notice is to be given prior to entering
dwellings or enclosed garden under occupation. As per sec 13 any damage
caused during such assessment shall be compensated to the owner.

Page | 10
L&B/Chapter-1/Law-Real Estate

Social Impact Assessment is mandatory but its report has a shelf life of 12 months and
must be redone if the preliminary acquisition notification is not issued within such
period. The appropriate government after recording in writing the reason may extend
the period of 12 months.
A rehabilitation and resettlement scheme have to be drawn up and duly publicized for
calling for objections and for information. This scheme must take account census details
and following particulars of:

• Lands and immovable properties being acquired of each affected family;


• livelihoods lost in respect of land losers and landless whose livelihoods are
primarily dependent on the lands being acquired;

• A list of public utilities and Government buildings which are affected or likely to
be affected, where resettlement of affected families is involved;

• The amenities and infrastructural facilities which are affected or likely to be


affected, where resettlement of affected families is involved; and
• any common property resources being acquired

The plan must give particulars of the rehabilitation and resettlement entitlements of each
land owner and landless whose livelihoods are primarily dependent on the lands being
acquired along with a timeline of execution of the Scheme. Where resettlement of
affected families is involved - then a list of Government buildings to be provided in the
Resettlement area and the details public amenities and infrastructural facilities to be
provided must be collected.
Within a period of twelve months from the date of the publication of preliminary
notification, a declaration stating the district or other territorial division in which the land
is situated, the purpose for which it is needed; its approximate area shall be made along
with a declaration of an area identified as the "resettlement area". Along with it the
declaration the Collector shall publish a summary of the Rehabilitation and Resettlement
Scheme. Any period or periods during which the proceedings for the acquisition of the
land were held up on account of any stay or injunction by the order of any Court shall be
excluded from the period of 12 months. The appropriate government can extend the
period of 12 months.
The declaration and summary of resettlement etc shall be publicized in the same manner
as listed under publication of preliminary notification detailed above. Once published
this declaration shall be conclusive evidence that the land is required for a public
purpose. Prior to the declaration, the land requisitioning agency must deposit an
amount, in full or part, as may be prescribed by the appropriate Government toward the
cost of acquisition of the land. If the application for acquisition itself specified stages for
the rehabilitation and resettlement, then all declarations shall be made according to the
stages so specified.
Public Hearing must be conducted in every affected Gram Sabha and Municipality where

Page | 11
IOV-Registered Valuers Foundation

more than twenty-five per cent of the land belonging to that Gram Sabha or Municipality
is being acquired

Within 12 months of publication of declaration, the Collector must make the award
unless the period is extended by the State government.
As per Sec 25 any land acquisition process initiated under the earlier 1894 act but not
completed can be lapsed in some conditions. If award was made under earlier Act then
the earlier Act shall apply even in the courts. If award was made but most did not take it
then the new Act will apply. Where an award has been made five years or more prior to
the commencement of this Act but the physical possession of the land has not been
taken or the compensation has not been paid the said proceedings shall be deemed to
have lapsed.

Rule 20 of Right to Fair Compensation and Transparency in Land Acquisition,


Rehabilitation and Resettlement (Compensation, Rehabilitation and Resettlement and
Development Plan) Rules, 2015 describes the manner of return of land which remains
unutilized for a period of five years. It states the same shall be returned to the original
owner or owners or their legal heirs, as the case may be, or to the Land Bank by issuing a
notice to the Requiring Body for whom the land was acquired and by giving an
opportunity of being heard and by passing necessary order in writing by the Collector in
this behalf for this purpose. The land is taken over by the Collector first prior to
transferring.

CASE LAWS IN LAND ACQUISITION

The Divisional Bench of Supreme Court in the Working Friends Co-operative House
Building Society Ltd. Vs the State of Punjab and Ors., (CA 8468/ 2015 decided on Oct.
12, 2015) considered the question "whether the compulsory acquisition of the ... land
under the Land Acquisition Act, 1894 lapses in view of the provisions of Section 24(2)" of
the new Act of, 2013". It was held that the acquisition proceedings initiated by the land
acquisition notifications dated 12th November, 1992 and 21" July, 1993 followed by the
award dated 22nd February, 1995 have lapsed only in so far as the appellant is
concerned. It is based on a conclusion of fact "The admitted position is that the
compensation of Rs. 35,52,528/- was neither paid to the appellant i.e. the Working
Friends Cooperative House Building Society Ltd., nor was it deposited in the Reference
Court. It was admittedly deposited in the Government Treasury of the State". The deposit
was made only after the Act came into force and was done perhaps with a view to get
over the provisions of Section 24(2) of the Act. It observes that even if it were deposited
in the reference Court on 26th June, 2014 it would not have aided as under any
circumstances and cannot be taken as "deemed payment".

Page | 12
L&B/Chapter-1/Law-Real Estate

A three-judge bench of the Apex Court in Union of India and Ors vs. Shiv Raj &Ors CA
NOS. 5478-5483 of 2014 (Arising out of S.L.P.(C) Nos. 24297-24302 of 2007decided on 7
May, 2014) - "The facts and circumstances which have arisen in this appeal are that the
land, the subject matter of the appeal, stood notified under Section 4 of the Act 1894 on
25.11.1980. The other persons whose land had also been acquired by the same
notification had challenged the validity of the notification under Section 4 of Act 1894 by
filing the writ petitions and its validity was upheld by the judgment and order dated
15.11.1983. It was during the pendency of the acquisition proceedings that the present
appellant had purchased the land vide registered sale deeds dated 6.5.1985 and
24.5.1985". ..."... various orders in various litigations pending before the High Court had
been passed. The writ petition filed by the present appellant was dismissed vide
impugned judgment and order dated 17.12.2004.”... "In view of the fact that the other
land covered by the same notification ...Review Petition etc. had been filed, which was
dismissed on 27.7.2007". A point rose before it was that many stays had been issued by
High Court etc. To which it opines "... It is evident from the orders passed by the High
Court that it had granted stay of dispossession during the pendency of the writ petition
as well as the review petition, though no interim order has been passed by this court.
The respondent did not take possession of the land in dispute though award had been
made in the year 1987-1988, and the High Court had decided against the appellant in
the year 2007. Thus, a period of 7 years has lapsed without any stay of proceedings and
yet no action has been taken by the respondents in pursuance to the award". The court
concluded the acquisition was hit by sec 24(2) of the new Act.

The period during which the proceedings for acquisition of land was held up on account
of any stay or injunction issued by any court is excluded for the purpose of calculation of
five years period.

DETERMINATION OF THE MARKET VALUE OF LAND

Under the earlier 1894 Act, the Collector decided the quantum of compensation payable
to those displaced. The present Act provides a clear process.

The date for determination of market value shall be the date on which the notification
has been issued under section 11. The Collector shall adopt the following criteria in
assessing and determining the market value of the land including all assets attached to
the land, namely:

1) The market value, if any, specified in the Indian Stamp Act, 1899 for the registration
of sale deeds or agreements to sell, as the case may be, in the area, where the land is
situated; or

Page | 13
IOV-Registered Valuers Foundation

2) The average sale price for similar type of land situated in the nearest village/ vicinity
area. This will be based on sale price in the sale deeds or the agreements to sell
registered for similar type of area in the near village or near vicinity area during
immediately preceding three years of the year in which such acquisition of land is
proposed to be made. One-half of the total number of sale deeds or the agreements
to sell in which the highest sale price has been mentioned shall be taken into
account. However, any price paid as compensation for land acquired under the
provisions of this Act on an earlier occasion in the district shall not be taken into
consideration. Any price paid, which in the opinion of the Collector is not indicative
of actual prevailing market value may be discounted for the purposes of calculating
market value. The market value so calculated shall be multiplied by a factor one for
urban areas and two for rural areas.

3) Consented amount of compensation as agreed upon under sub-section (2) of


section 2 in case of acquisition of lands for private companies or for public private
partnership projects, whichever is higher.

Where the market value cannot be determined for the reason that:

o The land is situated in such area where the transactions in land are
restricted by or under any other law for the time being in force in that
area; or

o The registered sale deeds or agreements to sell as mentioned in clause


(a) of sub-section (1) for similar land are not available for the immediately
preceding three years; or

o The market value has not been specified under the Indian Stamp Act,
1899 by the appropriate authority,

4) The State Government concerned shall specify the floor price or minimum price per
unit area of the said land based on the price calculated in the manner specified in
sub-section (1) in respect of similar types of land situated in the immediate adjoining
areas

5) If the body that requires the land offers its shares to the owners of the lands (whose
lands have been acquired) as a part compensation, for acquisition of land, such
shares in no case shall exceed twenty-five per cent of the value nor can it compel
land owners to accept share.

6) The land or property of an educational institution established and administered by a


religious or linguistic minority shall be such as would not restrict or abrogate the
right to establish and administer educational institutions. This requirement was
inserted vide Art 30(1A) in the Constitution of India in 1978.

7) For determination of value of things attached to land or building and other


immovable property or assets attached to the land or building which are to be
acquired, use the services of competent professionals:

Page | 14
L&B/Chapter-1/Law-Real Estate

o For buildings - engineer or any other specialist in the relevant field, as


may be considered necessary by him.

o Trees can be assessed by use the services of experienced persons in the


field of agriculture, forestry, horticulture, sericulture, or any other field, as
may be considered necessary by him.

o Value of the standing crops damaged during the process of land


acquisition, may use the services of experienced persons in the field of
agriculture as may be considered necessary by him.

AS PER THE FIRST SCHEDULE ONTHE MINIMUM COMPENSATION


FOR LAND OWNERS AND TENANTS

Compensation package Manner of determination of value


component
Multiplier Factor of the In the case of urban areas - one and In the case of Rural areas - One
market value to two based on the distance of project from urban area, as may be
notified by the appropriate Government [Vide MORD Notification
dated 9th February, 2016 S.O. 425(E) - .
[F No.13011/04/2015-LRD]

Solatium Equivalent to one hundred percent of the market value of land


A thing given to someone as multiplied by the appropriate factor mentioned above plus value of
Compensation/ Consolation assets attached to land or building

Final award in rural/ urban Market value of land multiplied by the appropriate factor for rural/
areas urban areas plus value of assets attached to land or building
mentioned plus solatium as above

ACQUISITION OF PART OF HOUSE OR BUILDING(S.94)

The provisions of this Act cannot be invoked for severing land and acquiring a part only
of any house, factory or other building when the owner desires that the whole of such
house, factory or building shall be acquired. Whether it is or is not part of a house,
manufactory or building within the meaning of this section will be decided by the
Authority to whom the Collector refers the matter which will decide whether the land is
critical to "full and unimpaired use of the house, manufactory or building"

If due to severing of a part of the land being acquired from remainder of the land of a
owner if the appropriate Government is of opinion that the claim is unreasonable or
excessive, it may, at any time before the Collector has made his award, order the
acquisition of the whole of the land of which the land first sought to be acquired forms a

Page | 15
IOV-Registered Valuers Foundation

part. No fresh declaration is needed even if this additional area is included in the award.

AWARD FOR LAND ACQUISITION

An award under this Act is computed for each eligible family and consists of the
following elements:

1) Market value as computed above for land, building and things attached to land and
building. This is the compensation amount- Solatium which is an amount equivalent
to one hundred per cent of the market value and given in addition

2) twelve per cent per annum on such market value computed for the period
commencing on the date of publication of the notification of the Social Impact
Assessment study and till the date of the award of the Collector or the date of taking
possession of the land, whichever is earlier
3) 25 percent of the total compensation can be through shares in the Requiring Body if
the projected affected family consent
4) If land is acquired for urbanization then 20 percent of the acquired land must be
reserved for landowning project affected people and offered to them at a price
equal to cost of acquisition

5) If a person previously displaced is being displaced again in another project from the
rehabilitation site - they will be entitled to 75 percent additional compensation.

EXEMPTION FROM INCOME TAX,STAMP DUTY AND FEES

As per sec 97 no income tax or stamp duty can be levied on any award or agreement
made under this Act, except under section 47 and no person claiming under any such
award or agreement shall be liable to pay any fee for a copy of the same on or after the
5th September 2011 as per Sec 47 amongst others provides when any person other than
a specified person is purchasing land through private negotiations for an area equal to
or more than such limits specified for applicability of the Act, he must apply to the
Collector. On application Collector shall pass individual orders.

If the same land is acquired within three years from the date of commencement of this
Act, then, forty per cent of the compensation paid for such land acquired shall be shared
with the person(s) who was the owner as on 5th, 2011.

Page | 16
L&B/Chapter-1/Law-Real Estate

Figure 1: The format of Land Acquisition Award specified in the Rules

FORM VI
See rule III
Land Acquisition case No:

1 Name of the Project -


2 Number and date of declaration under which the land is to be acquired

Situation and extent of the land in hectares, the number of field plots on the survey map,
3
the village in which situated with the number of mile plan if any.

Description of the land, i.e., whether fallow, cultivated, homestead, etc. If cultivated, how
4
cultivated? Source of irrigation

5 Names of persons interested in the land and the nature of their respective interests.
6 Aadhaar No. of such persons
7 Amount allowed for the land itself, without trees, buildings etc., if any
8 Amount allowed out of such sum as compensation for the tenants interested in the land.
9 Basis of calculation:
10 Amount allowed for trees, houses or any other immovable property
11 Amount allowed for crops.
12 Additional compensation on the market value under section 30(3)
13 Damages under section 28 of Act 30 of 2013
14 Solatium u/s 30(1)
15 Total of amounts
Particulars of abatement of Government Revenue, or of the capitalised value paid, the date
16
from which the abatement takes effect.
Apportionment of
the amount of Amount
Serial Name of Aadhar Bank A/c
17 compensation. payable *Remarks
No. claimants No. No.
to each
Area (in hectares)

Date on which possession was taken u/s. 38(1) and 40(I) of Act 30 of 2013.

If under section 40(1) the number and date of the order of Government giving authority to
do so.

Date: Signature

Bank account details to be collected in all cases where Aadhaar number is not available
or Aadhaar is not seeded in the bank account of the claimant.

Page | 17
IOV-Registered Valuers Foundation

2. BUILDING RULES AND REGULATIONS


Building Rules and Regulations of Local Bodies as well as Development Control Rules &
Regulations of different urban development authorities for feasibility of
Development/Redevelopment on the Land – Rules for Open Space, FSI and Plinth Area
Restrictions.

SOURCE OF BUILDING LAW ANDRULES

Building laws and rules are derived from different sources:

1) National Building Code, 2016

2) Model Building Bye Laws

3) Master Plan & Zoning Rules

1) NATIONAL BUILDING CODE, 2016

The National Building Code of India (NBC), a comprehensive building Code, is a national
instrument providing guidelines for regulating the building construction activities across
the country. It serves as a Model Code for adoption by all agencies involved in building
construction works be they Public Works Departments, other government construction
departments, local bodies or private construction agencies. The Code mainly contains
administrative regulations, development control rules and general building requirements;
fire safety requirements; stipulations regarding materials, structural design and
construction (including safety); building and plumbing services; approach to
sustainability; and asset and facility management.

The Code was first published in 1970 at the instance of Planning Commission and then
first revised in 1983. Thereafter three major amendments were issued to the 1983
version, two in 1987 and the third in 1997. The second revision of the Code
was in 2005, to which two amendments were issued in 2015.Due to large scale changes
in the building construction activities, such as change in nature of occupancies with
prevalence of high rises and mixed occupancies, greater dependence and complicated
nature of building services, development of new/innovative construction materials and
technologies, greater need for preservation of environment and recognition of need for
planned management of existing buildings and built environment, there has been a
paradigm shift in building construction scenario. Considering these, a Project for
comprehensive revision of the Code was taken up under the aegis of the National
Building Code Sectional Committee, CED 46 of BIS and its 22 expert Panels; involving
around 1 000 experts. As a culmination of the Project, the revised Code has been
brought out in 2016 as National Building Code of India 2016 reflecting the state-of-

Page | 18
L&B/Chapter-1/Law-Real Estate

the-art and contemporary applicable international practices. The comprehensive NBC


2016 contains 12 Parts some of which are further divided into Sections totalling 33
chapters (see Annex 1).The salient features of the revised NBC (see Annex 2) include,
apart from other changes made, the changes specially in regard to further enhancing our
response to meet the challenges posed by natural calamities. The major changes
incorporated in this third revision of the Code are as follows:
• Provisions for association of need based professionals and agencies have been
updated to ensure proper discharge of responsibilities for accomplishment of building
project.
• With a view to ensuring ease of doing business in built environment sector, a detailed
provision for streamlining the approval process in respect of different agencies has
been incorporated in the form of an integrated approval process through single
window approach for enabling expeditious approval process, avoiding separate
clearances from various authorities.
• Further, with a view to meeting the above objective, the provision on
computerization of approval process has been detailed, enabling online submission of
plans, drawings and other details, and sanction thereof, aiding in speedier approval
process.
• The mechanism of ensuring certification of structural safety of buildings by the
competent professional and peer review of design of buildings, have been further
strengthened.
• Requirements for accessibility in buildings and built environment for persons with
disabilities and the elderly have been thoroughly revised and updated.
• Provisions on fire and life safety have been thoroughly revised to meet the challenges
of modern complex building types including the high rises.
• Latest structural loading and design and construction codes including those relating
to wind load, earthquake resistant design of buildings, steel design and foundations
have been incorporated with a view to ensuring structural safety of buildings
including against a disaster.
• Provisions relating to all building and plumbing services have been updated keeping
also in view the latest international practices as related to the country.
• Provisions have been updated to ensure utilization of number of
new/alternative building materials and technologies to provide for innovation in the
field of building construction.
• Construction management guidelines have been incorporated to aid in timely
completion of building projects with desired quality in a safe manner within the
budgeted cost.
• Guidance has been provided for making buildings and built environment energy
efficient and environmentally compatible, through the newly introduced and updated
chapter on sustainability, namely Part 11 ‘Approach to Sustainability’

Page | 19
IOV-Registered Valuers Foundation

• New chapters have been added on structural use of glass; escalators and moving
walks; information and communication enabled installations; solid waste
management; and asset and facility management.

The Code has been published in two volumes containing all the Parts and Sections.

ANNEX 1

CONTENTS OF NBC 2016


VOLUME 1
PART 0 INTEGRATED APPROACH – A PRE-REQUISITE FOR APPLYING THE PROVISIONS
OF THE CODE
PART 1 DEFINITIONS
PART 2 ADMINISTRATION
PART 3 DEVELOPMENT CONTROL RULES AND GENERAL BUILDING REQUIREMENTS
PART 4 FIRE AND LIFE SAFETY
PART 5 BUILDING MATERIALS
PART 6 STRUCTURAL DESIGN
Section 1 Loads, Forces and Effects
Section 2 Soils and Foundations
Section 3 Timber and Bamboo
3A Timber
3B Bamboo
Section 4 Masonry
Section 5 Concrete
5A Plain and Reinforced Concrete
5B Prestressed Concrete
Section 6 Steel
Section 7 Prefabrication and Systems Building and Mixed/Composite
Construction
7A Prefabricated Concrete
7B Systems Building and Mixed/Composite Construction
Section 8 Glass and Glazing
VOLUME 2
PART 7 CONSTRUCTION MANAGEMENT, PRACTICES AND SAFETY
PART 8 BUILDING SERVICES
Section 1 Lighting and Natural Ventilation
Section 2 Electrical and Allied Installations
Section 3 Air Conditioning, Heating and Mechanical Ventilation
Section 4 Acoustics, Sound Insulation and Noise Control
Section 5 Installation of Lifts and Escalators and Moving Walks

Page | 20
L&B/Chapter-1/Law-Real Estate

5A Lifts
5B Escalators and Moving Walks
Section 6 Information and Communication Enabled Installations
PART 9 PLUMBING SERVICES (INCLUDING SOLID WASTE MANAGEMENT)
Section 1 Water Supply
Section 2 Drainage and Sanitation
Section 3 Solid Waste Management
Section 4 Gas Supply
PART 10 LANDSCAPE DEVELOPMENT, SIGNS AND OUTDOOR DISPLAY STRUCTURES
Section 1 Landscape Planning, Design and Development
Section 2 Signs and Outdoor Display Structures
PART 11 APPROACH TO SUSTAINABILITY
PART 12 ASSET AND FACILITY MANAGEMENT

ANNEX 2

Salient Features of National Building Code 2016 (NBC 2016)


1) Detailed provision for streamlining the approval process in respect of different
agencies in the form of an integrated approval process through single window
approach thereby avoiding separate clearances from various authorities, with a view
to ensuring ease of doing business in built environment sector.
2) Progressive computerization of approval process, for enabling online submission
of plans, drawings and other details, and sanction thereof.
3) Updated mechanism of ensuring certification of structural safety of buildings by
the competent professional and peer review of design of buildings.
4) Defining the roles and responsibilities of all professionals and contractors involved in
a building construction project.
5) Comprehensive planning norms for minimum amenities to be provided in a
city/town.
6) Detailed provisions relating to requirements for accessibility in buildings and built
environment for persons with disabilities and the elderly.
7) Planning and development norms, such as, Transferable Development Rights (TDR)
and Accommodation Reservation (AR).
8) Provisions for underground or multi-storeyed parking as also mechanized
parking of vehicles.
9) Norms for solar energy utilization.
10) Requirements for buildings on podium for ensuring fire and life safety in such
buildings.
11) Fire and life safety in modern complex buildings including the high rises, glazed
buildings, atria, commercial kitchen and car parking facilities.

Page | 21
IOV-Registered Valuers Foundation

12) Updated structural design provisions for wind and seismic loads, imposed load
due to helipad, and blast loads, for safe design and construction of buildings with
due focus on ductile detailing.
13) Latest research and development inputs and provisions on concrete, steel and
masonry buildings with a view to ensuring disaster resilient buildings.
14) Assessment of liquefaction potential of a site and ground improvement techniques
for maximum utilization of land resources including at seismically vulnerable sites.
15) Updated provisions on engineered use of bamboo in housing and other building
construction.
16) Promotion of use of agricultural and industrial wastes including construction and
demolition wastes in building construction without compromising the quality and
safety.
17) Inclusion of provisions on self compacting concrete, high performance concrete and
steel fibre reinforced concrete.
18) Updated provisions on prefabricated construction technique for speedier
construction.
19) New chapter on structural use of glass in buildings.
20) New and alternative building materials, and technologies for building
construction such as, reinforced masonry, confined masonry building construction
and masonry wall construction using rat-trap bond.
21) Construction project management guidelines for timely completion of building
projects within the budgeted cost with desired quality.
22) Habitat and other welfare requirements for workers at construction site.
23) Inclusion of modern lighting techniques such as LED and induction light and their
energy consumption.
24) New provisions on compact substations and updated provisions on installation of
energy meters.
25) Comprehensive provisions relating to lightning protection of buildings.
26) Provisions on aviation obstacle lights; electric vehicle charging and car park
management.
27) Protection of human beings from electrical hazards and against fire in the building
due to leakage current.
28) Use of refrigerants for air conditioning addressing zero ozone depletion
potential (ODP) and ultra-low global warming potential (GWP).
29) Inclusion of new and energy efficient options of air conditioning, heating and
mechanical ventilation, such as variable refrigerant flow system, inverter technology,
district cooling system, hybrid central plant using chilled beams, radiant floor
components, and geo-thermal cooling and heating.
30) Thrust on envelope optimization using energy modelling, day lighting simulation,
solar shade analysis and wind modelling software to optimize the air conditioning

Page | 22
L&B/Chapter-1/Law-Real Estate

load.
31) Air conditioning, heating, and ventilation (HVAC) provisions considering adaptive
thermal comfort conditions for energy efficiency.
32) Provisions pertaining to metro train ways and metro stations with respect to fire
and life safety; and air conditioning, heating and ventilation for metro stations.
33) HVAC requirements for data centres and healthcare facilities; refrigeration for cold
stores; efficient strategies for winter heating using reverse cycle operation, solar
heating systems, ground source heat pump and electric heat pump; and modern
system of mechanical ventilation for industries, commercial kitchen and
underground car parking.
34) Updated provisions on building automation system to include the latest practices
for web-based monitoring and control of performance parameters.
35) High speed lifts for tall buildings.
36) New chapter on escalators and moving walks for comfortable and safe movement
of people.
37) New chapter on information and communication enabled installations in
buildings.
38) Updated provisions on water supply, drainage and sanitation for modern high rise
buildings and complexes.
39) Provisions relating to swimming pools covering hygiene and safety.
40) Updated provisions on rainwater harvesting.
41) New chapter on solid waste management covering various solid waste
management systems within the building and building complexes.
42) Updated provisions on piped gas supply in houses, and in hospitals for medical
purposes.
43) Promoting quality of outdoor built environment through updated provisions
on landscape planning, design and development.
44) Promoting sustainability in buildings and built environment in tandem with relevant
sustainable development goals.
45) New chapter on asset and facility management to cover provisions relating to
management of building assets and associated services, also covering
responsibilities of occupants for maintenance of facilities, such as structures,
equipment and exterior property.

Page | 23
IOV-Registered Valuers Foundation

2) MODEL BUILDING BYE LAWS 2016

Building Bye-Laws are legal tools used to regulate coverage, height, building bulk, and
architectural design and construction aspects of buildings so as to achieve orderly
development of an area. They are mandatory in nature and serve to protect buildings
against fire, earthquake, noise, structural failures and other hazards. In India, there are
still many small and medium sized towns which do not have building bye-laws and in the
absence of any regulatory mechanism, such towns are confronted with excessive
coverage, encroachment and haphazard development resulting in chaotic conditions,
inconvenience for the users, and disregard for building aesthetics, etc. It is in this context,
TCPO has made an effort to prepare “Model Building Bye-Laws- 2016” for the guidance
of the State Governments, Urban Local Bodies, Urban Development Authorities, etc
which is an improvement over the previous Model Building Bye Laws brought out in
2004.
In 2003, the Ministry of Urban Development desired that Model Building ByeLaws be
prepared, in view of Bhuj Earthquake that occurred in 2001, to lay focus on structural
safety of buildings and for the guidance of the State Governments. Accordingly, the
MBBL 2004 incorporated the provisions of structural safety and other provisions like
rainwater harvesting and waste water recycling, solar assisted heating, barrier free public
buildings and fire safety. The Bye-Laws were circulated to all the State Governments and
Union territories and out of 36 States and UTs, wherein 22 States and UTs have
undertaken comprehensive revision of their respective Building Bye-Laws since 2004.
The Reason deter for revising the Bye-Laws are as under:
• Growing Environmental concerns

• Increased Safety and Security measures

• Technological Developments

• Swachh Bharat Mission

• Focus on Ease of Doing Business In 2015,

It was further desired by the Ministry of Urban Development that the Model Building
Bye-Laws, 2004 needs to be revised and updated keeping in view the emerging issues
like Norms for Rooftop Solar PV Installation, Segregated sanitation facilities for visitors in
public buildings, Additional provisions in Building regulations for natural hazard prone
areas, Conservation of heritage sites including heritage buildings, heritage precincts and
natural feature areas, Bye-laws for safe use of glass, barrier free environment for
disabled, children and old persons and Mitigation of the effects of electromagnetic
radiation on built spaces.

Page | 24
L&B/Chapter-1/Law-Real Estate

The Draft Model Building Bye-laws, 2016 were circulated to Central Governments
agencies/ institutes like National Disaster Management Authority, Bureau of Indian
Standards, National Building Construction Corporation, Delhi Development Authority,
National Capital Region Planning Board, Indian Institute of Public Administration,
Municipal Corporation of Delhi (South), Housing and Urban Development Corporation,
Schools of Planning and Architecture, State Town and Country Planning Departments,
selected Urban Development Authorities, selected Urban Local bodies and associations
like CREDAI and NAREDCO for obtaining their comments and suggestions.

A National Workshop on Draft Model Building Bye Laws, 2016 was held on 18.2.2015
mainly to obtain comments / suggestions from the State Town and Country Planning
Departments/Urban Development Authorities/ Municipal Corporations and Schools of
Planning and other related stakeholders like Association of Real Estate Developers and
private consultants.

The workshop was attended by 110 participants and comments/suggestions on


following issues emerged out in the day long deliberations-
• Norms for High Rise Buildings (Chapter 5)
• Norms for differently abled, senior citizens and children (Chapter 8)
• Sustainability and Green buildings (Chapter 10)
• Streamlining building pla approvals, Ease of Doing Business (Chapter 13)
• Norms for Low income housing
• Parking Norms (public parking space for vehicles in CBD)
• Flexible FAR prescriptions
• Incorporation of new terms in definitions
• Updations of all IS Codes of BIS
• Updating fire provisions with respect to staircase and exits.

A review meeting was held under the chairmanship of Secretary (UD) on 06.07.2015
wherein Joint Secretary (UD) made a presentation on the draft MBBL, 2016. In the
meeting, it was decided that the draft may also be circulated to related Ministries
involved in granting NOCs for construction projects. Comments were received from the
Ministries of Culture, Consumer Affairs, Food and Public Distribution, Housing and Urban
Poverty Alleviation, Power, Water Resources and River development and Ganga
Rejuvenation and Civil Aviation. The relevant comments received from all the concerned
have been duly incorporated in the MBBL, 2016. On18th March, 2016, a revised set of
Model Building Bye Laws, 2016 (‘2016 Bye-Laws’) was released.

Page | 25
IOV-Registered Valuers Foundation

THE SALIENT FEATURES OF MBBL- 2016 ARE GIVEN AS UNDER –

Safety and security (Chapter-6): includes Structural Safety, Disaster management as


per Prof. Arya Committee Report and BIS Codes including Structural Design Basis
Report (SDBR) for various building types. Prevention measures against “Soft Storeys”
in multi-storeyed buildings and Proof Checking of Structural Design for buildings.

• Barrier Free Environment: Provisions for Differently abled, Elderly and Children
including Site development, Access Path/ Walk Way, Parking, Building
requirements, Stair, Lifts, Toilets, Drinking Water, Refuge and signage.

• Environmental Concerns: Additional Chapter-10 incorporated for Green Buildings


and Sustainability provisions, Rainwater Harvesting, Wastewater Reuse and
Recycle and installation of Solar Roof Top PV norms.

• Adoption for Modern Construction Technology: Additional Chapter-5


incorporates Definitions, Structural Safety and other provisions for Highrise
Building regulations with Parking, Peripheral Open Spaces including set-backs.
Disaster Management & Fire Safety.

• Swachh Bharat Mission: Revised Norms for adequate toilet facilities for women
and Public Conveniences in Public Buildings and Mandatory Provisions for
Segregated toilet facilities for visitors in Public Buildings.

• Ease of Doing Business: Additional Chapter-14 includes Provision for Online


Building plan approval process adopting automated systems of plan scrutiny,
generation of reports/approvals and integrated systems of intimation of approval,
Compliance report from drawings for automatic generation of Completion
certificate and integration of various clearances at Master Plan levels and
introduction of Integrating “Single Window” process, Empowering Architects,
Outsourcing non discretionary verification jobs and formulation of Citizen’s
Charter.

• Rain Water Harvesting: Additional Chapter-9 Indicative Provisions for harvesting


in various types of buildings along with responsibility of ULBs for RWH in public
spaces, provisions for Enforcement and Monitoring.
• Effects of Communication Technology: Additional Annexure –I, Draft Guidelines
for mitigation of Electro Magnetic Radiations in built spaces by identifying
emission appliances/ sources. Guidelines for safe spacing of indoor and outdoor
appliances and equipment.

3) MASTER PLAN AND ZONING RULES

Where the Town and Country planning Acts are operational usually the
Directorate of Town and Country Planning oversees planning and development in urban
and rural areas by way of issuance of Master Plans prepared for the urban centers and
notified rural areas by indicative Land Use Plans. Such Directorates exist in almost all
states viz Andhra Pradesh, Odisha, Assam, Tamil Nadu, Chhattisgarh, Haryana, and

Page | 26
L&B/Chapter-1/Law-Real Estate

Himachal Pradesh. Jharkhand has a Town and Country Planning Organization (TCPO)
under the Urban Development Department and the Master Plans for Jharkhand was
prepared for the Urban Development Dept by a private agency.

The Town and Country Planning Organization (TCPO), technical arm of the Ministry of
Urban Development, Government of India, is an apex technical advisory and consultant
organization on urban and regional planning strategies and monitoring of central
government schemes and development policies.

Master Plan derives authority from the parent Act. In Jharkhand Master Plan is a
document prepared under the Jharkhand Regional Development Authority Act. 2001.
The Delhi Development Act 1957 gives sanctity to the master plan and zonal
development plans. As per sec 14 of Delhi Development Act use of land or building other
than in conformity with the Master Plan is prohibited. The current version as on the date
of registration of Sale Deed or date of application for sanction of plan applies. There is
no vested right to purchaser for claiming NOC for sanction of building under earlier
Master Plan on the basis of date of Sale Agreement or otherwise. Further no sanction
had been communicated to applicant under earlier plan (Dewan and Sons Investments
(P) Ltd. vs. DDA, AIR 1997 Del 388). In case of Mumbai earlier the Development Plans
were issued under Sec 26 (1) Maharashtra Regional and Town Planning Act, 1966. Later
these were issued under the Metropolitan Development Authority constituted under the
Mumbai Metropolitan Region Development Authority Act 1974.

Master Plans do not exist in every state/ city. But especially after Jawaharlal Nehru
National Urban Renewal Mission (JNNURM) the larger cities included under JNNURM
has Master Plans which amongst others divide the city into zones to regulate the type of
usage the land in that zone can be put to. There is usually no penalty in every state for
such violations. The Zoning and Development Promotion Regulations issued by
Hyderabad Metropolitan Development Authority has following zones:

• Residential Use Zone

• Commercial Use Zone, including commercial along notified commercial roads;


strip commercial along roads as earmarked in the master plan and areas
earmarked as Commercial use in the Master Plan

• Multiple Use Zone, including areas covered in Transit-Oriented Development


(TOD) zone

• Public and Semi - Public Use Zone

• Work Centre Use Zone

• Open Space Use Zone - Parks, Playgrounds, Exhibition grounds, Green buffer
zone

Page | 27
IOV-Registered Valuers Foundation

• Water bodies Use Zone - River, stream, Nalah, Storm Water Drains, Lakes

• Transportation Use zone (Road, rail, Airport, Bus depots, Terminals, Workshops,
Truck terminals, Warehouses, Parking areas/Parking lots/Parking complex

• Special Reservation Use zone

• Sites specifically earmarked as heritage conservation - buildings and


precincts/areas, rocks & hillocks/Natural heritage

• Defence/Military lands

• Burial grounds, Cremation grounds etc

• Special Area Development Plan (SADP) areas

Hyderabad's zoning provision overtly clarifies the proposed activities and uses in the
proposed buildings/premises in addition to the conformity with these regulations shall
be in conformity with the Hyderabad revised building rules, 2006 provisions. The
notification provides where there is an inconsistency between the building rules and the
zoning regulations, the provisions of the zoning regulations shall prevail

In Bangalore the BDA layouts are completely planned layouts, with access roads,
planned transportation hubs, major and minor roads of necessary width, sites set apart
for schools and hospitals, etc... There are 62 layouts. ... Sites allotted under the BDA
scheme may only be used for the specific purpose that they are planned for – residential,
commercial, or for providing social infrastructure.

Draft Development Control Regulations for Development Plan applicable in


municipal areas of Pune, every application for a building permission/ commencement
certificate shall be accompanied by the following "for verifying the proof of ownership.

• Latest 7/12 extract (or the property register card) indicating date of issuance
within six months, showing the area in words and figures

• Measurement Plan issued by authority, having date of demarcation within 4 years,


the only exception being recently sanctioned plotted layout.

• Title Search report by advocate who is enrolled with the Bar council.

• Other documents including no objection certificate as requisitioned by the


Municipal Commissioner"

These municipal provisions are mandatory since these requirements set out in executive
orders are issued under powers allowed by the parent act. OswalAgro Mills Ltd. Vs.
Hindustan Petroleum Corporation Ltd. etc Civil Appeal No. 10933 of 2013 the suit
land had been indicated as being earmarked for housing since the 1992 plan. SC through
its majority judgment directed the Municipal Commissioner to reconsider the application

Page | 28
L&B/Chapter-1/Law-Real Estate

made by OswalAgro Mills Ltd. keeping in view the objections raised by the Police
Department, Ministry of Petroleum, Ministry of Environment and Intelligence Bureau and
the Security Control Regulations issued by the State of Maharashtra.

But master plans may not integrate well with plans of other authorities like the Regional
Development Authority, any Mineral Area Development Authority (MADA) or the Mines
Board.

EXISTING BUILDINGS USE

As per the bye-laws of the Ranchi Municipal Corporation, an Existing Buildings is a


building, structure or its use as sanctioned/approved/ regularized by the competent
authorities. The existing buildings are classified into different types as follows.

Residential For normal residential purposes with or without cooking or dining or


Buildings both facilities. It includes one or two or multi-family dwellings,
apartment houses and flats.
But the definition under the relevant Act must be referred to avoid
confusion. For instance, as per Delhi Rent Control Act for the limited
purpose of computing Standard Rent public hospitals, educational
institutions, public libraries, reading rooms and orphanages are
considered as residential premises.
Institutional Used for medical or other treatment or care of persons suffering
Buildings from physical or mental illness/disease or infirmity, care of infants,
convalescents or aged person and for penal or correctional detention
in which the liberty of the inmates is restricted. Institutional buildings
ordinarily provide sleeping accommodation for the occupants and
include hospitals, sanatoria, jails, prisons, mental hospitals,
reformatories.
Educational These shall include any building used for school, college or day care
Buildings purpose for more than an hour per week and involves assembly for
instruction, education or recreation.

Assembly where people congregate or gather for amusement, recreation,


Buildings religious, patriotic, civil, travel and similar purposes. E.g. theatres,
motion picture houses (Cinema), assembly halls, auditoria, exhibition,
halls, museums, skating rinks, gymnasium, restaurants, places of
worship, dance halls, club rooms, passenger stations, and terminals
of air, surface and other public transportation services, recreation
places and stadium.

Page | 29
IOV-Registered Valuers Foundation

Business These shall include any building or part of a building which is used
Buildings for transaction of business, for keeping accounts and records for
similar purposes, doctor’s service facilities, City halls, Town halls are
classified in this group in so far as principal function of these is
transaction of public business and the keeping of books and records.

Mercantile/ Office, shops, stores, market, for display and sale of merchandise
Commercial either wholesale or retail (includes stores and service facilities
Buildings incidental to the sale of merchandise and located in the same
building); hotels, dormitories and lodging houses shall also come
under this category.
Industrial products or materials of all kinds and properties are produced/
Buildings manufactured E.g. assembly plants, Laboratories, power plants,
smoke houses, refractory, gas plants, mills, dairies, factories etc.

Storage whole or part of building used for storage and sheltering of goods
Buildings: e.g. warehouse, cold storages, freight depots, transit sheds, store
houses, garages, aero plane hangars, truck terminals, grain elevators,
barns and stables.

Hazardous highly combustible or explosive materials are stored handled


Buildings:

The category of existing building use is a very material consideration when allowing
conversion to other building use.

In, ‘Nihal Chand Lallu Chand Pvt. Ltd. vs. Pancholi Cooperative Housing (AIR 2010 SC
3607)’ Supreme Court held that any requirement for giving an undertaking contrary to
Development Controlled Regulations for Greater Bombay 1991(DCR) will not be binding
either on the flat purchasers or the Society.

Generally, the additional area in the precincts of monument under Archaeological Survey
of India is demarcated and fenced/ bounded. But future development needs, around the
site needs to be kept in mind and development rights restricted of the properties nearby.

Existing properties may be notified for being vacated since lying within certain distance
of shoreline/ river line etc.

Some specific provisions are:

1) PARKING SPACE

Providing parking for visitor's vehicle is increasingly a requirement. Development Control


Rules of Greater Mumbai - in Table 15 Regulation 36 - requires that 25 percent (earlier

Page | 30
L&B/Chapter-1/Law-Real Estate

10 percent) of the parking space has to be kept for visitor's cars.

According to Regulation 36 of the Development Control Regulations (1991) for Greater


Bombay, whenever a property is developed or redeveloped, parking spaces have to be
provided in accordance with the guidelines in the DCR. Before a builder can develop or
redevelop a project, he needs to obtain the development permission and a
commencement certificate. "To get these approvals, he needs to submit a building plan,
which includes the details of the parking spaces

OS Bajpai's case supra clarifies that parking space in stilt area of building cannot be sold
by builders.

2) OFFSET

Supreme Court decided in Anil Sen Gupta and Dhirendra N Basak 1998 SCC 547 the
offset to be kept from the side wall privately committed to the selling Cooperative
Society is a private covenant, even if the quantum provided in it is higher than that
specified in the applicable building rules. Court clarified that on a plain reading the
building rule R 40 of Bengal Municipal Act, 1932 only specifies the minimum norm of
offset and anything higher in a private covenant must be honoured.

3) NO OBJECTION CERTIFICATE

Approval or permissions are needed under various Act Works of Defence Act, 1903, The
Explosives Act (for e.g. petrol pumps, stock depots of firecrackers, explosion), State
Pollution Control Board, National Green Tribunal etc. Permission may also be required
from Station Commander of the local defence establishment of Armed Forces, Border
Security Forces or Central Reserve Police Force if construction is within a specified
distance of those establishments. Airport Authority of India may need to approve if
location is within a certain distance or within the takeoff or landing pathway.

The Pune's Development Control Regulation, 2013 in point 6.2.13 requires application for
development permission to be accompanies by No objection Certificate obtained from
the authorities like "Civil Aviation Authorities, Railway, Directorate of Industries,
Maharashtra Pollution Control Board, Central Pollution Control Board, District Magistrate,
Inspectorate of Boilers and Smoke Nuisance etc."

Page | 31
IOV-Registered Valuers Foundation

3. RENT CONTROL LAWS


Sections pertaining to Occupancy Rights of Tenants, Freezing of Rent and Protection
against Eviction of Tenant and its effect on value of property.

Rent and tenancy is regulated by state acts for e.g. some of the state level acts are
enumerated below:

1) West Bengal Premises Tenancy Act, 1997 as amended in 2002 initially notified on
28th December, 1998 and applicable to Kolkata, Howrah and some other areas
(which repealed West Bengal Premises Tenancy Act, 1956)

2) Tamil Nadu Buildings (Lease and Rent Control) Act, 1960

3) Jharkhand Building (Lease, Rent & Eviction) Control Act, 2011

4) Bihar Building (Lease, Rent and Eviction) Control Act, 1982 (which replaced The Bihar
Building (Lease, Rent and Eviction) Control Act, 1977)
5) Maharashtra Rent Control Act, 1999 (operational since March 2000. Earlier three
Acts were operational in the state of Maharashtra: Bombay Rent, Hotel and Lodging
House Rent Control Act for Bombay region; the Central Provinces and Berar Act were
prevailing in the area of Vidarbha and Hyderabad House (Rent, Eviction and Lease)
Control Act 1954 for Marathwada Region.

6) Haryana Urban (Control of Rent and Eviction) Act, 1973

7) The Himachal Pradesh Urban Rent Control Act, 1971 (Act No. 23 of 1971)

8) The Jammu and Kashmir Houses and Shops Rent Control, 1966
9) Uttar Pradesh Urban Buildings (Regulation of Letting, Rent and Eviction) Act, 1972

10) Delhi Rent Control Act, 1958 is still applicable in Delhi - it applies to tenancies with
Rs. 3500 or less paid as monthly rent. Another Act Delhi Rent Control Act 1995 was
enacted by Parliament and received presidential assent but was not implemented (as
it had triggered a major agitation by tenants in Delhi) and steps initiated to repeal it
in 2013 through introduction of a bill in the Rajya Sabha has not progressed. The
1958 Act is the subject of a petition by an association of landlords with Advocate
Shoba Agarwal as a member and being heard by Delhi High Court on enacting 1995
Act. But central government in 2015 has said it wishes to repeal the 1995 Act and
enact another.

These legislations are enacted to regulate the rent payable be tenant. Many acts seek to
prevent it from exceeding standard rent, regulate repair and maintenance of the
property and also regulate eviction of tenants. Rent Control Acts of different states favor
the tenants. The Rent Control Act generally only applies on lease agreements of at least
12 months, and hence an eleven-month agreement helps landlords to take a pre-

Page | 32
L&B/Chapter-1/Law-Real Estate

emptive measure of avoiding filing eviction suit against tenant to claim back possession
of premise. Sec 106 of Transfer of Property Act has limited application where ever a State
level legislation regulating rent and lease prevails.

LANDLORD/ TENANT DEFINED

Landlord - As per the West Bengal Premises Tenancy Act, 1997 the definition includes
any person who, for the time being, is receiving, or is entitled to receive, the rent for any
premises, whether on his own account or for another as a trustee/ guardian or court
receiver or on behalf of another. The term 'Landlord' also includes one who would have
received the rent as described here if the premises were let to a tenant.

The term "premises" means whole of part of any building or any hut or part of a hut that
is let out separately, and includes the gardens, grounds and out-houses appurtenant to
it. It also includes any furniture/ fixtures/ fittings supplied by the landlord for the use of
the tenant (but do not include a room in a hotel or a lodging house).

A lease for running a factory comprising costly machineries for the purpose of
manufacturing is not a premise - Alliance Jute Mills v Alliance & Co., 82 CWN 1055;
Venkaya v Subba Rao, AIR 1957 AP 619

Any space on roof parapet on the outer wall let out for hoarding or advertisement is not
a premise for which a tenancy can be granted- Durjendra v K. Shaw, AIR 1953 Cal 147:
ILR (1954)1 Cal 201.

Tenant refers to a person who has to pay rent or on whose behalf rent is payable for any
premise or would have had to pay rent but for a special contract. It includes

1) Any person continuing in possession after termination of his tenancy


2) If tenant dies, for residential properties, the definition also includes, for up to five
years from the date of death members of his family, who were his dependents
ordinarily staying with him and owning no other residential premise (namely his son,
daughter, parent and the widow of his predeceased son). The definition includes
spouse and no bar of five years applies. They shall also have a right of preference for
tenancy in a fresh agreement on payment of fair rent.
3) In respect of premises let out for non-residential purpose his spouse, son, daughter
and parent who were ordinarily living with the tenant up to the date of his death as
members of his family, and were dependent on him or a person authorized by the
tenant who is in possession of such premises
4) Does not include any person against whom any decree or order for eviction has
been made by a Court of competent jurisdiction

Page | 33
IOV-Registered Valuers Foundation

PREMISES EXEMPTED FROM PROVISIONS OF TENANCY ACT

Some premises are exempted from the provisions of rent control/ tenancy regulation as
per the West Bengal Premises Tenancy Act, 1997 as

1) As per Sec 3, the Act does not apply to any premises owned by the Central or State
Government, or a Government undertaking or enterprise, or a Cantonment Board or
a local authority or other statutory body which is not a local authority
2) Any tenancy created by the Central Government, or any State Government in respect
of the premises taken on lease or requisitioned by that Government

3) Any tenancy where the lease with due consent of the tenant has been registered
under the Registration Act, 1908 (16 of 1908), after the commencement of this Act,
and the fact of such consent has been recorded in the instrument so registered
4) If premise is rented to a foreign mission or international agency; or a tenancy
created by a foreign mission or an international agency, either by way of lease or
otherwise
5) A residential premise located within limits of Kolkata or Howrah Municipal
corporation for which Rs. 6500/- monthly rent is payable or monthly rent of
Rs.3500/- to other areas where the Act extends. While for a non-residential premise
located within limits of Kolkata or Howrah Municipal Corporation for which Rs
10000/- as monthly rent is payable or Rs. 5000 as monthly rent in other areas to
which this Act extends. If the tenancy is for dual purpose then respective rate shall
apply in proportion to respective areas

Central and State government properties are exempted in most Act. But state acts may
have specific provisions too. For example, as per the Jharkhand Building (Lease, Rent &
Eviction) Control Act, 2011 the provisions of tenancy contained therein shall not apply to
any building belonging to Jharkhand State Shwetamber Jain Trust Board or Jharkhand
State Digamber Jain Trust Board or Wakf Board or Hindu DharmikNyas Board or any
other Religious and Charitable Trusts or institution run by the Central or State
Government.

OBLIGATIONS OF LANDLORD AND TENANT

1) OBLIGATION OF LANDLORD

• Issuance of a written receipt in the prescribed form signed by landlord/


authorised agent on receiving the rent or any charge for the maintenance of
premises
• Keep the premises in good and tenantable condition and ensure due
maintenance of essential supply/ service
• No further charge can be claimed if tenant authorised (within tenancy
provision) way sublets the premise held by him

Page | 34
L&B/Chapter-1/Law-Real Estate

2) OBLIGATIONS OF TENANT

• Pay rent to the landlord or his authorised agent within the prescribed period.

• Use the premise for what it was let out to the tenant

• The landlord or his authorised agent will be allowed to enter upon the
premises and inspect the condition after due service of a notice on tenant of
the intention to do so

• Not add, alter the premises without the written consent of the landlord

• Without written consent of the landlord not to sublet, or transfer or assign his
right in the tenancy wholly or partly

• Pay charges on amenities of the premises at the rate of ten per cent of the
fair rent or the agreed rent and pay his share of municipal tax as an occupier
of the premises as required in the appropriate Municipal Act (in Bengal it is
Kolkata Municipal Corporation Act, 1980 or the West Bengal Municipal Act,
1993.

"Occupier" includes any person at that time liable for paying or actually paying to the
owner the rent or fee or any part thereof or paying damages on account of the
occupation of any land or building, and also includes a rent-free tenant: However an
owner living in or otherwise using his own land or building shall be deemed to be the
occupier thereof.

PROTECTION OF UNLAWFUL EVICTION OF TENANTS

"One half of the lis between landlord and tenant would not reach courts, if tenant agrees
to pay the present prevalent market rate of rent of the tenanted premises to the
landlord. In that case landlord would also be satisfied that he is getting adequate, just
and proper return on the property. But the trend in the litigation between landlord and
tenant shows otherwise. Tenant is happy in paying the meagre amount of rent fixed
years ago and landlord continues to find out various grounds under the Rent Acts, to
evict him somehow or the other."

(Obiter dicta in Mohammad Ahmad &Anr vs. Atma Ram Chauhan & Ors.CA No.4422 of
2011 [Arising out of SLP.(C) No.6319 of 2007 decided by Supreme Court Justices Dalveer
Bhandari and Deepak Verma]

The West Bengal Premises Tenancy Act says its provisions will prevail over other acts in
matters of suits for eviction of tenants filed by the Owner as the Courts have been barred
from passing any order/ decree in favour of the landlord against the tenant normally. So
if rent control act was not applicable on a premise, an owner can proceed against a

Page | 35
IOV-Registered Valuers Foundation

tenant on expiry of the notice of eviction served under Section 106. But once rent control
applies, the owner must prove grounds permitted under rent control act entitling him to
evict the tenant. The grounds on which landlord can file a suit for eviction of tenant
usually after serving a month's notice on the tenant as per the West Bengal Act are:

1) Without consent of owner in writing if the tenant has sublet/ assigned/ parted with
the whole or part of the property or used it for purpose other than purpose it was
rented.

2) Not paid rent for three months in a period of 12 months or thrice in a three year
where rent is not payable monthly.

3) Where the premises are required by the landlord for building or rebuilding or adding
or altering substantial portion and which cannot be carried out without the premises
being vacated. Or if the landlord or any beneficiary of the premise needs it for self-
occupation and the landlord does not have any suitable accommodation within the
same Municipal Corporation/ Municipality or in any other area within ten kilometres
from such premises where this Act extends. However, if the competent court deems
that ordering vacating only a part of the premise for building/ altering etc or for self-
occupation will serve the purpose of the landlord it may accordingly order if the
tenant/ subtenant agrees. If subtenant agrees but tenant does not then too
subtenant can continue in part of the portion.

4) Where the tenant has given notice to quit but not given vacant possession of the
premises to the landlord

5) Tenant or person residing therein have violated provisions of sec 108 of the TPA (like
provisions of clause (m) to restore property in the condition received and repair
within three months defects caused by lessee according to directions of lessor;
clause (o) i.e. use property as permitted in the lease or clause (p) i.e. not erect any
permanent structure without lessor's permission of section)
6) Allowing use of premise for immoral or illegal purpose:

• Tenant is in default and has failed to prevent deterioration of the premises


• Tenant is causing nuisance or annoyance to the neighbor including the landlord;
• For tenant who have rented for residential purpose and if the tenant has acquired or
constructed, or allotted, a house or flat (one year is allowed for vacating the
premises)
• If the landlord is a member of the Armed Forces and requires it for occupation of his
family and produces a certificate of the prescribed authority certifying he is serving
at a non-family post or in special conditions
• If the rented property is not used and kept locked by the tenant or persons claiming
through him for 10 out of 12 months.
• Where landlord got the property via transfer, he may file a suit within a year of such
transfer for the recovery of possession of the premises on the ground of needing it
for building or rebuilding or addition or alteration or for own occupation
• A two-year bar on litigation on property to which an earlier Act applied

Page | 36
L&B/Chapter-1/Law-Real Estate

• When a tenanted premise is transferred then the transferor's right to evict a tenant
on grounds of subletting as per Sec 13(1)(a) not supported by tenancy deed is also
transferred.

In Mohammad Ahmad & Anr vs. Atma Ram Chauhan & Ors.CA No.4422 of 2011 [Arising
out of SLP.(C) No.6319 of 2007] Supreme Court Justices Dalveer Bhandari and Deepak
Verma formulated "guidelines and norms …so as to minimize landlord-tenant litigation
at all levels". Its ratio was when rent is paid on time despite any agreement between the
parties to the contrary; the landlord cannot evict the tenant for five years. It recognises
need of premise for personal use of landowner as an exception to the five-year rule.
Further rent must be increased by ten percent every three years or as agreed by the
parties. Being a Supreme Court judgement, it will apply across the country and to the
courts hearing related matters Hence it is important to review the guidelines as
formulated in the case which are as follows:

• The tenant must enhance the rent according to the terms of the agreement or at least by
ten percent, after every three years and enhanced rent should then be made payable to
the landlord. If the rent is too low (in comparison to market rent), having been fixed
almost 20 to 25 years back then the present market rate should be worked out either on
the basis of valuation report or reliable estimates of building rentals in the surrounding
areas, let out on rent recently.
• Apart from the rental, property tax, water tax, maintenance charges, electricity charges
for the actual consumption of the tenanted premises and for common area shall be
payable by the tenant only so that the landlord gets the actual rent out of which nothing
would be deductible. In case there is enhancement in property tax, water tax or
maintenance charges, electricity charges then the same shall also be borne by the tenant
only.
• The usual maintenance of the premises, except major repairs would be carried out by the
tenant only and the same would not be reimbursable by the landlord.
• If any major repairs are required to be carried out then in that case only after obtaining
permission from the landlord in writing, the same shall be carried out and modalities
with regard to adjustment of the amount spent thereon would have to be worked out
between the parties.
• If present and prevalent market rent assessed and fixed between the parties is paid by
the tenant then landlord shall not be entitled to bring any action for his eviction against
such a tenant at least for a period of 5 years. Thus, for a period of 5 years the tenant shall
enjoy immunity from being evicted from the premises.
• The parties shall be at liberty to get the rental fixed by the official valuer or by any other
agency, having expertise in the matter.
• The rent so fixed should be just, proper and adequate, keeping in mind, location, type of
construction, accessibility with the main road, parking space facilities available therein
etc.
• Care ought to be taken that it does not end up being a bonanza for the landlord.

Page | 37
IOV-Registered Valuers Foundation

These are some of the illustrative guidelines and norms but not exhaustive, which can be
worked out between landlord and tenant so as to avoid unnecessary litigation in Court.

Supreme Court of India in Ranjit Kumar Bose &Anr vs Anannya Chowdhury &AnrCA
No. 3334/2014 held that Section 6 of the Tenancy Act overrides a contract between the
landlord and the tenant … that Sec 6 of the Tenancy Act is one such law which clearly
bars arbitration in a dispute relating to recovery of possession of premises by the
landlord from the tenant. Since the suit filed by the appellants was for eviction, it was a
suit for recovery of possession and could not be referred to arbitration because of a
statutory provision in Section 6 of the Tenancy Act. This upholds decision in Natraj
Studios (P) Ltd. v. Navrang Studios & Anr,1981 1 SCC 523 which concluded that
arbitration agreements between parties whose rights are regulated by the Bombay Rent
Act cannot be recognized by a court of law since the exclusive jurisdiction is vested in
Bombay Small Causes Court.

In Rahman JeoWangnoo vs. Ram Chand (AIR 1978 SC 413) Supreme Court held that
the provision contained in the West Bengal Premises Tenancy Act, 1956 mandates the
court to consider whether partial eviction as contemplated therein should be ordered or
the entire building should be directed to be vacated. Supreme Court also held in
Anamika Roy v JatindraChowrasiya&ors CA No. 4539 of 2013 that in a three-bed room
flat with one kitchen, it is too harsh to consider partial eviction and order to divide a flat
between parties. Besides partial eviction meeting the needs of the building owner had
also not been argued by the defendant at any stage.

Dr Suhas H Pophale v Oriental Insurance Company Limited and its Estate Officer
Deals with the applicability of the Public Premises (Eviction of Unauthorized Occupants)
Act, 1971 and the State's rent control act. DrPophale was the tenant of Indian Mercantile
Insurance Co. (MIC) and was a lawful tenant under the Bombay Rent Act 1947 (now
Maharashtra Rent Control Act, 1999). MIC was merged with the Oriental Insurance
Company a government company, with effect from 1 January 1974 and the management
of MIC was taken over by the Central Government in 1971. Eviction notice was served on
DrPophale. For any premises to become public premises the relevant date will be 16
September 1958 or whichever is the later date on which the concerned premises become
the public premises (as belonging to or taken on lease by Public Authorities like OIC). All
those persons failing within the definition of a tenant occupying the premises prior
thereto will not come under the ambit of the Act and cannot therefore be said to be
persons in "unauthorized occupation". If possession of their premises is required, that
will have to be resorted to by taking steps under the respective State Acts.

PROVISION REGARDING RENT

1) Despite any contract between parties, rent in excess of fair rent cannot be charged
2) Fair rent is to be paid within the time fixed by the contract or, in the absence of any
such contract, by the fifteenth day of the next month following the month for which
it is payable, provided the tenant may pay the rent payable for any month at any
time during such month before it falls due.

Page | 38
L&B/Chapter-1/Law-Real Estate

3) Other than one month's rent, no other premium in addition to rent can be charged
by the landlord at the time of grant, renewal or continuance of a tenancy of any
premises, whether in cash or in kind in addition to rent without the previous
permission of the Controller appointed under the Act. Any advertisement setting a
requirement for any payment in excess of a month’s rent as advance is banned
4) The tenant or any other person acting on his behalf cannot claim or receive any
payment for relinquishment, transfer or assignment of the tenancy

5) Sale of furniture on premise to the tenant cannot be made a condition for the grant,
renewal or continuance of the tenancy of such premises. Any advertisement
advertising such a requirement is also banned

6) Where any sum in excess of provisions of the Act is paid, on application to the
Controller within a period of six months from the date of such payment, the
Controller can direct the landlord to refund such sum or adjust against future dues.

FIXATION OF FAIR RENT (SEC 17)

1) The landlord or the tenant can apply to the Controller for fixation of the fair rent of
any premise.

2) For a property constructed and let out after the year 1984 - fair rent will be six and
three-fourth per cent per annum of the amount of the actual cost of construction
and the market price of the land on the date of commencement of construction

3) The cost of construction of premises includes the cost of water supply, sanitary and
electric installation and as per the Public Works Department rates of the State
Government as applicable for the area. A ten percent variation (+/-) is allowed by the
Controller.
4) For computing the market value of land, the controller is to consider only that
portion of the site on which the premises were constructed and as an amenity, add a
further sixty per cent of the appurtenant vacant land.

5) As per sec. 17(3) of the West Bengal Premises Tenancy Act, for premises constructed
in or before the year 1984,
a. If a tenancy subsists for twenty years or more, amount considered for fair rent, is
at most three times (in case of residential property) and five times (in case of
commercial purposes) the rent as on 1.7.1976, and then in case of residential
purpose only deducting the increase, if any, in the manner provided in Schedule
II

b. tenancy subsists for ten years or more but less than twenty years, at most two
times (in case of residential property) and three times (in case of commercial
purposes) the rent as on 1.7.1986 and then in case of residential purpose only
deducting the increase, if any, in the manner provided in Schedule III
c. Or in both cases by accepting the existing rent if such rent is more than the

Page | 39
IOV-Registered Valuers Foundation

increased rent determined according to that Schedule. If above processes do not


apply then the fair rent shall be such as would be reasonable, having regard to
the situation, locality and condition of the premises and the amenities and
considering rent payable in respect of other similar premises.
6) In case the landlord contravenes provision, tenant may file a complaint under Sec. 28
of the Act.

7) As per the statute the fair rent initially fixed shall be automatically increased by five
per cent every three years unless the state government notifies another rate of
increase. Fair rent as fixed by the Controller takes effect from the month of tenancy
next after the date of application.

8) Any increase of rent by the owner has to be preceded by a notice as provided in the
Act to the tenant. Usually the Rent Controls Acts have provisions like the landlord or
the tenant can approach Rent Control Court for revision of rent at an interval of
years (three or five years) .
9) Where the landlord does not accept any rent, the tenant shall remit the rent to the
landlord by postal money order within fifteen days of such refusal. If the postal
money order is returned to the postal authority as undelivered, either on account of
the landlord having refused to accept the payment thereof or for any other reason,
the tenant may deposit such rent with the Controller within fifteen days from the
date on which it is so returned to the tenant. If there is doubt as to the person or
persons to whom rent is payable, the tenant may deposit such rent with the
Controller. Deposit to Controller has to be supported by an affidavit of the tenant
identifying the property, address of landlord and the reason for depositing the same
with the Controller. The Controller then informs the tenant the fact of payment.
10) According to Sec 4(2) and (3) of the West Bengal's Act, a landlord is bound to keep
the premises in good condition. The low rent cannot be an excuse to not repair the
premise. He is bound to take measures to ensure essential supply or service
comprised in the tenancy. For effecting essential repairs, tenant may make an
application to the rent controller under Sections 35 and/or 36 of the Act.

11) Supreme Court has held in Ranjit Kumar Bose &Anr vs Anannya Chowdhury &Anr CA
No. 3334/2014, in matters of tenant eviction the state act shall prevail. Cases under
Rent Control Act are heard by the Rent Control Act where constituted and generally
Munsif's Court executes the order passed by the Rent Control Court.

CONSEQUENCE OF RENT CONTROL IN INDIA


Following adapted from an interesting analysis in a working paper by (Dev & Dey, 2006)
are some of the consequences of Rent Control and thus point to the need for change.

1) Rent control places an arbitrary cap on income operates as disincentive for people
with resources and otherwise capable of to investing in creating rental housing

2) Rent control introduced under stress situation like war, partition of country leading

Page | 40
L&B/Chapter-1/Law-Real Estate

to heavy influx of people tends to take on permanent character.

3) Leads to rapid deterioration of houses as it has eroded all interest in investing in the
upkeep of the existing apartment

4) Fear of losing control on property brought under rent control forces owners to
withdraw their vacant premises from tenancy market
5) Difficult to evict tenant in rent control premise and selling house with tenant reduces
the marketability of the title

6) Property tax is reduced as it is computed as a percentage of Annual Ratable value or


other rent computation as the income that the property fetches gets capped under
rent control act irrespective of the locality or demand for housing there

7) State must invest much to administer rent control act and enforce its provision, itself
earning little revenue from the taxes

8) Operation of tenancy persisting across generations in India leads to locking out of


the house property from the market

9) Selective applicability of rent control reduces its ambit - rent control may not apply
to new properties for a period of say ten years
10) Landlords refuse to issue rent receipt in a bid to prevent creation of proof of tenancy
and ends up fueling black money economy

Where the Act permits, the landlord can improve the property without the consent of the
tenant, in turn increasing the standard rent and thus the burden on the vulnerable

MESNE PROFITS OF PROPERTY

Is collective term for profits which a person in wrongful possession of a property actually
receives or with ordinary diligence could have received together with interest thereon.
But if any improvements to property were made by the person in wrongful possession
and profits accrued due to such improvement then it is not included. Prayer of mesne
profits is prayed for in conjunction with a decree for possession and mesne profits
under Order 20 Rule 12 of the Civil Procedure Code, 1908 on following grounds:

Where a suit is for the recovery of possession of immovable property and for rent or
mesne profits, the court may pass a decree—
1. for the possession of the property;
2. for the rents which have accrued on the property during the period prior to the
institution of the suit or direction an inquiry as to such rent;
3. for the mesne profits or directing an inquiry as to such mesne profits;

Page | 41
IOV-Registered Valuers Foundation

4. directing an inquiry as to rent or mesne profits from the institution of the suit
until—
i. the delivery of possession to the decree holder,
ii. the relinquishment of possession by the judgment debtor with notice to
the decree holder through the court, or
iii. the expiration of three years from the date of the decree, whichever
event first occurs.

Where an inquiry is directed under clause (b) or clause (c), a final decree in
respect of the rent or mesne profits shall (b)

4. RIGHT OF WAY, SECTION 52 -LICENSE


UNDER THE INDIAN EASEMENT ACT, 1882
A license is a personal right granted to a person to do something upon immovable
property of the guarantor and does not amount to the creation of an interest in the
property itself. It is purely a permissive right and is personal to the guarantee. It creates
no duties and obligations upon the person making the grant and is therefore, revocable
except in certain circumstances expressly provided for in the Act itself. The license has
no other effect than to confer a liberty upon the licensee to go upon the land which
would otherwise be unlawful.

Section 52 of the Indian Easement Act defines license as follows: “Where one person
grant to another, or to a definite number of other persons, a right to do, or continue to
do, in or upon the immovable property of the grantor, something which would, in the
absence of such right, be unlawful, and such right does not amount to an easement or
an interest in the property, the right is called a license.”

CHARACTERISTICS OF LICENSE

1) No transfer of interest:- A license is a permission to do some act which, without


such permission, would be unlawful.

2) No interest in accretions: - A licensee has no interest in the property and therefore,


he acquires no right by accretion
3) Neither transferable nor heritable:- A license is neither transferable; nor heritable.

4) A license is a matter purely personal between grantor and grantee.

5) Section 52 of Easement Act does not require any consideration, material


or non-material, to be an element of the definition of license, nor does it require
that the right under the license must arise by way of contract or as a result of
mutual promises.

Page | 42
L&B/Chapter-1/Law-Real Estate

6) The person who grants the license must be the owner of the property. The other
person who gets the permission must be a stranger or have no right in the property.

7) License creates no duties and obligations upon the person making the grant and is
therefore revocable except in certain circumstances expressly provided in the Act
itself.

8) A license is usually revocable by grantor, except in the two cases mentioned in the
section 60 of Easement Act.

9) A subsequent transfer of the property terminates a license.

10) A licensee cannot sue trespassers and strangers in his own name.

11) A license is terminated by death of either party.

5. SALIENT FEATURES OF RERA, 2016


Salient Features of Real Estate (Regulation & Development) Act, 2016 and Real Estate
Regulating Authorities Established under the Act.

The Real Estate (Regulation and Development) Act, 2016 aims to regulate and promote
the real estate sector by regulating the transactions between buyers and promoters of
residential as well as commercial projects. It also has provisions for establishing a
regulatory authority at state level called "Real Estate Regulatory Authority" (RERA) for
monitoring the real estate sector and adjudicating disputes relating to Real Estate
Projects. The main aim of the Act is to protect buyers and help investment in Real Estate
Sector.

MAIN OBJECTIVES

1) Enhance transparency and accountability in real estate and housing transactions;


2) Providing uniform regulatory environment to ensure speedy adjudication of disputes
and orderly growth of the real estate sector;
3) Boosting domestic and foreign investment in the Real Estate sector;

4) Promote orderly growth through efficient project execution and standardization;

5) Offer single window system of clearance for real estate projects.

SALIENT FEATURES OF THE ACT

• Establishment and incorporation of Real Estate Regulatory Authority (RERA) at every


State in India for monitoring and adjudicating disputes relating to real estate
projects (Section 20).
• Establishment of fast track dispute resolution mechanism for settlement of real

Page | 43
IOV-Registered Valuers Foundation

estate disputes through dedicated adjudicating officers and Appellate Tribunal


(Section 43 & 44).

• Registration of all real estate projects is made mandatory with RERA having
territorial jurisdiction over such projects. No sale in a real estate project can be made
without registration of the project with RERA (Section 3). RERA can also refuse to
register a project, if the same is not compliant with provisions of the Act.
Registration of a project can even be cancelled, in case, RERA receives any complaint
and the same is found to be correct after inquiry.
• It is mandatory for a promoter to upload details of proposed project on the website
of RERA, including details of registration, types of apartments or plots booked, list of
approvals taken and the approvals which are pending subsequent to
commencement certificate, status of the project, sanction plan, layout plan etc.
(Section 11).

• RERA shall approve or reject the application for registration within 30 days, failing
which it shall be deemed to have accepted the application for registration (Section
5).
• Any promoter shall not accept a sum more than ten per cent of the cost of the
apartment, plot, or building as the case may be, as an advance payment or an
application fee, from a buyer without first entering into a written agreement for sale
with such person and register the same. (Section 13)
• It has been made obligatory for the promoters to deposit 70% of the money
collected from buyers for a particular project in a separate account that will cover
the cost of land and construction and the same can be withdrawn only after
certification from an engineer, an architect and a chartered accountant. [Section
4(2)(l)(D)].
• It is now obligatory for all the promoters to obtain insurance in respect of title of the
land and buildings and construction of every project. (Section 16).

• The promoter shall not transfer or assign his majority rights and liabilities in respect
of a real estate project to a third party without obtaining prior written consent from
at least 2/3rd no. of Allottees, except the promoter, and without the prior written
approval of RERA [Section 15(1)].

• Both promoter and buyer are liable to pay equal rate of interest in case of any
default from either side (Section 2 (za)(i)).

• The promoter shall compensate the buyer in case any loss caused to him due to
defective title of the land, on which the project is being developed or has been
developed, in the manner as provided under this Act, and the claim for
compensation under this subsection shall not be barred by limitation provided under
any law for the time being in force. (Section 18)

• An aggrieved person may file a complaint with RERA, as the case may be, for any
violation or contravention of provisions of this Act or rules and regulations made

Page | 44
L&B/Chapter-1/Law-Real Estate

there under against any promoter, buyer or real estate agent. (Section 31)

• During the pendency of enquiry, RERA can restrain any promoter, buyer or agent
from continuing with the act complained of. (Section 36)

• A person aggrieved by any direction or decision or order made by RERA or by an


adjudicating officer under this Act may prefer an appeal before the Appellate
Tribunal having jurisdiction over the matter. (Section 43)

• If a promoter continues to violate the provisions of Section 3, he shall be punished


with imprisonment for a term which may extend to three years or fine which may
extend to ten percent of the estimated cost of the project or both [Section 59(2)].
• If a promoter fails to comply with orders or directions of RERA, he shall be liable to a
penalty, which may extend up to five percent, of the estimated cost of the project as
determined by the Authority (Section 63).

• If a promoter fails to comply with the orders or directions of the Appellate Tribunal,
he shall be punished with imprisonment for a term which may extend to three years
or fine, which may extend up to ten percent of the estimated cost of the project, or
with both (Section 64).
• Where an Offence under this Act has been committed by a company, every person
who, at the time, the offence was committed was in charge of, or was responsible for
the conduct of business of the company, as well as the company, shall be deemed to
be guilty of the offence and shall be liable to be proceeded against and punished.
(Section 69)

• No civil court shall have jurisdiction to entertain any suit or proceeding in respect of
any matter which RERA or the adjudicating officer or the Appellate Tribunal is
empowered by or under this Act to determine and no injunction shall be granted by
any court or other authority in respect of any action taken or to be taken in
pursuance of any power conferred by or under this Act. (Section 79)

REGULATORY FRAMEWORK – REAL ESTATE (REGULATION AND


DEVELOPMENT) ACT, 2016

• Establish the Real Estate Regulatory Authority for regulation and promotion of the
real estate sector.
• Ensure sale of plot, apartment of building, as the case may be, or sale of real estate
project, in an efficient and transparent manner.
• Ensure protect the interest of consumers in the real estate sector.
• Establish an adjudicating mechanism for speedy dispute redressal and also to
establish the Appellate Tribunal to hear appeals from the decisions, directions or
orders of the Real Estate Regulatory Authority (RERA)
• Regulates transactions between buyers and promoters of residential real estate
projects

Page | 45
IOV-Registered Valuers Foundation

• Establishes state level regulatory authorities called Real Estate Regulatory Authorities
(RERAs)
• Residential real estate projects, with some exceptions, need to be registered with
RERAs
• Promoters cannot book or offer these projects for sale without registering them.
Real estate agents dealing in these projects also need to register with RERAs
• Registration, the promoter must upload details of the project on the website of the
RERA. These include the site and layout plan, and schedule for completion of the real
estate project
• Amount collected from buyers for a project must be maintained in a separate bank
account and must only be used for construction of that project. The state
government can alter this amount
• Right to Legal Representation on behalf of Client by Company Secretaries or
chartered accountants or cost accountants or legal practitioners
• Imposes stringent penalty on promoter, real estate agent and also prescribes
imprisonment.

TRANSFER OF PROPERTY ACT (TPA), 1882

The Transfer of Property Act came into existence in 1882. Before that, the transfer of
immovable property was governed by principles of English law and equity. The preamble
of Act sets out the objectives of the legislation. Scope of this Act is limited. It applies only
to transfer by the act of parties and not by operation of law. Also, this Act deals with a
transfer of property inter vivos, i.e., a transfer between living persons. It contains transfer
of both movable and immovable property, but a major portion of the enactment is
applicable to the transfers of immovable properties only. The Act is not exhaustive.

STATUTORY DEFINITIONS (SECTION 3)

• IMMOVABLE PROPERTY

Definition in Section 3 is not exhaustive. It says only that ‘immovable property’ does not
include standing timber growing crops or grass. Definition of immovable property in
Section 3(26) of General Clauses Act, 1897, is also not exhaustive. It defines immovable
property as it shall include land, benefits to arises out of land, and things attached to
earth. Thus, we find that while Transfer of property excludes certain things. General

Page | 46
L&B/Chapter-1/Law-Real Estate

Clauses Act includes certain things under the head ‘immovable property’. By combing
both definitions, we may say that, the term includes land, benefits to arises out of lands,
and things attached to the earth, except standing timber, growing crops and grass.

1. Land: It means a determinate portion of the earth’s surface, which may be covered
by water, the column of surface above the surface, the ground beneath the surface.
All the objects which are on or under the surface in its natural State are included in
the term land. Also, all objects placed by human agency on or under the surface with
the intention of permanent annexation are immovable property, e.g., Building, wall,
fences.

2. Benefits to arise out of land: Apart from physical point of view, every benefits arise
out of land is also regarded as immovable property. Registration Act also includes as
immovable property benefits to arise out of land, hereditary allowances, right of way,
lights, ferries and fisheries. In Anand Behera v. State of Orissa, AIR 1956 SC 17, the
right to catch away fish from chilka lake, over a number of years, was held to be an
equivalent of profits a pendre in England and a benfits to arise out of land in India.
Similarly, a right to collect a rent and profits of immovable property, right to collect
dues from a fair or heat or market on a land are immovable property.

3. Things attached to earth: Section 3 of transfer of property defines the expression


‘attached to earth’ as including (1) things rooted in the earth, (2) things embedded in
the earth, (3) things attached to what is so embedded, and (4) chattel attached to
earth or building.

• Things rooted in earth include trees and shrubs, except standing timber, growing
crops and grasses (Section 3, TPA). Whether tress regarded as movable or
immovable depends upon the circumstances of the case. If the intention is that
trees should continue to have the benefit of further sustenance or nutriment by
the soil (land), e.g., enjoining their fruits, then such tree is immovable property.
But if the intention is to out them down sooner or later for the purpose utilising
the wood for building or other industrial purpose, they would be timber and of
accordingly be regarded as movable property (Shantabai v. State of Bombay, AIR
1958 SC 532) determining whether the tree is movable or immovable, the
intention if party is important if the parties intend that the tree should continue to
have the benefit of further nutriment to be afforded by soil, the tree is immovable
property. But if intention is to withdraw the tree from land, and the land is
providing it only as a warehouse, it is to be treated as movable property.

• Things embedded in earth: It includes such things as house, buildings, etc.,


however certain things like an anchor imbedded in the land to hold a ship is not a
immovable property’ to determine whether such things are movable or

Page | 47
IOV-Registered Valuers Foundation

immovable property, depends upon circumstances of each case and there are two
main conditions to indicate intention:
1. the degree or mode of annexation, e.g. tie-up seats fastened to the floor
of cinema halls are immovable property on brick-work and timber and
tapestries;
2. the object of annexation, for, e.g., Blocks of stone placed one on the top
of other without any mater or cement for the purpose of forming a dry
wall, will become part of land, so immovable property, but not the stones
deposited in the builder’s yard.

• Things attached to what is so embedded must be for the permanent beneficial


enjoyment of the to which it is attached, as section says for, e.g., door and
windows of a house are immovable property to be permanent, like electric fans or
window blinds, they are movable property.

• Chattel attached to earth or building if a chattel, i.e., movable property is attached


to earth or building, if is immovable property. The degree, manner, extent and
strength of attachment are the main features to be regarded in determining the
question. Standing timber, growing crops and grasses are regarded as severable
from land and they are regarded as movable property. However if they and the
land on which they stand is sold, such standing timber, growing crops or grasses
will pass to purchases.

4. Standing timber: The word standing timber includes Babool Tree, Shisham, Nimb,
Papal Banyan, Teak, Bamboo, etc. The fruit berating tree like Mango, Mahua,
Jackfruit, Jamun, etc., are not standing timber, and they are immovable properties
( Fatimabibi v. Arrfana Begum, AIR 1980 All 394). But if intention is to cut them down
sooner or later for the purpose utilising them as timber, and not to use them for the
purpose of enjoying their fruits, they are regarded as movable property. (T.A.
Sankunni v. B.J. Philips, AIR 1972 Mad 272).

5. Growing crops: Growing crops includes creepers like pan, angoor, etc., millets
(Wheat, Sugarcane, etc.), Veg like Lauki, Kaddo, etc. These crops don’t have any own
independent existence beyond their final produce.

6. Grasses: It can only be used as fodder, and no other use is possible. Therefore it is
movable. But a contract to cut grass will be an interest in chattel, so is immovable
property. The following has been judicially recognised as immovable property:

• Right to collect rent of immovable property.

• Right to dues from a fair on a piece of land.

• A right of fisheries.

Page | 48
L&B/Chapter-1/Law-Real Estate

• A right of terry.

• A right of way.

• Hereditary offices.

• The interest of a mortgagee in immovable property.

7. Minerals: Upon transfer of immovable property, things not only rooted to it, but
also anything found deep down below the property goes along with the transfer. All
minerals below the land sole are immovable property.

• MOVABLE PROPERTY

Transfer of property does not define movable property. In General Clauses Act, it is
defined as “Property of every description except immovable property”. Some examples
are right of worship, royalty, machinery not attached to earth which can be shifted, a
decree for arrear of rent, etc.

1. Attestation

Attestation has been defined in Section 3 of Transfer of Property Act as under:—


”attested”, in relation to an instrument, means and shall be deemed always to have
meant attested by two or more witnesses each of whom has seen the executant sign or
affix his mark to the instrument, or has seen some other person’s sign the instrument in
the presence and by the direction of the executant a personal acknowledgment of his
signature or mark, or of the signature of such other person, and each of whom has
signed the instrument in the presence of the executant, but it shall not be necessary that
more that one of such witnesses shall have been present at the same time, and no
particular form of attestation shall be necessary.

To ‘attest’ means to sign and witness any fact of execution by the executant. It means
that a person has signed the document by way of testimony of the fact that he saw it
executed.

Following are the essential requisites of a valid attestation-

1. There must be two or more attesting witness.

2. Each witness must see—(a) the executant sign or affix his mark (thumb
impression) to the instrument; (b) or, see some other person sign the instrument
in the presence, and by the direction, of the executant; (c) or, receive from the
executant a personal acknowledgment of his signature or mark or of the
signature of such other person.

3. Each witness must sign the instrument in the presence of executant.

Page | 49
IOV-Registered Valuers Foundation

4. Each witness must sign only after the executant is complete.

5. It is not necessary that more than one of such witnesses should be present at the
same time.

6. No particular form of attestation is necessary.

7. Attester Should be sui generis, i.e., capable of entering into contract.

8. The witness should have put his signature animoattestandi (intention to attest).

9. A. person who is a party to transfer cannot attest it as a witness. The object of


attestation is to protect the executant form being required to execute a
document by other party thereto by force, fraud, or undue influence, a party to
the transaction cannot laid down in Kumar Harish Chandra v Banshidhar
Mahanty, AIR 1965 SC 1738

Effect of invalid attestation it makes the deed of transfer of property involved, and
therefore no property invalid and therefore no property passes under it. The document
cannot be enforced in the court of law.

2. Notice

A person is said to have a notice of a fact when he actually knows that fact, or when, but
for wilful abstention from an inquiry or search which he ought to have made, or gross
negligence, he would have known it (Section 3).

Section 3 of Transfer of Property Act enumerates three kinds of notices—

• Actual or express notice

• Constructive or implies notice

• Imputed notice.

Actual notice: A person is said to have actual notice/express notice of a fact if he


actually knows it. It must be definite information given in the course of negotiations by
person interested in the property. A person is not bound to attend vague rumours.

Constructive notice: It is a notice which treats a person who ought to have known a fact,
as if he actually knows it. A person has constructive notice of all the facts of which he
would have acquired actual notice had he made those inquiries which he ought
reasonably to have made. Constructive notice has roughly been defined as knowledge
which the court imputes to a person upon a presumption so strong that it cannot be
rebutted that the knowledge must be obtained.

Legal presumption of knowledge arise when—(1) There is wilful abstention from an


enquiry or search. It means wilful or deliberate abstention to take notice of a fact which a
reasonable man would have taken in the normal cause of life. It is such abstention from

Page | 50
L&B/Chapter-1/Law-Real Estate

enquiry or search as would show want of bona fides in respect of a particular transaction.

Illustration

a) A contract to sell his house to B. The house is on rent and B knows that the
tenants have been paying the rents to C. B has constructive notice of the right of
C to take rents from the tenants.
b) A propose to sell his property to B, who at the same time knows that rents due in
respect of the property are paid by the tenants to a third person X. B will be fixed
with notice of the right of X. [Hunt v. Lack, (1902) 1 Ch 429.]

c) A refuse a registered letter, which contains information relating to property which


A propose to purchase. A will be deemed to have notice of the contents of the
latter. [Ismail Khan’ v. Kali Krishna, (1901) 6 Cal WN 134]

Gross negligence: Negligence means carelessness or omission to do such act which a


man of ordinary prudence would do. Doctrine of constructive notice applies when a
person, but his gross negligence would have known the fact. Mere negligence is not
penalised. It should be high degree of neglect. In Hudston v. Vincy, (1921) 1 Ch 98, Eve J.
said, “Gross negligence does not mean mere carelessness, but means carelessness of so
aggravated a nature as to indicate a attitude of mental indifference to obvious risk.” It
can be described as ‘a degree of negligence so gross that a court of justice may treat it
as evidence of fraud, impute a fraudulent motive to it and visit it with the consequences
of fraud’.

In Ltoyds Bank Ltd. v. P.E. Guzders and Co. Ltd., (1929) 56 Cal 868, a person A deposited
title deeds of his house in Calcutta with Bank. N to secure the loan he had taken from the
bank. Subsequently, A represented the Bank that intending purchases of the house
wanted to see the title deeds. The bank returned the deeds to A who deposited the
deeds with the plaintiff bank in order to secure a loan. It was held that the Bank N, on
account of gross negligence in parting with the deed has lost its prior rights with respect
of the house.

In Imperial Bank of India v. U. Raj Gyaw, (1923) 50 IA 283, a purchases was informed that
the title deeds were in possession of a bank for safe custody and omitted to make any
inquiry from bank It was held that he was guilty of gross negligence and was deemed to
have notice of the rights of the bank which had the custody of the deeds.

Registration as notice: Explanation I to Section 3 provides that ‘where any transaction


relating to immovable property is required by law to be and has been effected by a
registered instrument, any person acquiring such property or any part, or share or
interest in such property shall be deemed to have notice of such instrument as from the
date of registration, ‘Thus any person interested in the transaction which is registered

Page | 51
IOV-Registered Valuers Foundation

under the provisions of the Indian Registration Act, 1908 cannot plead that he has no
notice of the transfer made under the deed.

In order that, registration may be treated as constructive notice of its content, following
conditions must be satisfied:

1. The instrument must be compulsorily registrable.

2. All the formalities prescribed under the Registration Act are duly completed in
the manner prescribed.

The instrument and particulars must be correctly entered in the registers.

After registration, document becomes a public document and the title can be confirmed
in the Registrar’s office.

Actual possession as notice of Title: Explanation II of Section 3 provides that, “any


person acquiring any immovable property or any share or interest in such property shall
be deemed to have notice of the title, if any, of any person who is for the time being in
actual possession thereof. “Thus in order to operate as constructive notice, possession
must be actual, i.e., de facto possession. It amounts to notice of title in another, e.g., A
leased a house and garden to B who takes possession of the properties. A then sells the
said properties to C. C is deemed to have constructive notice of B’s rights over these
properties, i.e., C cannot plead that he had no knowledge (notice) of the fact of B’s
possession on the properties [Deniels v. Davison, (1809) 16 Ves 240].

Imputed notice

Explanation III to Section 3 provides that, “A person shall be deemed to have had notice
of any fact if his agent acquires notice thereof whilst acting on his behalf in the course
of business to Which that fact is material:

Provided that, if the agent fraudulently conceals the fact, the principal shall not be
charged with notice thereof as against any person who was a party to or otherwise
cognizant of the fraud”

This is based on the maxim Qui facit per aliumfacit per se, i.e., he who does by another,
does by himself. In MohoriBibee v. D. Gliosh, (1903) 30 Cal 539, held that although the
principle was absent from Calcutta and did not take part in the transaction personally, his
agent in Calcutta stood in his place for the purposes of the transaction and the acts and
knowledge of the latter were the acts and knowledge of the principal.

3. Actionable Claim

Section 3 of the Transfer of Property Act, 1882 defines actionable claim as, “it means a

Page | 52
L&B/Chapter-1/Law-Real Estate

claim Actionable Claim to any debt, another then a debt secured by mortgage of
immovable property or by hypothecation or pledge of movable property or to any
beneficial interest in movable property , not in the possession, either actual or
constructive, of the claimant, which the civil courts recognise as affording grounds for
relief, whether such debt or beneficial interest be existent, accruing, conditional or
contingent.”

Thus, according to Section 3, actionable claim means—

1. a claim to unsecured debt, or

2. a claim to beneficial interest in movable property not in possession of the


claimant.

Debt: A debt is an obligation to pay a liquidated sum of money. The amount of money
must be certain, otherwise it is not debt. For an actionable claim, a debt must be
unsecured debt, for secured debts are excluded from the definition of actionable claim,
e.g., A owes Rs. 10.00 to B. B’s claim is an actionable claim.

Debits secured by a mortgage of immovable property or by a pledge of movable


property are excluded from the definition of actionable claim, because they are secured
debt.

Claim to beneficial interest not in possession of the claimant: Actionable claim


includes a claim to any beneficial interest in movable property, not in the possession of
the claimant, interest or the right of possession of claimant is recognised by the court.
(However a claim for demands, i.e., for an unascertained. sum of money or a claim for
mesne profits does not come within the definition if actionable claim), e.g., A agrees to
sell to B bales of cotton deliverable on a future day B has a beneficial interest in the
goods and it is an actionable claim.

Some instance of actionable claim:


• Claim for arrears of rent.
• Claim for the money due under insurance policy (Shamdas v. Savitri, AIR 1937
Sind 24)
• Claim for the return of earnest money.
• A share in partnership.
• maintenance allowance payable in future.
• fixed deposit in a bank.
• right to the proceeds of business.
• Hire-Purchase agreement.
• Instances of claims which are not actionable:
• A copyright,

Page | 53
IOV-Registered Valuers Foundation

• Claim for mesne profits,


• Secured debt,
• Right to get damages,
• A debt which has passed into a decree.

Transfer of Actionable Claims

Actionable claims are transferable properties; thus it can be transferred by way of sale,
mortgage, gift or exchange. Section 130 of Transfer of Property Act provides the mode
of effecting the transfer of actionable claims, and its effect. It provides—

The transfer of an actionable claim whether with or without consideration shall


be effected only by the execution of an instrument m writing signed by the transferor or
his duty unauthorized agent, shall be complete and effectual upon the execution of
such instruments, and thereupon all the rights and remedies of the transferor, whether
by way of damages or otherwise, shall rest in the transferee, whether such notice of the
transfer as is hereinafter provided be given or not:

Provided that every dealing with debt or other actionable claim by the debtor or other
person from or against whom the transferors would, but for such instrument of transfer
as aforesaid, have been entitled to recover or enforce such debt or other actionable
claim, shall (save where the debtor or other person is a party to the transfer or has
received express notice thereof as hereinafter provided) be valid as against such
transfer.The transferee of an actionable claim may, upon the execution of such
instrument of transfer as aforesaid, sue or institute proceedings for the same in his own
name without obtaining the transferor’s consent to such suit or proceeding and without
making him a party thereto.” Section 131 of the Act provides that the notice should be in
writing signed. It provides:

“Every notice of transfer of an actionable claim shall be in writing signed by the


transferor or his agent duly authorised in this behalf, or in case the transferor refuses to
sign, by the transferee or his agent, and shall state the name and address of the
transferee.”

Section 132 of the Act provides the liability of transferee of actionable claim. It reads,
“The transferee of an actionable claim shall take it subject to all the liabilities and equities
to which the transferor was subject in respect thereof at the date of the transfer”. ,

E.g., A transfers to C a debt due to him by B, A being then indebted to B. C sues B for the
debt due by B to A. In such suit it is entitled to set off the debt due by A to him; although
C was unaware of it at the date of such transfer.

Page | 54
L&B/Chapter-1/Law-Real Estate

• TRANSFER OF IMMOVABLE PROPERTY: SALE, MORTGAGE,


GIFT, EXCHANGE, ASSIGNMENT, CHARGE, LIEN, TENANCIES/
SUB-TENANCIES.
Concept of Moveable and Immoveable Property

As per the Securitisation and Reconstruction of Financial Assets and Enforcement of


Security Interest Act, 2002 (SARAFESI) the term "property" means -

1. immovable property;

2. movable property;

3. any debt or any right to receive payment of money, whether secured or


unsecured;

4. receivables, whether existing or future;


5. intangible assets (know-how, patent, copyright, trade mark, License, franchise or
any other business or commercial right of similar nature like goodwill)

The SARAFESI Act although little out of place here but the ambit of the term “property"
in it is relevant for our understanding. However, valuation for secured lending centres
more on immoveable property and certain moveable assets like plant and machinery and
vehicles.

The term immoveable property is defined in several other statutes: Sale of Goods Act,
1930, General clauses Act 1897 (Sec 3(26)), Registration Act, 1908 (sec 2(6), Income tax
Act, 1961 (sec 269UA (d)), Transfer of Property Act, 1882 (sec 3). The definitions vary
from one statute to another in terms of the detail covered.

The General Clauses Act states Immoveable property includes land, benefits to arise out
of land and things attached to the earth or permanently fastened to anything attached
to it. Hence, land per se and mines, agriculture and forests being benefits from land are
covered. Any machinery fastened securely to land would also be covered but not
wooden poles etc. that is similarly fixed.

The sec 3 of Act TPA defines immoveable property as something that does not include
"standing timber, growing crops or grass". The definition is criticised for focusing on
what it is not, rather than focusing on defining it but it stresses on severability from the
earth as a paramount consideration in classifying moveable or immoveable property.
This Act deals with transfer of property as a whole and covers moveable property too. In
some cases, like the transfer of moveable property, transaction is complete with delivery
of property to the buyer/ recipient but not so in case of immoveable property. When the
Registration Act 1908, and Stamp Duty Act and Transfer of Property Act are read

Page | 55
IOV-Registered Valuers Foundation

together in case of immoveable property it is not enough to give possession, registration


of the transfer document and the payment of due stamp duty is also necessary. But the
party in possession and supported by unregistered sale deed, does have some rights
stemming from the doctrine of part performance (Sec 53A of the Transfer of Property
Act) read with Sec 49 of the Registration Act (i.e. an unregistered document can at best
be admitted as secondary evidence under the Indian Evidence Act to prove title) and
cannot be dispossessed without due process of law.

Transfer of Property Act, 1882 requires registration of all such deeds which transfer real
estate of value of over Rs. 100/-. The Promoter's Act requires registration of all the Deed
related to transfer of flat from Promoter to purchaser and some other agreements too.

• TRANSFER Right to keep possession of property belonging to


another person till the debt owed is discharged.
Transfer of Property Act, 1882 is crucial to understanding the laws of transfer of Real
Estate and provides for transfer through sale, exchange, mortgage, lease, lien and gift.

Some crucial points of pertaining to definition of transfer under section 5 of


Transfer of Property Act are:

1. Governs transfer of property between one or more living persons on either side
(inter vivo). The section itself clarifies that the concept of living persons includes
company or other registered or unregistered association of persons or body of
individuals. However, it further clarifies that TPA provisions are not in derogation
or contradiction of provisions in other statute also affecting transfer of property.

2. Transferor of property can also be the transferee (recipient) of the property either
alone or along with other living persons. For e.g. if the property is transferred to
a Trust created to manage the property for the benefit of the transferor and who
is also a Trustee

3. The line "conveys Property in present or future" in the definition leads to some
confusion. One court decision held that "present or future" qualifies the term
"conveys" - as determined by SC in 1955 in Jugal Kishore Saraff v Raw Cotton Co.
Ltd AIR 1955 SC 376. But in 1969 SC conceded there can be contract to transfer
future property under sec 40 & 43 of TPA in Bharat Nidhi Ltd. vs. Takhatmal AIR
1969 SC 313. Thus, property in question may be in existence or of a future
property (from will).

4. It is deliberate and is the result of the intention of the parties involved (i.e. the
Act does not cover situations like transfer of property due to forfeiture of
property given as security in mortgage, when property is impounded on any
order of court, intestacy (succession following of holder dying without making a
will or a trust for management). However, a point worth noting here is - TPA also
does not cover "family settlement"/"family arrangement" or a "family partition"
(which also occurs at the intention of the parties who have pre-existing title or

Page | 56
L&B/Chapter-1/Law-Real Estate

claim on property). A family settlement between all eligible adults only tries to
pre-empt and prevent future disputes around a property by determining the
rights of successors or those having other bases for claim in the property and
hence is usually not transfer under TPA. In a partition, one person who is a joint
owner gets some specific property in lieu of his rights on common property and
thus no new property is really being transferred and only pre-existing joint title is
being severed and reallocated, that too is not a transfer under TPA]

5. Can cover transfers of entire bundle of rights (as in sale) or only carved out part
of total rights (e.g. leasehold, mortgage).

The transfer of property is dealt in other Acts as well. The Allahabad High Court in Misc
No. 5937 of 2015 Sahara Grace Apartments Owners Assoc vs. State of U.P. has stated the
"provisions of the Transfer of Property Act, 1882, in view of Section 29 of the U.P.
Apartment Ownership Act, 2010, shall in so far as they are not inconsistent with the
provisions of the Act apply to the transfer of any apartment together with its undivided
interest in the common areas and facilities appurtenant thereto made by the owner of
such apartment, where such transfer is made by sale, lease, mortgage, exchange, gift or
otherwise as they are applied to the transfer of any immovable property." That is TPA
shall apply only if not inconsistent with the Apartment Act. Some Acts overtly assert the
applicability of TPA.

• WHAT CAN (NOT) BE TRANSFERRED?

This Sec 6 of TPA does not really details what can be transferred under it. This section
says everything can be transferred but then goes on to list what cannot be transferred.
One's most relevant to real estate valuation only are described below:

1. Spec successionis i.e. the mere expectation/ possibility of succeeding as heir to a


property of a living person at a future date. This future right is transferable in UK
but TPA bars it in India. In the same class is the chance of being left a legacy by
some living person which is similarly barred. Even if the living person whose
property is the subject matters gives consent, even then the right to the property
is not transferable.

2. Lessor and lessee are governed by the lease deed which may provide for
conditions that the lessee has to honour for continued enjoyment of the
property. It will also provide that any of these conditions are subsequently
breached then the lease stands cancelled (i.e. lessor gets right of re-entry into the
leased property). The mere possibility of a condition being breached by lessee
and property reverting to lessor is a mere chance and therefore cannot be
transferred by lessor. However, if the whole interest in the property including the
interest that will revert to lessor on the end of the lease period is being
transferred then such right of re-entry will also stand automatically transferred.

Page | 57
IOV-Registered Valuers Foundation

3. As we shall see in the chapter on easement - there are two properties involved in
an easement - the dominant heritage and servient heritage. The property, owner
of which is enjoying the easement right, is the dominant heritage and the
property on which that right is asserted is the servient heritage. Easement right is
defined in Easement Act "a right which owner or occupier of certain land
possesses as such for the beneficial enjoyment of that land, to do and continue
to do something or to prevent and continue to prevent something being done in
or upon or in respect of certain other land, not his own". An easement right by
itself cannot be transferred unless the dominant heritage is also transferred.

4. Any interest in property that is restricted to personal enjoyment cannot be


transferred. E.g. If B has a life interest in i.e. right of maintenance from a property
but no right to transfer the property, then the right of maintenance cannot be
transferred by B to anyone else.

5. A right to receive maintenance cannot be transferred, but any arrears not paid
can be transferred. However, an annuity i.e. when a fixed amount is payable, is
transferable. Maintenance amount can vary depending on the needs of that time
period and hence is not quantifiable. Since such a charge can be placed on
income accruing from a property hence in that sense it is relevant to
understanding real estate.
6. Mere right to sue is not transferable. Sue means to pursue a matter legally in
appropriate court/ forum. So merely transferring right to recover mesne profits is
not valid. But if the property is sold together with right to recover mesne profits
then it is valid.

In Amirtham vs. Sarman AIR 1991 SC 1256 - Supreme Court has clarified if a property
belonging to a minor is transferred by his guardian without legal necessity or for
meeting minor's wants/ benefit and also without court's permission then within three
years of the minor attaining majority he can file a suit to set aside the transfer. If instead,
on attaining majority the said minor transfers the right to sue for his alienated property
to a third person - then that transfer is not hit by sec 6 for a period of time till three
years after minor attains majority. The transferee from minor X is competent to file a suit
within three years of the minor's attaining majority to set aside the sale.

Page | 58
L&B/Chapter-1/Law-Real Estate

• PERSONS COMPETENT TO TRANSFER

Sec 7 gives essential competency to execute a transfer of property deed.

As per definition the persons competent to transfer are the following

1. Persons competent to contract: to understand which we have to refer to sec 11 of


the Indian Contract Act which in turn enumerates a person may contract if he has

a) attained the age of majority (i.e. a minor cannot contract and thus cannot
transfer his property by any method)
b) is mentally sound and
c) not disqualified under law from contracting

2. Entitled to transfer: person having title to the property including for example
coparceners in a joint property if a partition between the holders is first affected; a
Hindu father/ grandfather or great-grandfather of the joint property held
respectively with sons, grandsons, great-grandsons to pay off a personal debt of the
senior that was contracted earlier and is not immoral.
3. Includes any person not holding title to the property but having valid authority to
transfer - e.g. (1) vide duly executed power of attorney a person is permitted to
alienate property of another then it is valid (2) a member of a housing cooperative
registered under WB Cooperative Societies Act, 1973 does not own the flat he
purchased but can nominate a person to take his flat and his share in the
cooperative in return for money (3) Managing Partner does not own the property of
the Partnership firm but if the deed permits it he can alienate the property by way of
mortgage etc.

Since we are on the transferor's competency it may be good to see limits, if any, on who
may be a transferee under the Transfer of Property Act

• Sec 136 - prohibits judges, legal practitioners, mukhtar and other officers
connected with the courts of justice from purchasing any actionable claim (and
thus cannot be transferee).
• Order 21, rule 73 Civil Procedure Code prohibits decree holders from bidding in
auction sale without prior permission of the executing court.

Therefore, a minor can be a transferee of a property so long the minor is not required to
covenant in return to comply with some terms/ conditions. Hence a lease deed in favour
of minor is not valid if the child has to agree in return to pay rent at required interval etc.

Courts have held a de facto guardian i.e. one who is not a natural guardian, nor
appointed by court cannot transfer property of a minor unless it is for necessity or
benefit of minor or minor consents to it on attaining majority (Hari Satya Banerjee vs
Mahadev Banerjee AIR 1983 Cal 76 (81/80) DB)

Page | 59
IOV-Registered Valuers Foundation

• OPERATION OF TRANSFER
Sec 8 deals with "Operation of Transfer". As per the Act in normal course when a transfer
is done the following will be presumed to be included, in case of:

1. Land - the easement annexed thereto, the rents and profits thereof accruing after
the transfer, and all things attached to the earth;'

2. House - the easement annexed thereto, the rents and profits thereof accruing after
the transfer, lock, key bars, doors, windows and all things provided for permanent
use"

3. Machinery fixed to earth - include the moveable parts also

This summarization in the Transfer of Property Act helps to reduce the drafting load.
Only if express intentions to mean something else is manifest from the deed justifying
limiting the meaning of house/ land or such an implication is apparent on plain reading
will the alternate meaning will apply. For e.g. Sarkar, 2000 asserts citing case decisions in
A Jogendra vs Miasa 51 IC 360 held that when grantee of a land for a certain term in
years transfers his interest, the right to renewal of the grant also goes to the transferee
along with the land unless the express or implied intention is different.

Subsequent court decisions have expanded the term land to include "trees, shrubs and
saplings" as on date of agreement. Other Acts like Easement Act also clarifies the
conception of house/ land that with transfer of dominant heritage the easement benefit
will also change hands

This is best understood by the statement "A vendor cannot convey a title better than
what he has" - meaning what rights the vendor (whether as the seller or the lessor or the
mortgagor etc.) possesses only that he can transfer, he cannot transfer a title better than
what he has. So, vendor can transfer all the interest he possesses and any legal ability to
enforce his interests along with the property.

Page | 60
L&B/Chapter-1/Law-Real Estate

Vested and Contingent interest


Vested Interest Contingent Interest
Governing sections Sec - 19 of TPA and Sec 119 of Indian Sec 21 of TPA
Succession Act, 1925.

When transfer This is a certainty. In fact, has already Transfer is possible but not
happens happened and often even certain to occur.
communicated to transferee.
Note: despite being a certainty some
additional event in future may cause
the dissipation of the vested right.

Transfer of interest It covers both situation - present right The right to enjoyment takes
when takes effect to currently enjoy or is present right to effect on the happening or not
future enjoyment happening of some future
uncertain event or the doing or
not doing of some activity
described in the transfer
instrument of the property

Example 'A' transfers a life interest in A transfers life interest in


property to 'B' via will/ family property to B via will/ family
settlement and after B's death the settlement and after B's death
property will go to C and D. But if the property will go to B's
B's has a child the property will go children. In her lifetime B enjoys
to her child and not to C & D. In this the property. But her children
case C & D have vested interest till only have interest contingent on
no child is born to B. The time a their surviving their mother B.
child is born to B their vested So, while the death of B is
interest is dissipated/ eroded into certain, the order of death of B
nothingness. and her children is not known.

Vested interest is defined in Sec - 19 of TPA and Sec 119 of Indian Succession Act,
1925. In case of vested interest while the property is transferred but the transferee's
interest does not take immediate effect nor is it clear from the transfer deed when it will
take effect. The transfer is certain. Even if the transferee expires before getting the
property, his claim survives till the time the necessary event occurs and his heirs and
assignees can step into his rights.

It does not mean that existence of vested interest is inferable automatically where the
enjoyment of the property rights is postponed to sometime in future. Nor does it mean

Page | 61
IOV-Registered Valuers Foundation

that a prior interest is given to another. If income from property is not given to the
beneficiary but accumulated till the happening of a future event that too does not mean
that existence of vested interest is confirmed.

Contingent interest - the property is transferred but it will take effect on fate of an
uncertain event or action. Interest is vested once the desired event/ action take place.
Sec 21 itself states if a property is to be transferred to B when B attains a certain age and
if till then her needs are to be met from the income of the property then it's not a
contingent transfer.

• SALE
As per sec 54 of TPA Act: Sale is a transfer of property in exchange for a money value -
the price may have been paid, promised to be paid or partly paid and partly promised.
The completion of payment is not a condition precedent to transfer of any property.

Sale Transfer under TPA is not always absolute.

1. Sec 39 of the TPA deals with third person's right of maintenance from the
property or to get a provision for marriage, these rights would be enforceable
against the transferee who had due notice of it.
2. Sec 7 defines who may contract to transfer a property and one condition by
reference to the Contract Act, 1972 is that the person must be a major. Hence a
minor cannot transfer. His guardian can transfer to meet his needs/ in
furtherance of his benefit or it can be alienated on permission of the court.
Within three years of attaining majority the minor has a right to apply to court to
repudiate the sale of his property restoration of his property to him.

The date of sale is crucial in determining liability for charges and taxes. It is payable by
seller till that date, unless otherwise agreed. The seller is also entitled to rents and profits
till ownership transfer to buyer during the process of transfer.

Since the liability of stamp duty led to people undercutting the price recorded in sale
deeds, government per force introduced - Circle Rates/ Guideline rates. The following is
not from TPA and relevant to it but is none the less important in understanding concept
of Sale at market price. Other methods are also used to dodge the requirements of
paying appropriate stamp duty.

In Suraj Lamp & Industries (P) Ltd. vs. State of Haryana the Apex court called for an
end to the 'pernicious practice' of sales via Sale Agreement/General Power of
Attorney/Will which it said are resorted to 'avoid payment of stamp duty and registration
charges on deeds of conveyance and/or to avoid payment of capital gains on transfers,
and thus to invest 'black money' while avoiding payment of `unearned increases' to

Page | 62
L&B/Chapter-1/Law-Real Estate

Development Authorities. Submissions by the states of Delhi, Haryana, Punjab and Uttar
Pradesh were that such sales led to loss of revenue and increase in litigation due to
defective title. This decision will not affect transfers before the date of the instant
judgment. It will also not affect genuine transactions involving Power of Attorney to
immediate/ close relatives (spouse, son, daughter, brother, sister or a relative) to manage
his affairs or to execute a deed of conveyance. It also continues to permit land owner to
execute development agreement with a land developer or builder giving Power of
Attorney empowering the developer to execute agreements of sale or conveyances in
regard to individual plots of land or undivided shares in the land relating to apartments
in favour of prospective purchasers.

• RIGHTS AND LIABILITIES OF BUYERS AND SELLERS

Unless the buyer and seller decide on a modified or different set of rules in part or in whole
the provisions of sec 55 TPA which defines Rights and Liabilities of Buyers and Sellers will
apply.

DUTIES OF SELLER

1. Update the buyer on the defect of the seller's title to the property which the
seller knows and the buyer cannot discover by ordinary diligence.
2. If requested by buyer, produce all title related and allied documents in his
possession or which is within his power to procure
3. Respond to the questions on the property/ title put by the buyer based on best
information known to him
4. On receipt of price vide payment or by other tender, to execute proper
conveyance deed provided by the buyer
5. Between the date of the deed and the date the property is delivered to the
buyer, the seller undertakes to take proper care of the property and related
documents as any other careful man would have done
6. Give possession to the property when required to the buyer or the representative
indicated by the buyer

7. Pay all charges or dues till the date of sale

By executing a sale deed the buyer is holding out that the interest is subsisting and he
has legal capacity to transfer it vide the deed (this is referred to in judicial decisions as
the "Statutory Warranty of Title" or also as "Covenant of Title" and this cannot be waived
by agreement of parties. Another attendant covenant read into this is the "Covenant of
quiet possession" which goes with the "Warranty of Title" - that is right to quiet
enjoyment by transferee on transfer of title).

Page | 63
IOV-Registered Valuers Foundation

When a Trustee or Receiver or some person holding fiduciary capacity is transferring


then he is also thereby holding out that the seller has not done anything that puts a
bar/limit on his capacity to transfer. He is also holding out that the property is not
encumbered in any manner other than what was declared.

The section clarifies that both the interest in the property and the transfer of the right to
its benefit is presumed on execution of the deed enforceable by anyone having the
interest.

Seller’s Rights

1. Seller retains rights to the profit/ rent of the property till the date of the transfer
unless prior rents are also specifically transferred.

2. If the property is transferred before whole of the agreed amount is received by


seller then the seller has a charge on the property so transferred. The buyer,
every subsequent transferee who got without payment and such other transferee
who has notice of this non-payment will be subject to the charge on the
property.

BUYER'S DUTIES

1. Disclose such aspects of the property that enhances its potential price/ value or
disclose the full extent of seller's interest in the property which buyer is aware of
but are unknown to seller.

2. To pay/ tender to the buyer or his representative the price of the property at
time/ place where sale was completed. If there is any encumbrance on the
property which was not disclosed (or sale was made on unencumbered basis)
then when paying he can retain the amount payable to the encumbrance holder.

3. After title has passed to the buyer he is liable to suffer for any subsequent loss or
destruction to the property which was not caused by the seller.

4. He is liable for all charges payable on the property. He is also liable to pay off
any encumbrance (both principal and interest) of which he had notice of when
purchasing.

Buyer's Rights

1. Right to all rents and profit and also benefit stemming from any improvement in
the property or increase in its value.

2. Buyer has a right to a charge being levied on the property against the seller and
others claiming through the seller in these following situations

Page | 64
L&B/Chapter-1/Law-Real Estate

i. If buyer has paid any part of the purchase money in anticipation of delivery
ii. Any earnest money paid
iii. Any cost awarded to buyer by the court
iv. However, this right to levy a charge will not hold if the purchaser had declined
to take possession either because he failed to pay full price or other conditions
set in contract would constitute "improperly declined to take possession".

• MORTGAGE
Mortgage (sec - 58) is transfer of interest in a particular, identifiable immoveable
property for securing money already advanced or to be advanced as loan or other type
of debt or any engagement that can result in a financial liability. The transferee is the
mortgagee while the person transferring is the mortgagor. It is not necessary that
mortgagor is always the owner of the property. He can be for example leaseholder with
right to mortgage for general or specified purpose. Sec 59A further clarifies that the term
mortgagor or mortgagee includes all people claiming through him unless it is specifically
barred.

Sec 58 of the Transfer of Property gives details of different types of mortgage. But Banks
resort to mostly following types of mortgage:

Simple Mortgage (sec 58(b)) property continues in possession of the borrower


(mortgagee), only a personal undertaking acknowledging the loan and agreeing to repay
on a specified date is with the mortgagor (bank).

Mortgage by deposit of title deed (sec 58(f)) where original title documents are
deposited with the mortgagee/ bank or an agent specified them to create a security. This
TPA Act provides for this option of mortgage in Kolkata, Chennai and Delhi. It leaves
discretion to the State government to extend provision to other towns as well.
English Mortgage (Sec 58(e)) - Mortgagor absolutely transfers the mortgaged property
with absolute ownership rights with intention to create a mortgage and also commits to
repay the amount on a certain day. But the absolute transfer is done subject to a
condition that - the transferee will return the property on return of the money by the
mortgagor.

Other mortgages detailed in this section of TPA are: Mortgage by Conditional Sale (58(c))
- the property is conditionally so - if the amount obtained against this mortgaged is not
returned by the specified date, the conditional sale shall become final. Instead if the
amount is repaid then the sale shall be cancelled, or it may provide that the property will
be retransferred back to the seller. Explicitly stating when the amount advanced is
returned the property will be returned to seller - only then the deed can operate as a one
of mortgage by conditional sale.

Page | 65
IOV-Registered Valuers Foundation

Mortgaging has led to the development of a special class of Property Buyers


known as “Vulture Buyers” in the United States who as per (Mitchell et al.,
2010) target owners of distressed properties worried about potential forced
sales. Such property owners who have fallen behind on their loan payments
know a foreclosure action (where the lender moves in to assert claim on the
property given as security) may destroy or severely damage their credit rating
and make future loans impossible. The vulture buyers are able to reap
handsome profits by purchasing the property directly from the distressed
owner either prior to or after a foreclosure action has been initiated merely
to help them retain their credit scores.

Comes from Civil law, Bankers in India are referring credit scores of potential buyers generated by
right of limited duration. Credit Information Bureau Limited (CIBIL) - the credit rating scores more to
Right to use the property know existing level of indebtedness. Do you think when CIBIL reports are used
& enjoy its fruits more to assess credit worthiness of return capability from past repayment
behavior will such a class of buyers also be seen in India.

Usufructuary Mortgage - in this possession of the mortgaged property is transferred to


the person advancing the money along with the right to collects rent, other income
accruing from the property till the repayment fully or partly of the money advanced and
interest on it. Usufruct mortgage is different from "profit a pendere" which is not a
mortgage or a lease but essentially a License to enter and only collect something from
within it e.g. to harvest crop/ pluck fruits.

Anomalous Mortgage - A mortgage that does not fall in any of the foregoing
categories is an Anomalous Mortgage.

Why the other types of mortgage are not resorted to by banks will be best known to the
financial institution. Banking Regulation Act (Sec 9 on Non-banking asset) states 'no
banking company shall hold any immovable property howsoever acquired, except such
as is required for its own use, for any period exceeding seven years from the acquisition
thereof". Perhaps some forms of mortgage like mortgage by conditional sale or
Usufructuary mortgage may not be as common with banks because of this provision and
the requirement.

Mortgages involving principal amount of Rs. 100 or more has to be executed by a


registered document which is signed by two witnesses (except a mortgage by deposit of
title deed which does not have to be registered). It is important to remember that by its
very definition, mortgage refers to an immoveable property. A pledge refers to a
"moveable property" which is delivered to the pagdee as a security. In case of default
in due repayment, in pledge, the lender i.e. the holder of the immoveable asset can sell
it. In case of hypothecation too moveable asset is involved but unlike in pledge it is not

Page | 66
L&B/Chapter-1/Law-Real Estate

delivered to the person taking the loan (i.e. lender), it remains with the hypothecator.

ICICI Bank Ltd. Vs. Sidco Leathers Ltd. DOJ 28/04/2006, SC held that the dues of the
workmen and the debt due to the secured creditors are treated pari passu with each
other, but that does not dilute the concept of inter se priorities amongst the secured
creditors. Such a valuable right, therefore, must be held to have been kept preserved.

The State's Debts ranks above bank dues: This is derived from the English rule of Crown
debt preceding others. In Central Bank Of India vs. State of Kerala DOJ 27/02/2009 the
Supreme Court held the Recovery of Debts Due to Banks and Financial Institutions Act,
1993 and the Securitisation and Reconstruction of Financial Assets and Enforcement of
Security Interest Act, 2002 (SARAFESI or Securitisation Act) do not create first charge in
favour of banks, financial institutions and other secured creditors.

As per Sec 48 of the Transfer of Property Act, 1882 Act, the claim of the first charge
holder shall prevail over the claim of the second charge holder. Hence property valuation
reports need to reflect the nature of the security and the value of the earlier charge on it.

Mortgage default insurance is meant to cover normal risks as also catastrophic risks -
latter being foreclosure occurring due to economic depression. The risk horizon is long -
15 years or more. Inflation here reduces the risk of mortgage insurer's loss (Blood, n.d.).

Credit Rating Information Services of India Ltd. CRISL as per (Blood, n.d.) has staff and
competency in rating new general lines insurers and carriers that choose to engage in
writing mortgage default insurance. CRISIL has rated HFCs, banks and builders.

Entities like Housing Development Finance Corporation (HDFC) amongst the largest
Housing Finance Company in India - are reported to have studied mortgage insurance
business anticipating deregulation of insurance sector and allowing of other entities to
engage in it, yet such mortgage default insurance is not common. As of now borrowers
are insured for default due to death through the financial institution mandating an
insurance policy.

• RIGHTS OF MORTGAGOR

Sec 60 TPA deals with the right to redeem the mortgaged property and associated
documents and a court case filed to enforce this right is called a suit for redemption. At
any time after the principal money has become due, the mortgagor has a right, on
payment or tender, at a proper time and place, of the mortgage-money, to require the
mortgagee (a) to deliver to the mortgagor the mortgage-deed and all associated
documents of the mortgaged property till then with the mortgagee, (b) to deliver
possession of the property where applicable to the mortgagor and at the cost of the

Page | 67
IOV-Registered Valuers Foundation

mortgagor either to re-transfer the title of the mortgaged property to him or to such
third person as he may direct, or to execute and if mortgage was vide a registered deed,
then also register a mortgage release deed.

For this act to apply the parties should not have acted to extinguish mutual rights
stemming from lease deed. If the time of payment of principal is passed or not fixed, the
mortgagee in equity is entitled to a reasonable notice before payment or tender of such
money necessitating release of the property/ documents.

When there is more than one mortgagor of the property, part release by one mortgagor
or any person interested through him by paying his proportionate share of the mortgage
amount is not allowed.

Sec 60A states that by virtue of his redemption rights a mortgagor may assign the
mortgage debt and transfer the mortgaged property to a third person; and the
mortgagee shall be bound to accordingly transfer to the third person as directed by
mortgagor. The section also details the right can be enforced by or the mortgagor or by
any encumbrance holder. But encumbrance holder's claim is prior to that of mortgagor
and between encumbrance holders, the requisition of a prior encumbrance shall prevail
over that of a subsequent encumbrance. Section does not apply to a mortgagee who
was or has been in possession.

As long as a mortgagor's right of redemption subsists, mortgagor shall be entitled at all


reasonable times after requesting can make copies at this cost or inspect the documents
relating to the mortgaged property which are in the custody or power of the mortgagee.

Sec 61 clarifies where two or more property are mortgaged to same mortgagee at the
option of the mortgagor, he can seek to redeem one first or can redeem both together.

Sec. 62 Right of Usufructuary mortgagor to recover possession on clearing dues.

In a Usufructuary mortgage, the mortgagor has a right to recover possession of the


property together with the mortgage-deed/ documents with the mortgagee on payment
of the mortgaged money from rents/ profits or if he is authorised to pay himself from
such rents and profits or if the period has expired but the money not recovered then
when mortgagor pays or tenders to the mortgagee the mortgage-money balance
thereof or deposits it in court.

As per sec 63where there is any in the mortgaged property in possession of the
mortgagee redemption shall, in the absence of a contract to the contrary, be entitled as
against the mortgagee to such accession.

Sec 63A where mortgaged property in the possession of the mortgagee has, during the

Page | 68
L&B/Chapter-1/Law-Real Estate

continuance of the mortgage, been improved, the mortgagor, upon redemption, shall, in
the absence of a contract to the contrary, be entitled to the improvement. If such
improvement was paid for by the mortgagee and was necessary to preserve the property
from destruction or deterioration or was necessary to prevent the security from
becoming insufficient, or was ordered by a government or a public authority, the
mortgagor shall, in the absence of a contract to the contrary, be liable to pay the proper
cost thereof as an addition to the principal money with interest at the same rate as is
payable on the principal, or, where no such rate is fixed, at the rate of nine per cent per
annum, and the profits, if any, accruing by reason of the improvement shall be credited
to the mortgagor.

Where mortgaged property is a lease, and the mortgagee obtains a renewal of the lease,
the mortgagor, upon redemption, shall, in the absence of a contract by him to the
contrary, have the benefit of the new lease.

Liabilities of Mortgagor

Sec 65 sets out the implied contracts by mortgagor. It states in the absence of a contract
to the contrary, the mortgagor shall be deemed to have contracted the following with
the mortgagee-

1. The mortgagor has the power to transfer the said interest in the property and the
interest continues to subsist

2. Should it be needed the mortgagor will defend his interest during the mortgage
and when the mortgagee is in possession of the property, then mortgagor will
help the latter to defend title
3. Till the time mortgagee is in possession of the mortgaged property, the
mortgagor must pay all public charges
4. If a leased property is mortgaged, the mortgagor is deemed to be holding out
that the dues payable and condition set under the lease deed have been duly
complied, including the condition about renewal of deed. Till the mortgagee is in
possession, the mortgagor shall clear all dues. For all past dues not paid, the
mortgagor will indemnify the mortgagee against all the claims that may ensue

5. Sometimes a mortgaged property is re-mortgaged again: in such case the


mortgagor undertakes to clear interest dues accruing under the prior
encumbrance on the property and also discharge the principal when due.

6. Rights of mortgagee may be enforced by every person in whom that interest is


for the whole or any part thereof from time to time vested for whatever reason.

Sec 65A deals with powers of a mortgagor who is in possession to further lease the
property and it shall be binding on mortgagee. The lease will take effect within six

Page | 69
IOV-Registered Valuers Foundation

months from date of leasing. The lease shall comply with local custom and rules.
Nopremium shall be paid or promised and no rent shall be payable in advance nor make
any commitment about process of renewals

In the case of a lease of buildings, whether leased with or without the land on which they
stand, the duration of the lease shall in no case exceed three years and including a
condition of re-entry if rent is not paid on time.

As per TPA a property offered as security is deemed insufficient unless the value of the
mortgaged property exceeds by one-third, or, if consisting of buildings, exceeds by one-
half, the amount for the time being due on the mortgage.

• RIGHTS OF MORTGAGEE

Sec 67 TPA read with the explanation deals with the right of mortgagee by conditional
sale or a mortgagee under an anomalous mortgage having right to foreclose, to
foreclose or sell the mortgaged property as per process set in TPA. To foreclose means
to take away the right of mortgagors to redeem their mortgaged property. Unless the
mortgage deed states otherwise, and subject to the conditions set in TPA, at any time
after the mortgage money has become due and before the money is actually paid or a
court passes a decree for the redemption of the mortgaged property, the mortgagee has
a right to obtain from the court a decree that the mortgagor shall be absolutely
debarred of his right to redeem the property, or a decree that the property should be
sold. Such a suit is called a suit for foreclosure.

Liabilities of Mortgagee

1. The mortgagee by conditional sale cannot institute a suit for foreclosure as such
before the happening of the condition.
2. A mortgagor who holds the mortgagee's rights as trustee or legal representative
of mortgagee, cannot institute a suit for foreclosure even if he had the right to
sue for a sale of the property as per trust deed/ mortgage deed
3. Mortgagee in case of a railway, canal, or other work in the maintenance of which
the public are interested, to institute a suit for foreclosure or sale
4. When there is more than one mortgagee in a deed, a suit for foreclosure cannot
be filed for one mortgagee's interest in the mortgaged property till the time they
have first separated severed their interests under the mortgage.

TPA also casts a duty on the person acquiring immovable property or any share/interest
by mortgage or otherwise as being presumed to have notice of the title of any other
person who was in actual possession of such property.

Page | 70
L&B/Chapter-1/Law-Real Estate

All sales, mortgages (other than by way of deposit of title deeds) and exchanges of
immovable property are required to be registered by virtue of the Transfer of Property
Act.
• CHARGE
Where immovable property of one person is by act of parties or operation of law made
security for the payment of money to another and when it does not amount to a
mortgage, the latter person is said to have a charge on the property. The provisions of
Simple Mortgage apply. Sec 100 deals with charge and it states this section does not
apply to the charge of a trustee on the trust-property for reimbursing expenses properly
incurred in the execution of his trust. No charge shall be enforced against any property
once it is transferred for consideration without prior notice of the existence of charge
(except as provided in law).

• LEASE

In Sec 105 TPA the term "lease" is defined. A lease of immoveable property is a transfer
of a defined right to enjoy such property, made for a certain time, or in perpetuity. It
could be clearly stated or implied and in consideration of a price (paid or promised), or
of money, a share of crops, service or any other thing of value, to be rendered
periodically or on specified occasions to the transferor by the transferee, who accepts the
transfer on such terms.

The transferor of the right to enjoy such property, is called the lessor, the transferee is
called the lessee. The price paid or promised is called the premium. The 'money, share,
service or other thing to be so rendered usually at predetermined periodicity in return is
called the 'rent'.

Lease License

Governing Sec 105 of Transfer of Property Sec 52 of Easement Act, 1882


statutory Act and also relevant sections
provision of the corresponding Rent
Control/ Premises tenancy Act
of the State governing lease of
premises
Granted to To an identified individual/ To an identified individual/ body of
body of individuals individuals

What is the Delivery of exclusive A person, permits another a right to do or


lesser/ grantor possession of the property continue to do upon a property which in
allowing along with a defined interest in absence of such right would be unlawful

Page | 71
IOV-Registered Valuers Foundation

the property. Transfer of Only a permission to use the immoveable


interest is the defining property of the grantor. No interest in the
distinction said property is created.
Normally without right to exclusive
possession. However even if defined
exclusive possession is granted other
elements of License like mere permission to
use/ no interest created is point of
determination
Nature of the The rights are assignable/ License rights are not attached to the
rights inheritable within terms of the property and do not travel along with
lease deed change of ownership of the underlying
property. It is granted to a person. Hence
License is not inheritable nor can it be
assigned. The licensee's right dies with the
license holder.
Unless specifically permitted or an integral
element of it, License rights cannot be
transferred nor exercised by assignees,
servants or employees of License holder.
Exception for example include a movie
ticket which gives License to enter theatre
to watch the chosen movie in a manner
acceptable to the theatre manager. Its
nature is such that it can be transferred. But
other Licenses on immoveable property are
not transferable.
Can be created not necessarily only by the
owner, but even by tenant/ leaseholder but
not exceeding their own tenure (period).
Life interest holder too can create but not
for a period beyond his life time

Page | 72
L&B/Chapter-1/Law-Real Estate

Fate on Continuation governed by If granted for specific period, on expiry of


termination provision of the lease deed lease period licensee loses right to continue
and otherwise the TPA. to occupy. It expires with death of the
holder.
Lease deed may get
automatically continued for On termination of license by passage of
the same period again on time, the exiting licensee can seek time to
termination of the lease period leave the property and also to remove
if monthly payment continues building materials etc.
to be received.
In some lease it may convert to If licensed property is transferred by the
year-on-year lease when licensee to some other person against the
payment continues to be terms of the License, the transferee is a
accepted. trespasser against the transferor/ property
It is possible to transfer the owner/ original grantor.
right of re-entry on expiry of
the lease or on breach/
violation subsequently of a
condition set during grant of
lease

Example - State continues to levy a • At times a license can be converted to


leasehold rent on flats lease by way of one deed - a developer
developed in land owned is granted license to build and then
by Delhi Development after completion will convert as
Authority leaseholder for part of the building to
run office or other purpose
• A person cultivates the land of another,
shares the produce of agriculture with
the owner but has no right on the land
• Right to cut grass, collect and take away
leaves, bamboo are examples of license
• Allowing person to continue in
possession till the time the land is used
for the exclusive purpose of running a
school

In distinguishing Agreement to Lease vs. Agreement for Lease Supreme Court has held a
confirming third party (in this case a company) in agreement to lease a property did not
create a lease subject to Bombay Stamp Duty Act 1958 as relationship of lessor and
lessee was not established between the company and the State. It was only an
agreement to lease and thus does not fall under sec 2(n) of Bombay Stamp Duty Act

Page | 73
IOV-Registered Valuers Foundation

1958 [State of Maharashtra vs Atur India (P) Ltd. 1995(2) Bom CR 31 (SC))

The disrepair for which a leaseholder or tenant is usually liable for mainly commercial
properties when he has agreed to return premises in good repair is called Dilapidations.
No property can usually be returned in the same condition as received. But when the
building continues in possession of leaseholder/ tenant, deterioration may continue. It is
important to track if the building is notified for being pulled down. For example, section
354 of the Brihan Mumbai Municipal Corporation (BMC) Act deals with removal of
dangerous structures authorises demarcating even private buildings and for evacuation
to prevent human lives from being compromised due to shaky structures.

A tenant or lessee has liability to maintain hedges, boundaries, doors, premises etc. It is
thus breach of lease covenants relating to the condition of a property, and the process of
remedying those breaches. Landlord can demand being made good for the loss or
demand actual repairs being done. This is of special significance in UK jurisprudence
where in the event of disputes appraisers review case laws to argue their preferred
position.

Lease deeds were traditionally executed by two ways: written document where the lessor
executes a lease patta with recipient signing or the recipient i.e. lessee executing a
kabuliyat admitting to the receipt or orally along with delivery of possession.

Section 106 of TPA gives the period of lease in the absence of contract or local law or
usage stating otherwise

A lease of immovable property for agricultural or manufacturing purposes is deemed to


be year to year that can be terminated by either side on six month notice

A lease of immovable property for any other purpose is a lease from month-to-month
that can be terminated by either side on fifteen days' notice.

It also states the start date of notice of six month/ 15 days will be counted from date of
receipt of notice by the other side. If the period of notice falls short of the required
period then it is not a failing if the suit seeking recovery has been filed with delay. The
notice has to be served in writing, duly signed by issuer and sent by post or delivered in
person to the other side or handed to one of his family members or servants at his
residence or affixed to a conspicuous part of the property.

• RIGHTS AND LIABILITIES OF LESSOR AND LESSEE

Rights and liabilities of lessor and lessee are given in Sec 108 TPA the rights and
liabilities of lessor and lessee. The rights are tempered by the liabilities applicable on

Page | 74
L&B/Chapter-1/Law-Real Estate

lessor/ lessee as are applicable to the property leased:

Liabilities of the Lessor

1. Must disclose to the lessee any material defect in the property considering the
usage to which it will be put and known to lessor and which the lessee could not
with ordinary care discover
2. When lessee asks to put lessee in possession of the property he must comply

3. On due payment of rent as agreed in the lease contract and complies with all
conditions set by lessor he may hold the property without interruption for the
time till the lease determines during the time limited by the lease. Anyone who
claims through the lessee as the lease interest is vested in him will be entitled to
the benefit of the lease deed.

Rights of the Lessee

1. During continuation of the lease period any improvement/ accession in lease


property shall be deemed to be included in his lease;

2. Due to climate elements like 'fire, tempest or flood' or due to attack by any army
or mob, if the lease property is wholly or partly destroyed or becomes unfit for
the purposes it was taken on lease, at the option of the lessee the lease can
cease/become void. This protection will not be available if the injury is result of
the wrongful act or default of the lessee, he shall not be entitled to avail himself
of the benefit of this provision;

3. If the lessor does not carry out necessary repairs, the lessee may get the repair
done and deduct the expense of such repairs with interest from the lease rent
payable to lessor, or otherwise recover it from the lessor;
4. The lessee can pay off compulsory liabilities of the lessor which if left unpaid will
be recovered from lessee being the occupier or from the property, and deduct
principle and interest paid from the lease rent payable to or otherwise recover
from the lessor;

5. Once the lease determines (i.e. ends) but lessee continues in possession, the
lessee may remove, all things which he has attached to the earth but he has to
restore the property in the state in which he received it;

6. If a lease whose period is not definite, terminates for no fault of the lessee, then
even after its end lessee or his representative shall have free access to the land to
harvest and carry them

7. The lessee may transfer absolutely or mortgage or sub-lease the whole or any
part of his interest in the property and his transferee can further transfer it
likewise. But sub-lease and sub-transfer will not abate the liability of the lessee to
comply with the terms of the lease. A tenant with an un-transferable right of
occupancy cannot transfer/ lease. A farmer of an estate which has defaulted in
paying revenue or estate of one whose assets are being managed by the Court of

Page | 75
IOV-Registered Valuers Foundation

Wards cannot also assign his interest as such tenant, farmer or lessee.

Liability of the Lessee

1. The lessee must disclose to the lessor any fact known to lessee as to the nature
or extent of the interest which the lessee is about to take, of which the lessor is
not, aware, and which materially increases the value of such interest;
2. The lessee is bound to pay or tender the premium or rent to the lessor or his
agent in this behalf

3. The lessee is bound to keep property in good condition and on the termination
of the lease to restore, the property in the same condition as it was in at the time
when he was put in possession, except dilapidation cause by normal wear and
tear or irresistible force,

4. Has to allow the lessor and his agents, at all reasonable times during the term, to
enter and inspect the condition of the property and if any information is given of
any defect in the property due to action or default of lessee or his agents he will
make good the loss within three months of such notice
5. On learning of any legal action against the property or encroachment or others
which amounts to interference with the lessor’s rights on the lease property,
lessee must notify the lessor within reasonable is bound to give, with reasonable
diligence, notice thereof to the lessor
6. Like an ordinarily careful person would use his own property, a lessee may use
the lease property and its products. Further he must not use or permit another to
use, the property for a purpose other than that for which it was leased - mining,
quarrying cannot be started if land was not leased for that purpose. Nor must he
cut or sell sanding timber or pull down or damage any buildings belonging to
the lessor. Broadly any act that is destructive or permanently injurious to the
property is not allowed.

7. Without the lessor’s consent, permanent structure, except for agricultural


purposes cannot be erected
8. On the determination of the lease, the lessee is duty bound to put the lessor into
possession of the property.

• EXCHANGE
Sec 118 refers to a specific type of mutual two-way transfers. In exchange two persons
are involved where each transfers their ownership of one thing for the ownership of
another's non-money item. This distinction is important - in sale a thing is exchange for
money or a promise of money. But in exchange it's more a barter of kind. Even if the
exchange deed mentions the value of the items being exchanged from either side, that
will not change the character of exchange. A transfer of property in completion of an
exchange can be made only in manner provided for the transfer of such property by sale.

Page | 76
L&B/Chapter-1/Law-Real Estate

Family partition, family settlement, handing over of property in any compromise do not
constitute an exchange since condition of mutual transfer of property by the two parties
is not fulfilled.

• GIFT AND HOW TRANSFER VIDE GIFT IS AFFECTED


As per sec. 122 TPA, a "Gift” is the transfer of certain existing moveable or immoveable
property made voluntarily and without consideration, by one person, called the donor, to
another, called the donee, and accepted by or on behalf of the donee. The gift has to be
to a defined number and definite number of people, to be a valid gift.

Acceptance must be made during the lifetime of the donor and while he is still capable
of giving to be a valid deed. Conversely, if the donee (recipient) dies before acceptance,
the gift is void. It is possible that gift was accepted and its acceptance can be proved but
the transferor soon after died and hence the transfer in interest was not registered via
mutation during lifetime of the transferor. Then the transfer vide gift is held as valid.

A transfer of property is critical to constitute a "gift". Thus creation of a trust and


transferring property to it for the benefit of someone is not gift to the recipient if the
trust generally gives beneficial interest to the beneficiary while the property remains with
the trust. But a gift to an idol or for religious institution is valid (Santi Swaroop vs.
Radhaswami Satsang Sabha AIR 1969 All 248). Under Mitakshara property system, the
co-parceners hold the property jointly. One or some of the co-parceners cannot transfer
their share without consent of the other co-parceners.

Sec. 123 clarifies transfer via gift deed of immoveable property is to be made only
through a registered instrument signed by the donor or his representative, and attested
by at least two witnesses.

In case of a gift of movable property, registration of the deed is optional. It can also be
done by delivery of the property in a manner goods in sales are delivered.

Since the days of Transfer of Property Act, the concept of "property" has evolved.
Transferable Development Rights (TDR) is only a tradable privilege that originates in one
property which can be transferred to another area and property:

TDR is a certificate issued against land acquired for public purposes either by the Central
or State Government in consideration of surrender of land by the owner without
monetary compensation and free from all encumbrances or by way of declared
incentives by State Government, which are transferable in part or whole. It is issued by
the ULB's Urban Local Body (Municipal body, Urban Improvement Trust, Urban
Development Authority).

Page | 77
IOV-Registered Valuers Foundation

Care is taken to back TDR rights legally. Government of Karnataka amended the
Karnataka Town and Country Planning Act, 1961 in order to empower the local bodies
(Corporations / Planning Authorities) to permit additional FAR for the land. Likewise, a
new sec 14B was inserted in the A.P Town and Country Planning Act 1920. The land
prohibited by order of any court or lands under acquisition are not eligible. TDR
certificate can have name of a nominee and is usable by nominee on the death of the
holder and is tradable.

Maharashtra regulations required part of the FSI increase is be used in remainder portion
of the same plot whose portion of which was acquired for road widening etc. If needed a
further FSI can be purchased from other holders and used

As per a note on TDR prepared by Ministry of Urban Development, GOI in Aug 2015 for
Telengana Govt., TDR is resorted to for the following reasons:

• To facilitate development of affordable houses under State Affordable Housing Policy


In lieu of Floor Area Ratio (FAR) granted as per the Policy - TDR shall be calculated on
the total plot area being reserved for affordable housing project (including Economic
Weaker Section EWS / Low Income Group LIG) subject to the norms as prescribed in
Affordable Housing Policy. The maximum TDR shall not be more than 1.5 times of the
total land area but in case of Affordable Housing projects the developer shall be
provided double of the permissible FAR (e.g. at the time of launch of Affordable
Housing Policy permissible FAR was 1.67/1.80 hence the double FAR for both the
cases will be 3.50) in case of slum rehabilitation scheme under Slum Development
Policy, 2012 maximum permissible FAR is 4.0 but TDR shall not be more than 1.5 and
developer shall consume the maximum FAR on the proposed project site provided all
planning parameters and provisions of Building Regulations are fulfilled. The
unutilized FAR subject to maximum 1.5 can be taken as TDR
• Development of Green spaces - Parks/ Open Spaces/Playgrounds /Water Bodies etc.
as per the provision of Master Plan/ Sector Plan.
• Development of Master Plan/ Sector Plan roads including road widening
• Development of Public Parking lots.
• Development of City level Facilities/other public purposes as per Master Plan
proposals.
• Slum rehabilitation scheme under Slum Development
• In lieu of land surrendered for other purposes as specified by State Government

TDR policy can help to reduce time and money needed to acquire land - notification,
hearing, payment of compensation etc. By one estimate for the land acquisition for the
Peripheral ring road to be financed by a Japan agency, Bangalore Development Authority
needs Rs. 8100 crores a kind of resource it does not have. Often TDR is preceded by

Page | 78
L&B/Chapter-1/Law-Real Estate

reduction of the FAR applicable to the area. The idea is to introduce TDR: "For instance,
the floor area that includes balconies and staircases built on a 30ftX40ft plot should not
exceed 1200 sqft. To make any addition to this floor area, one will have to buy TDR at
twice the cost of guidance value as proposed in the new amendments." "Most parts of
Mumbai, where space is in high premium and has been traditionally a vertical city, the
FAR is 1. Bengaluru is growing horizontally. A FAR of 1 creates a huge market for TDR
making it a viable option for land losers”

Telangana government is considering this route for acquiring half an acre land for a 40
feet road along Banjara Hills of the Annapurna Studios in Hyderabad, owned by the late
Tollywood actor Akkineni Nagarjuna, which is part of 22 acres allotted at a nominal price
ranging from Rs 7,500 to Rs 8,000 per acre. The land is now worth up to Rs 30 crore per
acre.

TRANSFER OF IMMOVABLE PROPERTY, LEASE GRANTED BY


PRIVATE AND STATUTORY BODIES - IMPACT OF EACH ON
VALUATION.

1) LEASE

A lease of an immovable property is a transfer of a right to enjoy such property for a


certain time (express or implied), or in perpetuity, in consideration of a price paid or
promised, or of money, a share of crops, service, or any other thing of value, to be
rendered periodically, or on specified occasions, to the transferor by the transferee, who
accepts the transfer on such terms.

The price is called the premium. The money, share, service or other thing to be rendered
periodically is called the rent. The transferor is the lessor, and the transferee is the lessee.
The lease of immovable property is governed by Sections 105 to 117 of the Transfer of
Property Act.

2) LEASE GRANTED BY PRIVATE & STATUTORY BODIES – IMPACT OF EACH ON


VALUATION

The Transfer of Property Act does not define property but gives only a negative
definition of immovable property (Sec. 3). It states that "immovable property does not
include standing timber growing crops and grass". To determine precisely the meaning
of movability or immovability we have to resort to judicial decisions. The following
definition given by Justice Holloway in the 1972 Madras case of SukryKurdepa vs.
Condikul is accepted as a standard definition. Movability is the capacity in a thing to

Page | 79
IOV-Registered Valuers Foundation

suffer alteration with reference to its location without injury or damaging its
surroundings; while immovability is the characteristic of a thing to suffer alteration with
reference to its location by injuring or destroying its surroundings.

SECTIONS: 3, 5, 6, 7, 25, 53 AND 53A OF TRANSFER OF PROPERTY


ACT 1882.

1) SECTION 3

In this Act, unless there is something repugnant in the subject or context: -

1. “immoveable property” does not include standing timber, growing crops or grass:

“instrument” means a non-testamentary instrument:

“attested”, in relation to an instrument, means and shall be deemed always to have


meant attested by two or more witnesses each of whom has seen the executant sign
or affix his mark to the instrument, or has seen some other person sign the
instrument in the presence and by the direction of the executant, or has received
from the executant a personal acknowledgement of his signature or mark, or of the
signature of such other person, and each of whom has signed the instrument in the
presence of the executant; but it shall not be necessary that more than one of such
witnesses shall have been present at the same time, and no particular form of
attestation shall be necessary:

“registered” means registered in 6[ Bangladesh] under the law for the time being in
force regulating the registration of documents:

“attached to the earth” means:

• Rooted in the earth, as in the case of trees and shrubs;


• Imbedded in the earth, as in the case of walls or buildings; or
• Attached to what is so imbedded for the permanent beneficial enjoyment of that to
which it is attached:
• “Actionable claim” means a claim to any debt, other than a debt secured by
mortgage of immoveable property or by hypothecation or pledge of moveable
property, or to any beneficial interest in moveable property not in the possession,
either actual or constructive, of the claimant, which the Civil Courts recognize as
affording grounds for relief, whether such debt or beneficial interest be existent,
accruing, conditional or contingent:
• “a person is said to have notice” of a fact when he actually knows that fact, or when,
but for willful abstention from an inquiry or search which he ought to have made, or
gross negligence, he would have known it.

Page | 80
L&B/Chapter-1/Law-Real Estate

Explanation I.-Where any transaction relating to immoveable property is required by law


to be and has been effected by a registered instrument, any person acquiring such
property or any part of, or share or interest in, such property shall be deemed to have
notice of such instrument as from the date of registration or, where the property is not
all situated in one sub-district, or where the registered instrument has been registered
under sub-section (2) of section 30 of the Registration Act, 1908, from the earliest date
on which any memorandum of such registered instrument has been filed by any Sub-
Registrar within whose sub-district any part of the property which is being acquired, or
of the property wherein a share or interest is being acquired, is situated;

Provided that-

1. The instrument has been registered and its registration completed in the manner
prescribed by the Registration Act, 1908, and the rules made thereunder,

2. The instrument or memorandum has been duly entered or filed, as the case may
be, in books kept under section 51 of that Act, and

3. The particulars regarding the transaction to which instrument relates have been
correctly entered in the indexes kept under section 55 of that Act.

Explanation II.-Any person acquiring any immoveable property or any share or interest in
any such property shall be deemed to have notice of the title, if any, of any person who
is for the time being in actual possession thereof.

Explanation III.-A person shall be deemed to have had notice of any fact if his agent
acquires notice thereof whilst acting on his behalf in the course of business to which that
fact is material:

Provided that, if the agent fraudulently conceals the fact, the principal shall not be
charged with notice thereof as against any person who was a party to or otherwise
cognizant of the fraud.

Enactments relating to contracts to be taken as part of contract Act

2) SECTION 5

Defines transfer of property as a transaction whereby the possession or ownership of a


property undergoes a change by the acts of the parties to the said transaction. Examples
of such a change of ownership or possession may be in the shape of a sale (where the
ownership is totally effected), .a mortgage (where possession of the property is used) as
a security for payment of a loan or a lease (where possession of a property is enjoyed on
payment of a rent) among others.

Ownership, as seen earlier, consists of a bundle of rights. The rights which so make up

Page | 81
IOV-Registered Valuers Foundation

absolute ownership (e.g., right of possession or right of engagement) are called interests
in property and a transfer of property is either a transfer of absolute ownership or a
transfer of one or none of those interests.

Transferability is the general rule in the law of property and the right to property
includes the right to transfer the property to another person. Subject to this general
principle, the Transfer of Property Act lists certain properties which cannot be
transferred.

3) SECTION 7

Deals with competency to be a transferor. It states that a transferor must (a) be


competent to contract, and (b) have title to the property, or authority to transfer the
property if the property is not his own. In this connection, reference is made to Section
11 of the Indian Contract Act, 1872 which specifies the competency to contract and
states that a person is competent to contract if he is (a) of the age of majority according
to the law to which he is subject, (b) of sound mind, and (c) not otherwise disqualified
from contracting. It is thus clear that the power to transfer depends on the power to
contract, as without a contract to give and take, there can be no transfer at all. Further, it
is also a well-established principle of law that a minor lacks the competence to contract
and consequently the competence to transfer, and what is more, a minor does not even
have the competency to authorize another to enter into a contract on his behalf.

Although a minor cannot be a transferor, a transfer made to a minor is a valid one. In


other words, a minor lacks the competence to be the transferor but he possesses the
competence to be a transferee.

4) SECTION 8

of the Act confers stability on title and removes speculations in a transfer. It elaborates
the operation of a transfer and states that unless a different intention is expressed or
implied, a transfer of a property passes immediately to the transferee. Section 8 also
elaborates the legal incidents involved in the transfer. Some of these are:

1. Where the property is a land - the easements annexed thereto, the rents and
profits accruing after the transfer and all things attached to the earth.
2. Where the property is machinery attached to the earth - all movable parts
thereof.
3. Where the property is a house - the easements annexed to it, the rent accruing
after the transfer as well as locks, keys, doors, windows and all other things
provided for permanent use.

Page | 82
L&B/Chapter-1/Law-Real Estate

5) SECTION 25

Conditional transfer. —An interest created on a transfer of property and dependent upon
a condition fails if the fulfilment of the condition is impossible, or is forbidden by law, or
is of such a nature that, if permitted, it would defeat the provisions of any law, or is
fraudulent, or involves or implies injury to the person or property of another, or the
Court regards it as immoral or opposed to public policy. Illustration

1. A lets a farm to B on condition that he shall walk a hundred miles in an hour. The
lease is void.

2. A gives Rs. 500 to B on condition that he shall marry A’s daughter C. At the date
of the transfer C was dead. The transfer is void.

3. A transfers Rs. 500 to B on condition that she shall murder C. The transfer is void.

4. A transfers Rs. 500 to his niece C, if she will desert her husband. The transfer is
void.

5. Other sections under this head which deal in detail regarding the legal aspects of
the Transfer of Property Act are beyond the scope of this text.

FRAUDULENT TRANSFER

6) SECTION 53

1. Every transfer of immoveable property made with intent to defeat or delay the
creditors of the transferor shall be voidable at the option of any creditor so
defeated or delayed. Nothing in this sub-section shall impair the rights of a
transferee in good faith and for consideration. Nothing in this sub-section shall
affect any law for the time being in force relating to insolvency. A suit instituted
by a creditor (which term includes a decree-holder whether he has or has not
applied for execution of his decree) to avoid a transfer on the ground that it has
been made with intent to defeat or delay the creditors of the transferor shall be
instituted on behalf of, or for the benefit of, all the creditors.

Every transfer of immoveable property made without consideration with intent


to defraud a subsequent transferee shall be voidable at the option of such
transferee. For the purposes of this sub-section, no transfer made without
consideration shall be deemed to have been made with intent to defraud by
reason only that a subsequent transfer for consideration was made.]

Page | 83
IOV-Registered Valuers Foundation

SECTION 53A
Part performance.—Where any person contracts to transfer for consideration any
immoveable property by writing signed by him or on his behalf from which the terms
necessary to constitute the transfer can be ascertained with reasonable certainty, and the
transferee has, in part performance of the contract, taken possession of the property or
any part thereof, or the transferee, being already in possession, continues in possession
in part performance of the contract and has done some act in furtherance of the
contract, and the transferee has performed or is willing to perform his part of the
contract, then, notwithstanding that where there is an instrument of transfer, that the
transfer has not been completed in the manner prescribed therefore by the law for the
time being in force, the transferor or any person claiming under him shall be debarred
from enforcing against the transferee and persons claiming under him any right in
respect of the property of which the transferee has taken or continued in possession,
other than a right expressly provided by the terms of the contract: Provided that nothing
in this section shall affect the rights of a transferee for consideration who has no notice
of the contract or of the part performance thereof.

LAW RELATING TO INHERITANCE/ SUCCESSION

• MOHAMMEDAN: MUSLIM PERSONAL LAW

Muslims in India are governed by The Muslim Personal Law (Shariat) Application Act,
1937. This law deals with marriage, succession, inheritance and charities among Muslims.
The Dissolution of Muslim Marriages Act, 1939 deals with the circumstances in which
Muslim women can obtain divorce from their husbands and to provide for matters
connected therewith. These laws are not applicable in Goa state, and are also not
applicable to Muslims who have married under the Special Marriage Act, 1954

RULES OF INHERITANCE

1. A son gets double the share of the daughter wherever they inherit together.

2. The wife gets one-eighth of the share if there are children and one-fourth of the
share if there are no children. In case the husband has more than one wife, the one-
eighth share will be divided equally among all wives. The husband gets one fourth of
the share of his dead wife's property, If there are children and one-half if there are
no children.

Page | 84
L&B/Chapter-1/Law-Real Estate

3. If the parent has more than one girl children, only two-third of the property shall be
divided equally among girl children. If the parent has only one daughter, half of the
parent's property is inherited by her.

4. The mother gets one-sixth of her dead child's property if there are grandchildren,
and one-third of the property if there are no grandchildren.

5. Parents, children, husband and wife must, in all cases, get shares, whatever may be
the number or degree of the other heirs.

6. Slavery, homicide, difference of religion and difference of allegiance, exclude from


inheritance.

MAHR

Mahr can be defined as a mandatory payment, in the form of money (it can also be
anything agreed upon by the bride such as jewelry, home goods, furniture, a dwelling or
some land) or possessions paid by the groom, to the bride at the time of marriage, that
legally becomes her property. "Dower" is equivalent terminology to that of Mahr, and
can be termed as deferred Mahr, as "dower" refers to the payment from the husband or
his family to the wife, especially to support her in the event of his death.

Mahr is distinct from Dower in two ways:

1. Mahr is legally required for all Islamic marriages while dower was optional,

2. Mahr is required to be specified at the time of marriage (when a certain amount is


promised, if not paid immediately), while dower is not paid until the death of the
husband.
3. Mahr also can be classified as a form of "bridewealth", and is paid directly to the
bride and not to her parents.

Will

The law on Muslim wills is different from that of other Indians and this is because, under
Muslim Law, testamentary disposition of property is considered to be divine in nature
and draws on the Quran. The Muslim will is not governed by the Indian Succession Act,
1925. The Muslim personal laws in India, or the Shariat law, decrees certain rules and
regulations and ways in which an individual can dispose off his/her property.

CERTAIN IMPORTANT POINTS

1. Any person, who is 18 years of age or above, and is of sound mind, is eligible to
make their will, according to Shariat law.
2. The Muslim will can be made either orally or with the use of pen and paper.

Page | 85
IOV-Registered Valuers Foundation

There is no well defined format to draft a will. However, one needs to remember
that the instructions left in the document should be clear and concise, and
legible, to ensure it is followed after their death.

3. According to the Shariat law, a person can only leave one-third of their property
to anyone they wish. The remaining two-thirds will, by law, go to their heir or
heirs, equally shared between them.

For example, if A has total asset of Rs. 7 lakhs and he owes Rs. 50k of someone and let
his funeral expenses to be added of Rs. 50k than A can only left an amount of Rs. 6 lakhs
from which he can give an amount of Rs.2 lakh through WILL (one-third of the asset left),
and remaining Rs. 4 lakhs will go to his legal heirs.

1. Muslim law does not allow a property to be bequeathed to an unborn child.


However, in case the mother is pregnant with the child, and is born within six
months of the death of the person making the will, the child has all rights to
inherit it.
2. Muslim law decrees that a person can cancel the will at his convenience without
giving any reasons, anytime before his death.
3. At the time of making the will, an individual needs to pick the persons who might
execute his will. The request is taken into account at the time of disposal of
assets. The person picked as an executor of the will has the right to dispose off
the assets as specified in the will.

GIFT

A gift is a transfer of property in which the interest in the property will be transfer from
one living person to another, without any consideration and is purely gratuitous in
nature. This is the general definition that is accepted by all the religions, including
Muslim law. As per the Muslim Law, a gift is called as Hiba.

Under the Muslim Law, a gift is a transfer of property or right by one person to another
in accordance with the provisions provided under Muslim law. Hiba is an immediate and
unconditional transfer of the ownership of some property or of some right, without any
consideration or with some return; and the term ‘hiba’ and ‘gift’ are often
indiscriminately used but the term hiba is only one of the kinds of transactions which are
covered by the general term ‘gift’. The other types of gifts include Ariya, where only the
right to enjoy the property is transferred and Sadqah where the gift is made by the
Muslim with the object of acquiring religious merit.

Page | 86
L&B/Chapter-1/Law-Real Estate

• THE HINDU SUCCESSION ACT 1956, THE HINDU SUCESSION


(AMENDMENT) ACT, 205 (39 OF 2005)

The Act lays down a uniform system of inheritance equally applicable to persons
governed by the Mitakshara and Dayabhaga schools as also those in the Southern India
who are governed by the Marumakkattayam, Aliyasanthana and Nambudri systems of
Hindu law. The Act applies to all Hindus and the term Hindu includes Buddhists, Jains
and Sikhs.

It has further been extended even to those persons whose parents are Hindu, Buddhist, Jain
and Sikh and who are brought up as Hindus (Section 2). The Act does not apply to the
property of a person to whom the provisions of the Special Marriage Act, 1954 apply (Section
5).

1. Section 4 of the Act gives overriding effect to the provision of the Act. It
abrogates all the rules of the law of succession hitherto applicable to Hindus,
whether by way of any text, customs or usage, having force of law. Any other law
contained in the Central or State legislation shall cease to have effect in so far as
it is inconsistent with any of the provisions contained in the Act.
2. The Act has abolished impartible estate and the special mode of its succession.

3. The Act has extensively affected the entire concept of Mitakshara coparcenary
which was governed by the rule of survivorship. In this scheme female heirs did
not have any place and the property devolved only on the male heirs of the
coparcenary on the death of a male member under the Act. The rule of
survivorship has a limited application. It would apply only in those cases where a
male member on his death left coparceners only. Legal Right in ancestral property by birth

4. In case such male member of a Mitakshara coparcenary dies intestate leaving


behind a female heir mentioned in Class I of the Schedule, the property of the
deceased would devolve not according to the rule of survivorship but according
to the provisions of this Act, which provides for a specific share to such female
heir.

ORDER OF SUCCESSION
The order of succession provided by the Act is broadly based on the doctrine of
propinquity or nearness of blood and accordingly tine heirs are divided into four
categories instead of three, which are as follows:

Page | 87
IOV-Registered Valuers Foundation

1. Heirs in class I of the Schedule

Heirs in class II of the Schedule

i. Agnates
ii. Agnate means a person related to wholly through males either by blood or by
adoption. The agnatic relation may be a male or a female. One’s father’s brother, or
father’s brother’s son or father’s son’s son or father’s son’s daughter are agnates. So
the final relationship may be male or female but it must be through males. The
relationship can also be in ascending or descending line.
iii. Cognates
iv. Cognates means a person related not wholly through males. Where a person is
related to the deceased through one or more females, he or she is called a cognate.
Thus son’s daughter’s son or daughter, sister’s son or daughter, mother’s brother’s
son, etc. are cognates. Here also the final relationship may be male or female but
there is at least one female in that line of relationship.
v. Note that if there are more than one Widow’s , then they get one share only and then
divide it between themselves and a person immediate family will also be considered
as one unit only.

Page | 88
L&B/Chapter-1/Law-Real Estate

The property devolves firstly upon the sixteen preferential heirs mentioned in class I of
the Schedule to the Act and failing such heirs upon the second will prevail. The
outstanding feature of the above division is that the heirs in class I of the Schedule
inherit the property simultaneously and according to the doctrine of representation in
case of predeceased sons or daughter.

Another important point to be noted is that class I of the Schedule contained that a list
of sixteen heirs out of which eleven are females and five are males of which one male
can claim through a female. All of them inherit in equal shares. All the heirs in class II do
not succeed simultaneously but the heirs placed in one Entry would be entitled to inherit
simultaneously. In the case of third and fourth categories of heirs, i.e., agnates and
cognates, the rules of preference have been adopted in order to determine the priority.

1. The Act has abolished Hindu women’s limited estate and made her absolute
owner of the property irrespective of its source of acquisition. Any property
acquired by a Hindu female in any lawful manner whatsoever and possessed by
her became her absolute property and she enjoys absolute power to dispose of it
in a way she desires. (Section 14 of Hindu Succession Act, 1956).

Page | 89
IOV-Registered Valuers Foundation

2. The Act has also provided uniform order of succession with respect to property
of female Hindu. On her dying intestate her property shall devolve on her
children and husband and thereafter upon her parents and the heirs of parents.
In absence of any issues to her, the property inherited from her parents would
revert back to the heirs of parent instead of devolving upon the husband or heirs
of husband.

3. The right of child in womb at the intestate’s death and subsequently born alive
shall relate back to the date of intestate’s death. (Section 20)
4. The Act lays down some general rules of succession inter alia to the effect that
heirs related to a male or female intestate by full blood are to be preferred to
those related by half blood if the nature of relationship is the same in every other
respect (Section 18). Another rule is that if two or more heirs succeed to the
property of an intestate, they shall take their share per capita and not per stripes.
Such heirs take the property as tenants-in-common and not as joint tenants.
(Section 19)
5. The Act has thoroughly revised the law relating to exclusion from inheritance.
Section 28 discards all the grounds of exclusion based on physical defects,
deformity or disease. The disqualifications are confined to the case of remarriage
of a widow of a predeceased son, widow of a predeceased son of predeceased
son and widow of the brother.
6. Another disqualification stated in the Act relates to a murderer who is excluded
on the principles of justice and equity. Conversion is no longer a ground to
exclude a person from inheriting the property but a converts descendants have
been disqualified from inheriting the property of their Hindu relatives.
7. The right of illegitimate children to inherit the property of their mother has been
preserved but such children are disqualified to succeed to their father’s property.

The Hindu Succession (Amended) Act, 2005 (39 of 2005)

This amendment came into force on September 9, 2005 as the Government of India
issued a notification to this effect. The Act removed gender discriminatory provisions in
the previous Hindu Succession Act of 1956 and gave the following rights to daughters:

The daughter of a coparcener becomes a coparcener in her own right in the same
manner as the son. (Coparcener is a person who has equal right in the inheritance of an
undivided property.)

The daughter has the same rights in the coparcenary property as she would have had if
she had been a son;

The daughter shall be subject to the same liability in the said coparcenary property as
that of a son;

Page | 90
L&B/Chapter-1/Law-Real Estate

The daughter is allotted the same share as is allotted to a son;

According to Supreme Court of India, Hindu female inheritors not only has the
succession rights but also the same liabilities fastened on the property along with the
male members. A new Section (6) provides for parity of rights in the coparcenary
property among male and female members of a joint Hindu family on and from
September 9, 2005. This is a crucial date for the following reason:

This Act applies to a daughter of the coparcener, who is born before September 9, 2005
(and alive on 9 September 2005) on which date the amendment came into force. It does
not matter whether the daughter concerned was born before 1956 or after 1956 (when
the actual Act came into force) since the date of birth was not a criterion for application
of the Principal Act. And there is also no dispute about the entitlement of daughters
born on or after September 9, 2005.

• THE INDIAN SUCCESSION ACT, 1925


The Indian Succession Act, 1925 can be broadly be divided into two categories,
testamentary succession and intestate succession. Testamentary succession is applicable
when a written Will has been created. Intestate succession is applicable when a person
dies without a Will and his properties are to be distributed as per the applicable laws
based on his religion.

For Christians, the laws relating to both testamentary and intestate succession is as per
the Indian Succession Act, 1925.

For Jains, Buddhists and Sikhs, they have been given a wider meaning under the Indian
Succession Act. Hence, laws relating to testamentary succession as per the Indian
Succession Act, 1925 is applicable for them also. Intestate succession is applicable to
them as per the Hindu Succession Act, 1956.

• WILL & TESTAMENT

Making a Will helps ensure one's property devolves as wished and the right heirs receive
their fair shares. Under the Indian Succession Act 1925, a Will is a legal declaration of the
intention of the testator, with respect to his property which he desires to be carried into
effect after his death.
After the death of a person, his property devolves in two ways - according to his Will i.e.
testamentary, or according to the respective laws of succession, when no Will is made. In
case an individual dies intestate (no Will is made), the laws of succession come into
play. A Will is a legal declaration. Certain formalities must be complied with in order to

Page | 91
IOV-Registered Valuers Foundation

make a valid Will. It must be signed and attested, as required by law. A Will is intended
to dispose off property. There must be some property which is being given to others
after the death of the testator.
A Will becomes enforceable only after the death of the testator. It gives absolutely no
rights to the legatee (the person who inherits) until the death of the testator. It has no
effect during the lifetime of the testator.

A testator can change his Will, at any time, in any manner he deems fit. Every person of
sound mind, and not a minor, can make a Will. If a person is of unsound mind at the time
of making a Will, the Will is not enforceable.

A Will, obtained by force, coercion or undue influence, is a void Will as it takes away the
free agency of the person. A Will, made under influence of intoxication or in such a state
of body or mind, sufficient to take away free agency of the testator, is void.
A Will can be made at any time in the life of a person. There is no restriction on how
many times a Will can be made by a testator. However, only the last Will made before his
death is enforceable. A Will has to be executed by the testator, by signing or affixing his
thumb impression on it. It should be attested by two or more witnesses, each of whom
should have seen the testator signing the Will.

Though the registration of a Will is not compulsory, it can be registered with the sub-
registrar. If, at any time, the testator wishes to withdraw the Will, he can do so. A Will
also can be sealed and kept in safe custody.

On the death of the testator, an executor of the Will or a heir of the deceased testator
can apply for probate. The court will ask the other heirs of the deceased if they have any
objections to the Will.

If there are no objections, the court will grant probate. A probate is a copy of a Will,
certified by the court, and is conclusive evidence that the Will is genuine.

If a testator intends to make a few changes to the Will, without changing the entire Will,
he can do so by making a codicil to the Will. The codicil can be executed in a similar way
as the Will. Addition or supplement that explains, modifies or revokes a will or part of one

One must note that a Will or codicil is not unalterable or irrevocable. They can be altered
or revoked at any time. In case any objections are raised by any of the heirs, a citation
has to be served, calling upon them to consent. This has to be displayed prominently in
the court.

Page | 92
L&B/Chapter-1/Law-Real Estate

If no objection is received, the probate will be granted. It is only after this that the Will
comes into effect.

One can make some provision for a faithful servant, a nurse, a friend in need of money
and so on.
Further one can fulfill his spiritual desires like creating a trust, donating to good causes
like orphanages, temples, old age homes, hospitals, educational institutions, social
service organisations etc.
Any one of sound mind and not being a minor may dispose of his/her property by Will.
Registration of a Will is purely optional and it is not compulsory.
It cannot be ordinarily be tampered with, destroyed, mutilated, lost or stolen.
It is kept in the safe custody of the office of the Registry. If an unregistered Will is lost,
the testator's wish cannot be given effect as it will be difficult to trace the Will. Probate is
a document issued under the seal and signature of a Court officer, certifying that a
particular Will was proved, with a copy of the will annexed.
The Supreme Court has recently held that petition for probate or letters of
administration of the Will of a testator must be filed within three years from the date of
death of the testator. No probate is necessary for Christian and Muslim Wills.
Under Muslim law, male and female can make Will. Will by Pardanasin woman is also
valid but stronger evidence is needed to prove the genuineness of the same.
The executor is the most important person in the Will. An executor has a duty to collect
and realise the estate of the deceased, pay his debts and distribute the legacies as
mentioned in the Will by the testator. The duty of the executor is to probate the Will in a
manner known to law. The court shall grant probate only to an executor who has been
named in the Will.

All Wills can be revoked, either impliedly or expressly, either by conduct or by a specific
document. By conduct, the Will can be presented to be revoked by the testator.
For example, a testator may make bequest of property in his Will to a person, but he may
dispose of the said property even during his life time.
This is called implied revocation. Suppose, the testator makes a bequest of a vacant land
in his Will, but subsequently the testator himself constructs a dwelling house therein, in
such circumstance, the Will can be deemed to have been expressly revoked by the
testator.
It has been experienced that when there is a Will, painful litigation in the family of the
testator is prevented. Only in a very few cases, litigation crops up questioning the
genuineness of the Will.

Page | 93
IOV-Registered Valuers Foundation

SUCCESSION CERTIFICATE

Definition

A succession certificate is given to the successor of a deceased person who dies without
making a will in order to establish the authenticity of the successor and also to give an
authority over the deceased person’s debts and securities.

Who issues a succession certificate?

A succession certificate is issued by the district judge of the relevant jurisdiction. The
relevant jurisdiction would be where the deceased person ordinarily resided at the time
of his death or if no such place is available, the jurisdiction within which any property
belonging to the deceased may be found.

What are the particulars to be included in the petition for succession certificate?

The petition for the succession certificate made to the district judge must be signed and
verified by the applicant/successor and include the following details:

1. The date, time and place of the death of the deceased and death certificate will
be attached, if available.
2. The details of ordinary residence of the deceased at the time of his death; and if
such details are not available, then the details of the property that is within the
jurisdiction of the district judge to whom such an application is made.

3. The family or other near relatives of the person deceased and their respective
residences.
4. The rights of the petitioner.

5. The absence of any reason to invalidate the grant of the certificate

6. The debts and securities in respect of which the application for such a certificate
is made.

7. Prayer Clause.

Procedure to obtain a succession certificate

The legal procedure to obtain the succession certificate is mentioned as below:

1. Step 1: First draft the petition, verify and sign the same and submit it to the
district judge in the appropriate jurisdiction after paying the appropriate court
fees.

2. Step 2: The district judge will read and hear upon the application and if the same
is admitted, he shall fix a day for the hearing in respect of the same and also
send notice of the hearing to the defendants/other parties.

Page | 94
L&B/Chapter-1/Law-Real Estate

3. Step 3: After hearing all the parties concerned, the judge will decide if the
applicant is within his right to apply for the application and shall grant the
certificate to him if satisfied.

4. Step 4: The district judge may also require the applicant to provide a bond with
one or more sureties or any other security so as to make good any possible loss
arising out of the use or misuse of such certificate.

Validity of the Certificate

The succession certificate stands valid anywhere within India. However, where a
certificate has been granted to a person who is a resident of a foreign country, by an
Indian representative (as appointed by the government), of such foreign country, the
certificate will stand valid only if properly stamped.

What is the effect of the Certificate?

The main purpose of this certificate is to provide protection to all parties paying debts
where such payments are made in good faith. The certificate holder is also empowered
to receive any interest/dividend on the securities and negotiate or transfer such
securities as mentioned in the certificate. Thus all payments made to and by the
certificate holder on behalf of the deceased person will be legally valid. However, this
does not necessarily mean that the certificate holder is the owner of the securities or the
legal heir. The legal heir/heirs are determined by a separate procedure of law.

What are the differences between Succession Certificate and the Legal Heir
Certificate?

The following are the main differences between Succession Certificate and Legal Heir
Certificate:
Succession Certificate Legal Heir Certificate
Applicability To gain authority to obtain the debts To stake a claim as a rightful heir
and securities of the deceased where a to the estate of the deceased
will has not been drawn up
Contents Relationship of the applicant to the Lists all the legal heirs of the
deceased and the list of debts and deceased
securities sought.
Function Establishes authority of certificate Identifies and establishes living
holder to inherit debts and securities of heirs of a deceased person
the deceased and provides protection
to parties paying debts
Effect The holder may not be the ultimate The holder is entitled to inherit
beneficiary of the estate of the the estate
deceased

Page | 95
IOV-Registered Valuers Foundation

(This page is blank)


Page | 96
VALUATION OF REAL

ESTATE I 02
1. Cost, Price and Value
2. Types of Value
3. Basic elements of value
4. Factors affecting value
5. Highest and best use
6. Real Property
7. Annuities, Capitalization, Rate of Capitalization
8. Construction and use of tables
9. Urban Infrastructure and its influence
10. Real Estate market and its characteristics
11. Concepts of Green building

Page | 1
IOV-Registered Valuers Foundation

(This page is blank)


Page | 2
VALUATION OF REAL ESTATE I 02
1. COST, PRICE AND VALUE
Cost, Price and Value are three basic terms which are frequently used in valuation. These
three terms are distinctly different from each other and never substitute one another. We
must thoroughly understand the meaning of these words and should not use these words
in wrong context.
COST: Cost is the amount required to acquire or create the asset. When that asset has been
acquired or created, its cost is a fact. Price is related to cost because the price paid for an
asset becomes its cost to the buyer.
PRICE: Price is the amount asked, offered or paid for an asset. Because of the financial
capabilities, motivations or special interests of a given buyer or seller, the price paid may be
different from the value which might be ascribed to the asset by others.

VALUE: Value is an economic concept referring to the price most likely to be concluded by
the buyers and sellers of a good or service that is available for purchase. Value is not a fact
but an estimate of the likely price to be paid for goods and services at a given time in
accordance with the particular definition of value. The economic concept of value reflects a
market’s view of the benefits that accrue to one who owns the goods or receives the
services as of the effective date of valuation.

2. TYPES OF VALUE
Market Value: Market Value is the estimated amount for which an asset or liability should
exchange on the valuation date between a willing buyer and a willing seller in an arm’s
length transaction, after proper marketing and where the parties had each acted
knowledgeably, prudently and without compulsion. The definition of Market Value must be
applied in accordance with the following conceptual framework:

1. “The estimated amount” refers to a price expressed in terms of money payable for
the asset in an arm’s length market transaction. Market Value is the most probable
price reasonably obtainable in the market on the valuation date in keeping with
the market value definition. It is the best price reasonably obtainable by the seller
and the most advantageous price reasonably obtainable by the buyer. This
estimate specifically excludes an estimated price inflated or deflated by special

Page | 3
IOV-Registered Valuers Foundation

terms or circumstances such as typical financing, sale and leaseback arrangements,


special considerations or concessions granted by anyone associated with the sale,
or any element of value available only to a specific owner or purchaser.
2. “An asset or liability should exchange” refers to the fact that the value of an asset
or liability is an estimated amount rather than a predetermined amount or actual
sale price. It is the price in a transaction that meets all the elements of the Market
Value definition at the valuation date.
3. “On the valuation date” requires that the value is time-specific as of a given date.
Because markets and market conditions may change, the estimated value may be
incorrect or inappropriate at another time. The valuation amount will reflect the
market state and circumstances as at the valuation date, not those at any other
date.

4. “Between a willing buyer” refers to one who is motivated, but not compelled to
buy. This buyer is neither over eager nor determined to buy at any price. This buyer
is also one who purchases in accordance with the realities of the current market
and with current market expectations, rather than in relation to an imaginary or
hypothetical market that cannot be demonstrated or anticipated to exist. The
assumed buyer would not pay a higher price than the market requires. The present
owner is included among those who constitute “the market”.
5. “And a willing seller” is neither an over eager nor a forced seller prepared to sell at
any price, nor one prepared to hold out for a price not considered reasonable in
the current market. The willing seller is motivated to sell the asset at market terms
for the best price attainable in the open market after proper marketing, whatever
that price may be. The factual circumstances of the actual owner are not a part of
this consideration because the willing seller is a hypothetical owner.
6. “In an arm’s length transaction” is one between parties who do not have a
particular or special relationship, e.g., parent and subsidiary companies or landlord
and tenant, that may make the price level uncharacteristic of the market or inflated.
The Market Value transaction is presumed to be between unrelated parties, each
acting independently.
7. “After proper marketing” means that the asset has been exposed to the market in
the most appropriate manner to effect its disposal at the best price reasonably
obtainable in accordance with the Market Value definition. The method of sale is
deemed to be that most appropriate to obtain the best price in the market to
which the seller has access. The length of exposure time is not a fixed period but
will vary according to the type of asset and market conditions. The only criterion is
that there must have been sufficient time to allow the asset to be brought to the
attention of an adequate number of market participants. The exposure period
occurs prior to the valuation date.

8. “Where the parties had each acted knowledgeably, prudently” presumes that both
the willing buyer and the willing seller are reasonably informed about the nature
and characteristics of the asset, its actual and potential uses, and the state of the
market as of the valuation date. Each is further presumed to use that knowledge

Page | 4
L&B/Chapter-2/Valuation of Real Estate

prudently to seek the price that is most favorable for their respective positions in
the transaction.
9. Prudence is assessed by referring to the state of the market at the valuation date,
not with the benefit of hindsight at some later date. For example, it is not
necessarily imprudent for a seller to sell assets in a market with falling prices at a
price that is lower than previous market levels. In such cases, as is true for other
exchanges in markets with changing prices, the prudent buyer or seller will act in
accordance with the best market information available at the time.

10. “And without compulsion” establishes that each party is motivated to undertake
the transaction, but neither is forced or unduly coerced to complete it.
11. Investment Value: Investment Value is the value of an asset to a particular owner
or prospective owner for individual investment or operational objectives.
Investment Value is an entity-specific basis of value. Although the value of an asset
to the owner may be the same as the amount that could be realised from its sale
to another party, this basis of value reflects the benefits received by an entity from
holding the asset and, therefore, does not involve a presumed exchange.
Investment Value reflects the circumstances and financial objectives of the entity
for which the valuation is being produced. It is often used for measuring
investment performance.
12. Fair Value: The amount for which an asset could be exchanged between
knowledgeable, willing parties in an arm’s length transaction. Fair Value may also
be defined as the price that would be received to sell an asset or paid to transfer a
liability in an orderly transaction between market participants at the measurement
date. For purposes other than use in financial statements, fair value can be
distinguished from market value. Fair value requires the assessment of the price
that is fair between two identified parties taking into account the respective
advantages or disadvantages that each will gain from the transaction. In contrast,
market value requires any advantages that would not be available to market
participants generally to be disregarded.

13. Fair value is a broader concept than market value. Although in many cases the
price that is fair between two parties will equate to that obtainable in the market,
there will be cases where the assessment of fair value will involve taking into
account matters that have to be disregarded in the assessment of market value,
such as any element of special value arising because of the combination of the
interests. Examples of the use of fair value include: (a) estimating the price that
would be fair between a lessor and a lessee for either the permanent transfer of
the leased asset or the cancelation of the lease liability.

14. Special Value: An amount above the Market Value that reflects particular
attributes of an asset that are only of value to a special purchaser is called Special
Value. A special purchaser is a particular buyer, or a restricted class of buyers, for
whom a particular asset has special value because of advantages arising from its
ownership that would not be available to general purchasers in the market. Special
value can arise where an asset has attributes that make it more attractive to a

Page | 5
IOV-Registered Valuers Foundation

particular buyer than to the general body of buyers in a market. These attributes
can include the physical, geographic, economic or legal characteristics of an asset.
Market value requires the disregard of any element of special value because at any
given date it is only assumed that there is a willing buyer, not a particular willing
buyer. When special value is reported, it should be clearly distinguished from
market value.

15. The attributes of an asset that could be of value to a special purchaser include any
element of synergistic value that would be generated by its acquisition.

16. Synergistic Value: It is an amount that reflects particular attributes of an asset that
are only of value to a special purchaser. It is the result of a combination of two or
more assets or interests where the combined value is more than the sum of the
separate values. If the synergies are only available to one specific buyer then
Synergistic Value will differ from Market Value. The added value above the
aggregate of the respective interests is often referred to as “marriage value.”
17. Liquidation Value: Liquidation Value is the amount that would be realised when
an asset or group of assets are sold on a piecemeal basis. Liquidation Value should
take into account the costs of getting the assets into saleable condition as well as
those of the disposal activity. Liquidation Value can be determined under two
different premises of value:

• an orderly transaction with a typical marketing


• period, or
• a forced transaction with a shortened marketing period

A Valuer must disclose which premise of value is assumed.

Realisable Value: Market value minus all costs related to (1) holding costs during the
expected marketing period, (2) all selling costs related to disposition of the property and
(3) the cost of funds or rent loss during the anticipated marketing period. Holding costs
include, but are not limited to, real estate taxes, property insurance, liability insurance,
utilities, and normal repairs and maintenance. Selling costs include, but are not limited to,
brokerage commissions, closing costs, title work, and surveys. This is also called Net
Realisable Value (NRV).

Forced Sale Value: The term “forced sale” is often used in circumstances where a seller is
under compulsion to sell and that, as a consequence, a proper marketing period is not
possible, and buyers may not be able to undertake adequate due diligence. The price that
could be obtained in these circumstances will depend upon the nature of the pressure on
the seller and the reasons why proper marketing cannot be undertaken. It may also reflect
the consequences for the seller of failing to sell within the period available. Unless the
nature of, and the reason for, the constraints on the seller are known, the price obtainable
in a forced sale cannot be realistically estimated. The price that a seller will accept in a
forced sale will reflect its particular circumstances, rather than those of the hypothetical
willing seller in the Market Value definition. A “forced sale” is a description of the situation

Page | 6
L&B/Chapter-2/Valuation of Real Estate

under which the exchange takes place, not a distinct basis of value.

If an indication of the price obtainable under forced sale circumstances is required, it will be
necessary to clearly identify the reasons for the constraint on the seller, including the
consequences of failing to sell in the specified period by setting out appropriate
assumptions. If these circumstances do not exist at the valuation date, these must be clearly
identified as special assumptions.
A forced sale typically reflects the most probable price that a specified property is likely to
bring under all of the following conditions:

1. consummation of a sale within a short time period,

2. the asset is subjected to market conditions prevailing as of the date of valuation or


assumed timescale within which the transaction is to be completed,

3. both the buyer and the seller are acting prudently and knowledgeably,

4. the seller is under compulsion to sell,

5. the buyer is typically motivated,


6. both parties are acting in what they consider their best interests,

7. a normal marketing effort is not possible due to the brief exposure time, and

8. payment will be made in cash.

9. The value estimated by fulfilling all the above conditions is termed as Forced Sale
Value.

Salvage Value: The value of an asset that has reached the end of its economic life for the
purpose it was made is known as salvage value. The asset may still have value for an
alternative use or for recycling. It applies to a dismantled building but is not applicable to
land.

Scrap Value: This is the amount that is likely to be obtained for a portion or a component
of a material item when sold at the end period of its useful life. It is a kind of salvage value.
For example, if a building is demolished then certain old material that may be sold will
bring in money in the form of scrap value.

Residual Value: The anticipated value of an asset at the expiration of its useful life.

Reinstatement Value: Reinstatement Value means the cost of replacing or reinstating on


the same site, property of the same kind or type but not superior to or more extensive than
the insured property when new.

Page | 7
IOV-Registered Valuers Foundation

6. BASIC ELEMENTS OF VALUE -


MARKETABILITY, UTILITY, SCARCITY, AND
TRANSFERABILITY
• MARKETABILITY AND VALUE
The marketability of a property is determined by how ready it is to sell. Property’s capacity
to deliver services to meet human needs, house economic activities, and supply satisfaction
and amenities determine eagerness of the buyers to buy it. Only value of marketable real
estate depends upon marketability. Non-marketable real estate value does not depend
upon marketability. In case of marketable real estates, the more and more it becomes
marketable, the number of buyers will increase and the value will increase. Therefore, the
value of a property is directly proportional to its marketability.

The following defects make a property less marketable and thus reduce the value
considerably:

1. Outstanding mortgages/liens

2. Restrictive covenants
3. Outstanding future interests of others in the property, i.e. a "reverter".

4. Encumbrances
5. Easements on the property

6. Variations in the names of grantors and grantees


7. Variations in the chain of title

8. Adverse possession claims

9. Structural encroachments

10. Existing violations of an equitable servitude or covenant

11. Zoning restriction violations

• UTILITY AND VALUE UTILITY means the state of being useful, profitable,
or beneficial. Therefore, with the increase in utility the value of real properties increases.

• SCARCITY AND VALUE If demand of a commodity is high and price of the


commodity is high being short in supply then that particular commodity flows from other
parts of the market to that part where it is scarce and thus brings down the price in a
traditional economic market. Such a movement of price is possible because goods and

Page | 8
L&B/Chapter-2/Valuation of Real Estate

commodities can physically flow from one part of the market to other. But things are quite
different with the landed property. You cannot produce land and so the question of
increasing the supply of land through industrial production does not arise. Total area of
land is limited by nature and you have to get this thing accepted in any study of the real
estate market. So is the supply of land limited in a particular locality. When the demand for
land rises, the supply being fixed, the value of land also rises sharply. Therefore, with
increase in demand and fixed supply of land as well as other real properties (because of
stringent development control rules) a scarcity of land as well as built-up spaces is created
in the market. This results in increase in value of the properties in that specific area.
Therefore, value of properties is directly proportional to the scarcity of that property in the
real estate market.

• TRANSFERABILITY AND VALUE

In an economic market goods and commodities are traded freely and flown from one part
of the market to another in response to a demand and supply condition. But things are
quite different with the landed property. Landed property cannot be transferred physically
from one part of the market to another like other goods and commodities. As a result, even
when overall supply of developed land or real estate is increased through investment,
supply may remain low at specific pockets due to total non availability of developable land
in such areas. This characteristic of non-transferability of real estate causes a rise in value in
some locations while value may go down in other locations due to over-supply.

If we discuss ‘transferability’ of real estate as an inherent legal character of it, we may


declare that value of a real estate is directly proportional to its ease of transferability. The
more easily it could be transferred, the better value the owner of the estate could expect.

7. FACTORS AFFECTING VALUE - PHYSICAL,


ECONOMIC, LEGAL AND SOCIAL
• PHYSICAL FACTORS AFFECTING VALUE OF REAL ESTATE:

Location: It attributes most in determining the value of a real estate. The more and more
the real estate is located nearer to centres from where the residents can fulfill their every
day needs, the more and more would be the value.

Infrastructure: The ease of receiving water, electricity, roads, sewerage system, drainage
system, solid waste management etc. to the residents of a real estate project increases its
value.

Page | 9
IOV-Registered Valuers Foundation

Quality of Construction: The better the quality of construction the higher shall be the
value of that real estate.

Planning: The better the planning the better shall be the value of that real estate. For
example, if a real estate housing project provides parking facilities for the visitors shall fetch
more value than one not having this facility.

Amenities and facilities: The more the amenities and facilities (e.g. swimming pool,
jogging track, club, gymnasium, indoor sports centre etc.) the better shall be the value of
that real estate.

• ECONOMIC FACTORS AFFECTING REAL ESTATE


Inflation: Real estate is said to be a hedge against inflation. The characteristic of real estate
prices to overcome inflation makes real estate a suitable investment option in any
economic environment. During inflationary times investment in long-term government
securities or other fixed interest securities is not secured in real terms. The real value of
money associated with such investments falls with time causing a loss of capital. But value
of real estate continues to grow with time at a faster rate than the rate of inflation. As a
result, the real value of money in real estate investment is maintained in real terms.
Therefore, during inflationary times there is an increase in demand for investment in real
property from a section of investors to maintain the real value of money.

Population changes: As more and more people migrate in an area, there is an increase in
demand for land. But since total area of land is fixed by nature, the per capita allocation of
space for any use reduces. Supply of land being low and demand remaining high the value
of land sharply rises

Economic Growth: The economic growth is indicated by the growth rate of GDP, industrial
production, employment data etc. Studies indicate that when these indicators are vigilant
the economy of the country is of very sound health and real estate demand is high.
Conversely, when these indicators give low values the economy slows down in growth and
demand for landed property shrinks. Economic growth leads to the increase in real value of
the income of people. Owing to the rising income people are able to spend a larger
percentage of their earnings on home loans. Thus there is a rise in demand for residential
real estate. The converse is true of a time of slowing down of economic growth. Thus when
economy slows down and there is rise of unemployment rate or fear of unemployment,
lesser number of people bids for houses and demand shrinks.

Page | 10
L&B/Chapter-2/Valuation of Real Estate

• LEGAL FACTORS AFFECTING REAL ESTATE

Clear Title: Marketable title (real estate) is a title that a court of equity considers to be so
free from defect that it will legally force its acceptance by a buyer. If one cannot produce a
clear title of deed to the property, then the prospective buyer should expect to lose in a
specific performance action. Absolute clear title of a real property fetches maximum value
for it.

Rights and interests: Freehold right fetches the maximum value of a property followed by
leasehold right, assignor’s right, right of a life interest. A tenant or a licensee doesn't have
any valuable right of a real property. Freehold rights are also sometimes curbed by many
legal factors.

Covenants: Covenants agreed between two parties in a contract has a substantial effect on
valuation. Negative covenant viz. ‘not to sublease’ substantially reduces the value of
leasehold right of a property.

Statutes: Existing statues e.g. building bye laws, acquisition act, rent control laws, taxation
acts, transfer of property act etc. have a great effect on valuation of real estates. For a small
change in Floor Area Ratio (FAR) / Floor Space Index (FSI) makes a considerable change in
the value of real estate in urban areas.

• SOCIAL FACTORS AFFECTING REAL ESTATE


Customs: Social customs sometimes override the Acts which also has substantial effect on
valuation.

Rearrangement of family structure: Joint family structure in India is now shifting towards
nuclear family leading to increase in demand for individual flats.

Urge to reside close proximity to celebrities: Urge to reside close to celebrity’s house
increases demand vis-à-vis value in local basis.

8. HIGHEST AND BEST USE, VALUE IN USE,


VALUE IN EXCHANGE
Highest and Best Use Highest and best use is the use, from a participant perspective, that
would produce the highest value for an asset. Although the concept is most frequently
applied to non-financial assets as many financial assets do not have alternative uses, there
may be circumstances where the highest and best use of financial assets needs to be
considered. The highest and best use must be physically possible (where applicable),
financially feasible, legally allowed and result in the highest value. If different from the
current use, the costs to convert an asset to its highest and best use would impact the

Page | 11
IOV-Registered Valuers Foundation

value. The highest and best use for an asset may be its current or existing use when it is
being used optimally. However, highest and best use may differ from current use or even
be an orderly liquidation. The highest and best use of an asset valued on a stand-alone
basis may be different from its highest and best use as part of a group of assets, when its
contribution to the overall value of the group must be considered. The determination of
the highest and best use involves consideration of the following:

1. To establish whether a use is physically possible, regard will be had to what would
be considered reasonable by participants.

2. To reflect the requirement to be legally permissible, any legal restrictions on the use
of the asset, e.g. town planning/zoning designations, need to be taken into account
as well as the likelihood that these restrictions will change.

3. The requirement that the use be financially feasible takes into account whether an
alternative use that is physically possible and legally permissible will generate
sufficient return to a typical participant, after taking into account the costs of
conversion to that use, over and above the return on the existing use.

4. Market value should be the result of the highest and best use of the property that
maximizes its potential. Such highest and best use should have regard to what the
market participants have considered best before making a bid and what is
physically possible, legally permissible and financially feasible.
5. As a matter of fact, on many occasions the market value shall be the existing or
current use value of the asset. But if an alternative use is possible, and that is legally
permissible then cost of putting the property in that use by conversion or
redevelopment should have an impact on the value of the asset and if the most
benefits generated as a result of conversion or redevelopment exceeds the previous
benefits arising before conversion or redevelopment and is considered sufficient
then that alternative use might lead to the highest and best use.

6. So, in order to estimate the market value of the property, in its highest and best
use, the Valuer has to consider all the steps described in the preceding two
paragraphs. He is to be conscious of the fact that an alternative use of a property
may lead to the highest and best use provided they meet the criteria set down in
the preceding paragraphs.

Value in use This term refers to the value of an individual asset which is a component of the
going concern. It is the estimated amount that reflects the specific contribution of the
particular asset to the going concern entirety. Value in use can include elements of
personality (chattels, plant, machinery, equipment, etc.) and also encompasses concepts of
deprival of use, etc. Value in use is responsive to market prices through comparative or
investment/ capitalization approaches based on market and/ or business, industry/ trade,
etc.

Value in exchange It is the amount of goods and services which we may obtain in the
market in exchange of a particular thing. In other words, it is the price of a particular good

Page | 12
L&B/Chapter-2/Valuation of Real Estate

which can be sold and bought in the market. For instance, if one kg of rice can be obtained
in exchange of one dozen of banana, then we may say that value of one kg of rice is equal
to one dozen of banana. Thus, value-in-exchange depends on two things:

1. Time: Value-in-exchange depends on time element. That is, with the change in time
value- in-exchange for a commodity in respect to other commodity will vary.

2. Place: Value-in-exchange also depends from place to place. Value-in-exchange for


a particular commodity varies from one market to other markets. Hence, it varies
according to time and place. Again, one commodity may have immense use value
but no exchange value or vice versa. For example, water has immense use value but
not exchange value. On the contrary, diamond has huge exchange value but no use
value.

9. REAL PROPERTY: RIGHTS AND INTERESTS IN


REAL ESTATE, TYPES OF OWNERSHIPS AND
TYPES OF OCCUPANCY IN REAL ESTATE
Real estate and Real Property Real estate and real property are much discussed in all
knowledgeable circles in modern times. The International Valuation Standards Council
(IVSC) has defined real estate as a physical asset which includes land and all things that are
a natural part of the land e.g. trees and minerals as well as things that are attached to the
land by people, e.g. buildings and site improvements. Whereas, real property has been
defined by the IVSC as all the rights, interests and benefits related to the ownership of real
estate. Real property is a legal concept distinct from real estate which is a physical asset.
This distinction between real estate and real property has not been rigidly followed in the
use of the terms in the property and valuation circles in this country. However, real estate is
an economic commodity and is produced and supplied for purchase and sale in an
economic market but unlike other economic goods and commodities it has some peculiar
characteristics that distinguish it from other goods and commodities. These peculiar
characteristics often dominate the price and hence value of real estate. We will take up
their short introduction in the following paragraphs.

The peculiar characteristics of real estates are:—

1. Fixity in supply: Total quantity of land is fixed in supply by nature. Land is not
produced in factory by human effort. So, if the population in an area increases it
has the effect of putting an upward pressure on price because only those that can
pay more by competition will be able to procure it.

Page | 13
IOV-Registered Valuers Foundation

2. Non-transferability: Land/real-estate cannot be transferred physically from one


place to another unlike other goods and commodities. Real estate is a commodity
that is produced and consumed in situ. So, if demand for real estate in an area is
increasing (may be due to a population growth or increased income of people)
land/real-estate being non-transferable supply cannot be augmented to meet the
demand and so the price rises. Of course, in the short run supply can be somewhat
increased by more intensive development of existing properties. E.g. old dilapidated
properties are demolished, and site re-developed with greater quantity of floor
area. But ultimately a point is reached when demand overtakes the supply. This
again has the effect of pushing up the price of real estate.

3. Durability: Real estate is a highly durable commodity and land is virtually


indestructible. So, life of real estate tends to be perpetual. Again, real estate is
produced and consumed in situ. Once therefore, real estate has been produced it
cannot be withdrawn from the market except by demolition. So, there is a risk of
capital loss if prediction of the market trends could not be properly made and
market downturns after the constructional works are complete.

4. Heterogeneity: Goods and commodities as produced in factories are often


homogeneous in all aspects. But, real estate is developed in situ, each site as well as
the buildings having their special features. Even when in the same building block
some difference must exist between two apparently identical apartments. As a
matter of fact no two properties are identical in all respects in the whole world. This
characteristic of real estate is called heterogeneity. Heterogeneity makes
comparison of value between two properties difficult and requires experience and
expertise of the Valuer.

Bundle of rights Real estate ownership is associated with a number of rights often referred
to as the “bundle of rights” or a “bundle of sticks”, each stick representing a property right.
The real estate market is also referred to as the land market or the property market. The
paradox of the land market is that it does not deal in land. Because, what a transferred in
real estate transactions are nothing but property rights. Real estate is physically non-
transferable unlike other commodities. Only the rights associated with real estate are
transferred. When real estate (that is, the rights associated with real estate) is transferred, a
conveyance deed on appropriate stamp paper is drawn. A deed of conveyance describes
fully the rights, privileges and encumbrances of the real estate unit transferred. It is the
transferred rights, privileges and encumbrances that dictate the market value of the real
estate unit as much as the physical aspects of the land and buildings do. A.M. Honor
identified 11 sticks in the bundle of property rights. These are: the right to possess, the
right to use, the right to manage, the right to the income of the thing, the right to the

Page | 14
L&B/Chapter-2/Valuation of Real Estate

capital, the right to security, the rights of incidence of transmissibility, right to absence of
term, the prohibition of harmful use, liability to execution and the incident of residuality.

The right to possess This right empowers the owner to exclusive control of the property
physically or excluding others from the benefit or use of the property. This right has been
described by Honor as the “foundation on which the whole superstructure of ownership
rests”. Indeed, from a very early stage of evolution of ownership of property rights,
possession has been regarded as the nine points of law.

The right to use the right to use acknowledges the liberty of the owner to use and
enjoyment of the property personally.

The right to manage This right acknowledges power of the owner to decide how and by
whom the property should be used.

The right to the income This right empowers the owner to receive the benefits derived from
the property by allowing it to be used by others on parting with possession and personal
use.

The right to capital This right entitles the owner to alienate the property, that is, he can sell
and give away otherwise to another person. This right also entitles him to appropriate,
waste or to modification or destruction of the property.

The right to security This right empowers the owner to retain his rights against
expropriation, that is, acquiring by state or other authorities.

The power of transmissibility This power entitles the owner to bequeath the property that
is, allow to pass on the property to somebody upon death.

The absence of term This right recognizes the perpetual continuity of the ownership right
making it of indeterminable length.

The prohibition of harmful use This property stick imposes a duty on any person using the
property not to use it in a manner so that it causes harm to others.

Liability to execution This stick inflicts a liability on the owner for allowing the property to
be taken away from him for the repayment of a debt.

The incident of residuality. However, the owner may grant some rights to other persons
and yet may remain an owner of the property because he still owns many other rights left.
For example, if the freehold owner of a property grants an easement right to pass through
his land to the owner of the neighboring property, the owner of the easement right is his
neighbor having an enforceable right on the servient property. Yet the freehold owner
remains an owner of the property by virtue of owning the remaining rights. Similarly, when

Page | 15
IOV-Registered Valuers Foundation

a freehold owner mortgages his property to a bank and then allows his near relative to
occupy the property he still remains a owner as to the property subject to the terms of the
mortgage.

• THE PRINCIPAL TYPES OF INTEREST IN REAL ESTATE


The principal types of interest in real estate / property are as follows:

Freehold interest A freehold interest is of perpetual duration and is the highest form of
ownership prevailing in this country and many other countries like UK and the USA, etc. The
bundle of property rights comprising of all the 11 property incidents/sticks are available in
the highest form of freehold ownership. However, the freehold owner may grant rights over
his property to any other person, such as granting an easement right to walk through his
land to access the property by a neighbour. Or, he may grant a leasehold right to a lessee
forgoing his right to occupy the property and granting many other advantageous rights
according to the lease agreement and reserving some others for his own use as negative
covenant. In each case the freeholder will still remain the owner because many other rights
will still be left for his exercise. But, the value of the interest in the property will reduce
depending upon the nature of the encumbrance.

Leasehold interest A leasehold interest usually lasts for a term of years which may be a
short term, a long term, or a very long term. The rights enjoyed by a lessee depend upon
the lease agreement between the lessor and the lessee. The grantor of the lease is called
the lessor who may also be the freeholder or may be called the landlord. Whereas, the
lessee may also be called the leaseholder or the tenant. The freeholder may curtail any of
his property interests described above as bundle of rights. The agreement between the
lessor and the lessee determines whether the lessee has been granted a marketable title.
The rights and liabilities of the lessor and lessee are determined by a written agreement. In
the absence of a written agreement, local usages determine the rights and liabilities. When
both a written agreement and local usages are absent The Transfer of Property Act (1882),
section 108 determines the rights and liabilities of the lessor and lessee. Theserights and
liabilities under a lease are binding on both lessor and lessee. Some of the standard forms
of rights that can be of importance to the lessee are as follows. The right to sub-let; the
right to assign or transfer of the lease; the right to mortgage the leasehold interest; the
right to develop the property and construct buildings, etc. The freeholder’s or the lessor’s
right to receive back the property from the lessee after the lease is over is called reversion.

The life interest the freeholder of a property may grant a life interest in the property to any
person which last so long as the person lives. Like the leasehold interest therefore the life
interest is also another terminable interest in a property. However, whereas the leasehold
interest is for a definite term to be mentioned in the lease agreement, a life interest is for

Page | 16
L&B/Chapter-2/Valuation of Real Estate

an uncertain duration and hence of unsound value. Obviously, there is no power of the
recipient of a life interest to bequeath any property on his death. Whereas, the owner of a
freehold interest may grant an interest in property even on his death, because a freehold
interest is of perpetual duration. Even, a lessee may bequeath an interest to a person on his
death where the leasehold interest is of very long term duration and extends far beyond
the life of the lessee when the lease agreement does not provide anything to the contrary.
The legal successors of the lessee can step in his shoes and become the owner of the
remaining part of the leasehold interest on the death of the lessee. But this opportunity is
not available to the holder of life interest as he does not hold any right in the property on
his death and all his rights ceases there. If the freeholder, at the time of granting the life
interest grants a right to the life interest holder to transfer his property to another person
then the life interest holder can transfer his right in the life interest to an outsider for the
term of his life only. His interest ceases on his death and then the transferee loses his title
in the property. Life interests are valued according to the tables based on mortality of
people at various ages.

Co-ownership right The following are the rights and liabilities of the co-owners as decided
by Punjab and Haryana High Court in Sant Ram Nagina Ram case:

1. A co-Owner has an interest in the whole property and also in every parcel of it.
2. Possession of the joint property by one co-owner is in the eye of law, possession of
all even if all but one are actually out of possession.

3. A mere occupation of a larger portion or even of an entire joint property does not
necessarily amount to ouster as the possession of one is deemed to be on behalf of
all.

4. The above rule admits of an exception when there is ouster of a co-owner by


another. But in order to negative the presumption of joint possession on behalf of
all, on the ground of ouster, the possession, of a co-owner must not only be
exclusive but also hostile to the knowledge of the other, as, when a co-owner
openly asserts his own title and denies that of the other.
5. Passage of time does not extinguish the right of the co-owner who has been out of
possession of the joint property except in the event of ouster or abandonment.

6. Every co-owner has a right to use the joint property in a husband like manner not
inconsistent with similar rights of other co-owners.
7. Where a co-owner is in possession of separate parcels under an arrangement
consented to by the other co-owners, it is not open to any one to disturb the
arrangement without the consent of others except by filing a suit for partition.
8. The remedy of a co-owner not in possession, or not in possession of a share of the
joint property, is by way of a suit for partition Or for actual joint possession, but not
for ejectment. Same is the case where a co-owner sets up an exclusive title in

Page | 17
IOV-Registered Valuers Foundation

himself.

Where a portion of the joint property is, by common consent of the co-owners, reserved
for a particular common purpose, it cannot be diverted to an inconsistent user by a co-
owner; if he does so, he is liable to be ejected and the particular parcel will be liable to be
restored to its original condition. It is not necessary in such a case to show that special
damage has been suffered.

Easement right An easement is a right which the owner or occupier of certain land
possesses, as such, for the beneficial enjoyment of that land, to do and continue to do
something, or to prevent and continue to prevent something being done, in or upon, or in
respect of, certain other land not his own. By default a property owners attains the right of
way, the right to air or right to light, the right to build, the right to the uninterrupted flow
of water. All these are known as the examples of a property owner's easement rights. The
Indian Easement Act says that if a person has enjoyed these over a period of time, they
have a valid right without any restriction, almost as though it were a privilege.

Transferable Development Rights (TDR) Transferable development right means transfer the
right to develop a land to government, local authority or corporation. So, when an owner of
land transfers his rights of developing a land to a government, local authority, corporation
or government use the same land for infrastructure projects such as road widening, metro
rail projects, park, garden, schools or may be for making new roads or for any other
projects of public utility. DRC (Development rights certificate) will then be issued to owner
of the land, the main purpose of whole process is to acquire the required amount of land in
hassle free manner. Now this DRC will allow the land owner an additional built up area in
return of the area for which he has relinquish his rights and enables him to develop the
given area by himself of transfer his rights for consideration. DRC issued to land owner if
transferable is known as transferable development rights (TDR), which can be transferred to
another entity.

Rights under adverse possession Adverse possession is a legal principle under which a
person who does not have legal title to a piece of property—usually land (real property)—
acquires legal ownership based on continuous possession or occupation of the land
without the permission of its legal owner. The Limitation Act, 1963, is a key piece of
legislation, elaborating on adverse possession. The Act prescribes a period – 12 years for
private properties and 30 years for government-owned ones – within which you have to
stake claim on your property. Any delay may lead to disputes in the future. The principle on
which the Limitation Act is based is that 'limitation extinguishes the remedy, but not the
right'. This means that in case of an adverse possession, the original owner may have the
title over the property but he loses the right to claim such right through a court of law.

Page | 18
L&B/Chapter-2/Valuation of Real Estate

Some essential requirements to be proved for claiming under adverse possession are:

1. Hostile possession: The intention of the possessor of the property must be to


acquire rights through means of adverse possession. These rights are acquired at
the expense of the rights of the original owner. There must be an express or implied
denial of the owner's title by the possessor. Constructing a boundary wall around
the property can be means of asserting this possession.
2. Public knowledge: The public at large must be aware about the possession of the
claimant. This condition is put in place so that the actual owner has adequate
means to know that someone is in possession of his property and gets reasonable
time to act. However, one is not bound to inform the original owner about it.
3. Actual possession: There must be actual possession throughout the period of
limitation. Physical acts like harvesting crops, repairing the building, planting trees,
erection of shed, etc, could be means through which actual possession can be
determined. The possessor could not claim possession over the property without
being physically possessing it.

4. Continuity: The possessor must be in peaceful, unbroken, uninterrupted and


continuous possession of the property. Any break in the possession will extinguish
his rights.

5. Exclusivity: The possessor must be in sole possession of the property. The


possession cannot be shared by different entities or persons for the claimed time
duration.

• TYPES OF OCCUPANCY IN REAL ESTATE


Tenancy does not create any valuable right over the property but provided only occupancy
right. Tenancy in India is governed by different rent control legislations in different states.
The rent control legislation is not a central act but is a state act. Therefore, its structure and
purviews are different for different states. Moreover, a number of states have abolished this
legislation. Therefore, to estimate value of the tenant’s occupancy right, if there be any,
Valuer shall have to be thorough enough about that specific act prevailing in that state. The
detail of tenancy is elaborately discussed in the section ‘Rent Control Laws: Sections
pertaining to Occupancy Rights of Tenants, Freezing of Rent and Protection against
Eviction of Tenant and its effect on value of property” of the subject ‘Law – Real Estate’.

License Where one person grants to another, or to a definite number of other persons, a
right to do, or continue to do, in or upon the immovable property of the grantor,
something which would, in the absence of such right, be unlawful, and such right does not
amount to an easement or an interest in the property, the right is called a license. A license
may be granted by anyone in the circumstances and to the extent in and to which he may
transfer his interests in the property affected by the license. The grantor of license is called

Page | 19
IOV-Registered Valuers Foundation

‘Licensor’ and the acceptor is called ‘Licensee’. Unless a different intention is expressed or
necessarily implied, license to attend a place of public entertainment may be transferred by
the licensee; but, save as aforesaid, a license cannot be transferred by the licensee or
exercised by his servant or agents. When the grantor of the license transfers the property
affected thereby, the transferee is not as such bound by the license Licensee’s rights on
revocation: Where a license is revoked, the licensee is entitled to a reasonable time to leave
the property affected thereby and to remove any goods which he has been allowed to
place on such property. Licensee’s rights on eviction: Where a license has been granted for
a consideration, and the licensee, without any fault of his own, is evicted by the grantor
before he has fully enjoyed, under the license, the right for which he contracted, he is
entitled to recover compensation from the grantors. In short, license does not create any
type of right over the property. Even it does not even create any occupancy right.
Therefore, valuation of licensee’s right is not possible.

10. ANNUITIES, CAPITALISATION, RATE OF


CAPITALISATION, YEARS’ PURCHASE,
SINKING FUND, REDEMPTION OF CAPITAL,
REVERSIONARY VALUE
Annuity It is defined as the Net Annual payment (Return on investment) for the capital
invested in an immovable property or any other form of investment. Rent from land or
house, interest on bank fixed deposit or yield on government security are the examples of
Annuity.

Capitalisation the conversion of a periodic income to an equivalent capital value is known


as capitalisation.

Rate of Capitalisation the return represented by the income produced by an investment,


expressed as a percentage is known as rate of capitalisation. To illustrate, if a person
deposits Rs. l,00,000 in Bank fixed deposit and Bank offers interest of 8% on FD., rate of 8%
is called rate of capitalisation. If an investor yields 8 % return on his investment in a rented
house property, by way of net rent income, 8 % is called rate of capitalisation.

Years’ Purchase Years purchase is defined as capitalised value required to be paid once and
for all, in order to receive annual income of Re.1 for specified period of time at specified
rate of return. If ‘C’ is capital value and ‘N’ is the net income per annum of a property,
Years’ Purchase (Y.P) = C / N.

Annual Sinking Fund It can be defined as an annual recurring fund (amount) required to be

Page | 20
L&B/Chapter-2/Valuation of Real Estate

set aside every year, for a given period of time, at the given rate of interest, to recoup
capital invested in a property, interest or return from which would cease after a given
period of time.

Rate of Redemption of capital This rate of return is normally adopted when income is a
terminable income. To recoup (get back) the capital invested in such type of property (like
leasehold property where income would cease after some years), this rate of interest is
adopted. This rate is also called as Rate of Recoupment of Capital or Accumulative rate of
interest.

Reversionary Value This is generally defined as the present amount of value which is
obtained after a specified period. For calculating the reversionary value of land, the present
rate of land is usually adopted at a certain rate of interest for a deferred period of time. For
example, suppose a building stood on land and it was proposed to demolish the building
after four years. It was estimated that cost of demolition would offset the value of scrap
material to be sold. Land value at present is `10,00,000. Then considering 8% compound
interest, reversionary value of land would be 0.735 x 10,00,000 i.e. `7,35,000.

11. CONSTRUCTION AND USE OF


VALUATION TABLES
Introduction: The valuation tables are used as tools of Valuers in drawing their conclusion
on carefully considered estimate of worth of landed property. Without their help the Valuer
in each case would have to be involved in long laborious calculations of mathematical
processes. However, the invention of computers has made the task of entering into
laborious calculations rather simple. But that does not dispense with the use of valuation
tables altogether. Indeed, for practical purposes the tables are still useful tools to Valuers.
The valuation tables are based on compound interest principles as the derivation of their
formulae will indicate in the calculations that follow under this chapter. The use of the
tables cuts short the risk of mathematical errors and that is why they are extensively used
by Valuers and their use is likely to be continued in future.

Amount of `1 table The purpose of this table is to indicate the amount to which an
investment of `1 shall accumulate at a given rate of compound interest after a given
number of years. Starting simply, let us assume that interest rate on an investment of `1 is i
at the end of 1 year (where i is a fraction of ` 1).

Then at the end of 1 year, the amount to which ` 1 is accumulated is `(1+i)

At the end of two years, the amount accumulated is `(1+i)(1+i) or `(1+i)2

Page | 21
IOV-Registered Valuers Foundation

At the end of three years, the amount accumulated is `(1+i)2 (1+i) or `(1+i)3

Similarly, at the end of n years the amount accumulated is `(1+i)n

Therefore, the amount of `1 at the end of n years at i rate of interest is (1+i)n

Problem Mr. Ramaswami, an investor, has invested an amount of ` 1,000,000 in purchasing


an urban site on taking a loan at 9% compound rate of interest from a bank. What amount
will he have to repay to the bank after three years when he finds the purchaser for his land?

Solution The amount to which ` 1 is accumulated at i rate of compound interest after n


years is ` (1+i)n

Given that, i=0⋅9 and n=3

So, the amount to which his loan is accumulated

= 1,000,000×(1+i)n

= 1,000,000 (1+0⋅09)3

= 1,000,000×1⋅295

= ` 1,295,000

Therefore, the investor has to refund an amount of ` 1,295,000 to the bank out of the
proceeds of the land.

Present value of `1 table This table is intended to indicate the present value of ` 1 to be
received after a given number of years at a given rate of compound interest.

As before, assume that interest rate of an investment of ` 1 is i at the end of 1 year. (i is a


fraction of ` 1). It is known from Amount of ` 1 Table that,

` 1 invested today accumulates to (1+i) at the end of 1 year and,

` 1 invested today accumulates to (1+i)n at the end of n years.

So, if (1+i)n is the amount after n years, its present value today is ` 1.

Therefore, if `1 is the amount after n years, its present value today is `1/(1+i)n
L&B/Chapter-2/Valuation of Real Estate

Therefore, present value of `1 to be received after n years is 1/(1+i)n

Problem Mr. Datta, a developer plans to develop a large area of land and estimates the
residual value of the site to be ` 10,000,000. But the site will be handed over to him only
after 3 years. What is the present value of the site considering interest rate at 12% per
annum?

Solution Present value of ` 1 = 1/(1+i)n

Page | 22
L&B/Chapter-2/Valuation of Real Estate

Where, i is the interest rate on ` 1 and n= number of years.

Here, n=3 years" and i=0⋅12

Therefore, present value = 1/〖(1+0⋅12)3 × ` 10,000,000

= 0.712× ` 10,000,000

= ` 7,120,000

Therefore, present value of the site = ` 7,120,000

Amount of `1 per annum table This table is purported to indicate the accumulated
amounts of a series of installments of ` 1 each invested at the end of each year for a given
number of years at a compound rateof interest.

Let the rate of compound interest on ` 1 be I per annum.

Let n be given number of years for the series.

Then the first dose of ` 1 invested at the end of year 1 shall accumulate to (1+i)(n-1) after n
years.

Similarly, the second dose of ` 1 invested at the end of year 2 shall accumulate to (1+i)(n-2)
after n years, and so on …

So, if S denotes the summation of all the accumulated amounts of installments /doses of ` 1
for n years deposited as above the amount to which this sum Sis accumulated is given by
the expression,

S= (1+i)(n-1)+(1+i)(n-2)+(1+i)(n-3)… …+ (1+i)(n-r) +... …. (1+i)2+(1+i)+1

or, S= 1+(1+i)+(1+i)2+...(1+i)(r-1)+⋯+(1+i)(n-3)+... … (1+i)(n-2)+(1+i)(n-1)

This is a G.P. series the summation of which is given by the formula,

S= a(Rn-1)/(R-1)

where, a= first term; R= common ratio; n= number of terms.

Here, a=1; R=1+i; n= number of terms.

Therefore, S=(1((1+i)n-1))/(1+i-1)=[(1+i)n-1]/i

Therefore, amount of ` 1 per annum = [(1+i)n-1]/i

Problem Mr. Chopra, a builder, took 3 years to construct a six-storey building. He incurred
an expenditure of ` 2,000,000 during the first and second years each. During the third year
he expended an amount of ` 2,500,000. What is the total cost at the end of the third year if
he has borrowed capital at 14% interest rate? Assume all costs to have been incurred at the

Page | 23
IOV-Registered Valuers Foundation

end of the respective year.

Solution For all the three years he has a common expenditure of ` 2,000,000. In addition, he
expended another ` 500,000 during the third year.

So, amount of ` 2,000,000 per annum for three years = `2,000,000 x [(1+i)n–1]/i

where, i=0⋅14 and n=3

Therefore, amount of ` 2,000,000 per annum for 3 years

= ` 2,000,000 x [(1+0⋅14)3–1]/(0⋅14)

= (` 2,000,000×0⋅482)/(0⋅14)

=` 2,000,000×3⋅443

=` 6,886,000

Add another ` 500,000 for the third year

=`6,886,000 + `500,000

=`7,386,000

Therefore, total cost of the builder at the end of the third year = `7,386,000

Annual Sinking Fund Table This table indicates the amount which is to be deposited
annually at the end of each year for a number of years to be accumulated to ` 1 at a rate of
compound interest.

Let A be the amount which is to be deposited at the end of each year for n years. Let i be
the interest rate on ` 1 in one year.

Then the series of payments will accumulate to ` 1 in the following manner as shown below.

1= A(1+i)(n-1) + A(1+i)(n-2) + A(1+i)(n-3) +⋯ A(1+i)(n-r) + ...+ A(1+i)3 + A(1+i)2 +


A(1+i) + A) or,

1= A [1 + (1+i) + (1+i)2 + (1+i)3 + ⋯ + (1+i)(n-3)+ ……+ (1+i)(n-2) + (1+i)(n-1)]

The R.H.S. is a G.P. series of n terms.

The summation of a G.P. series is obtained from the formula S=[a(Rn-1)]/(R-1)

where, S denotes the summation of the series, a the first term, R the common ratio
and n the number of terms.

or, `1 = [A(1(1+i)n-1]/(1+i-1)= A (1+i)n-1]/i

or, A= i/(1+i)n-1

Page | 24
L&B/Chapter-2/Valuation of Real Estate

Therefore, the amount of annual sinking fund = i/[(1+i)n-1],which is just the inverse of the
amount of ` 1 per annum table.

Problem Mrs. Singh took on the lease of a house for 20 years. She paid a premium of `
923,230 at the beginning of the lease period and paid a reserved rent of ` 10,000 per
annum. The full rental value for the lease is ` 120,000 per annum (net). What is the amount
of sinking fund she should set aside from her income from the leasehold so as to
accumulate to the amount of premium paid at 2½% compound rate of interest after 20
years?

Solution Let S be the amount of sinking fund per annum to be set aside from Mrs. Singh's
income on subletting the property that accumulates to ` 1 at 2½% compound rate of
interest after 20 years.

Therefore, S=i/[(1+i)n-1]where i=0⋅025 and n=20

or, S=(0⋅025)/[1⋅02520-1]=0⋅039148

Therefore, to redeem ` 923,230 the total amount to be set aside

= ` 923,230 × 0⋅039148

= `36,143 per annum.

Present Value of `1 per annum at single rate (Years’ Purchase Single Rate) table

This table indicates the present capital value of a series of income of ` 1 to be received at
the end of each year at a compound rate of interest for a given number of years.

Let I be the interest on ` 1 for 1 year and n be the number of years.

Then, present value of ` 1 to be received after 1 year =1/(1+i)

Similarly, for any number of years n the present value of ` 1 to be received after n years is
given by 1/(1+i)n

So, the summation of a series of present values of ` 1 to be received at the end of each year
for n years is given by,

1 1 1 1
𝑆= + 2
+ 3
+. . . + +⋯
1 + 𝑖 (1 + 𝑖) (1 + 𝑖) (1 + 𝑖)𝑟
1 1 1
…+ + +
(1 + 𝑖) (𝑛−2) (1 + 𝑖) (𝑛−1) (1 + 𝑖)𝑛

Where, S= summation

But this is a G.P. series with 1/(1+i)as the first term and the common ratio is also 1/(1+i).The

Page | 25
IOV-Registered Valuers Foundation

number of terms is n.

The summation is given by the formula S= a(Rn-1)/(R-1) where, a=first term;

R=common ratio; and n = number of terms


1 1 1 1
( )×( −1) −1 1−
(1+𝑖)𝑛 (1+𝑖)𝑛 (1+𝑖)𝑛
Therefore, 𝑆 =
1+𝑖
1 = −𝑖
= 𝑖
−1
1+𝑖

1
1−
(1+𝑖)𝑛
Therefore, present value of ` 1 per annum (single rate) = 𝑖

The expression is also called years' purchase (single rate).

Problem What is the present value of a net income of ` 100,000 per annum for 15 years at
8% interest rate?

Solution Present value of ` 1 per annum


1 1
1− 1− 2⋅17217
(1+𝑖)𝑛 (1+0⋅08)15
= 𝑖
= 0⋅08
= 0⋅25377

Therefore, present value of ` 1 per annum (years' purchase) =8⋅5596

Therefore, present value of `100,000 per annum = `100,000 × 8⋅5596

= ` 855,960

Present Value of `1 per annum at dual rate (Years’ Purchase Dual Rate) table

In valuing a property by the income capitalisation technique the Valuer is often asked to
value a terminable income such as a leasehold interest. Obviously, the difference between a
perpetual income flow and the terminable income is that the former lasts in perpetuity
whereas the latter comes to an end after a number of years. Therefore, a freehold income is
not directly comparable with a leasehold income which lasts for a given number of years
only. The rate of capitalisation of freehold income cannot therefore be comparable as an
investment return with that of a leasehold income. In order to make a leasehold income
comparable to a freehold income, an amount is set aside from the leasehold income
annually so as to accumulate at a given rate of interest (rather low) to a capital amount of
value being equal to the value of the leasehold interest at the end of the lease period. In
this way the terminable income flow from a leasehold interest can be perpetuated and thus
the rate of capitalisation from the leasehold may now be compared with that of a freehold
interest. The amount thus set aside so as to be accumulated at a low rate of compound
interest to a capital sum is called the “sinking fund”. And the low rate is called the
“accumulative rate of interest”. The return on the capital or the “remunerative rate of
interest” is that rate of interest that the spendable income works out on the capital value of

Page | 26
L&B/Chapter-2/Valuation of Real Estate

the lease. All leasehold interests are valued by the dual rate principle. Obviously, all
incomes from leaseholds have two parts—one part is spendable income which caters to the
remunerative rate of interest and the other part is accumulated—this part caters to the
accumulative rate of interest.
Let, 𝑉= present value of ` 1 per annum for 𝑛 years (dual rate)
Let, 𝑖= the interest rate on ` 1 per annum or remunerative rate
of interest
Let, 𝑠= the annual sinking fund for redemption of𝑉
𝑖1
=
(1 + 𝑖1 )𝑛 − 1

where, i1= accumulative rate of interest and n = number of years.

Then each unit of ` 1 income shall have two components as follows.


1= 𝑉 × 𝑖 + 𝑉 × 𝑠=𝑉(𝑖 + 𝑠)
1
Therefore, 𝑉=
𝑖+𝑠

But V = present value of `1 per annum for n years (dual rate) = years' purchase (dual rate).
1
Therefore, years' purchase (dual rate) =
𝑖+𝑠

For capitalisation of income from leasehold interest in all cases, the net income of the
1
lessee should be multiplied by
𝑖+𝑠

Problem A lessee took on lease of a house for 30 years on an exclusive rent of ` 10,000 per
annum 15 years back. The property is worth ` 35,000 per annum exclusive of all outgoings
today. What is the value of the lessee's interest?

Solution
Gross rental value = ` 35,000 p.a.
Less rent paid by lessee = ` 10,000 p.a.
Net profit rent = ` 25,000 p.a.
Y.P. @ 7½% & 2½% for 15 years = 7.647
Value = ` 191,175

Therefore, value of lessee's interest = ` 191,175

Mortgage Installment table

The annual mortgage installment for redemption of `1 in year’s time at single rate of
interest is given by Annuity `1 will purchase which is equal to i+s where, i= remunerative
rate of interest s= sinking fund installment at accumulative rate of interest i for a period of

Page | 27
IOV-Registered Valuers Foundation

n years (please note that both the remunerative rate of interest and the accumulative rate
of interest is the same in this case).Therefore, amount necessary per month for redemption
(𝑖+𝑠)×100
of `100 = 12
which is the basis of mortgage installment table.

Problem What is the monthly installment to be paid for redemption of `100,000 on


mortgage at 9% if the loan is to be repaid in 15 years time?

Solution
(𝑖+𝑠)×100,000
Equated monthly installment = 12

Where i=0.09, s=0.034059 (the annual sinking fund to accumulate to `1 at 9% interest


in 15 years).
(𝑖+𝑠)×100,000 (0.9+0.034059)×100,000
Now, =
12 12

= `1,034 which is the monthly installment for redemption of `100,000 in 15 years.

12. URBAN INFRASTRUCTURE AND ITS


INFLUENCE ON VALUE OF REAL ESTATE
Infrastructure is the fundamental facilities and systems serving a country, city, or other area,
including the services and facilities necessary for its economy to function. Infrastructure is
composed of public and private physical improvements such as roads, bridges, tunnels,
water supply, sewers, electrical grids, and telecommunications (including Internet
connectivity and broadband speeds).The infrastructure is important for faster economic
growth and alleviation of poverty in the country. The adequate infrastructure in the form of
road and railway transport system, ports, power, airports and their efficient working is also
needed for urban economy. A distinguishing feature of infrastructure is that while the
demand-supply gap in case of other factors can be met by importing some of them, the
deficiency of infrastructure cannot be made up through imports. Because location-based
the need for relevant infrastructure facility can be met through development of its capacity
in the domestic economy. Economic Survey of India for the Year 2013 -14 declared, “Rural
economic growth in recent years has put enormous pressure on existing infrastructure
particularly on transport, energy and communication. Unless it is significantly improved
infrastructure will continue to be a bottleneck for growth and obstacle to poverty
reduction”.

Influence of Power Infrastructure Power or energy is a crucial input into all economic
activities and therefore rapid economic growth is possible only if adequate power is made
available everywhere. It is essential for growth of industry, commercial business as well as

Page | 28
L&B/Chapter-2/Valuation of Real Estate

for household-lighting. Uninterrupted power supply attracts investors to set industries in


those areas which on the other hand generates employment giving rise to demand for real
estate. Therefore, urban areas having adequate power facility show higher rate of value of
all types of real estates. Housing areas having no threat of power failure is always more
lucrative than areas having poor power supply records. Therefore, with adequate power
supply, the market value of all real estate increases.

1. Influence of Road infrastructure

2. Road is another important infrastructure which is essential for movement of goods,


raw materials and fuel. The cities having better road network show higher degree of
development. Besides movement of goods, inadequate road facility results in
congestion of traffic during pick hours of the day as well as night. This not only
results in loss in working hours but one of the main root causes environmental
pollution in urban areas. Therefore, different parts of a city having different network
of road shows distinct difference in values of real estates. The areas having
adequate road facility shows more demand for space than areas having inadequate
road network.

3. Influence of Railway Infrastructure


4. Railway transportation system is the most effective environment- friendly and
economically-efficient transport system in India. In the last few decades, railway in
the form of ‘Metro Railway’ becomes the most efficient and convenient transport
system for the city dwellers of Indian cities. The sub-urban railway system in most
of the metro cities protects the urban local government from excessive pressure of
population. Because of this railway system, cities are expanding every day its city
limit. The smooth and hassle free communication through metro railway increases
the demand for spaces on and around the metro railway channels in every cities.
Therefore, same type of units on and around a metro channel always carries better
value than their counterparts situated in other areas.
5. Influence of Water Infrastructure

6. Water is going to be the number one determinant for home buyers in this decade.
Due to alarming fall of ground water level, most of city government ban on
extracting water from ground water resources. Therefore, the only resource of
potable water in cities becomes the rivers flowing through or around the city. Rain
water harvesting, that is why, becomes a mandatory regulation for the new building
owners of most of the urban areas in India. As a result all the housing units having
the facility of uninterrupted water supply for 24 hours a day and 365 days in a year
are asking for premier price. Thus, good water infrastructure facility increases the
value of a real estate in India to a good extent.

7. Influence of Social Infrastructure

8. Social infrastructure can be broadly defined as the construction and maintenance of


facilities that support social services. Social infrastructures are created to increase
social comfort and act on economic activity. These are schools, parks and

Page | 29
IOV-Registered Valuers Foundation

playgrounds, structures for public safety, waste disposal plants, hospitals, sports
area, etc. Social infrastructure has a positive impact on the value of urban real
estate. People love to stay as close as possible to these social infrastructures.
Therefore, closeness to social infrastructure increases the value of real estates in
urban areas.
9. Indian Government Policies for urban infrastructure development

10. Housing Policy

11. The National Urban Housing & Habitat Policy 2007 (NUHHP-2007) has been
formulated keeping in view the changing socio-economic parameters of the urban
areas and growing requirement of shelter and related infrastructure. The Policy
seeks to promote various types of public-private partnerships for realizing the goal
of “Affordable Housing for All” with special emphasis on the urban poor. Given the
magnitude of the housing shortage and budgetary constraints of both the Central
and State Governments, the NUHHP-2007 focuses the spotlight on multiple stake-
holders namely, the Private Sector, the Cooperative Sector, the Industrial Sector for
labour housing and the Services/ Institutional Sector for employee housing.
12. Jawaharlal Nehru National Urban Renewal Mission

13. The JNNURM was launched in 2005 as the first flagship scheme of the Ministry of
Housing and Urban Affairs. JnNURM implemented by MoHUPA has two
components e.g. Basic Services for Urban poor (BSUP) and Integrated Housing and
Slum Development Programme (IHSDP) which aimed at integrated development of
slums through projects for providing shelter, basic services and other related civic
amenities with a view to providing utilities to the urban poor.

14. The Mission was initially for a seven year period i.e. up to March 2012 which was
extended upto March 2014 for completion of the already approved projects. During
March 2013, the Mission period was extended by one more year i.e. upto March
2015 to complete ongoing works. 65 Mission Cities identified based on urban
population (Census 2001), cultural and tourist importance was covered under BSUP
and the remaining cities were covered under IHSDP (887).

15. It supported 63 cities (7 mega cities, 28 metro cities and 28 capital cities and towns
of historical/religious importance) across the country through perspective building
City Development Plans (CDPs) for specifying infrastructure gaps relating to water,
sanitation, sewerage, drainage and roads on the one hand and deficiencies in
housing and basic services on the other hand.
16. Swachh Bharat Abhiyan

17. Swachh Bharat Abhiyan is a movement across India spearheaded by Government of


India launched on 2ndOct,2014. The earlier schemes like Total Sanitation Campaign,
Nirmal Bharat Abhiyan have been integrated into it. The Swachh Bharat Abhiyan
(Urban) is an initiative of the Ministry of Urban Development. It's focused on
elimination of opendefecation, eradication of Manual Scavenging. It advocates
modern and scientific Municipal Solid Waste Management. It seeks to change

Page | 30
L&B/Chapter-2/Valuation of Real Estate

behaviors around sanitation and its linkages with public health. Necessary capacity
augmentation of ULB's and creating an enabling environment for private sector
participation in Capex (capital expenditure) and Opex (operation and maintenance)
was also incorporated.

18. Atal Mission for Rejuvenation and Urban Transformation

19. Atal Mission for Rejuvenation and Urban Transformation (AM RUT) was launched
on June 25, 2015. It is built on the idea that Infrastructure creation should have a
direct impact on the real needs of people, such as providing taps and toilet
connections to all household - a learning from the earlier JNNURM Mission.

20. SMART Cities Mission

21. SMART Cities Mission was launched on June 25, 2015 with 75 percent share of
centre and balance to be financed by the respective state. It targets 98 cities. Smart
City advocates multiple points: Mixed land use in area based developments and
increase land use efficiency in 'unplanned areas' through a range of compatible
activities and land uses including flexibility in land use and building byelaws to
adapt to change; Also seeks to expand housing opportunities for all. An interesting
thrust is on creating "walkable localities" where road network is to be assessed for
different types of uses - vehicles/ public transport, cyclists and pedestrians. It will
focus on promoting a variety of transport options in a Transit Oriented
Development (TOD). Reduce congestion, air pollution and resource depletion, and
ensure security. Also seeks to preserve and develop open spaces - parks,
playgrounds, and recreational spaces to improve the quality of life of citizens,
reduce the urban heat. It will offer online services especially using mobiles to bring
about accountability and transparency in service monitoring and seek feedback of
people.
22. National Heritage City Development and Augmentation Yojana

23. National Heritage City Development and Augmentation Yojana (HRIDAY) was
launched on January 2015 to preserve the heritage character of 12 cities - Ajmer,
Arnravati, Amritsar, Badami, Dwarka, Gaya, Kanchipuram, Mathura, Puri, Varanasi,
Velnakanni and Warrangal - facilitating economic growth and urban planning in an
inclusive manner. The interventions include development of sanitation facilities,
roads, public transportation, parking and citizen services. The project will end in
March 2017 and has a total outlay of Rupees five hundred crores.

Page | 31
IOV-Registered Valuers Foundation

13. REAL ESTATE MARKET AND ITS


CHARACTERISTICS, INVESTMENT IN REAL
ESTATE, FACTORS INFLUENCING DEMAND
AND SUPPLY SCHEDULE IN REAL ESTATE
• REAL ESTATE MARKET AND ITS CHARACTERISTICS
In economics, a market is defined, not as a fixed place having a demarcated area where
goods and commodities are exchanged. A market is a device by which buyers and sellers
come in contact with one another to form a price. A market may not be located in a single
location. E.g. Indian diamonds may have a worldwide market. Similarly, software produced
in India may have a market in the USA and other western countries. A price is formed in a
market by the interaction of demand and supply of a commodity. This is equally true in the
case of real estate market. However, the real estate market is divided in three tiers, viz. local
market, national market and international market. Each tier of the market again has its own
segment of real estate such as local market of residential real estate, local market of
commercial real estate, local market of retail real estate. Again, national real estate market
has national market of residential real estate, national market of commercial real estate and
national market of retail real estate etc. Similarly, international real estate market has each
segment as international market of residential real estate, international market of
commercial real estate and international market of retail real estate etc. Other types of real
estate may have similar segregation in the three tiers of market.

The efficiency of a market depends on accurate price information of any commodity at


different parts of the market and free movement of commodities from one part of the
market to another. In the case of real estate however, these conditions are not fulfilled.
Information on price of real estate is difficult to obtain as the transactions in real estate are
very much restricted in information, being only available to the buyers and sellers. The lack
of transparency in transactions is a character of real estate anywhere in the world. Again,
real estate cannot be geographically transferred from one place to another. Therefore, if
there is an augmentation of demand in a location, since supply cannot be increased in
response immediately, the price rises in that location. That is why real estate market is said
to be sluggish in response. Furthermore, the dealing cost of real estate is high because of
the presence of a considerable stamp duty which is required for investment in real estate.
Also, the information on price of a real estate may require an analysis by a valuer thus
making it costly for an investor to invest in real estate. Again, investment / purchase of real
estate require large funds unlike investment in securities in the capital market. So, if interest

Page | 32
L&B/Chapter-2/Valuation of Real Estate

rate for borrowed funds is high or there is a capital crunch in the economy, investment in
real estate suffers causing a shortage in supply. All these factors make hindrance to
demand and supply and make real estate an imperfect market for the reasons as already
stated.

Investment in Real Estate

In the investment market there are always a number of alternative opportunities for
investment. The investor who would prefer capital gain from the investment would risk
investing his capital in the equity shares of first-class companies whereas the investor who
would prefer safe and secure income from the investment would like to investment in long-
term government securities. Between the two extremes of highly risky equity share
investment and safe and secure government securities however there are a number of
alternative opportunities with different degrees of risk and security. Real property is one
such vehicle of investment in the totality of the investment market scenario. The role played
by real property is dependent on the various aspects of real property as a channel of
investment. We discuss them as follows.

Management aspect

Real property investment is always associated with trouble and cost of management. Thus
for management of real property the services of a lawyer, an architect, a Valuer or a real
estate agent are necessary from time to time. Further, there may be involvement in
litigations day to day involvement in enforcing lease covenants, collection of rents etc, all
parts of hazards of management. Comparatively, investment in equity share is less
troublesome and costly to manage. Therefore, in order to induce an investor in the
investment in real property there must be an incentive of higher rate of return than the
yield from equity share investment.

Again, the investor can invest in long-term government securities to obtain a risk-free
long-term income. Against this, there is the prospect of rental growth and appreciation of
capital value as the incentive for investment in real property in spite of the hazards of
management.

Legal aspect

Law interferes with real estate investment in a number of ways. Thus, rent control acts may
severely affect the prospect of rental growth of the property. Further, taxation laws may
affect real property investment. Examples are property tax, capital gains tax, betterment
levy etc. Again, town planning acts may adversely affect real property investment through
restrictions imposed on land uses, density etc. As a result the investors should expect
higher yield from real property investment to compensate against this.

Page | 33
IOV-Registered Valuers Foundation

Liquidity

Real property is an illiquid asset. The time taken for real estate disposal is a lengthy
process. Further, for transfer of property one may have to incur cost on lawyers' fees, stamp
duty etc. Comparatively, the time taken for disposal of shares or cost of transfer of share
are much less than that of real property. Hence, in order to induce an investor to invest in
real property one must expect to receive a higher yield.

Size of investment

Real property is indivisible into very small components. Generally, a huge amount of capital
is associated with an investment in real property. Comparatively, a much smaller amount of
capital is required in order to invest in shares. To compensate against this the real property
investment should have higher yield.

Lack of central market

There is no central market for real property for the stock exchange market. Furthermore,
the transactions in real property are often of closed type and one is required to incur a high
amount of expenditure to have full knowledge of the transaction. These add to the market
imperfection of real property market and have the effect of increasing the yield from real
property investment.

It is clear from item number (1) to (5) above that cost and hazards of management and
market imperfections cause return from real estate to be higher than risk-free securities.
The benchmark of interest rate in the investment market is the yield from long-term
government securities (also called the gilt-edged securities or the risk-free securities). So,
all investments should have ideally produced higher return than the yield from government
securities. But in case of real estate, the prospect of capital appreciation and income
appreciation act to set off against the higher return required than that of the government
securities. That is why, real estate in first-class location tends to indicate lower yields than
the risk-free government securities. This phenomenon is known as the reverse yield gap.

• FACTORS INFLUENCING DEMAND IN REAL ESTATE


The price of any commodity in a market is fixed by the interaction of demand and supply.
So also in the property market, demand and supply interact to fix the prices. Any study of
factors affecting value or prices in the property market therefore amounts to the
examination of the factors affecting demand and supply of landed property.

In the demand side the following factors are relevant:

1. Population changes

Page | 34
L&B/Chapter-2/Valuation of Real Estate

2. Economic growth
3. Accessibility
4. Inflation
5. Change in taste and choice of people

Population changes

As more and more people migrate in an area, there is an increase in demand for land of all
types such as residential purposes, commercial uses, industrial uses, open space and
recreational needs etc. But since total area of land is fixed by nature, the per capita
allocation of space for any use reduces. Supply of land being low and demand remaining
high the value of land sharply rises.

Economic Growth

As an economy grows there is more investment in infrastructure, industries, commercial


sectors and services. The living standard of people rises and per capita income grows. As a
result there is more demand for residential, educational, shopping and other commercial
facilities. So the overall demand of land increases. Existing land resources are developed
and redeveloped and value of land rises.

Accessibility

As a nation undertakes great road-building programs, opening up of new railway


communications, establishing aerodromes etc, there is an overall change of accessibility.
The remote parts of a country become connected with urban centers through improved
accessibility. As a result demand for land for all uses in the remote districts increases. Again
re-planning and re-development of existing road networks within an urban area improves
accessibility and increases demand for land for various uses.

Inflation

During inflationary times investment in long-term government securities or other fixed


interest securities is not secured in real terms as the real value of money associated with
such investments falls with time causing a loss of capital. But value of landed property
continues to grow with time at a faster rate than the rate of inflation. As a result, the real
value of money in landed property investment is maintained in real terms. Hence, during
inflationary times there is an increase in demand for investment in landed property from a
section of investors to maintain the real value of money. Landed property therefore is said
to offer a hedge against inflation.

Change in taste and choice of people

With increased standards of living, the large joint families came to be fragmented as

Page | 35
IOV-Registered Valuers Foundation

people chose to live in small family units. As a result, demand for small flats rose
phenomenally.

There was large scale demand for flats in high-rises that gave rise to intensive development
or redevelopment of urban sites.

• FACTORS INFLUENCING SUPPLY SCHEDULE IN REAL ESTATE


1. Property development is a time-consuming process. As a result, supply of landed
property cannot be increased suddenly. In the short run, therefore, when demand
of land in an area increases, say by population growth, supply of property cannot
be increased. As a result, the demand being high and supply remaining unchanged,
price rises sharply.

2. When the demand for landed property increases in an area in the long run, re-
development of under-developed sites and development of virgin sites take place,
thus contributing to the fresh supply of goods in the property market. But since
total land in an area is fixed by nature, there is a limit up to which supply can be
augmented. So, in the long run also, demand continues to overtake supply and
prices of landed property continue to rise.

3. Landed property being a durable good, its supply cannot be reduced once
constructed if the demand suddenly falls. As a result, property prices fall and in
order to avoid further capital loss, the suppliers may be compelled to sell property
at a reduced price.

4. Flow of funds to the property market from the capital investment market is largely
responsible for property development. If therefore the flow of fund in the property
market is restrained due to imperfections in the capital investment market, the
supply of property will fall and prices will rise due to a growing demand.

5. Enactments may be a hindrance to the supply of property in the market. Thus, a


high stamp-duty may restrain the investor from investment in property for
development if price incentive is not high enough to cover the cost. Again, rent
restriction acts may stand in the way of investment in property sector for letting
causing a shortage of supply of rental accommodation.
6. The decision of planning authority may control the supply of land in the land
market. The planning authorities may allocate land for some use by use zoning
through the outline development plan of an urban area. Again, they may control
the intensity of development of land by density zoning. If the authority restricts the
use of land by use zoning, the supply of such land use is reduced in the area and
prices may rise due to a growth of demand.

Page | 36
L&B/Chapter-2/Valuation of Real Estate

11. CONCEPTS OF GREEN BUILDING

• INTRODUCTION

Green building is one of the fastest-growing movements in the housing industry today.
More than just a passing trend, the green building movement is a response to the pressing
environmental problems prevalent in our society and provides tangible environmental,
community, and economic benefits, improving health and public spaces and lowering
building and energy costs.

Buildings have major environmental impacts during their life. Resources such as ground
cover, forests, water, and energy are dwindling to give way to buildings. Resource-intensive
materials provide structure to a building and landscaping adds beauty to it, in turn using
up water and pesticides to maintain it. Energy-consuming systems for lighting, air
conditioning, and water heating provide comfort to its occupants. Hi-tech controls add
intelligence to ‘inanimate’ buildings so that they can respond to varying conditions, and
intelligently monitor and control resource use, security, and usage of fire fighting systems
and other such systems in the building. Water, another vital resource for the occupants,
gets consumed continuously during building construction and operation. Several building
processes and occupant functions generate large amounts of waste, which can be recycled
for use or can be reused directly. Buildings are thus one of the major pollutants that affect
urban air quality and contribute to climate change. Hence, the need to design a green
building is to address all these issues in an integrated and scientific manner. It is a known
fact that it costs more to design and construct a green building compared to other
buildings. However, it is also a proven fact that it costs less to maintain a green building
that has tremendous environmental benefits and provides a better place for the occupants
to live and work in. Thus, the challenge of a green building is to achieve all its benefits at an
affordable cost.

A green building depletes the natural resources to a minimum during its construction and
operation. The aim of a green building design is to minimize the demand on non-
renewable resources, maximize the utilization efficiency of these resources when in use, and
maximize the reuse, recycling, and utilization of renewable resources. It maximizes the use
of efficient building materials and construction practices; optimizes the use of on-site
sources and sinks by bioclimatic architectural practices; uses minimum energy to power
itself; uses efficient equipment to meet its lighting, air conditioning, and other needs;
maximizes the use of renewable sources of energy; uses efficient waste and water
management practices; and provides comfortable and hygienic indoor working conditions.
It is evolved through a design process that requires input from all concerned – the

Page | 37
IOV-Registered Valuers Foundation

architect; landscape designer; and the air conditioning, electrical, plumbing, and energy
consultants – to work as a team to address all aspects of building and system planning,
designing, construction, and operation. They critically evaluate the impacts of each design
decision and arrive at viable design solutions to minimize the negative impacts and
enhance the positive impacts on the environment.

Green building encompasses a wide range of renewable construction materials, energy and
resource-efficient building techniques, and an overriding philosophy of sustainable
development. The most significant green building practices, commonly referred to as the
“six elements of green building,” relate to site, water, energy efficiency, indoor air quality,
materials, and operations and maintenance.

Check list of Green Building evaluation

The following are the check list of green building evaluation:

Site planning For sustainability of land, two ideas behind green site planning
and development are: (i) to protect and restore habitat and (ii)
to maximize open space, providing social and environmental
benefits.

Water efficiency Water efficiency (e.g., water use reduction, landscaping) considers
the potable water as well as storm water and waste water
management.

Energy and By energy and atmosphere, we mean renewable energy sources as


atmosphere well as ozone depletion. Green building considers the conservation
of energy in the building’s design by closing the building envelope,
integrating energy-efficient mechanicals, electricals and fixtures,
landscaping to assist in shade or solar access, or using renewable
energy sources such as solar, wind, or geothermal alternatives.

Indoor air quality Indoor environmental quality means air quality, emissions and
passive heating. Green building design focuses on mitigating the
negative effects of combustion-based appliances and moisture.
Green buildings use low or no VOC paint to provide better indoor
air quality.

Material and Materials and resources mean reuse, recycling of materials as well
resources as use of renewable materials. The materials used have a significant
effect on indoor air quality. Green buildings use materials that are
less toxic than their conventional counterparts. Additional

Page | 38
L&B/Chapter-2/Valuation of Real Estate

considerations include what the material is made of, how it is


manufactured, and where the material originates. Green buildings
use floor coverings that have low volatile organic compound (VOC)
concentrations and are durable and recyclable.

Operation and Innovation and sustainable design includes measures to control


maintenance water and energy consumption along with the use of durable
materials and designs that are meant to lower maintenance costs
while lengthening the lives of building components.

Whole building approach

Green building has popularized a new style of design called the “whole building approach.”
The whole building approach is elementary to green building. This approach views all of a
building’s parts as an integrated system. The goal of the whole building approach is to
ensure that the different parts of a building work with, rather than against, one another.
Whole-systems thinking include:

1. the use of native landscaping


i. to reduce required maintenance (mowing and irrigation)
ii. to preserve and enhance groundwater and wildlife habitat

2. day lighting
i. to reduce the need for artificial lighting and energy consumption
ii. to improve worker comfort and productivity

3. convenient access to public transportation


i. to reduce amount of space allocated for parking
ii. to foster reductions in vehicular miles traveled (VMTs) and carbon usage

Green Building Valuation

Revenue, occupancy, operating expenses, and risk are the four determinants of higher
value of a green building.

Revenue
In today’s real estate market, especially office property market, rental premiums are
emerging in green buildings as many of today’s best tenants are increasingly willing to pay
a premium for green spaces. For these tenants, leasing green space is an opportunity to
demonstrate a commitment to sustainability, attract the best employees, and improve
productivity. For owners not pursuing certifications, investments in green features in
conventional buildings can also establish position in the marketplace and keep rents up. In
those circumstances where you have an underperforming property, investments in green
features may result in better positioning of the asset within the spectrum of other

Page | 39
IOV-Registered Valuers Foundation

properties in the marketplace. A skilled valuation professional can assist in understanding


whether investments are truly a green “premium” or necessary retrofit to keep up with the
market.

Occupancy

Occupancy premiums can lead the case for green investments. If it can be determined that
the green features will result in higher occupancy than an otherwise similar building, it can
be an acceptable feature for increases in value.

Operative expenses

The most effective determinant for higher value of green buildings is their very low utility
bills. In premier markets with high rents, energy savings may appear relatively small as a
percent of operating income. By contrast, in markets with lower rents, energy costs
represent a much larger percentage of income; improving cash flow. Operational savings
on maintenance and reserves also add value

Risk

Green building value also shows up in the risk-mitigating protections these assets offer to
owners. In the valuation and underwriting process, high-performance buildings can offer
hedges against changing consumer preferences as well as new laws and increasing energy
prices. Recognizing the relative safety of green assets, major insurers may offer discounts
for green buildings.

Green building rating system

A green building rating system is an evaluation tool that measures environmental


performance of a building through its life cycle. It usually comprises of a set of criteria
covering various parameters related to design, construction and operation of a green
building. Each criterion has pre-assigned points and sets performance benchmarks and
goals that are largely quantifiable. A project is awarded points once it fulfils the rating
criteria. The points are added up and the final rating of a project is decided. Rating systems
call for independent third party evaluation of a project and different processes are put in
place to ensure a fair evaluation. Globally, green building rating systems are largely
voluntary in nature and have been instrumental in raising awareness and popularizing
green building designs. Building Research Establishment’s Environmental Assessment
Method (BREEAM) of United Kingdom, Comprehensive Assessment System for Building
Environmental Efficiency (CASBEE) of Japan, Leadership in Energy and Environmental
Design (LEED) of United States and Green Rating for Integrated Habitat Assessment
(GRIHA) of India are the internationally acclaimed green building rating systems.

Page | 40
INCOME APPROCH TO

VALUE I 03
1. Relation between Income and Value
2. Valuation of Property affected by rent control
3. Derivation of Yield rate from market derived data
4. Remunerative Rate of interest and accumulative rate
of interest
5. Types of Rent
6. Determination of Market rent and standard rent
7. Lease
8. Valuation of lessor's interest
9. Real Estate as an investment
10. Investment Decisions
11. Profit Methods

Page | 1
(This page is blank)
Page | 2
INCOME APPROACH I 03
1. RELATION BETWEEN INCOME & VALUE
• INCOME APPROACH
1. The income approach considers the income that an asset will generate over its
remaining useful life and estimates value through a capitalization process. This
process applies an appropriate yield, or discount rate, to the projected income
stream to arrive at a capital value. The income stream may be derived under a
contract or contracts, or be non- contractual, e. g. the profit generated from
either the use of or holding of the asset.
2. Two commonly used methods that fall under the income approach are income
capitalization, where an all risks yield is applied to an income stream, or
discounted cash flow where the cash flows for future periods are discounted to
a present value.

The income approach can be applied to liabilities by considering the cash flows
required to service a liability until it is discharged.

• INVESTMENT METHOD

This method is also called the investment method of valuation. Under this method the
net income flow from the property is capitalized at an appropriate rate percent or yield
in order to find the capital value. Properties with monthly tenants or leases are valued
by this method. Owner-occupied properties or vacant properties can also be valued by
this method provided there is adequate rental evidence.

Consider a person seeks an income of INR 1,000 per annum from an investment. He
wishes this to be an 8% return on his investment. What is the amount he has to invest?
Obviously,(100/8)*1000= INR 12,500 is the amount from the investment for an annual
income flow of INR 1,000.
From the above example, we may conclude, capital value= (100/Yield)*net income per
annum or simply value = (100/Yield)*net income per annum.

If instead of considering return on INR 100 we consider return on INR 1 then it can be

Page | 3
IOV-Registered Valuers Foundation

shown that value = (1/Yield on INR1)*net income per annum.

It is usual to express return on INR 100 by whereas return on INR1 is expressed by i.e.

So, value = (100/I)* net income per annum

Or, value= (1/i)* net income per annum

The expression 100/I or 1/I is called “years’ purchase” or simply Y.P. for a perpetual income
flow. So, value= Y.P.*net income

The model of valuation, value = Y. P. x net income accepts that the income from the
property is a perpetual flow. Correctness of this assumption may be called into question
because a property, particularly the building does not last in perpetuity. The building
gradually deteriorates and falls to zero value. Hence no income from it could be perpetual.
However, from merely practical point of view it is arguable that the income from the
building may fall at the remote end of its life but the value and hence the income from the
land gradually rises thus maintaining the perpetual income flow of the property.

Again, from the valuation tables it can be shown that V.P. or present value of INR per
annum at a given rate percent say 6% for 30 years is 13.756, for 50 years is 15.762 while for
100 years is 16.618 and in perpetuity on 16.667.

So, instead of lasting in perpetuity if the building lasted for 30 years only it accounted
for82.6% of its total value. If the building lasted for 50 years only it accounted for about
94.6% of its value.

So, instead of lasting in perpetuity if the building lasted for 50 years only it could account
for about 95% of its value. So, only the remaining 5% would come from the remaining part
of the building's life considering if it lasted in perpetuity.

The model of valuation V= (100/Yield)* net income has two variables.

1. The income.

2. The yield.

• THE INCOME
The income from a property is the rent which is paid by the tenant when the property has
actually been let out. If the property is vacant then its potential income would mean the
rent at which it can be let out in the market.

Gross rent is the gross amount which is paid by the tenant to the landlord for occupying
the property. The landlord is to incur some expenditure towards maintenance of the

Page | 4
L&B/Chapter-3/Income Approach to Value

property and continuation of the income from the property. To name some of these
expenditures are repairs and maintenance, property tax, management expenses, insurance
etc. These expenditures are collectively called outgoings.

When outgoings are deducted from the gross rent, the result is called the net rent. Rental
value of a property at any point of time would mean the rent at which the vacant property
could be let out in the open market.

A property has been let out at exclusive rent means under the tenancy agreement, the
tenant will bear the burden of all outgoings. Inclusive rent under a tenancy agreement
means the tenant will pay the gross rent only and the landlord will bear the burden of all
outgoings.

Fully repairing and insuring lease means a lease where the tenant bear the burden of all
outgoings. The common types of outgoings are as follows.

• REPAIRS
The actual amount of expenditure on account of repairs depends upon the state of repairs,
maintenance and other aspects of the property. For a relatively recently constructed
property, an allowance of 10% on gross rent can be taken for repairs. For an older property,
an allowance of the order of 10% to 20% can be taken.

However, there is no hard and fast rule regarding the amount to be allowed forrepairs. It
entirely depends on the condition and state of repairs of the property. Apart from the
recurring allowance for repairs, a property may be in a very poor state of repairs requiring
immediate capital expenditure for renovating the property. In such case a capital amount
should immediately be deducted from the market value of the property arrived at.

• PROPERTY TAX
Property tax is levied by the municipal authority on the property. The actual tax levied by
the authority should be deducted from the gross income. However, provisions may be kept
for a future revision of the tax.

• INSURANCE
Insurance of a property is done for keeping a provision against any accidental damage, say
loss by accidental fire. Generally, for new buildings the provision is 1% of the gross rent per
annum. However, for older buildings, whose reconstruction cost is higher in case of fire
damage, the insurance premium may be as high as 3%.

Page | 5
IOV-Registered Valuers Foundation

• MANAGEMENT
The owner of the property is to undertake a lot of hazards and costs for managing the
property. Thus, he has to collect the rents and enforce the tenants' liabilities on account of
the various covenants of the tenancy agreement. He may also be involved into litigations.
For a property with a single tenant the cost of management may be as low as 5% of the
gross rent. Whereas for a property with multiple tenancies the management cost may be as
high as 10%.

• SERVICE CHARGES
Service charges are levied when a block of flats is let out to a number of tenants. In such
cases some services for common benefit of the tenants is to be rendered by the landlord.
Costs of the services provided by the landlord is charged by the landlord from the tenants.
Thus costs of maintenance of lifts, lighting of common parts of the building, payment of
wages of caretakers etc. are incurred by the landlord and charged from tenants over and
above the outgoings.

• VOIDS
Where there is a possibility of the property remaining unoccupied for a period an allowance
may be made on account of voids by deducting a reasonable percentage from gross rent.
However, alternatively a higher yield may be taken for capitalisation of the income expected
from the property.

• THE YIELD
We have already discussed on yield from an investment in our studies of the capital
investment market. The valuer should be able to recognize that the income from a property
in a first-class location should be capitalised at a relatively lower yield. This is because the
rental growth of a property in a first-class location is very fast and average rate of growth of
rent. So, the income is highly secured whereas in a secondary location the rental growth is
low and below average rate of growth. So, rent from a secondary location should be
capitalised at a relatively higher rate percent.

Page | 6
L&B/Chapter-3/Income Approach to Value

2. VALUATION OF PROPERTY AFFECTED BY


RENT CONTROL ACTS
Important effect of Rent Act was that value of all the tenanted properties fell in the market
substantially and it remained at artificially low level for a very long period of time lasting
about 40 to 50 years.

Persons desiring to invest funds in sound securities with fair return, stopped building
houses for rental income as rented immovable property was no more considered sound
security but was treated as liability or diminishing asset.

Rent Control Acts were introduced in different states of India as a temporary measure to
fight with acute housing shortage in urban areas in post-independence periods. But it
remained on status quofor about five decades.

Very recently different governments started rethinking about need for remedial measures
for the damages caused by the Rent Control Act. Even Supreme Court has expressed its
opinion in case of Prabhakaran Nair V/s. State of Tamil Nadu that the laws of landlords and
tenants must be made more rational, humane, certain and capable of being quickly
implemented. In case of M.P. Acharya V/s. Govt. Maharashtra Supreme Court held 1998 to
enact equitable Rent Act.

The new trend or rethinking of having balanced Rent Control Legislation is evident from the
amendments proposed in Rent Act by some governments. Central Government totally
scrapped age old Delhi Rent Control Act of 1958 and introduced and got approval of Delhi
Rent Act, 1995.Government of Maharashtra amended Rent Act by new Rent Act of 1999.
However new Act is far away from equitable provisions.

To understand impact of Rent Control Legislation on property valuation, it is necessary to


study various provisions of Rent Act, old as well as new provisions has to be examined.

Before we examine effect of new provisions, let us first study effect of old provisions
because several old provisions are still remained in amended acts.

Almost of Rent Acts have the provisions for protecting tenants against ejectment from the
premises if tenant regularly pays rent and taxes.

All Rent Acts, till recent amendments also have provision to freeze rent at date of first
letting. Initial rent cannot be increased except permitted tax increases.

If we examine effects of these two important provisions it will enable us to understand


impact of Rent Act on valuation. The combined effect of these two principal provisions, was,

Page | 7
IOV-Registered Valuers Foundation

that land on which the building was constructed never reverted back to the landlord.

Even after expiry of physical life of building, it could not be demolished. If the building
collapsed on its own, even then tenancy rights of tenants did not extinguish but it remained
in force. (Vide Tribhovandas V/s. Chimanlal, Gujrat High Court GLR/1971/556). Landlord
proposing new building on such cleared land was bound by law to give premises to tenants
of collapsed building on rental basis. Similarly tenants of structure washed away due to
flood or collapsed under earthquake had first right as tenants in the new building built on
the plot. (Vide Krishna Laxman V/s. Narsingh Rao, Bombay High Court BLR/1973/29).
Tanants in collapsed building of Gujrat will have similar protection.

From valuation point of view, these two provisions totally extinguished value of land to
owner in all Rent Act affected areas provided the property was fully developed. As there
was no reversion of land back to landlord, there was no reversionary value of land. Land
value was artificially brought to zero due to tenancy and Rent Act. This fact was duly
endorsed by Calcutta High Court in two cases. C.I.T. V/s. Ashima Sinha, 116 I.T.R and C.I.T.
26 and C.I.T. V/s. Anupkumar Kapoor, 125 I.T.R.684 of 1980.

What remained was only frozen rent income from the property and hence under rental
method, only structure is valued on the basis of rental method. Under rental method land
value is adopted at Nil value in Rent Act affected areas.

There is a widespread misconception about rental method and reversionary value of land.
Several valuers now believe that under rental method Reversionary value of land can never
be taken. This is not the correct approach.

This misconception is perhaps due to the views expressed by the Calcutta High Court in
case of C.I.T. V/s. Anupkumar Kapoor (125 I.T.R. 684 of 1980). In that case buildings were
wholly let to tenants and buildings were more than 100 years old. The Court rightly rejected
addition of Reversionary value of land because tenants were protected against ejectment
under Rent Control Act. However, reasons given by the Court for rejection were not proper.
It was stated “Mr. Pal was unable to place before us any authority in support of
Reversionary method of valuation. We are not able to appreciate and understand either the
principle or logic behind this Reversionary method applied by Valuation Officer. He has
taken value of land twice. Once in arriving at figure by yield method and again in applying
reversionary method. This in our view was wholly wrong.”

It is necessary that rental method be understood in its proper perspective. Merely because
Counsel failed to show authority in a particular case to establish theory of reversion does
not mean that such a method or approach is not existent. Before rejecting approach of
reversionary value of land for all purposes, we must examine situations under different Acts.

Page | 8
L&B/Chapter-3/Income Approach to Value

Under Transfer of Property Act, rent is not frozen for ever. It is governed by tenancy
contract between landlord and tenant. If it provides for review after 3 years, rent is frozen
for 3 years and then increased or decreased. There is no life time tenancy under T.P. Act. If
agreed rent is not paid, tenant is ejected. If the building has outlived its physical life, tenant
could be ejected for reconstruction of the building. There is no liability to rehouse old
tenants in the new building. Even otherwise where there is no Rent Control Act, alternative
premises are readily available at the market rent and no one would like to risk his life by
saying in old dilapidated building.

We are not able to visualize these circumstances because provisions and effect of Rent Act
has brainwashed our mind and thinking process. If we see and visualize the situations
prevalent in foreign countries, it becomes very easy to understand this aspect. There,
tenants are easily ejected. Sometimes tenants themselves move out for better
accommodation with more amenities and lower rents elsewhere. No one would take up
premises in very old building even at low rents. Land with vacant old building will thus
automatically vest back to the landlord. It is therefore proper and fair to assume
reversionary value of land under such circumstances.

On the other hand provisions of Rent Act prohibits ejectment and rent increase. It is a case
of life time tenancies and even thereafter. Land never reverts back free to the landlord. It is
therefore proper to take reversionary value of land at Nil value in Rent Act affected
premises.

From economic point of view, the Rent Act converts property asset into a liability. It ignores
physical life of building. Even very old buildings are not permitted to be demolished and
burden of repairs involving prohibitive costs is on landlord. Thus Rent Act makes rented
property a diminishing asset.

A point of caution is necessary while adopting Rental Method for properties not affected by
Rent Act. Valuer will have to be very careful in adopting rate of capitalization for net yield
and also rate of reversion for assessing differed value of land. Valuer will have to be up to
date in knowledge of expected yield in the market in different forms of investment. Little
variation or error in rate of capitalization is likely to give incorrect results.

Let us now consider one by one the impact of various provisions of Rent Control Act
Legislation on valuation of property.

Most important provision of all Rent Acts is provision of prohibition to eject tenants from
the premises.

The prohibition of ejectment of tenants under Rent Act has resulted into unforeseen

Page | 9
IOV-Registered Valuers Foundation

benefits to tenants of under developed properties.

Another provision is freezing of rent forever. The provision of freezing of rent in Rent Act
has benefited tenants tremendously. Due to inflation, prices of all commodities has
increased including salary income and business income but rent remained same resulting in
gain to tenants at the cost of landlords. This provision has also severe impact on valuation
of property. Let us now study impact of some of the new provisions in Rent Control Act. As
per Section 3(1) of Delhi Rent Act 1995, the provisions of Rent Act are not applicable to
premises yielding rent more than Rs.3,500/- per month. Section 3(1)(b) of Maharashtra Rent
Control Bill 1993 provides that Rent Act shall not be applicable to premises let to Public
Limited Companies having paid up share capital of more than rupees one crore. The impact
of these provisions would be that landlord will be permitted to increase contractual rents
after 3 to 5 years period on the basis of market rents prevalent then. Another advantage
would be that the landlord will be able to eject all tenants in building when physical life of
the building is over and it is uneconomic to retain the structure for any longer period. The
value of such properties will involve two components.

1. Capitalize value of net yield on the basis of future flow of rental income i.e. based
on future life of the building.
2. Reversionary value of land also based on future life of the building and present rate
of land prevalent in the locality.

3. These values can be easily worked out as detailed in earlier paras.

In case whether valuation is done by rental method with reversionary value of land, it may
be necessary to cross check valuation by means of adopting land and building method.

Another welcome provision in New Rent Acts is the provision to increase rents of old
existing structures which were frozen for last four to five decades.

From valuation point of view, some of the provisions of amended Rent Act are really worth
study and discussion.

Section 7(1) of Delhi Rent Act provides that Standard Rent of rented premises be calculated
at 10% of the cost of land and building construction. Section 8(2) of Maharashtra Rent
Control Bill provides that Net Standard Rent shall be fixed by the Court on the basis of 15%
return on landlord’s investment in land and building.

First of all having fixed rate of return on investment for all years is not proper. Capital
market in 1990 may not be same as in 1996 or in year 2000. For buildings constructed in
these different years one should not assume that all investors (Landlords) would expect
same return on investment. Again word ‘investment’ would create problems.

Page | 10
L&B/Chapter-3/Income Approach to Value

Secondly, if rent of new building is fixed at 10% yield as per Rent Act and in same year if
Valuer values rented property by capitalizing yield at 12% or 15%, the value of the property
would depreciate in same year immediately after investment. It will have same effect of
keeping lower rent for rented premises than the Standard Rent.

Another important point for consideration is the word ‘investment’ and cost used in
Bombay and Delhi Acts respectively. It is very likely that investment or cost of land to
landlord in many cases may be low due to inherited land or old purchases much prior to
year of development. If land is purchased in year 1980 and plot is developed in 1996,
landlord would be deprived on return on price rise in the market for 16 years. Similarly, if
building is built in 1990 and kept vacant for six years and first let in 1996, he may not be
able to claim rent on the basis of its likely cost in 1996. This point needs discussion from
valuation point of view for fixation of rent.

Section 5 of Delhi Rent Act 1995 has one very unusual and harsh provision about
inheritability of tenancy.

Section 5(1) provides that in the event of death of tenant the tenancy shall pass on to
tenant’s spouse, children or parents only for a period of 10 years from tenant’s death and
thereafter the tenancy shall extinguish.

Section 5(2) provides that if successor owns separate house, he has to surrender the rented
premises to the landlord within one year of tenant’s death.

Section 5(3) provides that in case of non-residential premises the tenancy shall not pass to
successors and it has to be surrendered within one year to tenant’s death to the landlord.

These provisions are harsh and controversial. However, leaving aside social aspect and
fairness of these provision, we have to examine and study it from valuation point of view.

If we examine the provision, two aspects are important from valuation point of view. First is
that on tenant’s death, premises would revert back as vacant premises to landlord within
specified period. Second is probable life span of tenant.

It is generally observed that value of property is more if property is is available on vacant


procession. If in one building all the tenants are of very oldage then its value in the market
will be more than the similar adjoining building having all young tenants. Thus Valuer will
do good to enquire about age of tenants and assess the likelihood of having the vacant
property at time of valuing the property.

This may appear amusing but Valuer may have to do such an inquiry, to value the property.
If tenants age is known, Valuer can easily make an assessment of having vacant possession.

Page | 11
IOV-Registered Valuers Foundation

Valuer then could predict with fair degree of accuracy when rental premises would vest
back to the landlord.

If tenant is saying 80 years age, as per mortality table, his future life span prospects are only
5 years. On death of tenant, after 10 years period, the premises would be required to be
surrendered to the landlord by successors. Hence 15 years from today, premises would be
available vacant to the landlord. Such premises could be valued on the basis of the rental
income for 15 years and adding reversionary value of the premises for 15 years deferment
period. Such a method of valuation may appear to be a wild guess methodology, but time
may come when we may have to resort to such methods of working.

There are some other important amendments also made in the Revised Rent Act. One such
provision is worth mentioning. Section 9 of Maharashtra Rent Bill provides that application
for fixation of Standard Rent should be made within one year of date of occupation of the
premises. Section 13 of Delhi Rent Act1995, provides two years duration for such
application. These provisions would be a boon for stopping unnecessary litigation and
would reduce burden of the Small Causes Courts and Rent Controllers to a great extent.
Misuse of judiciary for personal revenge would stop. From valuation point of view these
provisions has limited effect. Once application period is over Valuer need not work out for
verification whether contractual rent is higher or lower than Standard Rents.

Out of several vital factors affecting rental values of premises following two are important
parameters for serious consideration.

• From Landlords point of view: Expected Returns on investment.

• From Tenants point of view: Paying capacity of tenant.

It will be seen that Rent Act under new amendment form will require great expertise in the
Valuation Methods. New valuation techniques will have to be developed. Valuer cannot
blindly capitalize net rental yields to get value of the property. Effects of various new
provisions will have to be studies by the Valuer before arriving at the final conclusion about
methodology and final value of the property.

LICENSED PROPERTY UNDER THE EASEMENT ACT 1882


The term license is very common in today’s world. Even a layman knows the term and to
him, a license signifies a transaction of some sort between some specific parties. But the
legal and jural meanings are rarely considered. Earlier license was considered as the
landlord’s consent to the licensee to use his land, where the licensee could use the land
without trespass. In modern times, a license can be considered as validation by the licensor
to the licensee to carry out activities on his property, which without entering the property of

Page | 12
L&B/Chapter-3/Income Approach to Value

the owner would be considered illegal.

LICENSE UNDER INDIAN LAW


Under the Indian Law, a license is governed by the Indian Easement Act, 1882. Section 52 of
the Act defines license as a permission by one person to the other or a group of people to
carry out any activity on the property of the grantor, which without such permission from
the grantor would be considered unlawful. Interpreting section 52, it can be said that when
a person is given the right to use a particular property for certain use, while the possession
and control of the property are with the owner, the person will be considered as the
licensee. A license can be granted to only a definite number of people, as a license is a
personal right given to the licensee. Section 56 of the Easement Act also states that a
license cannot be transferred by a licensee or the right under the license be exercise by his
servants or agents. In the judicial pronouncement of Associated Hotels of India Ltd. v RN
Kapoor, the Apex Court gave a definition of a license. The Court stated that when a
document only gives the right to use a certain property in a certain way, it’ll be considered
as a license. The licensee is entitled to use the premises only in a specific manner. Without
the permission, his activities would be considered unlawful. The essential features of a
license can be thus summarized as-

1. A license does not create an interest in the property. It acts only as permission
which created a personal right with regards to the property.

2. A license authorizes certain acts on the property which would be otherwise


unlawful.
3. A license cannot be assigned or transferred to some third party.

TYPES OF LICENSE
License are generally of two types-

1. Bare license

2. License coupled with a grant.

BARE LICENSE
A bare license can be defined as a personal consent which is granted without any
consideration. A bare license can be revoked at any time. A bare license acts as a defense to
the act of trespass. If a person is allowed to enter a property for some particular purpose,
but on entering the property does something else, it’ll be considered as an act of trespass.
When one party grants another party to carry on some activities on the land without
changing the nature of the land, it is a bare license.

Page | 13
IOV-Registered Valuers Foundation

License coupled with grants

A license coupled with grant or interest means that the permission is given to carry on
activities in the property or premises of the other for the purpose of earning some profit
and exploit the interest given in the land. For instance, the government gives license to
contractors to enter a forested area to collect timber. A license coupled with a grant is
assignable, and it also cannot be revoked at any time. The parties must give each other
some reasonable time before the permission is revoked.

Granting of license and Implied & Express license

Under the Indian law, the provisions governing the granting of a license are section 53 and
section 54 of the Easement Act. Section 53 talks about the question of who may grant a
license. Section 54 states that a license may be granted in an express or implied manner.

Section 53 of the Act states that any person can grant a license who in a particular situation
can transfer his interest in the property which is affected by the license. In simpler words a
person cannot grant license rights if he does not have sufficient legal interest in the
property. The power to grant a license co-exists with the power of transferring of interest.
Section 53 also states that a person can grant a license to the extent he can transfer his
interest in the property. Interpreting this part, it can be said that if a person is not the owner
of property, but he has some interest in that property, even he can grant a license to the
extent of his interest. Therefore, even a co-tenant or a mortgagee can grant a license to a
third party. A tenant can also grant a license, but this right is limited only to the extent of
his interest in the property, i.e. the tenancy rights. A tenant cannot transfer the interest
which goes beyond the term of the lease.

Section 54 of the Indian Easement Act states that a grant of license may be express or
implied. It depends on the conduct of the grantor. Many times it so happens that the owner
of a property creates an agreement for easement, but it may turn out to be an implied
license. Thus, the owner of a property should take proper care to differentiate between the
permission he is giving, because due to his behavior the opposite party may get a license
even though a formal agreement was never created. But in such a case, the co-tenant or
mortgagee should be in the sole possession and enjoyment of the property.

Implied License

A license may be an implied license due to the conduct of the owner of property, who may
allow some other person to carry on activities on his property. For instance, a shopkeeper
allows a customer to enter the shop and enter into a transaction with him. An implied
license may also rise due to the reasonable belief of the licensee that the licensor has

Page | 14
L&B/Chapter-3/Income Approach to Value

consented to certain acts on his property.

Express License

In the case of express license, some direct authorization is given to the licensee to carry on
activities on the property. Express license are more specific than implied license, as in the
case of express license specific terms and conditions are mentioned.

Difference between license and lease

The concept of lease is governed by the Transfer of Property Act, 1882. Section 105 of the
Act defines what is meant by a lease. According to the provision lease means a transfer of
the right to enjoy the property, for a fixed time and in lieu of some consideration or price.
The person who transfers the rights is known as the lessor and the person to whom the
rights are transferred is known as the lessee. The question which arises here is that what the
difference between a lease and a license is? In simple words, it can be said that the
difference lies in the intention of the parties, and the fact that whether any exclusive
possession has been given or not. In the case of Section 52 of the Easement Act, no
exclusive possession is given to the opposite party. Therefore, if exclusive possession is
missing, it cannot be a lease and will be considered as a license. The difference between the
two can be summarized in the following points-

1. In the case of a lease, the right to enjoyment is transferred, i.e. exclusive possessory
rights. In the case of a license, a mere permission is granted without any transfer in
interest or rights.
2. A lease can be both heritable and transferable. But on the other hand, a license is
neither heritable not transferable.
3. In the case of a lease, the parties are entitled to any accession or improvement
made to the property. In the case of a license, no such entitlement is there.

4. In the case of a contract of lease, the lessee has the right to protect the possession
in his own name. In the case of a license, the licensee does not have the right to
defend the possession in his own name as no proprietary rights are transferred.

Difference between license and easement

An easement is similar to a license, but easement is a bit trickier. In a case of easement also
there is a consent or permission of the owner of the property. But here the easement holder
gets some interest in the property. For instance, if X grants permission to his neighbor Y to
plant flowers on a patch of land on his property during his lifetime it’ll be considered as an
easement right. So it can be said that where a license is just a personal right, an easement is
a right pertaining to the property. Also, in the case of a license, the licensee gets only right

Page | 15
IOV-Registered Valuers Foundation

in personam. But easement rights are rights in rem and can be enforced against the whole
world. An easement can be both positive and negative in character, but a license cannot be
negative in character.

Leasehold Properties under Transfer of Properties Act 1882.

As per sec. 105 of the Transfer of Property Act 1882, A lease of immovable property is a
transfer of a right to enjoy such property, made for a certain time, express or implied, or in
perpetuity, in consideration of a price paid or promised, or of money, a share of crops,
service or any other thing of value, to be rendered periodically or on specified occasions to
the transferor by the transferee, who accepts the transfer on such terms. Lessor, lessee,
premium and rent defined.— the transferor is called the lessor, the transferee is called the
lessee, the price is called the premium, and the money, share, service or another thing to be
so rendered is called the rent.
Necessity: The lessee needs to ensure that lessor has title to the property being leased.
Further, the lessee should take a declaration from the lessor to ensure that the property is
not encumbered. The lessor needs to take permission from the society or the governing
authority of the building.
Also, the lessee has to understand the terms and conditions under which such permission is
granted. It should be ensured that all previous dues like telephone and electricity bills have
been paid and no bill is due.
The agreement should be signed by the lessor himself or his authorised power of attorney.
The society share certificate or equivalent should be seen. The lessee should verify the
society share certificate in order to ensure that the lessor has the title to the property.
This becomes all the more important as in most transactions the lessee is expected to pay a
security deposit to the lessor.

Important Points of Lease Agreement


1. Facilities to be included for the rent Monthly maintenance and society charges
2. Parking charges
3. Payment of house tax Rent escalation clause
4. Routine repair expenses
5. Fixtures and fittings to be provided by the lessor and their maintenance
6. Grounds for termination of the agreement
7. Notice period to be given by either party to terminate the lease
8. Registration expenses of lease deed
9. Renewing the deed
10. Security deposit payable
11. Interest on the security deposit
12. Advance rent payable Access to common areas
13. It needs to be ensured that the interests of both lessor and lessee are adequately
protected.

Page | 16
L&B/Chapter-3/Income Approach to Value

14. The mutually-agreed terms and conditions should be laid down in black and white.
This will reduce the chances of disputes at a later stage.
15. Unearned Increase.

Essential elements of lease: -


• Immovable property;
• Right to enjoy such immovable property;
• The ascertainable duration of the lease;
• The parties that is the transferor and the transferee who are competent to make
contract;
• The consideration that is premium or rent.

Example on Lease:

Date of commencement of lease 10-Oct-43


Lease rent 240 pa
Date of Valuation 1-Apr-19
Lease period 99.00 years
Used period of lease 75.47 years
Balance period of lease 23.53 years

Rights of lessor
To receive rent up to end of lease period
Reversionary value of land

To receive rent up to end of lease period


Rate of interest for accumulation = 6.50%
Rate of Interest for reversion = 8.50%

Present value of rent = 1,841.72

Reversionary value of land


Plot area = 808 sqm
Rate adopted 130000 per sqm
Market value 105,040,000

Lessors interest 15,411,556

Value to lessor = Present Value of rent + Value of Reversion of land


= 15,413,397

Page | 17
IOV-Registered Valuers Foundation

3. DERIVATION OF YIELD RATE FROM MARKET


DERIVED DATA
Thus, Valuer is sometimes required to find out the amount required to be set aside every
year for a certain number of years to recoup capital which is terminable after some time. In
case when income (lease rent) from leasehold properties (land), ceases with the maturity of
the lease (if there is no renewal clause), it is necessary to allow for recoupment of initial
capital invested by the lessee in addition to return on capital investment. Similarly, in case
of capital investment in building, which is a depreciable asset, it is necessary to provide for
return of capital invested in building and also for recoupment of capital invested
(accumulated sum in form of Annual Sinking Fund Amount).

4. REMUNERATIVE RATE OF INTEREST AND


ACCUMULATIVE RATE OF INTEREST.
The rate of interest adopted for return on capital investment is called 'Remunerative rate of
interest'. The rate of interest adopted for working out Sinking Fund i.e. recoupment fund is
called 'Recoupment rate of interest'. It is also known as 'Accumulative Rate of Interest’.

Remunerative rate of interest (for perpetual income) is always higher than accumulative rate
of interest (interest rate for recoupment of capital invested in property having terminable
income).

Remunerative rate of interest is decided by comparing market yield rates from similar other
sound securities like government security available in the market. Recoupment rate of
interest on the other hand is decided by comparing it with lowest possible maintainable
interest rate. The rate of recoupment should be so selected that at that rate interest income
is guaranteed and said income will not fall even in a very long period of time for the
investment (say 40 to 80 years time period). Hence this rate is generally low and is
compared with interest rates offered by Banks on amounts

YEARS PURCHASE
This term is very frequently used by the Valuers in their day to day calculations and
estimations of values of income fetching properties. Years purchase is defined as capitalised
value required to be paid once and for all, in order to receive annual income of Re. 1 - for
specified period of time at specified rate of return.

Page | 18
L&B/Chapter-3/Income Approach to Value

Years Purchase YP =Capital Value x Rs.1


Rate of Capitalization

5. TYPES OF RENT
LEASE RENT
It is the rent charged by the land owner (Lessor) to the land tenant (Lessee) for use of land
under specified and mutually agreed lease terms and conditions. It can be monthly rent or
annual rent. It is also called 'Ground Rent'. The ground rent may be fixed for the entire
period or may increase at fixed intervals of years or may be reviewed at the time of renewal.
The words lessor, lessee, lease rent are sometimes used for the letting of the built-up
(Building with appurtenant land) premises also. Company lease agreements for office
premises or premises for the commercial use so invariably use these words.

RACK RENT
It is the actual full rental value (Gross Rent) receivable from the property in a year. It may be
rent for land or for both land and buildings.

VIRTUAL RENT
It is the virtual gross rental value to the lessor receivable from the lessee for the leased out
property. In some cases, lessor receives some fixed lump sum amount called premium from
the lessee in advance, in the beginning of the lease itself. This premium amount is nothing
but an advance rental for the property. Thus returns to the lessor are divided in two or
more parts. Gross Rent actually received from lessee during the lease period together with
notional rental value that is receivable on a lump sum amount(premium) received from the
lessee, constitutes total rental or the virtual rent of the property.

Sometimes improvements are done on the property by lessee; this would ultimately belong
to the lessor as per lease agreement. In such a case, notional rental value on such capital
improvement works also becomes a part of the virtual rent. Thus virtual rent is the sum of
the total of actual rent received from lessee during lease period, notional rent receivable on
premium amount and notional rent (annual equivalent) receivable on capital sum invested
by lessee in the property.

HEAD RENT
A head lease is the original lease between a tenant and a landlord. In such a lease, the
overall contractual responsibility is given to one identifiable tenant called the head lessee. It

Page | 19
IOV-Registered Valuers Foundation

is a primary lease under which subleases have been granted. It is also termed as
primary lease or chief lease.

WHAT IS PAGDI SYSTEM?


Pagdi system is alike any other renting system that is prevalent across the world. This
“pagdi-kirayadar” system was launched before independence in order to avoid paying
excessive taxes to the British. Under this system, verbal agreements played a major role in
property transfers where in the tenants were given a slip of rent payment and further
tenants made payments in full to the landlord.

Simply put, the system akin to any other renting system that is prevalent across the world
i.e. landlord and a tenant. In this system the only deviating factor is that the tenant
becomes a part owner of the house and not of the land. This tenant continues to pay rent
to the owner as long as he is not sub-renting the premises. Additionally, the tenant has the
option to sell the said property while giving a percentage of the gross amount to the
owner. This percentage varies from anywhere between 30-50%.

In case of sub renting, the old tenant (now a landlord) and the owner of the property shall
share the rental amount between them, typically at 35:65 ratio. This gives provision for the
owner to make some money off his asset while avoiding taxes. The old tenant benefited,
the new tenant rented a premises with low rent.

To understand the system better, we provide you with an example: In Pagdi system, if an
existing tenant wants to sell his home for Rs 10 lakh, he is compelled to pay anywhere
between 3 to 5 lakh to the owner. Furthermore, the purchaser of the land is said to be a
tenant. Needless to say, he enjoys limited rights because landlord collects minimal rent and
provides the receipts accordingly.

Redevelopment of the property

There are three parties involved when redevelopment of the property is considered –
owner, tenant and builder/developer. As the landlord holds the records of the property, he
possesses complete ownership to sell the property. Once the property is redeveloped by
the builders/developers, the landlord takes the profit out of the property at once and
permanently. Once the profits are withdrawn, the landlord has no authority over the
property. The tenants who are in possession of the said flat/property after the
redevelopment becomes the sole owner of the property.

What’s State Government currently proposing in Maharashtra and why the protests?

Very recently state government proposed to make amendments to Maharashtra Rent

Page | 20
L&B/Chapter-3/Income Approach to Value

Control Act, 1999 – aimed at residential and commercial tenants who reside on properties
above 847 sq ft and 547 sq ft where rent across several Mumbai buildings are controlled by
the Pagdi system. This proposal was also put forth in a bid to phase out a “first generation”
rent control system introduced in 1947 because even today in South Mumbai’s several rent
controlled flats, tenants often pay as little as Rs 300-Rs 500 as a rent even when the market
rate is as high as Rs 20,000-Rs 60,000.

These rent controlled buildings are disheveled and decrepit because these nominal rents
are absolutely failing to motivate landlords to spend on maintenance.

Though most owners and landlords agree that this system is unfair, then tenants who have
occupied large flats in Mumbai are against this State government-proposed latest plans of
phasing out the old system and moving towards Model Tenancy Act proposed by the
Central Government. If the amendment is passed and implemented, then tenants occupying
the properties above 847 sq ft and 547 sq ft will have to pay rents based on the current
market rates which is more than 200 times the rents that they are paying currently.

PROFIT RENT
When Head Lessee subleases a property, he normally charges higher rent than 'head rent'.
This increased rental is called Improved Rent. The difference between head rent and
improved rent is called Profit Rent. If 'A' leases property to 'B' for Rs. 2,000/month and 'B' in
turn subleases to 'C' for rent of Rs. 3,000/month, Rs. 2,000 is head rent, Rs. 3,000 is
improved rent and difference i.e. Rs. 1,000 is profit rent.

CONTRACTUAL RENT
It is the rent mutually agreed between the landlord and the tenant under the tenancy
contract, which may be written or verbal. The rent may be for use of land alone or for land
and buildings together. This rent mayor may not include property taxes and other
outgoings. Tenancy contract would fix the agreed norms and conditions.

STANDARD RENT
The concept of rent is established under the Transfer of Property Act that governs and
regulates relationship between landlord and tenant. However, since independence oflndia
and due to the post World War situations in our country, most of the states thought fit to
bring out Rent Restriction Act for protection to tenants against exploitation and eviction.
Almost all rent acts has a provision of controlling rent which is nothing but freezing of rent
at the level as on the date of first letting. This rent can never be increased even after several

Page | 21
IOV-Registered Valuers Foundation

years. This freezing of rent and protection against eviction under Rent Acts were contrary to
provisions of Transfer of Property Tax.

Hence, these Rent Control Acts are treated by the courts as special legislation superior to
Transfer of Property Act. Under the Rent Control Acts, the norms for working out fair rent
payable by the tenant to the landlord were fixed. This is called as Standard Rent. Standard
Rent can be defined as the rent fixed by the Court for land or land with building (premises)
in accordance with the provisions of the Rent Control Act. Most of these Rent Acts defines
the terms Standard Rent and also clarifies what it includes and what it excludes under
different situations. The landlord cannot charge or receive from the tenant, any amount in
excess of this legal rent i.e. Standard Rent.

MARKET RENT
It is the highest rent that is receivable for the property, by the landlord, in the open market
after considering all advantages as well as disadvantages of the property, as well as market
conditions (demand and supply), in the prudent manner. It can be more than Standard Rent
and in some cases may be even lower than Standard Rent.

CONCESSIONAL RENT
When landlord gives premises on rent to some relatives or friend at a token or nominal rent
which is much below ruling rent in the locality, it is called concessional rent. It is always
lower than the standard rent and market rent of the premises.

MONOPOLY RENT
Some property has unique location in the locality. This advantage of location can be
exploited by charging monopoly rent to the occupant. This rent is normally higher than
ruling market rent in the locality. The only hotel in front of railway station or only single
office building in residential zone is the example of monopoly situation.

There are many other types of rents such as Situation rent, Intangible rent, sitting rent,
nominal rent, etc.

Page | 22
L&B/Chapter-3/Income Approach to Value

6. DETERMINATION OF MARKET RENT &


STANDARD RENT
Rent is the payment made by the tenant to the land lord for the use of land and/or
building. This is normally referred as contractual rent. Under transfer of property act 1882,
the term rent means money payable to land lord by a tenant for the use and occupation of
the premises.

Market Rent - Market rent is the amount a landlord might reasonably expect to receive,
and a tenant might reasonably expect to pay, for a tenancy. It needs to be similar to the
rent charged for similar properties in similar areas. Market rent is a useful guide for
landlords when they're deciding what the rent will be.

Gross Rent - Gross rent is the amount of rent payable periodically including all out goings.

Rack Rent – It is an excessive or extortionate rent, or the full rent of a property, including
both land and improvements if it were subject to an immediate open market rental review.

Virtual rent – Virtual rent is also the full rental value of the premises. It may be classed as a
rack rent. Virtual rent is the rent received under a lease plus the rental equivalent of any
‘premium’ and improvements made to the premises per annum.

Virtual rent = Annual equivalent + Rent actually paid + cost of improvements (if any)

Standard Rent - It is the rent which the land lord is expected to receive from the tenant.
Standard rent is only provided if the property is under Rent Control Legislation. If the rent
control act is applicable in the area then standard rent will be determinable under the
provisions of the act. Standard rent can be determined either by using the method of
comparison or on the basis of the investment theory. In the method of comparison, the rent
is compared with other similar properties. This method may or may not yield proper results
depending on circumstances of the comparable properties. Comparable, in fact, may be
difficult to come by and unless a detailed objective analysis is done, considering situation,
amenities, area, nature of construction, and the like, it would be difficult to arrive at the
correct results. The method is summarized below:

1. Present value of the land is estimated.


2. Depreciated cost of construction is estimated taking account of depreciation,
obsolescence etc
3. Capital value of property is obtained by adding the above two items.

Page | 23
IOV-Registered Valuers Foundation

4. Rate of return, as provided in the appropriate Premises Tenancy Act (or other Acts),
on the value of land and cost of construction should be applied. This is the net
yearly return.
5. Add to this return, all outgoings, like repairs, insurance, collection charges, taxes,
sinking fund etc. This is the gross rent that can be realized by the owner of the
property.
6. Any amount paid by the tenant for municipal taxes etc must be deducted from the
gross rent.

Example –

Estimate the reasonable standard rate for a newly constructed house property in a co-
operative society. The land was sold on ownership basis.

1. Land cost paid to society = Rs. 2,00,000/-

2. Building Cost = Rs. 6,50,000/-


3. Amount spend for amenities = Rs. 60,000/-

4. Period of construction = 2 years

5. Society membership deposit = Rs. 5,000/- (refundable)

6. Service charges of society = Rs. 1,500 per month

Solution –

Return on land @ 8% = Rs 2,00,000 X 8%

= Rs. 16,000/-

Cost of construction = Rs. 6,50,000/-

Amount for amenities = Rs. 60,000/-

Rs. 7,10,000/-

Interest on above for half period of construction = 9% for 1 year on Rs. 7,10,000
= Rs. 63,900/-

------------------

Total cost of construction = Rs. 7,73,900/-

9% return on total cost of Rs. 7,73,900 = Rs. 69,651/-

Repairs and maintenance charges @1% = 90% X Rs. 7,73,900 X 1%

Page | 24
L&B/Chapter-3/Income Approach to Value

= Rs. 6,965/-

Insurance @0.25% = 90% X Rs. 7,73,900 X0.25% = Rs. 1,741/-

Sinking fund @4% on 90% of Rs. 7,73,900 for 80 years = Rs. 1,264/-

Total = Rs. 95,621/-

Add municipal taxes @25% of above = Rs. 23,905/-

Service charges of society @ 1,500 per month = Rs. 18,000/-

Interest on capital blocked with society @8% = Rs. 400/-

Total Standard rent = Rs. 1,37,926 per annum

7. LEASE LESSOR& LESSEE TYPES OF LEASE,


LEASE PROVISIONS & COVENANTS
LEASE-HOLD INTEREST
A lease-hold interest is usually subject to the payment of annual rent and to observe terms
and conditions i.e. covenants contained in the lease agreement. The person who grants
lease is called "lessor" and the person who takes property on lease is called "lessee". The
covenants of lease usually include three main conditions amongst other conditions,
regarding payment of rent reserved under the lease, regarding regularly carrying out annual
repairs and maintenance and manner in which the property reverts to lessor on expiry of
lease period. A lease where lessee undertakes to carry out all the repairs and to bear all
outgoings is known as "Full repairing lease".

TYPES OF LEASES
There are four principal types of leases in practice.

BUILDING LEASE
These leases are in respect of lands ripe for construction of buildings. In
such a lease, lessee undertakes -

1. to pay annual ground rent,

2. to construct building or buildings on the land,

3. to keep the buildings in good state of repairs and maintenance during the period of

Page | 25
IOV-Registered Valuers Foundation

lease and pay all outgoings, and

4. on expiry of lease term either pull down buildings and revert the land to its original
state or to handover possession of land with buildings thereon to the lessor.

5. These leases are usually for long term e.g. 99 years.

OCCUPATION LEASE
These are leases where land and buildings are leased out for occupation by the lessee. The
term of lease normally varies according to type of property and leases are usually for short-
term or for medium term. In the case of medium or long term leases, there is usually a
condition regarding periodic revision or review of lease rent.

SUBLEASE
Subject to the terms and conditions of head lease, a lessee may sub-lease the property for
any term less (at least by a day) than his own lease term. Sub-lease may be building sub-
lease or occupation sub-lease.

LEASE FOR LIFE


It is the lease granted for the duration of the life of the lessee or some other person. The
period of such a lease is uncertain and can be considered as may be indicated by mortality
tables or life expectancy tables. State Government, local authorities and Semi Government
agencies many a time give land for development to individuals or to entrepreneurs or to a
company by creating long lease and by charging initial full premium representing full
market value of land at a nominal lease rent of Re.1/- per year. Keeping in view long term
public interest, and in view of desire for development of under developed area, such lands
are sometimes given on long lease charging premium at less than the market value of land.
Such leases invariably consist of a clause of payment of unearned increase, in case of sale or
transfer of lease-hold interest in land. If lessee earns profit on land value, by way of such
sale, the lessor (Government) or leasing agency would have a share in profit (say 50% of
unearned increase in value of the property over initial premium payments, as per
Government Policy). This charge by the lessor, in case of sale or assignment of the lease, is
called payment for unearned increase. It is a charge on the property. In such leases, the
premium charged represents market value of land with or without concession and the lease
rent is nominal. Such leases are virtually grant of land but leases for names' sake for
observing polity of disposal of land by Government or by Planning and Development
authorities.

Page | 26
L&B/Chapter-3/Income Approach to Value

LEASE PROVISIONS AND COVENANTS


There are two parties of lease - lessor and lessee. Each has certain obligations towards each
other and society in general. Certain terms are agreed between two parties which are called
as covenants.

LESSORS’ BASIC OBLIGATIONS.

a) Covenant of quiet enjoyment.

The covenant of quiet enjoyment assures Lessees that they will have peaceful possession of
the property, free of interference from the acts or claims of the lessor or certain third
parties claiming rights through the lessor. The Lessees are protected from title disputes
between the lessor and anyone claiming superior title. Generally, even without a specific
covenant, quiet enjoyment is implied in any lessor/Lessee relationship. Condemnation of
the leased premises is generally not a violation of the lessor’s covenant of quiet enjoyment.
Furthermore, a covenant of quiet enjoyment does not require the lessor to repair leased
premises.

b) covenant of possession

Provided the Lessee is not in default, the covenant of possession gives the Lessee exclusive
possession and control of the premises. An lessor cannot enter the Lessee’s property unless
the lessor reserves specific entry rights in the lease. Ordinarily, the lease states that the
lessor can come onto the property to accomplish lessor obligations, such as to complete
any required repairs, without disturbing the Lessee’s business. In addition, the lessor may
specify the right to enter the premises near the end of the term to show the premises to
potential Lessees. Without these kinds of specifically drafted exceptions, the landlord
usually has no right to enter the premises, make repairs, or curtail a nuisance.

c) constructive eviction

Constructive eviction enables the Lessee to break the lease if the lessor interferes so much
with the Lessee’s use of the premises—such as failing to provide utilities—that the Lessee is
forced to leave.

d) warranty of fitness

A warranty of fitness ensures that the leased premises are suitable for the purposes
anticipated by the Lessee. If this warranty is included in the lease by the lessor, then its

Page | 27
IOV-Registered Valuers Foundation

application becomes a matter of contract interpretation. Unlike a warranty of habitability,


which may be implied in residential leases, a majority of jurisdictions have found that there
is no implied warranty of fitness in leases of property for commercial purposes.

LESSEES’ OBLIGATIONS.

a) Paying the Rent

One of the hallmarks of a lessor- lessee relationship is the obligation of the lessee to pay
rent by a certain interval. Many long leases also include provisions that increase rent at
certain intervals to adjust with the inflation.

b) Not to disturb other Lessees.

Not disturbing other Lessees means a Lessee’s business should not cause any problems for
neighbors, such as noise, nuisances, clutter in common areas etc.

c) Care for repair

A lessee’s care and repair duties depend on the terms negotiated in the lease and on the
nature of the leased property. Under a long-term lease for an entire building, the duty to
repair usually falls to the Lessee because the Lessee is in the best position to do the repairs.
On the other hand, in a multi-Lessee building, a landlord is usually obligated to make
repairs to the common areas, roof, and building exterior.

LEASE TERMINATION
The normal ways of termination of the leases area

1. Expiry of the term

A lease expires automatically when its term expires i.e. a lease of a term of 30 years
expires at the end of the 30 years from the start of the term

2. Disclaimer

Disclaimer occurs in cases of insolvency. A tenant’s liquidator or trustee in


bankruptcy may disclaim a lease if it is considered to give rise to a liability to pay
money or perform any onerous act or if the property is not readily saleable
(pursuant to insolvency statute law). A disclaimer extinguishes the lease and
releases the tenant from any further liability.
3. Enlargement

Page | 28
L&B/Chapter-3/Income Approach to Value

This is extremely rare. A lease granted for over 300 years, with at least 200 years to
run, may be “enlarged” by a tenant into a freehold estate.
4. Exercising a “break clause”

Break clauses are usually found in leases of commercial (as opposed to residential)
premises and provide for either party to determine the lease at one or more
intervals before the expiry of the term (e.g. at 3 yearly intervals or at a fixed date).

Premature termination of lease or surrender of lease:

1. Forfeiture

The right to forfeit a lease in the event of a tenant’s breach of covenant is usually expressly
provided for in the lease. The exercise of the right is generally subject to restrictions
implied in the law (e.g. in the case of breach of repairing covenants in a commercial lease).

2. Frustration

This is rare. A lease may be discharged by frustration. It occurs where the frustrating event
brings about a sufficiently fundamental change in circumstances as to justify treating the
lease as at an end.

3. Leasehold and enfranchisement


Certain statutory provisions enable a tenant to purchase the freehold reversion (and
thereby merge the lease bringing it to an end)
4. Merger

If a tenant acquires the landlord’s freehold interest, the lease “merges” into the freehold
and is extinguished.
5. Notice to quit
A periodic tenancy (as opposed to a lease for a fixed term) may be terminated by notice to
quit served by either party. The notice must be served and provide for termination on a
date at the end of one of those periods for which rent is payable. For example, a monthly
periodic tenancy is terminable on one month’s notice. In residential premises there are
additional statutory restrictions for the Notice to be enforceable.
6. Repudiatory breach

This is rare. Where circumstances are extreme, the court may rule that a breach of a
fundamental provision in a lease entitles the innocent party either landlord or tenant to the
lease to treat the lease as terminated (in addition being able to sue for damages).

7. Statutory termination

In some cases a specific statutory provision will operate to terminate a lease e.g. under
certain laws relating to residential property.
8. Surrender

Page | 29
IOV-Registered Valuers Foundation

A surrender can be by deed or be implied (by operation of law) for example by the tenant
removing all their possessions and vacating the property and returning the keys to the
landlord. Any surrender is only effective if it is accepted by the landlord. If not, the lease
continues.

Lessor's Interest
1. Lessor or freeholder has right
2. to receive lease rent fixed under the lease deed for the unexpired period of lease
and
3. to receive the property after expiry of lease period when the property reverts to
lessor. This second component of lessor's interest when the property reverts back to
the lessor is known as Reversionary Value as the property reverts to lessor after
the expiry of lease period.

The lessor has right to receive rent reserved under lease deed for the
unexpired period of lease. Where the rent reserved under the lease is well below the fair
market rent, it is most secured and hence remunerative rate for capitalizing rent reserved is
estimated at Y:z% to 1 % lower than the normal rate of interest corresponding to security
offered by property. If the difference between the market rent and rent reserved is less, still
the rent reserved isore secured than the market rent and the remunerative rate for
capitalizing such reserved rent is capitalized at about ~% lower than the normal rate of
interest corresponding to security offered by the property.

The value of the lessor's interest will therefore be the sum of -

1. Capitalized Value of rent reserved under the lease, at appropriate single rate of
interest for unexpired period of lease and
2. Right of reversion of property after expiry of lease period.

3. The presumption is often made that the lessor's interest plus lessee's interest add to
the fair market value of freehold property and that the fair market value of freehold
property can be apportioned into lessor's and lessee's interest in some fixed
proportion. This proposition is totally incorrect and wrong.

Lessee's interest

In the case of lease-hold properties, the lessee has a right to enjoy the difference between
the market rent and the rent reserved under the lease which is known as profit rent, for the
unexpired period of lease. The market value of lessee's interest is therefore estimated by
capitalizing profit rent for the unexpired period of lease. The accumulative rate of interest is
usually, taken at 2% to 3%. The remunerative rate of interest for capitalizing lessees profit
rent is usually taken about 1 % higher than the remunerative rate considered for
capitalizing freeholder's (lessor's) net income corresponding to security offered by the

Page | 30
L&B/Chapter-3/Income Approach to Value

property since the lessee's interest in a property is considered to be less secured than the
lessor's interest Upright.

Value on Reversion: Reversionary Value

In the case of lease-hold properties held on building lease or occupancy lease, the lessee's
interest being limited to enjoy the benefits from the land/property only during he period
fixed under the lease, ceases on expiry of the lease period and the property with or without
improvements or buildings reverts back to the lessor as per terms of lease.

Once the property reverts to lessor he has option to give it on further lease, keep vacant or
to use for self-occupation. Thus, lessor has right to derive benefits in perpetuity as the
owner of the property.

The market value of this right of the lessor (to get back the property) is estimated by
deferring market value of property as freehold for unexpired period of lease.

Let's consider a case of a property having unexpired period of lease as 15 years. Market
value of property in vacant possession to a freeholder on the date of valuation is say ~ 1 cr.
The lessor will get back the property after the expiry of lease period of 15 years. Therefore,
value of right of reversion will be ~ 1 cr. deferred for 15 years. The value of such reversion
of the property depends on the unexpired period of lease and in case the unexpired period
is too long i.e.50 years or more the capital value on reversion being deferred for longer
period becomes almost negligible, but still it is not zero.

8. VALUATION OF LESSOR'S INTEREST AND


LESSEE'S INTEREST
The simple case of valuation of the interests of lessor and lessee is where lease has been
granted at certain lease rent without charging any premium and has some unexpired period
of lease. It has been already stated earlier that the Lessor's interest comprises of two parts
viz. (i) to receive the rent reserved under the lease for unexpired period of lease and (ii)
reversion to land or land and building, as per terms and conditions of lease, after expiry of
lease. The lessee's interest is to receive profit rent (i.e. market rent - lease rent) for the
unexpired period of lease. The following examples will show how the values of lessor's and
lessee's interests can be estimated.

Where the rent reserved under this type of lease is the full rental value of land and
buildings, it is known as 'market rent' or 'rack rent' and where it is less than the full rental
value it is known as a 'head rent'. The difference between the rack/full rental value and the

Page | 31
IOV-Registered Valuers Foundation

head rent (such difference is known as 'Profit rent')when capitalized represents the value of
lessees interest.

Valuation of freeholds let out at full rental value

It has already been discussed that the valuation model V = Y. P. x net income presupposes
that the income lasts in perpetuity. This assumption holds good for a freehold property
income from which is of perpetual nature. It has also been discussed that even though the
building standing on the land does not last forever the argument that the income is
perpetual is correct. The reasons for this has been shown at the beginning of this discussion
on investment method on income approach of valuation. So, when a freehold property is let
out at full rental value or capable of being let out at full rental value it can be valued by the
model V = Y. P. x net income

It has also been shown that when the income received from the property is gross the
landlord is to undertake some expenditure collectively known as outgoings to arrive at the
net income. The net income is to be multiplied by Y.P. to arrive at the capital value. The
expression Y.P. stands for (100/yield). So the selection of yield is very crucial for the
valuation under the income capitalisation method.

It has also been discussed that greater the security of the income, lesser is the yield
acceptable. For a property situated in a first-class location, the initial rent rises at a very fast
rate. When the rate of rental growth is above average rate, the income is highly secured
and invites a low yield for capitalisation. In such locations where the rate of growth of
income is slow or even it may fall in the long run, the rate of capitalization or the yield is to
be taken as high.

We shall now proceed with actual valuation of freehold properties.

Example

A freehold shop property in a first-class location is worth a rent of INR200,000 per annum
(gross). What is the market value of the property?

Solution

Gross Rent = INR 200,000 P.A.

Less, outgoings:

Repairs @ 10% = INR 20,000

Management @ 5% = INR 10,000

Property Tax, say 15% = INR 30,000

Page | 32
L&B/Chapter-3/Income Approach to Value

Insurance @ 1% = INR 2,000 = INR 62,000 P.A.

Net Income = INR 138,000 P.A.

Y.P. in perpetuity at 4% = INR 3,450,000

NB: The yield for capitalisation has been taken very low at 4% because the property is in a
first-class location.

Valuation offreeholds subject to leases let out at below full rental value

When a freehold property is subject to a lease, then the leaseholder occupies the property
for a period depending on the tenure of the lease called the "term". After the term is over,
the leaseholder vacates the property and the property returns to the freeholder. This right
of receiving back the property at the end of the lease is called the "reversion". The valuation
is done in two stages. First of all the term is valued. And after that, the reversion is valued.
The valuation method is called the term and reversion valuation.

The rate of capitalisation of the term depends upon the length of the term and whether the
term rent under the lease is a fixed one for a long period. If the term rent is fixed for a long
period more than the normal review period (say 5 years) then it is to be capitalised at a
relatively high rate of interest. This is because the term rent though low yet being fixed is
hit by inflation and is not secured in real terms. However, if the term rent is fixed for a short
period (say 5 years) then it can be capitalised at a low yield. In such cases usually the term
and the reversion rents are capitalised at the same rate of interest.

After capitalising the term, the reversion is valued. The rent of the reversion is taken as the
current rental value and not the rental value that can be predicted at the end of the term.
The reversion is valued in perpetuity choosing an appropriate yield. Since the reversion
occurs after expiry of the term, it is to be multiplied by present value of INRl at the same
rate percent as the yield for capitalisation of the reversion to bring it at today's value.

Valuation of leasehold Interests

Leasehold interests are not so lucrative investments as the freeholds. Generally, the lessee is
restricted in his way of dealing with the property by restrictive covenants in a lease deed in
a number of ways. Some examples of such restrictions are not to sublet, not to assign, not
to redevelop, not to make any change of use etc. Further, in case the property is sublet and
if the subtenant fails to carry out any lease obligation the lessee remains liable for it to the
freeholder. Also, it is very difficult to dispose of the fag end of leases. Again, due to inflation
the provision for sinking fund made under a leasehold interest may fall short of the
targetamount required for redemption of capital. For all these reasons, leasehold interests

Page | 33
IOV-Registered Valuers Foundation

are considered less attractive thanthe freehold interests and the investment return of
leasehold may be taken at 1% higher than its freehold counterpart.

The leasehold interest is of limited duration and is not perpetual like the freehold interest.
As has already been discussed, provision for a sinking fund is made forvaluation of the
leasehold so that the capital value of the lease can be redeemed at the end of the lease
period. In this way, the leasehold income may be perpetuated and the investment return
can be compared with that of the freeholds. Leasehold interest is therefore valued at dual
rate of interest. One is a return of capital (calledthe remunerative rate of interest). Another
is a rate of capitalisation of the sinking fund (called accumulative rate of interest). The latter
is muchlower than the former rate varying between say about 1% to 4% only.

The difference between the rent at which the leaseholder sublets the property and the rent
which he pays to the freeholder is called the profit rent. The value of a leasehold is the
capitalised amount of the net profit rent. The value of the lease-holdis positive if the net
profit rent is positive.

Example

Mr. Bose has granted a lease of a shop premises to Mr. Sen at a rent of INR150,000 per
annum all burden of outgoings being shouldered by Mr. Sen. The lease has today5years to
run. The current full rental value of the property is INR300,000 per annum the tenant
undertaking all burden of outgoings. Considering the freehold yield at 4%on shop property
in this location what is the value of the lessee's (Mr. Sen's) interest?

Solution
Valuation of lessee's interest
Full rental value = INR300,000 per annum (net)
Less, rent paid by the lessee = INR150,000 per annum
Profit rent = INR150,000 per annum (net)
V.P. 5 years at 5% and 2)12% = 4.162
Value = INR624,300

Therefore, valuation of lessee's interest = INR624,300

NB: The leasehold yield has been taken at 1% higher than the freehold yield.

Leasehold property subject to a sublease Example

X has granted lease of a residential property to Va few years back which has still 50
yearsunexpired. The ground rent paid by V to X is INR10,000 per annum. V has sublet the
propertyto Z on INR20,000 per annum on a full repairing and insurance leasehas only 4

Page | 34
L&B/Chapter-3/Income Approach to Value

yearsunexpired. The full rental value of the property today is INR100,000 per annum the
tenant bearing burden of all outgoings. What is the value of V's (lessee's) interest? Freehold
residential property yield is 7% in this location.

Solution
Valuation of lessee’s interest
Term
Rent reversed to Z = INR 20,000 Per annum (net)
Less Ground rent paid by Y = INR 10, 000 Per annum
Y’s profit rent = INR 10,000 Per annum (Net)
Y.P. 4 years at 8% and 2.5% = 3.117
Value
= INR 31,170
Leasehold reversion

Rental to F.R.V. = INR 100,000 Per annum (net)


Less ground Rent = INR 10,000 per annum
Y’s profit rent = INR 90,000 per annum
Y.P. 46 years at 8% and 2.5% = 10.890
Value = INR 980, 100
P.V. of INR 1 @ 8% in 4 yrs = 0.7350
Value = INR 720,374

Therefore, term + leasehold reversion = INR31,170 + INR720,374 = INR751,544 which is the


value of V's (lessee's) interest.

Examples may be added later

9. YIELD FROM REAL ESTATE AGAINST OTHER


FORMS OF INVESTMENT
There are a lot of ways to invest the money. Like from real estate, stocks, Bonds/Bank Fixed
Deposits, CDs or even gold. The burden of placing your money in someone else’s hands is
not something to take lightly. At what point does one investment option become more
attractive than another? By examining real estate vs other investments one can help
determine which options are better at given time.

Deciding which investments are right for a portfolio will depend on a number of factors: the
value, risk tolerance, the potential return on investment, and the amount of control
involved. With any investment opportunity, there will be tradeoffs; however, real estate

Page | 35
IOV-Registered Valuers Foundation

represents a strong investment opportunity for those seeking to increase cash flow, save for
retirement, and build a strong financial portfolio.

In India for a long time real estate is popular mode of investment giving best returns for a
long term. The study suggests real estate has historically been a strong investment
opportunity for investors. This is because real estate allows investors to profit in more ways
than one. First, by purchasing real estate a person invests in physical asset that will
appreciate over time. Real estate also provides the opportunity to generate monthly cash
flow through rental income.

Aside from the potential for residential and commercial real estate returns, real estate
investing also offers the benefit of being relatively easy to break into. Most individuals grow
up somewhat familiar with the real estate market, and will have a strong understanding of
real estate when compared to the stock market or other investment opportunities

However, the real estate may take a long time to liquidate. Hence quick availability of funds
may not be possible as in the case of shares or gold.

REAL ESTATE VS STOCKS

Historically, stocks tend to increase both in profits and cash dividends over time. Investors,
therefore, have the chance to earn increased profits from stocks annually if the company
proves profitable enough. Investing in stocks also provides the chance to create a diverse
portfolio. By investing in mutual funds, investors can buy stock in multiple companies at a
time, which enables investors to spread out their investment capital and lessen the overall
amount of risk they take on.

While the benefits of investing in stocks are hard to ignore, there are a few tradeoffs
involved. The most obvious dilemma for investors is that stocks are unpredictable. Due to
market volatility, the price of stocks can fluctuate dramatically in a given period of time. For
some investors, watching their profits grow and shrink rapidly throughout the year can be
frustrating. In comparison, real estate represents a more stable investment opportunity for
investors unwilling to participate in risky nature of the stock market.

A final thought when considering the stock market vs real estate is the amount of
knowledge required to get started. The average person is unfamiliar with the inner workings
of the stock market, and while that is perfectly acceptable, it can act as a barrier to entry.
Many investors will find themselves at the hands of stock brokers or financial advisors. For
those who prefer to have more control over their portfolio, stocks can represent a
challenge.

Page | 36
L&B/Chapter-3/Income Approach to Value

REAL ESTATE VS BONDS/BANK FIXED DEPOSITS / BANK FIXED


DEPOSITS

There are three main categories of Bonds investors can work with: corporate, municipal and
government Bonds along with the fourth bank deposits. Investors earn revenue from
Bonds/Bank Fixed Deposits/deposits through interest as they come to maturity. One of the
biggest perks of investing in Bonds/Bank Fixed Deposits is that they are low risk. While
interest rates can fluctuate, Bonds/Bank Fixed Deposits are often considered more reliable
than other investment opportunities.

Despite their low risk nature, investors may find Bonds/Bank Fixed Deposits do not offer the
same profitability when compared to other investments. In many cases, the returns will be
significantly impacted by rate of inflation. For example, if you are earning three percent and
the inflation rate is one percent, your returns will have dropped by one third. On the other
hand, real estate typically benefits during times of inflation. As material and labor costs go
up, rent traditionally follows suit, leading real estate investors to realize higher profits
during these times.

While Bonds/Bank Fixed Deposits present a straightforward, relatively low-risk investment


opportunity, they may not offer the best returns when compared to real estate. Across the
board, rental properties typically outperform Bonds/Bank Fixed Deposits because of their
overall ability to generate cash flow, even during times of inflation or low interest rates.
However, just because there are disparities in the profit potential does not mean investors
should entirely rule out this option. For investors seeking to diversify an existing portfolio,
Bonds/Bank Fixed Deposits can provide a stable opportunity to do just that.

REAL ESTATE VS GOLD

Investing in gold is a simple process that works like this: Investors buy the precious metal,
and earn a profit only after selling it (once the gold has appreciated in value). While this
process sounds straightforward, it is truly a waiting game, especially when compared to real
estate investing. Real estate allows the opportunity to earn revenue while waiting on an
asset to appreciate in value, resulting in more cash flow overall.

As you examine real estate vs other investment returns, gold may not be the first thought
that comes to mind. It is often not the most profitable, nor the most popular investment
opportunity. There is something to be said about investing in gold; although, investors
looking for an asset to focus on may not find the returns they are hoping for by purchasing
gold.

Page | 37
IOV-Registered Valuers Foundation

Real Estate Capital Appreciation


Regular rental income
Liquidity time consuming
Low risk, high investment
Stocks Capital appreciation
Dividend income
Easy liquidity
High risk, needs study
Bonds / fixed deposits Safety
Regular term income
Easy liquidity
Low risk, low profits
Gold Capital appreciation
No regular return
Easy liquidity
Low profits

Real estate investing involves "the purchase, ownership, management, rental and/or sale
of real estate for profit Improvement of realty property as part of a real estate investment
strategy is generally considered to be a sub-specialty of real estate investing called real
estate development. Real estate is an asset form with limited liquidity relative to other
investments, it is also capital intensive (although capital may be gained through
mortgageleverage) and is highly cash flow dependent. If these factors are not well
understood and managed by the investor, real estate becomes a risky investment. The
primary cause of investment failure for real estate is that the investor goes into negative
cash flow for a period of time that is not sustainable, often forcing them to resell the
property at a loss or go into insolvency. A similar practice known as flipping is another
reason for failure as the nature of the investment is often associated with short term profit
with less effort.

ROLE OF REAL PROPERTY IN THE INVESTMENT MARKET

In the investment market there are always a number of alternative opportunities for
investment. The investor who would prefer capital gain from the investment would risk
investing his capital in the equity shares of first-class companies whereas the investor who
would prefer safe and secure income from the investment would like to investment in long-
term government securities. Between the two extremes of highly risky equity share
investment and safe and secure government securities however there are a number of
alternative opportunities with different degrees of risk and security. Real property is one
such vehicle of investment in the totality of the investment market scenario. The role played

Page | 38
L&B/Chapter-3/Income Approach to Value

by real property is dependent on the various aspects of real property as a channel of


investment. We discuss them as follows.

MANAGEMENT ASPECT

Real property investment is always associated with trouble and cost of management. Thus
for management of real property the services of a lawyer, an architect, a Valuer or a real
estate agent are necessary from time to time. Further, there may be involvement in
litigations day to day involvement in enforcing lease covenants, collection of rents etc, all
parts of hazards of management. Comparatively, investment in equity share is less
troublesome and costly to manage. Therefore, in order to induce an investor in the
investment in real property there must be an incentive of higher rate of return than the
yield from equity share investment.

Again, the investor can invest in long-term government securities to obtain a risk-free long-
term income. Against this, there is the prospect of rental growth and appreciation of capital
value as the incentive for investment in real property in spite of the hazards of
management.

LEGAL ASPECT

Law interferes with real estate investment in a number of ways. Thus, rent control acts may
severely affect the prospect of rental growth of the property. Further, taxation laws may
affect real property investment. Examples are property tax, capital gains tax, betterment levy
etc. Again, town planning acts may adversely affect real property investment through
restrictions imposed on land uses, density etc. As a result the investors should expect higher
yield from real property investment to compensate against this.

LIQUIDITY

Real property is an illiquid asset. The time taken for real estate disposal is a lengthy process.
Further, for transfer of property one may have to incur cost on lawyers' fees, stamp duty etc.
Comparatively, the time taken for disposal of shares or cost of transfer of share is much less
than that of real property. Hence, in order to induce an investor to invest in real property
one must expect to receive a higher yield.

SIZE OF INVESTMENT

Real property is indivisible into very small components. Generally, a huge amount of capital
is associated with an investment in real property. Comparatively, a much smaller amount of

Page | 39
IOV-Registered Valuers Foundation

capital is required in order to invest in shares. To compensate against this the real property
investment should have higher yield.

LIQUIDITY

Real property is an illiquid asset. The time taken for real estate disposal is a lengthy process.
Further, for transfer of property one may have to incur cost on lawyers' fees, stamp duty etc.
Comparatively, the time taken for disposal of shares or cost of transfer of share is much less
than that of real property. Hence, in order to induce an investor to invest in real property
one must expect to receive a higher yield.

SIZE OF INVESTMENT

Real property is indivisible into very small components. Generally, a huge amount of
capital is associated with an investment in real property. Comparatively, a much smaller
amount of capital is required in order to invest in shares. To compensate against this the
real property investment should have higher yield.

LACK OF CENTRAL MARKET

There is no central market for real property for the stock exchange market. Furthermore,
the transactions in real property are often of closed type and one is required to incur
a high amount of expenditure to have full knowledge of the transaction. These add to
the market imperfection of real property market and have the effect of increasing the yield
from real property investment.

10. INVESTMENT DECISIONS: DISCOUNTED


CASH FLOW TECHNIQUES, INTERNAL RATE
OF RETURN (IRR) & NET PRESENT VALUE
There are several criteria that have been suggested by economists, accountants and others
to judge the worthwhileness of the capital investments. In this topic we will discuss the two
discounted cash flow techniques viz. Net Present Value (NPV) & Internal Rate of Return.

Page | 40
L&B/Chapter-3/Income Approach to Value

Investment Criteria

Discounting Criteria Non Discounting Criteria

Net Present Value Payback Period

Internal Rate of
Accounting Rate of Return
Return

Benefit Cost Ratio Urgency

Net Present Value:

The Net Present Value of the project is the sum of present values of all the cash flows-
positive as well as negative- that are expected to occur over the life of the project. The
general formula for NPV is

n
NPV of a Project Ct
= ∑ -- Initial Investment
t=1 (1+r)t

Where,

Ct = Cash Flow at the end of year t

n = Life of Project

r = Discount Rate

To illustrate the calculations of NPV, consider the project that has following cash flow
stream at the end of year. The cost of capital r for the firm is 10%.
Year Cash Flow
0 -1000000
1 200000
2 200000
3 300000

Page | 41
IOV-Registered Valuers Foundation

4 300000
5 350000

The net present value of the proposal is as follows,


Year Cash Flow Discount Deferring Present Value
Rate Factor

0 -1000000 1.00 -1000000.00


1 200000 10% 0.91 181818.18
2 200000 0.83 165289.26
3 300000 0.75 225394.44
4 300000 0.68 204904.04
5 350000 0.62 217322.46
NPV -5271.62

The NPV represents the net benefit over and above the compensation for time & risk.
Hence the decision rule associated with Net Present Value Criteria is.

1. Accept the project if the NPV is positive.

2. Reject the project if the NPV is negative.

3. If NPV is zero it’s a matter of indifference.

Internal Rate of Return:

The internal rate of return (IRR) of a project is the discount rate which makes its NPV equal
to zero. Put differently it is the discount rate which equates the present value of future cash
flows with the initial investment. It is the value of r in the following equation.

n
Investment
∑ Ct
=
t=1
(1+r)t

Where,

Ct = Cash Flow at the end of year t

n = Life of Project

r = Internal Rate of Return (IRR)

In NPV calculations we assume that the discount rate (cost of capital) is known and

Page | 42
L&B/Chapter-3/Income Approach to Value

determine the NPV. In the IRR calculations, we set the NPV equal to zero and determine the
discount rate that satisfies this condition.

To illustrate calculation of IRR consider the cash flows of a project being considered.

Year 0 1 2 3 4 5

Cash Flow -1000000 200000 200000 300000 300000 3500000

The IRR is the value of r that satisfies following equation.

200000 200000 300000 300000 350000


1000000 = + + + +
(1+r)1 (1+r)2 (1+r)3 (1+r)4 (1+r)5

IRR Calculations:
Year Cash Flow

0 -1000000
1 200000
2 200000
3 300000
4 300000
5 350000
IRR 9.813%

The decision rule for IRR is as follows,

1. Accept: If the IRR is greater than cost of capital.


2. Reject: If the IRR is less than the cost of capital.

11. PROFIT METHOD OF VALUATION


INTRODUCTION
The 'profit method' of valuation, as the name suggests, consists of working out the profit of
business enterprise and capitalizing at an appropriate year's purchase. The investments
made in such business enterprises are with the sole motive of earning regular profit. The
state government, or in some cases the central government, issues licenses before such a
business can be started. These include cinemas, hospitals, hotels and restaurants.

Page | 43
IOV-Registered Valuers Foundation

The license is not only attached to the business but also specifies as to where the business
is to be carried out. For example, in case of a hotel the business embraces land, buildings
and ancillary improvements. There will also be furniture, fittings and fixtures, and goodwill
associated with the business premises. The value of the property, therefore, depends upon
the earning capacity of the business as a whole and not on the cost of construction of the
building or the value of land in the vicinity. Consequently, valuation of the property,
whether to be valued on behalf of the owner or the lessee, would include business goodwill
besides, of course, the physical assets.

The tangible assets would consist of land, buildings and structures, plant and equipment
and other physical assets. Tangible assets are those which can be physically touched.
Intangible assets, on the other hand, would include goodwill of the business, license
associated with the premises, quality of management, and so on.

Very often, the licenses for such businesses, by themselves, possess considerable value as
may be the case of a hotel or restaurant serving liquor. Therefore, in order to make realistic
valuation of such licensed premises, the Valuer must possess an in-depth knowledge of the
particular industry and make a clear investigation and analysis of the market.

VALUATION TECHNIQUE FOR USING PROFIT METHOD


Basically, the procedure consists of determining the annual profit, apportioning it into
tangible and intangible components and capitalizing each by an appropriate year's
purchase.

DETERMINATION OF PROFIT
Profit is determined by the relationship: Profit = Income - Outgoings

Income includes all receipts from sale such as food or liquor in a hotel or restaurant, or
from sale of cinema tickets in the case of a cinema, from advertisements, parking fees, etc.

The outgoings can be classified as follows:

1. Preliminary expenses

2. Working expenses
3. Depreciation and maintenance

4. Owner's profit

Preliminary expenses include entertainment tax, License fees, film hire charges in case of a
cinema and/or basic items necessary for running the business.

Page | 44
L&B/Chapter-3/Income Approach to Value

Working expenses would include salaries and benefits to staff, operating and running
charges of the business including municipal taxes, purchase of food and/or other
consumables for the hotel.

Depreciation and maintenance would include depreciation on building, furniture, fixtures


and other items used regularly. Also, an amount for a sinking fund can be provided for,
besides making an allowance for maintenance and repair.

Owner's profit could be anywhere between 15 and 20% or more and normally takes into
account interest on blocked capital, interest on working capital, trade profit, risk factor and
management skill, and the like.

However, interest on borrowed capital or income or wealth tax paid should not be included
in the expenses.

RATE OF CAPITALIZATION
After arriving at a figure for the profit, it is necessary to determine two factors:

1. The ratio of tangible profit to intangible profit.

2. The capitalization rate for each profit.

The tangible portion of the net profit is due to the capital assets or assets which can be
touched. The intangible portion of the profit is due to goodwill, management, License, etc.
Normally this would be in the ratio of the profit of 75 to 25 per cent or 70 to 30 per cent,
depending on the type of business being analyzed. This ratio will have to be determined by
the valuer considering facts and circumstances of the business and other relevant data.

The capitalization rate will have to be fixed taking into consideration the return on other
investments and prevailing market rates of interest.

HOTEL AND RESTAURANT INDUSTRY

In the case of hotels or restaurants which depend primarily upon the quality of customers,
the fundamental criterion to be realized is that it is a trading operation. The main object of
conducting the business is profit. In a restaurant certain goods are processed, reprocessed,
and served in a manner which depends upon management practices, location of the
premises, etc. For their profits hotels too depend, mainly for their clientele, on location,
management practices, facilities, services and conveniences to clients, type of construction
and related operations. Therefore, a hotel building probably represents the highest and
best use of the site itself. The quality of its management services also goes a long way in

Page | 45
IOV-Registered Valuers Foundation

maximizing its profits. Some of the salient points to be noted for valuing a hotel business
are as follows:

1. Location of the premises.

2. Characteristics of furniture, fittings and miscellaneous equipment.

3. Type of management and its efficiency.

4. Type of services provided.

5. Quality of rooms, as also the general construction of the premises.

6. Quality of food that is served.

7. General standard of cleanliness.

Thus, hotels and restaurants can best be valued on the basis of their trading potential
although the valuer must always make it a point to study occupancy rates, check for a
comparable property in the area and, in general, verify all aspects of the property. The
following example will probably serve as a good example on valuation of a hotel business.

12. CINEMA BUSINESS IN INDIA


A movie theatre in any town or city of the country is generally a landmark site. It tends to
become the center of social activity and a meeting place. However, with the advent of T. V.
and other similar sources of entertainment, the importance of cinema may have decreased
but the film theatres are still important locational sites in a town.

The market value of a cinema depends mainly on the earning capacity, irrespective of other
considerations such as the break-up price of land, structures, fittings and fixtures, etc. Of
course, these components do have an effect on the value. However, the profit method
would take in all these including the effect of goodwill and management.

Again, the gross income is from the exhibition of movies which depends on the movie
being exhibited, quality of the hall, seats, parking arrangements, aesthetics, environment,
sound system, general comfort and other such factors. Income will also be from
advertisements, parking stands, rent from show cases, rent from food stalls, and the like.The
outgoings would include entertainment tax, preliminary expenses for hiring of films, other
expenses pertinent to the business, depreciation, etc. Owner's profit should also be
considered.

Page | 46
L&B/Chapter-3/Income Approach to Value

VALUATION AS A GOING CONCERN AND OF GOODWILL

Certain businesses such as the ones stated above, i.e., the hotel business and the cinema
business are expected to continue in operation and have a 'going concern value'. This going
concern value includes a value apportioned to both tangible and intangible assets. As such,
there is also an element of goodwill value built into the overall valuation for such valuations
where the profit method of valuation is adopted.

MARKET VALUE OF GOODWILL


According to the Shorter Oxford Dictionary, goodwill is "The privilege granted by the seller
of a business to a purchaser of trading as his recognized successor; the possession of a
ready-formed connection with customers considered as a separate element in the saleable
value of a business."

Goodwill can include the market value to the extent that it is transferable. It may be
attached to personal qualities, reputations, brand names or similar items. However, goodwill
as such does not form a separate element in the sale value of a land or building. But the
special value associated with it is regarded as a part of market potential.

The goodwill of a business is capable of being transferred independently of the property


even though it may diminish substantially over time and space. As goodwill is a separate
element from the property, it may be practical to assess the value of the property and value
of the goodwill attached to it separately. This can be done by putting the property in
question through a valuation process where an alternative use of the premises is adopted.
Therefore, although such a basis is not compatible with existing circumstances of
continuous operation of the business, can nevertheless, give a separate valuation figure of
goodwill value.

Goodwill value can be estimated by considering the average net profit over a certain period
and then adjusting this in a number of ways:

8. Financial adjustments to normalize treatment of such items as capital, bad debts,


depreciation, valuation of stock, etc. and to exclude non-recurring items.
The value of services provided to the business by the proprietors and/or members
of their families.

The value of fixed assets owned by and/or employed in the business.

Page | 47
IOV-Registered Valuers Foundation

VALUATION AS A GOING CONCERN


The value of business as a going concern illustrated in the above examples should normally
be reflected in a balance sheet. In addition to the fixed assets, the assets include financial
assets and also intangibles amongst which may be included goodwill. In terms of
accountancy, the going concern value concept assumes the business as a continuing
operation. Under normal circumstances the going concern value cannot be apportioned to
any particular property assets. A fixed asset may be valued as part of a going concern in
which case the appropriate basis of valuation is open market value for existing use or
depreciated replacement cost.

Market transactions for buy-sell proposals usually, take place on a going concern basis. But
this may not be an appropriate method of valuation for some other purposes such as
taxation and mortgage.

VALUATION OF BRICK FIELD


Brick fields are normally located at some distance from a town but within limits of easy
transport to cater to the needs of the housing or construction industry or near newly
developing areas. Very often fallow lands are leased out to brick manufacturers to obtain
earth for bricks and to construct kilns. Brick manufacturing units normally work for short
term, say ten to fifteen years time, after which no more earth is profitably available from
such land as the deep excavation increases beyond economic level and excavated land
turns into a tank. A landfill operation may prove uneconomical in which case a suitable
option might be a fishery.

Value of such a brick field is usually calculated on the basis of ten years profit for the
lessee's interest. For the lessor's interest, the valuation basis is the present value of the
rental return and a reversion value to fishing or suitable other rights pertaining to tank thus
formed.

VALUATION OF FISHERIES
There are basically two kinds of fisheries viz. sweet water and salt water.

1. Sweet water fisheries are generally in the open land usually fallow land where water
is impounded by means of earthen bunds for rearing fish. More often than not, to
augment the growth of fish and if possible, sludge water obtained from effluent of
town sewers at the point of disposal from a treatment plant is occasionally added to
the water so impounded. Here small fish are reared to big size for commercial
purpose.

Page | 48
L&B/Chapter-3/Income Approach to Value

2. Salt water fisheries are always near a tidal river or estuary directly or indirectly
connected to sea. Small fish often in embryo are flown into such fisheries by means
of sluice gates in the channel connected to rivers during high tides. These fish grow
to their proper size within such fisheries enclosed by earthen bunds.

BASIS OF VALUATION
Fisheries are valued on the basis of profit as it is a commercial venture. Profits may not be
found from books of accounts as fisheries are generally established and run by people with
not much of an education background. In such cases an account of probable yield of fish
per acre at a particular location may be obtained from the Government Fisheries
Department as a guide for calculation. Normally the year's purchase for such fisheries
should be 10 to 12 on net profit.

Alternatively, in the absence of any data, value should be on the basis of land value and
depreciated value of bund plus expected value of fish with the fishery on the appropriate
date of valuation.

Page | 49
IOV-Registered Valuers Foundation

Page | 50 (This page is blank)


MARKET APPROACH TO

VALUATION I 04
1. Types of Market
2. Market Survey & Data Collection
3. Comparison of Sale Instances
4. Hedonic Model and Adjustment Grid Model
5. Land Characteristics
6. Hypothetical Plotting Schemes
7. Residue Technique
8. Valuation for Joint Venture development of property

Page | 1
IOV-Registered Valuers Foundation

(This page is blank)


Page | 2
MARKET APPROACH TO
VALUATION I 04
TYPES OF MARKET, DEMAND AND SUPPLY CURVE, BELL CURVE FOR OVERALL SALES
PERFORMANCE (PROBABILITY DISTRIBUTION).

1. TYPES OF MARKET
Like the commodity market there are 3 types of market conditions in Real Estate Market namely;

1. Buyers’ Market.
2. Sellers’ Market.

3. Stable Market.

Buyers’ Market: When number of units available to the purchasers is far in excess of demand it
is Buyers’ Market. Prices tend to decrease because of greater bargaining power of purchasers
and availability of plenty of alternative and substitute properties. Sellers compete with each
other to sale their units at lesser price. We can say buyer is the king in Buyer's market.

Sellers’ Market: When supply of units is less than demand for such units, prices tend to go up
due to competition amongst buyers to purchase these units even at higher price. Bargaining
power of purchaser reduces. Moreover, Real Estate Units are such that it cannot be brought into
market in short span of few days or few months like consumer products. It takes pretty long
time to increase the supply to meet with market demand. Hence in seller's market seller is king
and he will dictate terms.

Stable Market: Sometimes in market demand as well as equal supply also exists but due to
various reasons, purchasers stop purchasing or defer the purchase of premises. Artificial
stoppage of demand in the market prevails. The reasons could be likely changes in Government
policies or lack of paying capacity of people at large in the locality, to pay high or prohibitive
price of properties.

Page | 3
IOV-Registered Valuers Foundation

Expectations of people about likely fall in price of real estate in near future also results in Stable
Market conditions. Prices do not fall nor rise but remain stable year after year. No material
transaction takes place in the market. Stray sale transactions in market do not change stable
market situations.

OPEN AND CLOSED REAL ESTATE MARKET


When we are considering Market Approach, we should keep in mind certain basic aspects which
are operating in the real estate market. Some of these factors are same as prevalent in other
markets where as some other are special to Real Estate Market.

Types of real estate market

There are two types of real estate market. Open real estate market and closed real estate
market. Open market is like any other commodity market. All buyers and all sellers are welcome
to transact deal. There is no common market place for sale or purchase of real estate like
common market place for sale and purchase of other commodities and consumer goods. Open
market for Real Estate could be therefore defined as a notional forum or platform, where group
of buyers and group of sellers are present each one competing with other to buy or to sell and
where ample substitute properties are also available for sale and purchase. Monopoly aspect is
not present in such open market.

However, there are certain situations where a deal takes place in a closed market.

All types of buyers are not permitted to buy real estate in such closed market. Only special
buyers are allowed to buy. Community based Limitations e.gSale and purchase of property in a
Parsi Colony permitted only to Zoroastrians. Similarly, a Catholic society permitting deals
between Catholics only. In some societies sale is permitted only to vegetarians. Even for
commercial premises such closed market is possible. An office premises in Diamond Bourse at B.
K. Complex, Mumbai can be sold only to Diamond merchant and to no one else. There are many
other types of closed markets. In family partition disputes and partition cases of jointly owned
properties many a time court orders to call open bids from rival parties (Joint owners) to
purchase the property. It is a private auction. This is nothing but type of sale under a closed
market condition. Outsiders are not permitted to bid. If such private bids fails then the court
orders for sale by public auction i.e. in open market. It is a common experience that price of
property in closed market is less than its real worth in an open market.

While understanding this open market concept, we must also understand another concept
"MARKET IS SUPREME" for all marketable properties. Only for non-marketable properties
like Temples, Museum, Schools, Govt. buildings this axiom is not applicable. Market takes into
account all market forces namely economical, technical, legal, social, and political or any other

Page | 4
L&B/Chapter-4/Market Approach to Valuation

such factor. There may be an element of black money operating in the property market.

There may be threat from Slumlord or Mafia or Terrorist or Muscleman operating in the locality
to the transacting parties dealing with land transactions. Market considers all these aspects and
prevalent rates get fixed in the market after all these adjustments.

Even in commodity market axiom ‘Market is Supreme’ holds good. No one can control or over
power market singularly or collectively for long time. Hunt Brothers of U.S.A. trying to corner
silver of entire world and thereby control Silver market failed miserably. In our country stock
brokers like Harshad Mehta during 1990’s and Ketan Parekh during 2001 failed to control share
market for longer duration. Another feature of open market is the fact that persons in the
market are very ruthless. Market does not care for anyone and this includes government also. If
a person in distress is in urgent need to sell the property due to his personal difficulty then he
will get 20% to 30% lesser price in open market because buyers available in the market will
know that seller is in distress and purchasers in the market will enjoy exploiting seller in distress
by offering low price.

Another glaring example of this supremacy of market and ruthless behavior of transacting
parties in the market is the case of Rent Control Legislation implemented by most of the State
Governments throughout India. Under this Act, government tried to control Rental Market by
artificially levying control by freezing of Rent. This was grossly contrary to financial forces
operating in the real estate market. What ultimately happened in the market is an eye opener.
Market invented a novel scheme to take care of financial forces in the rental market. This
scheme of one time advance payment is popularly known by various names such as pugree,
pakidi, premium, salami, nazrana or key money under which the landlord was financially
rewarded in advance in lieu ofloss of rental income due to freezing of rent. In this way the trend
of parallel economy started in real estate market in India and Government failed against
supremacy of market.

DEMAND SUPPLY CURVE


While considering market (commodity market or real estate market) we always consider the
factors: supply and demand. These are most important and powerful factors affecting price level
in a market. Economist J. M. Keynes says that "Demand is not the function of current price but is
also function of future market price as well." The principle of supply and demand states that the
price of a commodity or service varies inversely with the increase in the supply of the item. This
means that more the supply less will be the price. The price varies directly with the increase of
the demand of the item in the market. This means that more the demand more will be the price.
This principle can be better explained by Supply and Demand Curve shown in the graph below.

Page | 5
IOV-Registered Valuers Foundation

In the demand and supply curve shown below. Price is plotted vertically as ordinate (Y Axis)
indicating: Price of the commodity against different quantity of the product available in the
market. Quantity of the goods available in the market is plotted horizontally as abscissa (X Axis)
indicating: Number of units.

The Demand Curve in the graph above indicates that demand for product is approximately 500
pieces for price of Rs. 400/- . As price reduces to Rs. 200, demand in market increases to 2500
pieces. When price falls down to Rs. SO/piece, demand in market rises to 9000 pieces. On the
other hand, supply curve indicates that at the price of Rs. 50/- No. the supply of product is
negligible. When price rises to Rs. 200, the supply of commodity increases to 6500 pieces and
when price in market rises to Rs. 400, the supply increases to 8500. However, the point of
intersection of the two curves gives us equilibrium price of the product or the fair price of the
product. Hence price of Rs. 125/piece would be an ideal price for the said product.

Only difference in real estate market is that supply of real estate units cannot be increased over
night or at short notice like that for commodity market. It takes quite some time to increase
supply of units to meet with increased demand in real estate market. Another reason for
restricted supply is that the real estate asset is not movable like consumer products which can
be easily transported from one place to other place.

In most of the market, supply of money is also important factor bringing in rise or fall of prices
in the market. More supply of money in Real Estate market will result in price rise of Land as well
as of ready flats. If money supply is reduced due to stringent finance policy then rates will fall.
However, flood of money supply in the market will result in high volatility in the market and
ultimate devastation. In 2005, 250 crores were poured in real estate market by foreign investors.
In 2007, this amount rose to 8749 Crores. Prices of properties rose very high and then in 2008

Page | 6
L&B/Chapter-4/Market Approach to Valuation

prices crashed down heavily in the Real Estate Market.

Another aspect that the Valuer should consider is to find out as to whether it is a boom period
or slack period (Slump or Burst period) running in the real estate market. After several years of
research in real estate market in different cities of the World, Dr. Fred Harrison of U.K concluded:
'there is a regular cycle of 18 years’ operating in the Real Estate market’. During the first 14 years
the market remains stable or goes on rising but there after the market is crashed and there is
recession in the market for a period of 4 years, before the next cycle starts. He named this
phenomena as 'Boom and Bust Cycle Theory’. In India, the Slump period (bubble bust or
recession) is after every 10 years.

Reasons for recession could be different but recession does take place at regular interval. In
1977, prices of flats decreased due to Emergency. Recession hit the market in 1986. In 1997,
there was a decrease or collapse in rates of flats because N.R.I. investors withdrew their
investment from real estate. In 2008, prices crashed in India because of bankruptcy of Lehman
brothers and collapse of American economy. If the Value rstudies this cyclical aspect their
estimation of trend in the real estate market is likely to be closer to the actual price variations.

Apart from the demand and supply aspect, it is interesting to study the determination of price of
a commodity in common market between a buyer and a seller. There are group of buyers in the
market who compete with each other to acquire the commodity. Each buyer determines highest
price he is prepared to pay, on the basis of his individual capacity to pay (wealth). He also
estimates a fair cost of the product and his need and urgency to acquire the said product. In fact
his attempt is to get the product on payment of minimum price. On the other hand, there are
groups of sellers in the market who compete with each other to sell their product. Each seller
determines lowest possible price for sale on the basis of same parameters viz. cost of products,
profit margin, capacity to withhold sale of product, need and urgency for sale. In fact,
expectation of seller is to get highest possible price for his product but he succumbs to the
market forces.

If offered price is higher than highest price determined by the buyer or it is lower than lowest
price determined by seller, the transaction or sale of product does not take place. But in all other
cases transaction takes place after haggle (Bargaining) for the price.

BELL CURVE

Please verify the exact requirement of Bell Curve here in IBBI Syllabus.
This process can be easily explained by the Bell Curve shown in figure number 2 above.

Page | 7
IOV-Registered Valuers Foundation

Buyers curve A-B-C shows


number of buyers at
different price levels. 'A'
price is offered by minimum
number of buyers in the
market. Transaction does
not take place if the said
price is an unviable price. At
'B' price, there are maximum
numbers of buyers who are
willing to buy. Transaction
may take place or may not
take place. At 'C' price again
there are minimum numbers
of buyers because said price
is considered very high by
buyers.

Sellers curve D-E-F indicates


a number of sellers at different price levels. At 'D' price there are hardly any sellers able to sale
their product because buyers will consider said price too high. At 'E' price there are maximum
numbers of sellers. Transaction mayor may not take place at this price. Again at 'F' price there
are minimum sellers because said price is considered too low by sellers.

Thus, most of the transactions take place in the market in the price range of 'F' to 'C'. Price '0'
can be said to be ideal price as it is average price of highest price that would be offered by a
willing buyer and lowest price that would be acceptable to any willing seller. It is said that in
perfect competition market, transaction takes place at ideal price '0'.

In real estate market, there is no such common market place as we have for commodity market.
However, these concepts do operate in Real Estate Market. There is a range of prices for any
specific property depending upon individual buyer's needs, their paying capacity and estimation
of cost of said property by each individual buyer. Demand and supply factor and factor for
availability of substitute property in the market operate in this market with equal force.

In order to arrive at fair price of a commodity in the market, buyer as well as seller consider
various characteristics of the product viz. its use, its physical benefit, service life, etc. Similarly for
Real Estate viz. land or land with building/s; buyer and seller consider various characteristics of
properties viz. its usage, its service life (for building) its resale value, benefit of infrastructure
amenity and civic amenities etc.

Page | 8
L&B/Chapter-4/Market Approach to Valuation

2. MARKET SURVEY &DATA COLLECTION,


SOURCES OF SALE TRANSACTIONS.
MARKET SURVEY & DATA COLLECTION

There are several sources from which sale instances of a particulars locality can be collected.

1. Sales recorded at the Registrar's office of the concerned district.

2. Information from local brokers/residents (Local Enquiry).

3. Advertisements in Newspapers.

4. Land Acquisition cases with award details.


5. Auction sale information from different authorities.

6. Various websites on Internet, giving details about sale offers of real estate (flat/Shop
etc.).

7. Valuers own Data Bank.

Each State Government at their District Registrar’s office maintain a sales register which is called
Index II Register. In this register, Registrar records all sale transactions that have taken place
during the year in entire district under his command (Jurisdiction). Each of these sales entries in
the register shows name of seller (transferor) name of purchaser (transferee), plot number,
survey number or city survey number of sale property, name of village in which said sale
property falls, date of execution of sale document and date of registration of sale document in
Registrar's office. This register also shows sale price or transfer consideration, area of property
sold, serial number of registration of the sale in sale register. Register also shows type of transfer
document viz. conveyance sale transaction or lease assignment or mortgage deed or release
deed etc. These sales are recorded in village wise list which are compiled in alphabetical order.
All sales in a particular village in a particular year are recorded in the register in chronological
order of dates and months of said year. Registrar maintains separate register for each year. Old
records of past 100 years sales in entire district are preserved and maintained by the Registrar.

These are made available to the public on payment of fixed inspection and search fees. Hence,
Valuer can obtain land or property transaction details of all sales in any locality, for any previous
year (even 60 to 80 years old) from Registrar's office of concerned State and District.

Page | 9
IOV-Registered Valuers Foundation

SOURCES OF SALE TRANSACTIONS

The Registrar's office apart from issuing certified Extract of Index II: Entries also issues
certified true photo copies of any sale documents of any year that is registered in Index II
register. Study of this document copy will enable valuer to understand background and
conditions of sale transaction. This document copy will normally indicate date of agreement to
sale when earnest money was paid, and date of conveyance or lease deed when balance
consideration was paid. Document will also show date of registration with registrar as endorsed
on sale deed. For unit rate of sale, date and year of agreement to sale should be generally taken
into consideration because that is the first date when sale price is agreed between seller and
purchaser.

That is the correct view for unit rate under the document. However, some valuers adopt date of
conveyance as the basis for unit rate fixation.

Another source of data collection for sale transactions in locality is to carry out local market
inquiry. Information from local brokers dealing in such real estate transaction is very useful.
Information from local residents and property owners will also be helpful. Particularly owners
who purchased or sold property in locality can give sale information. Local practicing architects
and engineers involved in planning and construction of buildings for their clients can also
furnish required data of ruling prices of land and buildings in the locality.

Advertisement in newspaper and business papers also give plenty of information on sale prices
offered and prevalent rates of various types of properties in the locality. Every valuer should
preserve at least monthly data of such newspaper advertisements in his own data bank for
present as well as future reference.

If information is available about land acquisition cases and award details for property acquired
in the locality, then, such details are also valid data for prevalent rate.

Auction sale rates by Income Tax Department by Govt. or Semi Govt. undertakings by Banks
custodian, Improvement Trusts, Court Receiver or Court Registrar also give sale rate information.
These are valid comparable sales.

In the case of Jumramal Son", the Allahabad H.C. held, "We find the ground given by the Tribunal
for not considering the rates of auction sale to be untenable. Auction signifies public sale of the
property usually conducted by bidding one after other which augment the price. On the approval
of the bid, a contract comes into existence. Certain auction rates have been quoted are higher than
the rates at which the property in question had been sold. The auction sales were therefore,
wrongly excluded by the Tribunal. In our opinion, the Tribunal committed an error in ignoring the
material evidence, the present would be fit case for setting aside its judgment. "

Page | 10
L&B/Chapter-4/Market Approach to Valuation

Past experience of valuer himself, his own data bank, his local valuer friend's opinion can also be
a source of prevalent trend in real estate market. Before undertaking comparison with sale
transactions, the valuer must check and study title deeds of subject property. Correct area of the
property and ownership title should be examined thoroughly and examine whether the sales are
genuine or otherwise.

There are four sources to check area of plot of land.


1. Area of the plot shown in Government Records called 7/12 Utara or area shown in Extract
from the Property Register Card of City Survey.
2. Area stated in conveyance deed (title deed).
3. Area shown in sanctioned plan (If building is erected on plot).
4. Area as per actual survey carried out on site by the valuer.

It may not be too harsh a statement that "Plot areas as per all these four sources generally
differ". Invariably, we find that area of plot shown in conveyance and government record do
not tally. Even if actual survey is conducted on site area as per actual survey also do not
compare well with conveyance figures or government records. Realizing such wide
disparities in plot areas; several municipality in the country have made a policy to allow
development in the plot, as per minimum area of plot under these three sources. Valuer
should also therefore follow either area as per Govt. record or minimum of three areas as
correct plot area for his value estimates.

Area of plot worked out from actual Plane Table Survey may differ from area worked out from
actual The odolite Survey. However, area of land arrived at by the odolite survey is more
authentic. Now advanced surveys by satellite images are also in vogue.

If as per actual survey the plot area is found to be much more than area shown in government
records; the property owner should be advised to get area corrected in Govt. Records after
getting resurvey of the plot done by Govt. Survey.

Unless such corrections in Govt. Records are carried out, it is risky and unwise to adopt higher
plot area as per actual survey. Since there may be possibilities of encroachment by the owner on
neighbor’s property. Only government survey will prove that there is no encroachment and area
of plot in fact is more than the old government record.

Like checking and verification of plot areas, there are four sources to check correct built-up floor
area or carpet area of the ownership premises. (Flats/ shops/ offices/ factory galas).
1. Area shown in purchase document.
2. Area shown in society's record/condominium record.
3. Area shown in sanctioned plan of the building.
4. Area as per actual survey on site by valuer.

Page | 11
IOV-Registered Valuers Foundation

3. COMPARISON OF SALE INSTANCES FACTORS


OF COMPARISON AND WEIGHTAGES FOR
ADJUSTMENT IN VALUE.
Sales Comparison Method (Direct market comparison method)

SALES COMPARISON

It is human tendency to compare and select. A customer intending to buy consumer product will
compare quality and price of such a product available in one shop with quality and price of a
substitute/ similar product of other brand available in the same shop or in another shop. He
would select the product only after comparison of both products from all angles including price.
It is said that 'to err' is human. Likewise we can also say that "to compare" is human.

In this process of comparison one important concept is worth study. This concept is that similar
looking product may or may not have same value. Similarity may be deceptive. Similar looking
two plots or two buildings may have quite different characteristics and different values also. This
important point is required to be thoroughly understood. The following story further provides
valuable insights into the concept of comparison and value of the product.

A tradesman (Saudagar) came to the Court (Darbar) of the King Akbar and produced three
similar beautiful dolls. He then demanded that wise person of the Court estimate fair value of
each of these 3 dolls. Akbar asked Birbal to do valuation. Birbal collected 3 dolls and asked for a
day to answer the question. Next day Birbal came to the Court and declared value of first doll as
Nil. He said that value of second doll is 1000 gold mohars (Gold coins) and further added that
the third doll is invaluable but he puts its value at 1 lac gold-mohars. Saudagar agreed with
valuation.

However Akbar asked for reasons for different valuation of the similar looking dolls. Birbal gave
practical demonstration. He put up a thin wire in ear of first doll it came out of the mouth of the
doll. Birbal said that this doll symbolizes a person (Lady as well as Gent) who listens to the secret
of her/his friend and tells (everyone about it and hence such person has no value in the society.
Wire put in ear of second doll came out of second ear. Birbal said such persons are less harmful
as they simply listen and then forget. They neither help the friend nor do they cause any harm to
the friend. Hence, he valued second doll more than first doll. When he put wire in third doll's ear
it did not come out but remained inside the doll. Birbal explained that such persons in the
society are invaluable because they not only keep the secret of friend with themselves, but they
also try to solve problems of friend.

Page | 12
L&B/Chapter-4/Market Approach to Valuation

There are three important lessons to be learnt from this old story.

1. The similarity of property may be deceptive. Valuer must thoroughly investigate


tofind differences of attributes between even similar looking properties.

2. Valuer should not immediately jump to conclusion. Case must be thoroughly studied
by asking for time for valuation and then only considered opinion about valuation
should be given.

3. End users of report should ask for reasons for given valuation and Valuer should
support and prove his valuation by giving demonstration or supporting evidence.

4. HEDONIC MODEL AND ADJUSTMENT GRID


MODEL UNDER SALES COMPARISON
METHOD.
Adhoc Comparison Technique / Hedonic Pricing Model Under sales Comparison Method,
most popular comparison technique is “Adhoc Comparison Technique”. This adhoc
comparison technique is very well known and it has been in use for several years. In fact
even today, many Valuers adopt this technique to save time and to do quick reporting of
valuation to the clients.

Under this model, price of the property is expressed by the formula.

P = f (STLA)

P = Price of the property in the market.


f = Stands for function of

S = Size or covered area of the premises.


T = Time factor at which asset is traded in the market.

L = Location of the property.

A = Age or physical conditions of the property.


It will be seen that out of thousands of factors which affects value of the property in real
estate market, only four factors are considered in this model. Under this adhoc comparison
model, the Valuer compares these four factors of both properties viz. sale instance property
and subject property. Valuer arrives at the rate to be adopted after doing this adhoc
comparison. There is no doubt that price of the property is the function of area, location
and time period. Location in respect with proximity of civic amenities is a major factor
affecting value. Similarly time factor indicates demand and supply position and

Page | 13
IOV-Registered Valuers Foundation

environment of market at relevant period of time which determines price of an asset.


However age factor is only for buildings and that too in a small way. It does not apply in
case of land value comparison where characteristic of land decides its value in the open
market. Similarly, size or area factor also is not a major or a very important factor for
building values but it does constitute an important attribute for land values. Demand for
more area is comparatively less than demand for smaller areas. Under this adhoc
comparison method the Valuer normally follows steps as detailed below.

Valuer first collects data of sale instances in the locality for the relevant period of time. Non
genuine instances of sales are excluded. ii. Rates and attributes of genuine sales are
compared with the attribute of subject property. All relevant factors and weightages
applicable are recorded. iii. Valuer then arrives at a concluding overall effect of these
comparison. It may be positive weightage over sale instance rate or negative weightage
over said rate. iv. Valuer then finally estimates the final rate for the subject property based
not only on his experience and expertise but also on the basis of overall impression. Hence
it is called adhoc analysis.

This method is categorized under direct market comparison method. This method has been
described as the simplest and most direct case of using comparable evidence to arrive at a
value conclusion. But comparison evidence should meet some criteria in order to be
acceptable for a valuation exercise. They are mostly applicable in the goods and
commodities market.

In a nutshell, these criteria are as follows:

First of all, all the comparable evidence should be comprehensive, that is, there should be a
number of comparable, not a single evidence of value. Secondly, the subject of valuation
and the comparable should be vey similar and ideally the comparable should be identical to
the subject of valuation. Thirdly, the transaction should be a current transaction reflecting
current market trends. Fourthly, it should be the outcome of an arm's length transaction in
the open market. Fifthly, it should be capable of verification as far as possible and sixthly,
the comparable evidence should corroborate the local market norms.
These criteria are unlikely to be totally fulfilled in the real property market situation. Firstly,
heterogeneity is an important characteristic of real property. No two properties are identical
in all respects. Even similar apartments in a development project differ somewhat in
orientation or finishes or other aspects. Again, external factors outside the property make
them further dissimilar. Location in respect of amenities and necessities make physically
identical properties different for a market transaction. Also, the same property may be
subject to different kind of property rights. So, different rights in respect of identical
properties may have been transacted in the market. The zoning laws again make identical

Page | 14
L&B/Chapter-4/Market Approach to Valuation

properties different. Further, transactions in real property market are less frequent than
those in other markets of goods and commodities. So, where does one find comparable at
a particular point of time? Finally, the lack of transparency in the property market
anywhere in the world make availability of comparable evidence still further difficult. For all
these reasons comparable evidence is difficult to find in the real property market.

A Valuer therefore is constrained in any approach of valuation to find suitable evidence for
comparison purpose. The available data in the real property market can only give an
overview and trends of the market from where he has to draw his own valuation
conclusion.

As already stated, real estate is an extremely heterogeneous commodity. The main aspects
that caused dissimilarities between properties are placed below.

INTERNAL OR PHYSICAL CHARACTERISTICS


These characteristics concern land and building comprising of the property. They include
such aspects as:

1. Land: size of land, frontage of site, depth of site, shape of site, directional facing

2. Building layout of building, design of building, internal planning, construction


method, specifications, area of floor, age and obsolescence etc.

EXTERNAL CHARACTERISTICS
These characteristics are external to the property. They have an overall influence on the
value of the property. They include such aspects as:

This gives accessibility to roads, services, public


(a) Location of the property
open spaces, proximity to amenities or undesirable
features etc.

Use zoning or other restrictions by planning


(b) Neighborhood characteristics
authorities, presence of slums, social aspects,
historic aspects 'amenities etc.

One-time payment, stage payment, time


(c) Transactional characteristics
aspects etc

The total number of factors affecting different aspects of real estate and causing
dissimilarities between them may be much more than this. In most cases however, Valuers
generally consider four principal aspects of real estate and make ~n ad hoc comparison

Page | 15
IOV-Registered Valuers Foundation

between them to arrive at a valuation conclusion. These four aspects are location, size,
time aspect &age and physical state of the property.

The first step to proceed with the ad hoc process is to collect suitable comparable as far as
possible within the same locality and as similar to the subject property as possible. The
comparable should be recent sale transactions as practicable. The attributes of the
comparable are then compared with those of the subject property. The Valuer is then to
note all relevant factors and give positive or negative weightages over the rate of the
comparable. Finally, the concluding stage of the comparison is reached by weighing the
positive weightages over the negative weightages. The outcome is to arrive at a final
answer with either positive weightage or negative weightage over the rate of the
comparable unit.

Following examples will indicate the valuation approach.

Example

A residential flat of 150sq.m. size is situated in a building on a 12m wide road. Another flat
of size 90sq.m. is situated in the same locality but in a building on a 20m wide road. The
interior planning of the former flat is superior to that of the latter though both are of the
same specifications. The latter flat enjoys the amenities of a swimming pool and a large
garden on the plot. The former flat was sold a year back at a price of INR40,000 per sq.m.
What is the market value of the latter flat today?

Solution
Considering the location, size, interior planning, amenities and time factor we allow the
following adjustments on the sale price of the comparable unit.

+10% for subject property being on 20m wide road as against 12m
of the comparable unit
+5% for size of subject property being smaller than the comparable
unit (90sq.m. as against 150sq.m.)
-10% for interior planning of subject property being inferior to
comparable unit
+10% for subject property enjoying amenity of a swimming pool and
garden as against none of the comparable unit
+12% For time taken of 1 year as the comparable unit was sold a year
back
27% (overall addition)

Page | 16
L&B/Chapter-4/Market Approach to Valuation

Therefore, value per sq.m. of subject property = INR 40,000 x 1.2 =INR50,800 per sq. m.
Therefore, market value of subject property = INR50,800 x 90 = INR4,572,000

Example

A plot of land of area 350sq.m. is situated on a 20m wide road in close proximity of a
cinema hall. Another plot of land measuring about 800sq.m. is situated on a 30m wide road
in the same locality in purely residential surroundings. The latter plot has a return frontage
on a 12m wide road on its west. The latter plot was sold 6 months back at a price of
INR22,400 per sq.m. What is the market value of the former plot?

Solution

Considering the location, size, accessibility and time aspect we allow the following
adjustments on the sale price of the comparable unit.

-10% for subject property being located on a road of smaller width than the
comparable unit.
-10% for subject property being located in close proximity of a cinema hall as
against residential neighborhood of comparable unit
+10% for subject property being of smaller size than the comparable unit
(350sq.m. as against 800sq.m.)
-10% for the comparable unit having a return frontage on another road
+6% for comparable unit having being sold 6 months back
-14% (overall deduction)

Therefore, value per sq.m. of subject property = INR22,40o x 0.86 = INR19,264


Therefore, market value of subject property = INR19,264 x 350 = INR6,742,400

As a matter of fact, however, ad hoc practice may be somewhat arbitrary. The proper
weightages of each attribute should be ascertained from an analysis of sale transactions in
the real estate market. Unfortunately, such research is entirely lacking in this country and
therefore ad hoc practice cannot be dispensed with altogether and the Valuer has to
depend more on his experience.

ADJUSTMENT GRID MODEL


An attempt has been made to make the approaches of sale comparison less ad hoc through
the introduction of this model. A number of sale instances as similar as possible to the
subject property is chosen as comparable. About four or five major attributes of the subject
property are then selected for comparison with those of the comparable. Each attribute is

Page | 17
IOV-Registered Valuers Foundation

classified under a suitable rank.

Rank (5) for attribute of highest quality.


Rank (3) for attribute of medium quality.

Rank (1) for attribute of poor quality

E.g., considering valuation of a flat under this approach.

Age aspect:
Rank (5) for buildings less than 7 years of age

Rank (3) for buildings 7-25 years of age

Rank (1) for buildings of age more than 25 years

Location aspect:

Rank (5) if civic amenities and services are within a short distance
Rank (3) if civic amenities and services are at a moderate distance

Rank (1) if civic amenities and services are available at a remote place

Specification aspect:

Rank (5) for best specification

Rank (3) for general standard specification

Rank (1) for poor specification

Size aspect: Rank (5) for flat area 60sq.m. or less


Rank (3) for flat area between 60sq.m. and 120sq.m.
Rank (1) for flat area beyond 120sq.m.

In practice, the number of attributes may be more although only four have been chosen
here. The Valuer may choose the appropriate number of attributes according to their
importance.

Now, all the comparable, their attributes with ranks and the subject property are placed in
the evaluation grid as indicated in the table.

Next, rank of each attribute of the comparable is compared with the rank of the said
attribute of the subject property and positive or negative weightages are given. For superior
rank of the attribute of a comparable negative weightage is given whereas for inferior rank
of attribute of the comparable positive weightage is given.

Then all weightages of all the attributes of each comparable are summed up to find the
resultant effect on the sale figure of each comparable. The finally adjusted sale figure of the

Page | 18
L&B/Chapter-4/Market Approach to Valuation

comparable are then considered for arriving at the value of the subject property.

Example

A residential flat is situated in a 5 years old building. The building is situated on a main road
and closed to civic amenities and services. The building is of good specification with marble
flooring and good internal planning. Area of the flat is 75sq.m. What is the value of the flat
today based on sales of three comparable given below?

Sale X: A flat measuring 130sq.m. sold @INR40, OOO per sq.m. about 6 months back. The
building is 18 years old and situated on a narrow lane away from civic amenities. The
building is of standard specification within mosaic floor tiles.

Sale Y: A flat measuring 100sq.m. sold @INR35, OOO per sq.m. a year back. The building is
20 years old and situated in a middle-class locality on a narrow lane but close to civic
amenities and services. The building is of poor specification.

Sale Z: A flat measuring 80sq.m. sold @INR45, OOO per sq.m. about 9 months back. The
building is 12 years old and situated on the main road in a locality dwelt in by a wealthy
community but away from civic amenities and services. The building is of good specification
and provided with marble flooring.

The evaluation grid is given below showing the adjustments

Evaluation Grid

SI Property Subject SaleX Sale Y Sale Z Remarks


# characteris property 130sq.m. 100 sq.m. 80sq.m.
tics 7Ssq.m. flat flat flat
flat
1 Sale data To be INR40,000/sq INR35,000/s INR45,000/s
evaluated . m. q q .m .
.m.
2 Time aspect Today +6 months +12 months +9 months
3 Adjustment +6% +12% +9% Allowance
for + + + 1% per
time INR2400/sq. IN INR4,050/sq month
m. m.
R4,200/sq. m.
.
4 Adjusted rate for INR42400/sq INR39,200/s INR49,050/s
time . m. q
.m . q
.m.
5 Location 1 3 3
aspect
Rank ... 5 +20% +10% +10%
weightage
6 Size aspect 1 3 3

Page | 19
IOV-Registered Valuers Foundation

Rank ... 3 +5% nil nil


weightage
7 Age aspect 3 3 3 18 - 5 = 13
Rank ... 5 -6.5% -7.5% -3.5% years
allowance 20 - 5 = 15
year
~% per years
12 - 5 = 07
8 Specification 3 1 5 years
aspect +10% +20% nil
Rank ... 5
9 weightage
Overall +28.5% +22.5% +6.5%
weightage
over +INR 12,084 + INR8,820 + INR3,188
rate for
adjustment
item 4
time at
10 Finally INR52,00 INR54484 INR48,020 INR52,238
adjusted
rate 0
INR/sq.m.
Therefore, value per sq.m. of subject flat = INRS2,000. Therefore, value of subject flat
= 75 x INR52,000 = INR3,900,000

The Valuer may take more comparable if available and choose more attributes of the
subject property for comparison with those of the comparable.

5. LAND CHARACTERISTICS AND ITS EFFECT


ON LAND VALUES
Earlier Economists Adam Smith and David Recardo talked about basic land characteristics
and theory of rent. Land was considered then as fixed supply commodity having mainly
agricultural use. These theories are now outdated. Land can now be generated in vertical
form by use of F.S.I. and T.D.R. concepts. More over Land use is not restricted to only
agricultural use. According to eastern cultures, there are five basic elements of nature.
These are land (earth) water, air, fire and space. All civilization and cultures are developed
along river banks having plenty of fertile land and enough water. All empires were land
oriented in the past due to agricultural activities. Even after industrial revolution, demand
for land never ceased, on the contrary demand for land increased. More and more control
and legislations on land in each country is the proof of this high demand. In new
millennium and even thereafter, mother land will continue to exert prime influence on
economic and cultural activities of mankind because with ever increasing population, land
will remain to be scarce even with vertical usage of land .The fact remains that land cannot

Page | 20
L&B/Chapter-4/Market Approach to Valuation

be created or produced easily like buildings and other commodities. This is the great
importance of land. Before we undertake study of land characteristics, we must first
understand what the ‘property’ actually means and what are the types of property.

Property means Things and Assets. There are two types of properties.

1. Tangible properties.
2. Intangible properties.

Assets are those which can be physically seen and touched like land, buildings, jeweler, car,
machineries etc. Intangible Assets are those which cannot be seen or touched but its effect
can be notionally seen and felt. It can be called a notional asset. Intangible Assets gives
legal right of ownership to the owner over non material assets. The examples of such assets
are Goodwill, Brand right, Copy rights, Easement rights, Intellectual property rights, Life
interest, Transfer of Development Rights etc. Tangible properties are again subdivided into
two parts.

1. Movable properties and

2. Immovable properties

Plant and machineries, cars, trucks, aircraft, jewelry are movable properties. Land and
buildings are immovable properties. These immovable properties are also called Real Estate
or Real Property. The ownership rights in an immovable property i.e. land or land with
building as per Transfer of Property Act comprises of following five major rights.

1. Right of possession in exclusion of whole world.


2. Right to user and enjoy the property permanently.

3. Right to alienate and destroy the property. (Land can be alienated and the building
could be destroyed.)

4. Right to lease, rent, sale, transfer or assignment, gift or give away under will. v. Right
to develop.

All these rights are popularly known as bundle or rights. Absolute ownership of Real Estate
means that the owner holds each and every right described above. However, owner may
not hold all rights but may be holding only partial rights in the real estate. He can part with
some of these rights for consideration or otherwise and still continue to own the property.
He can rent, lease or create life interest. He can also permit development on the plot by
third party, on condition, by giving power of attorney. Thus, valuation of real estate does
not mean valuation of actual physical form of land and buildings but valuation of rights of
the owner in land and buildings. Valuation of Real Estate therefore requires proper
assessment of value of rights of the owner, in the tangible property (Physical form of land

Page | 21
IOV-Registered Valuers Foundation

and buildings). These rights could be absolute or partial like right to receive house rent or
land lease rent without physical possession of the property. These rights could be in form of
an intangible property, like Life Interest in Real Estate. Valuer should examine rights of
owner and value these rights on relevant date of valuation. The values of land depends on
innumerable characteristics of land which may be categorized into four basic groups.

1. Economic Aspect.

2. Legal Aspect.

3. Technical Aspects.

4. Social Aspect.

Land Characteristics Land possesses innumerable characteristics. Some attributes of land


are good and it enhances value of land in the open market. Some attributes of land are bad
in nature and it reduces the value of the land in the property market. Due to these
innumerable characteristics of land, estimation of land value has become very difficult task
even for expert Valuer. Out of these many characteristics of land, we shall study only some
main attributes of land and shall study how it tends to increase or decrease value of land in
real estate market. Some of these attributes are inherent qualities of land itself where as
some other attributes of land are artificial or implanted attributes like land use laws,
building byelaws, F.S.I. norms, social beliefs, tenure laws etc. In present study, we shall
study only the following attributes of land. The study is restricted mainly to nonagricultural
use of land. The effects of these attributes could be better understood, if each attribute is
considered in isolation of other attributes of land.

1. Situation.

2. Orientation Location.

3. Tenure of land.

4. Infrastructure amenities.
5. Covenants in title.

6. Neighborhood.

7. State & Central Laws on land.

8. Frontage.

9. Building Bylaws

10. Depth.

11. Status of land


12. Accessibility.

Page | 22
L&B/Chapter-4/Market Approach to Valuation

13. Encumbrance.

14. Size (Plot area).


15. Soil condition.

16. Shape.

17. Climatic condition.

18. Road width.

19. Natural forces.

20. Land use (Zoning).

21. Ownership pattern

22. Topography.
23. Environmental aspects. Vista/View aspect.

24. Prestige Aspect Stigma Effect

25. T.D.R.

6. HYPOTHETICAL PLOTTING SCHEME:


This is more scientific and rational method of valuing very large size plots in one
ownership. Like belting theory, there are certain preconditions for applicability of this
scheme also.

1. If sales of large size plots in the locality are not available, then and then only this
scheme can be applied.
2. Plot should be in developing area of town where demand for housing site exists. It
should not be in fully developed C.B.D. area of town nor should it be in area where
there is no demand for housing plots.
3. The plot should be of sufficiently large size area so that it could be divided into
several small size plots, similar to plots in sale instances. Small plots made will be
with access from internal layout road of the scheme.

4. Depth of the original plot should be considerably more as compared to road


frontage (Width). It will be seen that large size plots outside town area have value
as agricultural land rather than potential N.A. use. Similarly large size plots in C.B.D.
area will be developed into tower structures rather than development of land by
dividing plots into small size plots.
5. In case of Sp. Land Acq. Officer Elura, Supreme Court held – “Value fetched by sale
of small extent land cannot be adopted for large extent land. Loss of land for road
and park, expenses for development should be deducted”.

Page | 23
IOV-Registered Valuers Foundation

6. Thus plotting scheme involve planning housing sites in plots, laying out of internal
roads and other utility services in the plot and providing requisite amenity area
(Garden or Recreation Space) in the plot.
7. The manner of working out values of large plot, by Hypothetical Plotting Scheme, is
given below.

8. In a large size plot, a valuer should first prepare a hypothetical layout, subdividing
the large land into small size plots as house sites. Such hypothetical layout (selected
from many alternative layouts) should be best possible layout with maximum area
in saleable plots after observing D.C. Rules.

9. The internal road network and amenity space should also be planned in the layout
as per provisions of development control rules applicable to the town.

10. Wasteful planning by several internal cross roads should be avoided. Normally 15%
to 20% of total plot area for internal layout roads is considered normal but planning
should be so economical which would consume minimum area of land in the road.

11. Similarly amenity space (Garden) should be centrally provided so as to benefit


maximum number of plots in the layout.
12. Generally as per D.C.rules, 10% or 15% of the total area is required to be provided
for Garden space. Area of the small plots (House sites) in the layout should be as
per general demand in the area. In small size towns 1000 sq.mts. plots may be in
demand and in large size city 500 sq.mts. plot may be in demand. It may be vice
versa also.

13. If plots for societies are in greater demand, larger size plots will be required. Plots
for personal bungalows could be as small as 200 sq.mts. for Row houses and 400
sq.mts. for independent bungalow sites. It all depends on local conditions and need
of persons in the concerned locality and market trend.
14. In this hypothetical layout, Valuer may also plan a combination of small size plots
and medium size plots. Say medium size plots along main road and small size plots
in interior.

15. Valuer should also see that all these plots also confirm to the standards prescribed,
under D.C. rules of the local authority, for layout and subdivisions. Having prepared
a layout plan, a Valuer should find out and consider prevalent rates of small size
plots in the locality.

16. Valuer should then estimate values of each of the subdivided plots in the layout
scheme, depending upon advantages and disadvantages of each of these
subdivided plots in the layout by comparing them with lands/plots involved in
instances of sale.

17. Plot along main road may have full value. Plots abutting on internal layout roads
may be 15% less in value than the rate of land estimated for plots along main road.
18. Some of the internal plots, in some cases, may even fetch same rate as main road

Page | 24
L&B/Chapter-4/Market Approach to Valuation

plots. The plots overlooking large central garden area of the colony, may fetch such
higher price.
19. Generally corner plots inside the colony layout, abutting on the colony layout roads,
may or may not fetch higher rate, in spite of return frontage.

20. All these rates have to be determined keeping in view general demand and
preferences given by prospective buyers of the locality, to various internal plots in
the proposed scheme. Plots over-looking garden may or may not fetch higher rate,
if demand in locality is not much or garden is not on wind ward side but on leeward
side. If topography of large plot is uneven, internal plots in low lying plots may sale
at greater discount than plots having level ground or higher ground. All these land
characteristics have to be considered while estimating land rates for plots in
hypothetical layout. All these plots in the planned scheme may not sale
immediately. Depending upon number of plots in the colony and general demand
and competition by other developers, sale of all plots may take 2 to 4 years period.

21. Investor and developer undertaking such a scheme is required to discount his offer
for land, to account for this delay in getting back his capital investment in the land.
Some plots will be sold initially and some in last year. Hence total receivable sum is
generally deferred for half the period of anticipated total sale period. This
deferment of value is for locking in period of the capital investment in land, during
which return on investment is not likely to be fully available. This deferment would
give present worth of full receivable sum (Total realization) by sale of all the plots.
22. The Developer/investor undertaking such layout development will be required to
incur expenses for providing infrastructure amenities in such a layout.

23. All these expenses must be deducted from total receivable amount from sale of
subdivided plots. Such development works can be completed in stages slightly
ahead of sale plots. The period for completing development works can be
considered, say, one year less than the period for sale of all plots and the cost of
development works be deferred for the average period for completing the
development works.

24. Most of local authority insists for providing following amenities and the Valuer must
assess its cost and deduct the same from total receivable sum.

25. To construct internal layout road of required width as per norms.

26. To provide road side storm water gutters to drain off rain water in monsoon and
avoid flooding in the plots and the layout area.

27. To provide street lights on internal roads as per prescribed norms.

28. To lay water mains in layout roads up to each of subdivided plots for supply of
water. 5. To lay underground sewerage mains in road to drain waste water from
each building on the plot up to municipal sewer main laid under main road. In case
of answered area, this need not be provided as each plot will be having its own
septic tank and soak pit. If central septic tank is planned, its cost should be

Page | 25
IOV-Registered Valuers Foundation

considered.

29. To lay electric cable in roads and provide sub-station for electric supply to each
plot. 7. To construct wall for garden plot and to develop garden for the use of the
residents. 8. In addition to cost of providing above amenities, the Valuer should
also deduct for the cost of Architects and Consultants fees for planning and getting
approval of layout as well as for survey and demarcation of plots on site and
execution of above stated amenities under his supervision.

From the total amount receivable by sale of all plots, Valuer should also deduct for
following expenses.

1. Developer’s profit at 10% to 15% of total sale proceeds.

2. Interest on borrowed capital, if any.

3. Developers own remuneration for spending his time and energy in project. (In
addition to developer’s profit)

4. Advertisements and brokerage charges which may be 1% to 2% of the sale price of


plots.

5. Sale documents charges and solicitors fees for conveyances of plots. These fees are
usually adopted at about 1% to 2% of sale price. If cost is to be borne 50% by the
purchaser, as per the prevalent trend in the locality, lesser amount can be deducted.

6. Registration and stamp duty charges. Normally this is borne by the purchasers but
in some areas 50% of this charge is borne by Seller/Developer. Following example
will explain how the value of large size plot is estimated with the help of
Hypothetical plotting scheme. Plan of hypothetical plotting scheme is also shown
with the solution.

Page | 26
L&B/Chapter-4/Market Approach to Valuation

Example : An investor desires to purchase a plot admeasuring 150m X 225m size in out-
skirts area of town. Prevalent rate of small size plots (400 sqm to 600 sqm plot area), in the
locality, abutting main road, is Rs. 1000/sqm. Garden area required to be provided is 10% of
total plot area. Adopt road width at 14 meter and 10 meters. Sale of all plots is likely to take
2 years. Adopt road cost at Rs. 200 per sqm, cost of laying services at Rs. 20/sqm. Architects
fees at 6% of cost of amenities. Assume Developer’s profit at 12% and expected rate of
return 9%. Ignore legal and stamp charges and adopt brokerage and advertisement charges
at 2% of sale price. Advice on fair purchase price of the plot by adopting plotting scheme
method of valuation. Also work out value of the plot by Belting theory.

Page | 27
IOV-Registered Valuers Foundation

Solution:A hypothetical layout is first prepared for the plot (vide fig.). From the fig.
Relevant details works out as under.

Area in m Area in m
Total Area of Plot = 150 x 225 = 33750 sqm
10% Garden Area = 97 x 35 = 3395 sqm
Internal Road area = as per fig = 6196 sqm
Group 'A' Plots: Plot Nos. (1) to (8) = (8 Nos)
Group 'B' Plots: Plot Nos. (9) to (26) = (18 Nos)
Group 'C' Plots: Plot Nos. (27) to (42) = (16 Nos)

In the above layout, Central garden of 97m X 35m with 3,395 sqm area is proposed. 14m
and 10m wide internal roads are also proposed. Total 42 plots are proposed which could be
divided into 3 groups for value purpose.

Group ‘A’ Plots : Along main road

8 nos. Plots each with size of = 17 m x 32 m x 8 4352 sqm

Group ‘B’ Plots: (Overlooking Garden):

12 nos. = 17 m x 32 m x 12 = 6528 sqm


4 nos. = 17 m x 42 m x 4 = 2856 sqm
2 nos. = 16.5 m x 35 m x 2 = 1155 sqm
Total = 10539 sqm

Group ‘C’ Plots : (Last zone )


6nos. = 17 m x 32 m x 6 = 3264 sqm
2 nos. = 24 m x 32 m x 2 = 1536 sqm
4 plots = 531 sqm x 4 = 2124 sqm
4 plots = 586 sqm x 4 = 2344 sqm
Total = 9268 sqm
Total area of all 42 plots = 24159 sqm

Area of internal roads-


2nos. = 14 m x 64 mx 2 = 1792 sqm
2 nos. = 10 m x 117 mx 2 = 2340 sqm
2 nos. = 10 m x 35 mx 2 = 700 sqm
1 no. = 10 m x 101 mx 1 = 1010 sqm
2 nos. = 0.79 m x 225 mx 2 = 354 sqm
Total = 6196 sqm
(18.6% of total plot)

Page | 28
L&B/Chapter-4/Market Approach to Valuation

As plots with main road frontage have value of Rs. 1000 per sqm, rate for ’A’ group plots is
estimated at Rs. 1000 per sqm. Rate of ‘B’ group plots is estimated at Rs. 900 per sqm and
rate of ‘C’ group plots is estimated at Rs. 800 per sqm. This discount of 10% and 20% is not
a fixed percentage and it could vary depending upon town, locality, demand and market
trend, evidenced by instances of sale of similar comparable plots.

Income from sale of plots:

Group Area in sqm Rate per sqm Value Rs.


A 4,352 1,000 4,352,000
B 10,539 900 9,485,100
C 9,268 800 7,414,400
Total realisation Rs. 21,251,500

As sale of plots will take about 2 years, value is differed or 1 year at 9% yield.

P.V. = Rs. 2, 12,51,500 x .917 = Rs. 1,94,87,625/-

= Say Rs. 1,94,88,000/- ..................................... (A) Income from property

Expenditure :
Area in sqm Rate per sqm Amount Rs.
Road construction 6,196 200 1,239,200
Water supply and electrification 24,159 20 483,180
Garden Development 3,395 10 33,950
1,756,330
Architects fee 6% 1,756,330 105,380
Developers profit 12% 19,488,000 2,338,560
Brokarage 2% 19,488,000 389,760
Total Rs. 4,590,030

(C) Net realisable value = 19,488,000 less 4,590,030


14,897,970 say 14,898,000

Belting theory

The value of the plot can also be estimated using belting theory. We would consider 1 st belt
of 30 m depth, 2nd belt of 45 m depth and third belt of 150m depth.

Land rate for 1st belt = Rs. 1,000 per sqm

Land rate for 2nd belt = Rs. 666 per sqm

Land rate for 3rd belt = Rs. 500 per sqm

Page | 29
IOV-Registered Valuers Foundation

( D) Value of land
Area Rate Amount Rs.
First belt = 150 x 30 = 4,500 1,000 4,500,000
First belt = 150 x 45 = 6,750 666 4,495,500
First belt = 150 x 30 = 22,500 500 11,250,000
Total Rs. 20,245,500 (D)

7. RESIDUAL TECHNIQUE & OTHER


DEVELOPMENT METHODS
The residual value can be defined as the assessment of a surplus. The surplus is that
amount which is available to pay for the land for an economic activity after making
allowances for expenses and profits. The residual method is normally adopted in valuation
for properties which have scope for development. It may be a vacant land site for
development or land with existing structures ready fordemolishing and rebuilding or for
refurbishment. Mathematically, the relationship can be expressed as follows:

S-D-P = L

Where,S = Sale proceeds

D = Cost of development

P = Profit

L = Surplus for land

Notes: (I) The technique primarily consists of looking at the property as value to the owner
rather than as a market transaction.

(2) The technique of residual valuation is applied when there is no established market for
theproperty and it is the Valuer’s job to assess the value of land as a development site.

(3) The method is popularly used where the developer is in a position to assess sales and
costs.

Page | 30
L&B/Chapter-4/Market Approach to Valuation

ROLE OF EVIDENCE IN SUPPORT OF METHODS OF VALUATION

The Comparison Method

This is the simplest and most direct case of using comparable evidence for the purpose of
valuation. If data of similar property transactions are directly available in the market then
such data can be directly used as comparable evidence. Thus by direct comparison the
value of a subject property can be estimated. As is the case of the goods and commodities
market this method is also most widely used in property valuation. However, direct
availability of comparable and use of this method are restricted in the case of industrial and
trade related properties or other specialized properties. The method is however applied
extensively in the residential and commercial sectors.

The Investment or Income Capitalization Method

The method is used where rental data of properties are available in the market. Since capital
value of a property is derived from the net rent and yield for capitalisation, these two data
are investigated in the market for applying in the valuation method. Obviously, comparable
evidence of rental values and yields are required to be made available for the purpose of
valuation. So, although the method is designated as the investment method, the basic
ingredients and steps are dependent on comparison or comparable evidence.

Similar comparable evidence is also required in the more advanced form of Investment
Method i.e. in the discounted cash flow techniques. Under these techniques, comparable
evidence on rental value, sale price, discounting rate and costs are needed to be
ascertained.

The Cost Approach

Under the cost approach, usually the value of the building and the value of the land are
separately estimated. The cost of the building is estimated first. In ascertaining the cost of
the building, the knowledge of current building cost of similar replacement buildings under
valuation is needed to be ascertained by comparison with similar building costs in the
market. This cost is then depreciated to arrive at the market value of the building.

Also the market value of the land is ascertained by comparison with land prices for similar
land available in the market.

The values thus arrived at are then clubbed up together to find the market value of the land
and building. So, under the cost approach also use of comparable evidence for the building
cost and the land value is involved.

We shall now examine each of the methods of valuation in greater detail.

Page | 31
IOV-Registered Valuers Foundation

Belting

Belting is resorted to when the valuer is asked to value a very large sized plot but the
comparable available in the market are only small plots. In such cases a direct comparison
of value per sq.m. or per s.ft. with the small sized plot may not be effective in estimating the
value of the large sized plot. Not only there is the question of difference in size but also the
question of fluctuation of land value of different parts of the plots comes into the picture.
For a plot of considerable depth there may be a fall in value of the back land away from the
road compared with the value of the front land just on the roadside. The back land may not
be suitable for a valuable land use when the front land is appropriate for such use. E.g.,
front land on a road may be suitable for a commercial use while back land can only be
suitable for residential use. Again, the front land may have the advantage of having direct
connection with services such as water supply, electricity or sewerage connection from the
main road. But for the back land, more cost is involved for having these connections over or
under a passage land from the main road. Further, a well designed building with adequate
air and light facility may be constructed on a roadside land apart from having direct
accessibility to the road. In some areas however, people may prefer to erect their residential
buildings away from main road on the back side plot to avoid traffic hazards, noise and
pollution. But in most of the cases, the front land is considered more valuable than the back
land. The land use character of the neighborhood dictates the rate of fall in value of the
back land as compared with the front land. E.g., in an area close to a railway station or a bus
terminus where a large number of pedestrians pass, the front land may be suitable for retail
shop development. The value of land there may fall sharply at a short distance, say 30ft.
from the road frontage. The rate of fall in the value of land away from the main road may
be less drastic in case of residential neighborhoods. No rigid rule can be set for the rate of
fall in value. In localities where small plots are sold in large numbers and people prefer to
live in small detached houses say on 200sq.m. plots or so the first belt land may have a
depth of 50ft. (lS.24m) only. The second belt land may be of depth between 50ft. (lS.24m) to
a distance of 125ft. (38.11m) from the main road. The rate of fall in value of the second belt
land may be 20% less than the first belt land. The recess lands can be taken at 10% less
than the value of the corresponding belt. Everything depends upon what is indicated by the
sale transactions in the surrounding area. It will be appreciated that belting is not a method
of valuation. It is a method of comparison to indicate the extent of land at different belts of
a large sized plot. The entire land area of the subject plot can be converted into equivalent
first belt units and a subject plot can then be valued in terms of first belt value of a small
sized plot.

The following example will clarify the exercise done by the valuer in applying the belting
method.

Page | 32
L&B/Chapter-4/Market Approach to Valuation

Example

A piece of land having a site area of approximately 1,265sq.m. abuts on a 15m wide public
road. Sale of plots in surrounding area indicates mostly small plots measuring 200s.ft. to
350s.ft. of around 40ft. (12.20m) depth to 50ft. (15.24m) depth only. This is a residential
neighborhood dominated by a wealthy community living mostly in two-storey bungalows.
Sale of land in the surrounding area indicates that land value falls by 20% at a distance of
50ft. (lS.24m) from the main road and by 30% at a depth beyond 125ft. (38.1m) from the
main road (from that of the front roadside land).

Land up to a distance of 50ft. (15.24m) from road may be taken as first belt land. Land
between 50ft. distance from road up to 125ft. distance (38.1m) from road may be taken as
second belt land. Land beyond 125ft. (38.1m) distance from main road may be taken as
third belt land. Recess land may be valued at 0.10 of respective belt value.

Front land situated in the first belt within a depth of 50ft. (15.254m) is fetching a price of
INR1,500 per sq.m. What is the average value per sq.m. of the 1,265sq.m. plot? What is the
value of the 1,265sq.m plot?

You are given the following data.

Position of land Area in sq.m.

First belt frontage 240

First belt recess 75

Second belt frontage 360

Second belt recess 180

Third belt frontage 250

Third belt recess 160

Total 1,265

Page | 33
IOV-Registered Valuers Foundation

Position of land Area in sq.m. Multipli Equivalent


er
for first
land belt
(sq.m.)
First belt frontage 240 belting
1 240
First belt recess 75 0.9 67.5
Second belt frontage 360 0.8 288
Second belt recess 180 0.8 x 0.9 129.6
Third belt frontage 250 0.7 175
Third belt recess 160 0.7 x 0.9 100.8
Total 1,265 1,000.9

Solution

Belting coefficient = 1009.9/1.265 = 0.7912

Therefore, average value of the plot = INR1,500 x 0.7912 = INR1,187per sq. m. (say)
Value of the plot = INR1,187 x 1,265 = INR1,501,555

8. VALUATION OF JOINT VENTURE PROPERTIES


Joint ventures (JVs), in the real estate sector, typically, take place between two developers
or between a developer and a land owner. In the JV between the landowner & developer in
India, the land owner contributes the land, while the builder undertakes the development
and marketing.

How does a joint venture agreement between an owner and


developer in real estate transactions work in India?
In India, a joint venture/JDA can happen when the land owner wishes to develop the land
via a partnership with a developer. The key aspect of this model is that the land owner
contributes his land and the developer undertakes the responsibility of obtaining approvals,
property development, launching and marketing the project with his financial resource. The
land owner expects much higher consideration in return for land given up for development
to the builder. Normally, consideration is discharged in the form of upfront payment,
sharing of gross revenue, sharing of constructed area or a combination of all three.

Mostly, cost of land includes a substantial part of the cost of a residential/ commercial
project which leads to a decrease in cost for the builder. In this case, the funds of the
builder do not get blocked and can be used to accelerate construction time and speed. The

Page | 34
L&B/Chapter-4/Market Approach to Valuation

association starts with a mutual agreement between the owner and the builder to develop
the projects. The land owner usually gets between 25 to 40 percent share in built up area,
and the rest goes to the builder. There will be a difference in the profit-sharing basis the
agreement. Also, the land owner has to execute an irrevocable general power of attorney
(GPA) in favour of the builder which should be registered on a stamp paper of appropriate
value with the registrar.

Recently, Indian government has given a new proposal in Union Budget on JDA which will
help to minimize the issues which the owner of land faces while paying the capital gains tax
in the year of transfer. As per the new proposal an individual or HUF entering into an
agreement for development of a project, the capital gains shall be chargeable in which the
certificate of completion is issued by the relevant authority or part of the project based on
the stamp duty valuation.

Sample calculation of Joint venture

Owner owns a land admeasuring 5,000 sqm. Land is sold at the rate of Rs. 40,000 per sqm.
What percentage of share a Developer should offer him for Joint venture. FSI in area is 1.1,
and 40% TDR to be purchased by developer @ Rs. 10,000 per sqm. Cost of construction is
Rs. 20,000 per sqm. Assume Overheads, profit and development cost.

Project calculation will be as under:


Calculation of Joint Venture -
1 Land cost sqm
5,000
Land Rate Rs. per sqm
40,000
Total Land cost Rs. Rs.
200,000,000 ……1
2 Building cost
FSI 5000 1.1 sqm
5,500
TDR 5000 0.4 sqm
2,000
Building area Total sqm
7,500
Rate for construction Rs. per sqm
20,000
Building cost Rs. Rs.
150,000,000 ……2
3 Other expenses
Land development cost Rs.
5,000 3,000 15,000,000

Page | 35
IOV-Registered Valuers Foundation

TDR purchase Rs.


2,000 10,000 20,000,000
Overheads - 5% of 5% Rs.
Construction cost 150,000,000 7,500,000
Rs.
42,500,000 ……3
4 Total Project Cost Rs. 1+2+3 Rs.
392,500,000 ……4
Profit 15% Rs.
58,875,000 ……5
4 Total sale estimated 4+5 Rs.
cost 451,375,000 ……6
Sale Prise of Flat Rs. per sqm
60,183
5 Apportionment of
cost
Total income of project Rs. per sqft
451,375,000
Owners share in JV 44.3090557%
200,000,000
Builders share in JV 55.6909443% Rs.
251,375,000

Page | 36
COST APPROACH TO
VALUE I 05
• Report -QUALITY, STRUCTURE, STYLE
• The report writing for the various purposes of valuation
• Contents of a report

Page | 1
IOV-Registered Valuers Foundation

Page | 2 (This page is blank)


L&B/Chapter-5/Cost Approach to Value

COST APPROACH
TO VALUE I 05
1. METHODS OF COST ESTIMATES FOR
BUILDINGS
COST APPROACH
The cost approach applies the basic economic principle that a buyer will pay no more
for an asset than the cost to obtain an asset of equal utility, whether by purchase or by
construction. Unless undue time, inconvenience, risk or other factors are involved, the
price that a buyer would pay for the asset being valued would not be more than the
cost to acquire or construct a modern equivalent. Often the asset being valued will be
less attractive than the cost of a modern equivalent because of age or obsolescence;
where this is the case, adjustments will need to be made to the cost of the modern
equivalent. This adjusted figure is known as the depreciated replacement cost.

REPLACEMENT COST OF BUILDINGS


On a particular date of valuation, the land and building method of valuation envisages
the determination of the replacement cost of the building less depreciation. The basic
cost of replacement can be determined by the various methods discussed below,

Method of Actual Accounts: If at the time of actual construction, accurate accounts


are kept, then cost of construction or current replacement cost is known. However, if
cost of construction is required on some other date for constructing exactly the same
structure, then the exact cost will not be available by this method. However, an
approximate cost would be available based on this method, if allowance is made for
adjustment due to time difference.

Page | 3
IOV-Registered Valuers Foundation

Item-wise Method: This involves estimating the quantities or items of work and
multiplying each item by the rate on the prevalent date on which replacement cost is
required. Generally, the item-wise rates are published year-wise by each local Public
Works Department (PWD) or the Central PWD (CPWD). These rates can be used as a
guide. Prevalent market rates may also be adopted.

Estimate by Plinth Area or Covered Area: This method involves calculation of the
plinth area or covered area (or as per modem trends, the 'super built area'). An
appropriate unit rate is then applied. The unit rate depends on factors such as the type
of construction and fittings installed and other factors.

Estimate by Considering Cubical Contents: This method, instead of considering areas


as in the previous case, takes the volume into account.

Building Cost Indices: A certain weightage is given to each building component. For
example, steel, bricks and mason's wages. A construction index is then constructed for
a particular place for a specific date by comparing with another or same place,
evaluated on a different date. Normally, government organizations, like the CPWD
publish indices for Delhi and other places with Delhi as base for January 1, 1970 or
October 1, 1976 or January 1, 1992 for each year. The index so obtained is then
multiplied by the plinth area constructed. There is sufficient scope in the method to
increase or decrease the cost if there is deviation from specifications. Further details on
this method are given in the Chapter titled 'Building Costs and Index Numbers'.

Plinth Area Rate Method: Plinth Area Rates as applicable on 1.10.2007(Reprint) were
last circulated under Memo no. SE(S&S)/EE-II/AE-III/ dated 19.7.2010 along with
annexure I to IV. Relevant cost indices with reference to base 100 as 1.10.2007 shall
continue to be applicable on these plinth area rates for works in progress etc. However,
the need for issuing fresh plinth area rates has been felt to account for rise in prices in
the last 5 years as well as the latest development in construction practices such as fire
safety measures for high rise buildings and also incorporating the green buildings
concept for three star rating of GRIHA. Accordingly, fresh plinth area rates with
reference to base 100 as on 1.10.2012 has been prepared for circulation in the
department. In future, Preliminary Estimates may be prepared on the basis of these
plinth area rates.

All the rates are based on data of actual expenditure for structures completed recently,
as received from various field formations. In case of any discrepancy in Hindi & English
versions,

English version will prevail. The latest plinth area rates as on 1.10.2012 is hereby issued

Page | 4
L&B/Chapter-5/Cost Approach to Value

with following annexures:

1. Annexure – I: Fresh Plinth Area Rates with base 100, as on 1.10.2012 (for
residential /nonresidential buildings, services and development).

2. Annexure – II: Broad specifications and scale of amenities for sanitary/Electrical


fittings for which plinth area rates are applicable.

3. Annexure – III : Memo no. 29/21/58/WI of 10/83 indicating the rules for
working out plinth area from plans, to be observed while adopting these plinth
area rates given in Annexure - I.

4. Annexure –IV: Proforma for calculating cost index for future cost index with
base 100 as on 1.10.2012 indicating revised weightages also.

5. Annexure –V: Specifications for buildings - Non Residential& Residential


Buildings.

2. LIFE OF BUILDING
Obsolescence is a phenomenon that is widely discussed in the literature, although
rarely in relation to buildings. In this paper parallels are drawn between obsolescence,
depreciation and discounting in order to develop a new method for predicting the
impact of building obsolescence based on measurable context factors. These factors
have physical, economic, functional, technological, social, legal and political
characteristics. Useful life is defined as discounted physical life, where the rate of
discount is determined from predicted future obsolescence. As part of the method, a
new tool for determining the physical life of buildings is presented. Using an adaptive
reuse paradigm to compare predicted useful life with actual useful life, a large number
of case studies is analyzed retrospectively. The findings demonstrate that the proposed
method is robust and that the concept of discounting physical life using obsolescence
as a discount rate is valid.

Life of building is in three main types,

1. Physical life
2. Economic life

3. Legal Life

1. Physical life: The length of time that it takes for an asset takes to become fully
depreciated, at which time it provides no additional use. The absolute physical
life is often taken into consideration when companies purchase assets. The
measure is typically associated with assets that have low risk of becoming

Page | 5
IOV-Registered Valuers Foundation

technically obsolete.

Economic life: The economic life of an asset could be different than its actual
physical life. Estimating the economic life of an asset is important for
businesses so that they can determine when it's worthwhile to invest in new
equipment, allocating appropriate funds to purchase replacements once the
equipment's useful life is met.
Legal Life: Duration of an intangible asset (such as a patent or copyright) as
allowed in law. In cases where the economic life is shorter than the legal life of
the asset, the former is used for computing amortization.

3. FACTORS AFFECTING LIFE OF BUILDING


FACTORS INFLUENCING THE LIFE
The life of structures depends on a variety of factors, such as (i) Purpose of use (ii)
socio-economic considerations (iii) Materials of construction (iv)Surrounding
environment; and (v) Degree of maintenance.

1. Purpose : Very long life of even up to 500 years or more would be desired for
monumental buildings such as temples and churches. Public buildings such as
town halls and parliament buildings could be expected to last for 100 to 200
years, whereas private structures such as offices and dwellings for perhaps 50 to
60 years.

2. Socio-economic considerations impinge on the above durations, some of


which tend to reduce the life spans of buildings, while others increase them.
The changing needs of various owners, and indeed the changing face of the city
or area in which the building is located, may cause a building to be obsolete
even before it ceases to be serviceable. In the context of the above proneness
to change, most investors or builders may not want to invest in a building with
an excessive life. On the other hand, owners sometimes try to use an existing
building over and above its life, because demolition and reconstruction may
force them to comply with new planning regulations. Also, once a building
exceeds a certain lifespan, the owner, or even other interested parties, may wish
to prolong its life further, if it is considered a national heritage.

3. The different materials of construction that are used in a building will give
rise to different rates of deterioration. In general, steel and reinforced concrete
will tend to deteriorate faster than masonry; and timber in internal
environments.

Page | 6
L&B/Chapter-5/Cost Approach to Value

4. Surrounding environment: Heat and moisture are environmental factors that


tend to accelerate deterioration. Where steel embedded concrete and structural
steel are concerned, a chloride environment, inclusive of proximity to the coast,
will significantly enhance corrosion. The environment a building is subjected to
will vary from external elements to internal elements and also from seaward
side to landward side.

5. Degree of maintenance :Different building elements may receive different


degrees of maintenance, depending on their accessibility and inspect ability.
The primary objective of maintaining a building is to preserve it in its initial
functional, structural and aesthetic states. This is to ensure that the building
continue to remain in such state and retain its investment value over a long
period of time.

4. TOTAL LIFE, AGE, ESTIMATING FUTURE


LIFE
All things living and non-living wear out with age. The building is no exception to it.
Ideally, the average lifespan of any concrete structure is 75-100 years. But, it is
considered that the average life of an apartment is 50-60 years. Their lifespan can be
improved by carrying out regular maintenance.

A house is a structure made of a combination of elements that degrades with time. Put
together, the environmental impact and the human usage both do their share of
damage. Apart from this, poorly designed homes degrade faster anyway.

Things which are under constant usages like water pipelines, power cables and other
allied services gets degraded over time and worn out after a fixed period of time. Apart
from this, window and door openings, poor construction quality, waterproofing,
painting, layouts of plumbing also result in ageing of a home before time.

The expected economic life of the building under normal occupancy and maintenance
conditions is considered to be as below:.

RCC Framed structure 75 – 80 years


Load bearing Construction 55 – 60 years
Semi permanent structure 30 years
Purely temporary structure 5 years

There are a couple of major financial considerations when making a real estate
investment. First, you want to estimate the property's potential to generate rental

Page | 7
IOV-Registered Valuers Foundation

income, which can usually be done by evaluating the rental histories of similar
properties nearby.

Future life of building : Various life assessment methods are considered to provide a
framework to enhance site inspection objectivity and quantify future service lives - a)
visual condition assessment, b) modification of a reference service life using factors, c)
scales of component condition, d) rates of deterioration, e) failure patterns, f)
probabilistic models and g) service life forecasting

GENERAL Procedure to do the Valuation of Building

1. Measure the Area of building.

2. Adopt suitable Replacement Rate of construction

3. Multiply the area by the unit rate to get the replacement value of the building.

4. Ascertain the age of the Building.

5. Estimate suitable total life of the Building.

6. Assume suitable % age for salvage value.

7. Calculate Depreciation by Straight line method.

8. Depn% = (Age / Total life) x (100 - % Salvage value).

9. If the age is not known or if the building has crossed its service life, estimate
future life and calculate the depreciation by using the formula.

10.

D = Depreciation % age multiplied by the Replacement value will be the


Depreciation Value.

Present Value = Replacement Value – Depreciation Value

Land value by Market approach and building value by cost estimation method for
owner occupied Bungalows, Factories, Public Buildings

There are situations when the combination of approaches is required to be used. e.g.
Factories, school, bungalow, public buildings etc. Such properties are not commonly
traded properties like flats, shops and offices. In such a cases combination approach is

Page | 8
L&B/Chapter-5/Cost Approach to Value

used. Land is valued by market approach. Land is commonly traded, hence market
approach is adopted, while for estimating building cost, cost approach is used i.e.
estimating replacement cost (cost of construction less depreciation)

Example: A factory building of 400 sqm is constructed on plot of 1,000 sqm. The plots
are traded at Rs. 8,000 per sqm. The cost of construction of factory shed is Rs. 10,000
per sqm. The building is 10 years old. The total estimated life of shed is 40 years. What
is the value? ( Assume salvage value 10%)

Solution:
Land component by market approach
Land area = 1,000 sqm
Rate adopted = Rs. 8,000 per sqm
Land component = Rs. 80,00,000/-

Building component by cost approach


Shed area = 400 sqm
Rate adopted = Rs. 10,000 per sqm
Replacement cost for new = Rs. 40,00,000/-
Rate of Depreciation = (100-10)%/ 40 = 2.25% per year
Depreciation = 2.25% x 10 = 22.5%
Depreciated value = 40,00,000 x (100% - 22.5%)
= Rs.31,00,000

Total Value = Land component + building component


= Rs.80,00,000 + Rs. 31,00,000
= Rs. 1,11,00,000

5. DEPRECIATION
The word 'depreciation' generally denotes physical deterioration, i.e., perishing due to
age, or simply wearing away. However, in valuation practice this word would embrace
not only physical deterioration but also functional or economic obsolescence. Broadly
speaking, depreciation is the loss, not restored by current maintenance, which is due to
all the factors causing the ultimate retirement of the property. These factors embrace
wear and tear, decay, inadequacy and obsolescence.

A reliable estimate for the allocation for depreciation or present value of property
should be based upon age, inadequacy, obsolescence, usefulness and other factors as
well as the physical state. Physical condition alone is neither depreciation nor the sole
measure of depreciation, in the sense of either cost or value. In valuation, the term
generally used is accrued depreciation which is the loss in value from reproduction or

Page | 9
IOV-Registered Valuers Foundation

replacement cost new due to all causes except depletion, as of the date of valuation.
This differs from accounting depreciation, which is the difference between the original
cost and current book value of an item. Accrued depreciation is measured as of the
valuation date and applies to improvements.

Accrued depreciation reflects the demand side of the market. The cost of construction
represents the supply side. Cost and value are most similar when improvements are
new and represent highest and best use. As structures age, they suffer physical
deterioration and obsolescence and, as a result, lose value relative to newer structures.
This loss in value is caused by a perceived diminished utility for the property. The
demand curve, therefore, shifts to the left (see Fig. 5.5) and sales prices are lower than
those of properties with new improvements. The true measure of depreciation, then, is
the effect on marketability and sales price. The valuer stimulates depreciation (loss in
value) by analyzing the market and subtracts depreciation from replacement cost new
to estimate the market value of improvements. Only items included in replacement cost
new can be depreciated".

CAUSES OF DEPRECIATION
There are in general four causes of accrued depreciation viz. physical deterioration,
functional obsolescence, economic obsolescence and technical obsolescence.

Physical Deterioration: Physical deterioration is the loss in value due to wear and tear
and natural forces. Practically every structure suffers natural physical decay. This may be

Page | 10
L&B/Chapter-5/Cost Approach to Value

due to tension, compression, friction and chemical changes in the materials used in
construction. Some causes of physical deterioration are normal use, breakage, neglect,
infestation of insects, moisture and the elements. Maintenance can slow physical
deterioration but not completely arrest it.

Physical deterioration may be classified as curable or incurable. Curable physical


deterioration occurs when the value added by a repair equals or exceeds the cost of
repair. A prudent property owner would make such repairs, which correct conditions
caused by delayed maintenance. Examples include leaky plumbing, broken windows,
cracked paint or plaster, a leaking roof, etc. Incurable physical deterioration is that
which, as of the date of valuation is not economical to repair or replace; that is, the cost
of repair exceeds the gain in value. Physical components that are not easily seen, such
as the structural framework, foundation, sub flooring and ceiling structures, suffer from
incurable physical deterioration.

The classification of depreciation as curable or incurable will vary with the age and
location of properties. Extensive renovations may be advisable in an improving
neighborhood but not in a declining one.

Functional Obsolescence: Functional obsolescence is loss in value due to inability of


the structure to perform adequately for the function for which it is used, as of the
valuation date. Functional obsolescence can result from changes in demand, design
and technology. It can take the forrn of deficiency like having only one bathroom when
two are required or need for modernization like an outmoded kitchen, etc. In any case,
potential buyers perceive a loss in utility. Therefore, the price offered may be lower due
to reduced demand.

Functional obsolescence can also be classified as either curable or incurable, depending


on whether the cost to cure is economically justifiable or not as of the valuation date.
Example of curable functional obsolescence include old-fashioned bathroom and
kitchen fixtures, an outdated hot water heater, fewer electrical outlets than required per
room, low-hanging pipes in commercial or industrial buildings, etc. In all these
examples, the increase in value from correcting the problem usually exceeds the cost.

Incurable functional obsolescence occurs when the cost of correcting the condition
exceeds the increase in value. Examples include outmoded design, poor room
arrangement, no garage (and no space to build one), inadequate column spacing in a
warehouse and inadequate frontage in commercial structure.

Sometimes an entire structure can be functional obsolete because of its location; for
example, a large, custom-built house in a remotely located neighborhood or a small,

Page | 11
IOV-Registered Valuers Foundation

poor-quality house in a high-priced neighborhood.

Economic Obsolescence: Economic obsolescence is loss in value resulting from


impairment in utility and desirability caused by external factors i.e. outside the
property's boundaries. Economic obsolescence often arises from changes in the highest
and best use of the property due to market shifts or rezoning government policies. It
may be the result of inadequate public services, lack of adequate facilities for trade in a
business district, narrow streets and heavy traffic in a residential neighborhood, or
proximity to inharmonious industrial or commercial land use. Economic obsolescence is
seldom, if ever, curable. An undesirable location can affect land as well as improvement
values. The effects should therefore be separated.

Technological Obsolescence: Old load bearing structures with thick walls are not
preferred now in the city areas. Every one now desires to stay in high rise R.C.C. framed
structures having thin partition and external walls. This is now possible due to
technological advancements. Timber structures are also now replaced by R.C.C. framed
or steel framed constructions. Wooden windows are replaced by aluminum windows.
Now modem technologies and planning concepts have made it possible to design and
erect even an intelligent building.

MEASURING FORMS OF DEPRECIATION


Depreciation is not an isolated effect. The existence of one form of depreciation can
cause others to occur. For example, if owners do not make repairs to a property that
has significant incurable functional obsolescence, physical deterioration will occur.
Although it may not always be necessary to distinguish and label different forms and
causes of depreciation, it is essential to account for all depreciation. Depreciation, like
land value and replacement cost new, must be estimated afresh each time a property is
revalued.

Curable physical deterioration: Curable physical deterioration is measured by the cost


to cure the defect. If, for example, a kitchen has been damaged by broken water pipes
and will cost Rs. 7,500 to repair, but this repair will increase the property's value by Rs.
7,500 or more, then Rs. 7,500 or the cost to cure is the amount of curable physical
deterioration. To be classified as curable, the defect must be in such poor condition
that it should be cured as of the date of valuation.

Incurable physical deterioration: Incurable physical deterioration is physical


deterioration that is not economical to repair. This usually includes all basic structural
items that are still functional, such as a deteriorated roof. Functional items should be
partially depreciated as of the valuation date. For example, assume an air conditioner

Page | 12
L&B/Chapter-5/Cost Approach to Value

having an economic life of 10 years and an effective age of 3 years. Using straight-line
depreciation (see below), physical depreciation can be calculated.

Functional Obsolescence: Curable functional obsolescence relates to defects within


the structure that should be corrected. In analyzing such conditions, it is necessary to
determine what is included in replacement or reproduction cost new. If physical
depreciation has already been applied to the items in question, then these cannot be
repeated again.

As an example of a curable functional deficiency, assume that the current market


expects air-conditioning but the property has none. The cost to add air conditioning to
the existing structure is Rs. 15,000. The cost to include air conditioning in the original
construction is Rs. 12,000. In this case, replacement or reproduction cost new has not
included air conditioning, and the measure of functional obsolescence is simply the
cost to cure: Rs. 15,000.

Incurable Functional Obsolescence: Incurable functional obsolescence is a condition


that decreases the utility of the property and is not economically feasible to cure as of
the date of valuation. Buyers who can live with the deficiency will accept it if
compensated by a lower purchase price or lower rent. This condition is best measured
by the comparable sales analysis. For example, consider a house with an outdated
design. If sales analysis shows that flats with this kind of design sell for Rs. 50,000 less
than otherwise comparable flats, functional obsolescence is Rs. 50,000.

Incurable functional obsolescence can also be measured by the capitalization of income


loss. In this method, the loss in value is computed by capitalizing the loss in income
associated with the obsolescence. Consider a retail store that has high ceilings, poor
placement of electrical fixtures and inadequate doorway sizes. These conditions cannot
be cured economically at the time of valuation.

A prudent buyer would realize that rents are adversely affected by these deficiencies. A
rental study can determine estimated rent loss. The present value of the loss in rent
over the remaining economic life of the building will provides the amount of functional
obsolescence.

Economic Obsolescence: Economic obsolescence is loss in value from forces external


to the property. These are considered incurable because nothing can be done to rectify
the defects. In analyzing of economic obsolescence, loss in value attributed to the land
should not be applied.

Like incurable functional obsolescence, economic obsolescence can be measured either


by comparable sales or capitalization of income. As an example of the former, assume

Page | 13
IOV-Registered Valuers Foundation

that a house is located on a busy street. Analysis of sales reveals that this condition
reduces market value by Rs. 50,000. The land is valued at Rs. 30,000 less than
comparable sites not located on a busy street. The additional loss in value ofRs. 20,000
is attributable to the improvements.

As an example of the measurement of economic obsolescence using income data,


assume that properties located on streets in the subject neighborhood and not affected
by heavy traffic rent for Rs. 6,000 per month. The subject property, which is affected,
rents for Rs. 5750 per month. The monthly rent loss is thus Rs. 250 (Rs. 6,000 - Rs.
5,750). Multiplying the monthly rent loss by a developed gross monthly rent, multiplier
of 100 (assumed) results in a Rs. 25,000 loss of value.

Methods of Calculating Depreciation

There are several methods of a calculating depreciation. Some of these are arbitrary
whereas others are based on theory. The methods have been divided into two broad
heads:

• Non interest methods

• Methods based on interest theories

Non Interest methods of valuation

Direct appraisal method, which is also commonly used is also unwarranted: It is


sometimes assumed that an expert value’s intuition can decide arbitrarily, by merely
inspecting a property, how much it has depreciated in value without applying any
particular process of reasoning or analysis or without reference to any cost data
applying to the property. This method, too, is arbitrary having no basis and perhaps will
not stand to close scrutiny in a court of law.

Arbitrary lump sum method: Although now practically abandoned, many enterprises
make arbitrary lump sum allocations as expense for depreciation. This method also has
no basis.

Constant percentage method: Depreciation as a percentage of revenue Estimating


cost depreciation as a percentage of revenue involves the same motive as found in the
arbitrary lump sum method. The percentage of gross revenue method seems not to
have been used much.

Sum of digits method: This method is also known as the sum of the years' digits
method. In this method, the sum of the arithmetic series of numbers from 1 to n is
used, n represents the probable life of a unit, as the denominator of a series of

Page | 14
L&B/Chapter-5/Cost Approach to Value

fractions. The numerator of the fraction for any specific period is the expectancy of life
as of that period. If six years is the estimated probable service life of a unit, the
denominator of the fractions is 1 + 2 + 3 + 4 + 5 + 6 or 21. The depreciation
allocations for the first to the sixth years are found by multiplying the depreciable cost
by the series of fractions; 6/21, 5/21, 4/21, 3/21, 2/21, and 1/21. Total depreciation over
a life span of six years = 6/21+5/21+4/21+3/21+2/21+ 1/21 = 21/21 or 100%

The sum of the years' digits method is an arbitrary method of allocation in which the
depreciation base is held constant and the yearly rate decreased. As with the declining
balance method, the result is to allocate the larger amount of depreciation to the first
year and to decrease the amount each succeeding year. The method has not been used
to any great extent, because it is difficult to apply to groups of units, and because the
declining balance method achieves similar results with greater ease and flexibility.

Declining balance method: This method is also known as 'constant or equal


percentage', or 'written down value' method of depreciation. The method is a variation
of the fixed percentage depreciation idea, in which the fixed percentage is applied to
the unallocated balance of the base instead of to the cost of the new base. In results,
the declining balance method closely parallels the sum of the digits method. When the
declining balance method is applied to a single property unit, the annual depreciation
is always a fixed percentage of the unit's unallocated base at the beginning of the year.

Let, n Expected life of the structure in years.


P Constant percent by which the structure loses its value each year.
C Value at the beginning (new).
F Value at the end oflife (final or scrap value), i.e., at the end of n years.
M Number of years after which value is to be found.
Vm Value after m years.

Value of the property at the end of the first year or beginning of the second year

= C - CP = C(1 - P)

Value of the property at the end of the second year or beginning of the third year

=C(1-P) – C(1-P)P
=(1 - P)(C - CP)

=(1 - P) C(1 - P)

=C(1-P)2

Straight line depreciation method: The declining balance and sum of the years' digits

Page | 15
IOV-Registered Valuers Foundation

methods distribute the depreciation more heavily in the early life of the unit than in the
latter periods. The straight line depreciation method allocates the depreciable base of a
property unit uniformly throughout its service life except when the estimate of service
life is changed. This fixed percentage depreciation method is more widely used in
depreciation calculations than any other. It is the one method most generally used for
determining depreciation for tax purposes and for profit and loss financial statements.
It is the method prescribed by most agencies.

Let, n Life of the property, in years


C Original cost or value at the beginning
F Final value at the end of useful life
D Depreciation

Then, D- (C-F)/n

METHODS BASED ON INTEREST THEORIES

Sinking fund method: It will be recalled that the sinking fund is an amount set aside
each year and invested at compound interest so that after a specified period one gets
predetermined value of asset less salvage. The sinking fund amount goes on
accumulating at compound interest and is meant to meet the future liabilities of
replacement. The difference in sinking fund and depreciation should be noted. Sinking
fund allows for recoupment of capital but in depreciation there is no such provision.

The sinking fund equation S= R/ {(1+r) n-1}

Where, RSinking fund rate


n = Total life of the structure or machine and the amount per annum, A, formula
A = (1+R) m -1
R

Page | 16
L&B/Chapter-5/Cost Approach to Value

PRACTICAL ASPECTS OF DEPRECIATION


The various methods discussed in the previous sections regarding estimating
depreciation of buildings or structures give different results. This may cause confusion
as to which method would be appropriate. The two common methods which are used
are:

The constant percentage method (declining balance method) is sometimes simply


referred to as the 'linear method' and Eq. (5.2), i.e., V = C(1 - P)" is used. In this case, no
scrap value is assumed.

In the straight line method the depreciation per cent D is calculated as follows:

D= (Total life - Future life) x (100% less salvage value)

Total life

This is a simple equation which is very often used for estimating depreciation of
existing buildings. For estimating depreciation by this method the total life, future life
and percentage salvage value are necessary.

Estimation of the Total Life of the Structure: There are no fixed rules for estimating
the lives of various types of structures which depend upon many factors such as quality
of construction, maintenance, etc.

Estimation of the Future Life of a Building: For estimating the future life of a building
a thorough inspection of the property, preferably by a structural engineer, is necessary,
since no mathematical equation or mechanical device is available for determining
future life. Some of the factors affecting future life are:

1. Condition of foundation - If the foundation has settled then the future life is
considerably reduced.

2. Type of construction material used - If the construction material is of superior


quality the life of the building may be prolonged.

3. If cracks have developed, the nature of the cracks should be studied and the
possible effect on the future life should be ascertained.

4. The condition of wooden parts such as doors, windows and rafters.

5. Condition of repairs and whether maintenance is periodically done. The general


condition of the building should be seen from all major aspects.

6. Whether the building is subject to any external forces such as passing of the
railway lines close by or effect of any vibration in the vicinity of the building, etc.

7. The future economic life of a building normally includes consideration of

Page | 17
IOV-Registered Valuers Foundation

physical condition and functional and economic or external obsolescence.

If a particular structure has been constructed in two parts and the condition of the
structures is different, then different future lives may be estimated for each portion.
Again, if a different foundation is used for upper floors when constructing on an old
existing building, then too, different future lives, i.e., depreciation, may be adopted.

6. REPRODUCTION COST/ REPLACEMENT


COST, DEPRECIATED REPLACEMENT COST
COST APPROACH: ACCOUNTING AND PLINTH AREA RATES
METHOD
Following extract are from the Guide Lines for Valuation of Immovable Properties, 2009.
Valuation Cell, Income Tax Dept., Ministry of Finance, Govt. of India (pg. 36)

ACCOUNT METHOD
If the assesse has maintained proper books of accounts wherein all details are correctly
mentioned duly supported by authentic vouchers and no defects are pointed out and
the books are not rejected then the figures shown therein have to be followed for
determining the cost. If the assesse has produced less vouchers for some of the
materials, the same is estimated and added at the market rates. Similarly, the quantum
of labor payment is assessed and if the assesse has maintained proper account, the
total cost is worked out on the basis of detailed produced by him. We rarely come
across such cases where the assesse submits complete technical accounting along with
justification statements of materials and labor. Such cases appear where the assesse is a
professional builder or has taken huge loans and payments made through financial
institutions. In such in tanks, the VOs should be more vigilant in pointing out the items
and specifications which may have got escaped from the assessee's submission of facts.
Such items can be valued and added separately. However this method yields to a near
to perfect valuation, if the accounts are correctly maintained.

PLINTH AREA RATES AND COST INDEX METHOD


This is a commonly used method for determining the cost of a building by comparing
with the known cost of a building. The cost of a building entirely depends on the major
factors –
1. The area and specification of the building

Page | 18
L&B/Chapter-5/Cost Approach to Value

2. The cost of materials and labor. The first one is covered by the plinth area rate
and the second one cost index.
The known cost of a completed building (Standard building) is divided by its plinth area
to arrive at the Plinth Area Rate (PAR). For determining the basic co t of a similar
building its plinth area i multiplied by the PAR. The extra cost involved in providing
richer specifications compared to the standard building, whose plinth area rate was
determined, is added to the basic cost to arrive at the completion co t. These are
usually termed as extra items. For instance, if PAR was determined for a building with
cement concrete flooring then for determining the cost of a building with marble
flooring, the additional cost involved in flooring is to be added. There are instances
where the additional cost due to richer specifications is more than the basic cost arrived
by multiplying the plinth area by the PAR.

The additional and extra items not covered by the normal plinth area rates should be
give separately under three sub heads i.e. (a) Richer specifications, (b) Extra items and
(c) Deduction items. Depreciation percentage should be based on future life / normal
expected life of such building and should be brought out in the Annex. The plinth area
rates (PAR for various structures including Auditorium, Grain Go down as issued by DG
(CPWD) and a format for computing Cost Price index are given in the Annex of the
guidelines (as applicable on 1-10-2007) but would need to be updated as per last
applicable details. The guidelines also provide a general caution not only for PAR -
reasons for ignoring any document submitted by the owner should be recorded e.g.
vouchers, valuation reports of registered valuer’ s, bill of quantity of materials, accounts
etc.
COST SUMMATION METHOD
In the cost summation method the cost of individual construction components (e.g.
foundation, walls, concrete, wood etc.) are computed on a per "unit-in-place" basis. The
total cost of each component is found by multiplying these varied per unit costs by the
number of units (sq. foot, length ete.), The component costs are then summed up and
combined with applicable indirect cost to obtain a total replacement cost estimate. The
unusual or special purpose properties may conveniently be valued on such basis of
costing.

By this method approximate estimates are prepared for type-designed structures or


units consisting several works after comparing with the past records of expenses for
such works. The estimate of a building is prepared considering the cost of building
structure, cost of internal water supply, sanitation and electrification works. But
allocation of cost for external services, if any, should be provided in the total estimate.

Page | 19
IOV-Registered Valuers Foundation

The external services include-

1. Clearing site, including the demolishing of any old building, damage of existing
roads or adjacent property that may occur due to construction of the building,
or

2. Forming site approach roads or paths,

3. Fences, gates, boundary wall, shades etc.

4. Laying of water supply, drainage, electric cables, gas mains, fire mains lines from
the property line to service main,

5. Turning or planting trees etc.

The costs to be computed may be either actual cost, or capital cost or prime cost. The
ACTUAL COST is the actual expenditure involved in order to complete a work excluding
profit but including all other incidental, establishment and travelling charges. All such
expenditure in connection with the work are accounted regularly in a register book and
are summed up after completion to know the actual cost. This is not the estimated cost
and generally maintained for the builder's own satisfaction.

The term CAPITAL COST is used to indicate the actual amount incurred in connection to
complete a work. The expenditure required for surveying, designing, planning,
'drawing, supervision, legal expenses, travelling, taxes, charges for electricity and water
supply fees, establishment, contingencies and all other miscellaneous charges required
in connection with the work are included in the capital cost. But the capital cost does
not include profit and interest due to investment of money. Cost of land where
required are also included in the capital cost.

PRIME COST is the net cost or purchase cost of articles at shop, and refers to the supply
of the articles only and not to the carrying out of work. No profit for such supply of
materials is reckoned. During preparation of an estimate it is not always possible to
specify the exact type or brands of materials required as in the case of door and
window or water supply and sanitary fittings according to the choice of the owner. The
owner may supply such articles or may instruct a contractor to purchase such articles
for the work. The fitting and fixing charges are paid separately. In order to execute such
items of work a reasonable amount is provided in the estimate as PRIME COST.

Page | 20
L&B/Chapter-5/Cost Approach to Value

QUANTITY SURVEY METHOD


In quantity survey method, all the various labor and material costs are separately
itemized, as INDIRECT COST. This method tends to produce the reproduction cost
estimate rather than a replacement cost estimate and may be useful to contractors
while bidding on construction projects.

The process involves finding out the quantities of works from the detailed
measurements of various items of works in a scheme. The estimator takes off the
various dimensions from the plan, sections and drawings and tabulates them in a
measurement sheet also called dimension paper.

BILL OF QUANTITIES or schedule of quantities is a complete list of all items of work


involved in connection with the estimate for a project together with the description,
quantity and unit of rate against each item of work. When the bill of quantities are
multiplied by their corresponding rates of the items and totaled it gives the estimated
cost of the project. Bill of quantities is also required to calculate the quantities of
different materials required for the project.

During execution of a project there may be certain type of works which are not
included in the schedule and cannot be paid by measurements viz., special type of
architectural works, dismantling partition wall under water, taking out root of trees
during earthwork in excavation for foundation, trenches etc. Costing on the basis of
actual quantity of materials and labor hours required to complete the job is denoted as
'Day Work'.

The prices are to include cost of materials, labor, overhead, supervision charges and
profit. The total cost may take into account all these items.

TRENDED ORIGINAL COST METHOD


The trended original cost method involves the trending over time of historical cost of
factors obtained from a construction cost index. It can be useful in the valuation of
complex- structures for which the comparative unit or unit-in-place methods would be
either inapplicable or difficult to apply. It is particularly necessary to have an up-to-date
construction cost index for the purpose and the original cost figures should be
accurate.

INDEX OF BUILDING COST indicates the comparison of cost for any specified item in
percentage basis over a certain year with respect to the price of a particular year known
as "base year". By this method the approximate cost of construction of a building may
be worked out.

Page | 21
IOV-Registered Valuers Foundation

Consider the year 1987 is based with datum = 1 for the construction cost for a specified
type of building at a place and the construction cost index at that place for same type
of building in the year 2014 is 2.50. So the index indicates the cost of construction in
the year 2014 is 150% more than that in the year 1987. There are various methods for
preparing the cost index number but any of the methods to estimate building cost
does not give realistic results because rise in cost for all the building materials and
labor are not uniform and vitiated by heterogeneity.

The above four methods are not to be taken as exhaustive. Other techniques of
estimating are also in vogue. One such is the BAY-METHOD. The approximate estimated
cost by Bay- method = Number of bays in the proposed structure x cost of one such bay.
Bays are compartments or similar portions of a structure. When the area of a structure
consists of similar cabins or parts such as a go down, a railway platform, factory shades
etc., these are built up with intermediate columns or with roof trusses on walls placed
at equal distances with same roofing on top of those supporting members. The area
may then be divided from center to center of the supports and each division treated as
a Bay. The cost of construction of one bay may give a reasonable estimate of all others.

This is a reliable estimating process but the following points should be considered:

1. End bay should be considered separately due to its end wall (Gable Wall).

2. Current rate per bay should be worked out from the previous recorded rate.

3. The location, specifications and drawings should be the same.

The cost data may be available from the sources of Government publications and ought
to be supplemented or modified with locally gathered cost data. The cost data should
also be updated or revised periodically. Regardless of source, cost data should be valid
against cost of bench-mark properties. Local modification is necessary to fine tune
them for the purpose of application in a particular case.

7. VALUE OF LAND
In estimating land value or the purposes of cost approach, the sales data constitute the
primary source. When data for sale of vacant land are acquired, the land residual
method can be resorted to. It means that the estimated construction values are
subtracted from sale price. The resulting land residual can then serve as substitute for
vacant plot sale in determining average unit value of land. Thereafter, adjustment on
account of size, depth, shape, utilities, ingress and egress and similar factors can be
made. Another method of land valuation may be his capitalization of land rent from the

Page | 22
L&B/Chapter-5/Cost Approach to Value

cost of development method. In the latter, it is necessary to estimate the present value
of projected net revenues from development and sale of land. Then, there may be the
subtraction method in which land values are simply calculated as a percentage of the
total value estimate for the entire property arrived at by the sales comparison approach
or income approach. Once estimated, the land values should be reviewed and updated.

In view of the known fact that the documents of transfer, very often than not contain a
gross undervaluation of the transacted property, whether in a sales transaction, or in a
gift or mortgage. This often poses a stumbling block in the way of relying on the
document of transaction for the purpose of valuation of land or property comprising
both land and building. In such cases, the best way to assess the correct market value
of property is to look at the transactions in land acquisition or value of registered
transactions of collateral security in mortgages to the Banks. Because, in such cases,
people have a tendency to over-value the land/property for obtaining higher rates of
compensation or of advances, as this cause may be. Whether the upper value of land or
property recorded in the document of conveyance and the lower value given in the
transactions registered, an approximation to the market value can be reached by
resorting to prudent 'plus' and 'minus' factors.

Value of land has to be arrived at for the year in which suit premises (Premises for
which contractual rent is in dispute in the Court) is first let. In case of any other
situation, date shall be as provided under the Act. In case of Maharashtra Act under
Section 7(15)(a)(ii) of Act of 1999, Licensee who acquired status of 'deemed tenant'
under earlier Act, rent has to be fixed as on 1-2-1973 though year of construction and
date of occupancy may be much earlier.

1. Valuer has to collect instances of sales of land in the locality for the relevant
year and compare with land of suit premises. Land characteristics and land
comparison technique are discussed in previous chapters. This will give value of
land appurtenant to the suit premises. Sometimes, this is not necessary as land
purchase document is available with landlord. Having determined cost of land,
valuer should add for expenses like brokerage charges, stamp duty and legal
expenses for the conveyance.

2. Valuer should also allow for interest on idle capital (Investment in land) because
the investment will start yielding return only after completion of the building
and occupation by tenants. Generally, interest is allowed for full period of
construction.

3. In case, rent has to be fixed for upper floor premises built after some years of
erection of ground floor, charges for brokerage, stamp duty may not be

Page | 23
IOV-Registered Valuers Foundation

allowed on land values. However, potential value of land pertaining to upper


floor would be permissible, even if land return on lower floor construction was
allowed earlier. This concept was approved in the case of Madhukar Tatke, by
the Bombay Court. Similar view was taken in case of RajnikantSheth by the
Gujarat High Court. The court held, "1 think it would be reasonable to allocate
the rise in the land value proportionately between all the floors in the case of a
building to which floors are added subsequently at the interval of a few years."
Land rate was Rs. 40/S.Y. in 1965 (Gr. Fl.) Rs. 70/S.Y. in 1969 (First Fl.) and Rs.
100/S.Y. in 1971 (Second Fl). Court allocated land rate of 40/S.Y. for ground
floor, Rs. 30/S.y' as price rise in land value for first floor and adopted 60/S.y' as
land rate (Price rise) for second floor even though in 1965 full land was
considered for return on land value for ground floor, by the court.

4. Land pertaining to the building can be worked out by taking into consideration
municipal by laws or development control rules for construction of a building.
In cities where FS.I. Concept is introduced, land area of suit premises will be
based on said rules. However, in small towns or in areas where rules of
construction are 1/3 built of the plot area, land area pertaining to suit premises
can be worked out by adopting 3 times buildup floor area of tenanted building.
This is because building is normally having some open spaces.

5. Madras Court allowed 50% extra land as amenity land over constructed portion
of land while considering land area of tenanted premises and the return on land
investments. If suit premises are only ground floor structure, it will be fair to
adopt land area as 3 times buildup area or buildup area plus all round open
space areas required under rules. This approach will be proper even in areas
where F.S.I. concept is introduced. Building cannot exist without open space and
hence for low rise building this is essential. It is important to note that whole
plot area cannot be adopted for valuation if FS.I. of plot is not fully utilized in
construction.

BUILDING COST

1. Building cost should also be worked out on the date of 1 stletting or as per
provisions of concerned Rent Control Act. The methodology of working out
building cost is already discussed is earlier chapters on cost approach.

2. The valuer should work out total buildup floor area of the building and also
carpet area of each and every premise in the building. This is necessary for

Page | 24
L&B/Chapter-5/Cost Approach to Value

apportionment of total rent and for allowing loading factors for different floors
and different users.

3. Fair cost of tenanted building could be estimated after considering specification


and amenities provided for the building and year of construction of the
building. The cost of construction can also be adopted as per landlord's record
if proper books of account and income tax records are maintained. However,
valuer appearing on behalf of tenants will not have access to these data and he
will have to resort to other methods of cost estimation discussed in Cost
Approach Chapter.

4. Having arrived at building cost, the valuer should add for cost of development
works like compound wall, compound pavement, security cabin, water tank
works. Having determined total cost, valuer should allow for interest on idle
capital. In this case interest is allowed only for 1/2period of construction
because on day one i.e. date of starting construction, investment is nil and on
date of completion full money are invested. Hence average period i.e. 1/2
period of construction is considered for working out returns on idle capital
investments in building and accessory works.

5. In case if building is built in much earlier year and l" letting is done subsequent
year, valuer should allow for depreciation on replacement cost in year of cost
estimation. In such case details of original investment in the building is not
helpful. In such case even interest on idle capital cannot be allowed as there will
be no idle capital. Only notional building cost in year of letting is estimated
whereas building is actually already built few years ago. Such situation arises
when building is initially owner occupied and it is rented at much later date.

6. In case where upper floor to the building is subsequently raised and the said
premises are rented out much later than the lower floor building; the valuer
should adopt total life of building as well as age of the building on the basis of
life and age of lower floor construction, for the purpose of working out
depreciation and sinking fund for the upper floor premises. This approach is
proper because upper floor cannot survive if lower floor collapses. However,
land cost and building cost should be as per the prices prevalent in year of
construction of upper floor.

7. In building cost, valuer should include all costs like civil work cost, plumbing
and electrification costs, cost of development works, cost of amenities like lift,
cost for Architects services and Structural engineers professional fees.

Page | 25
IOV-Registered Valuers Foundation

8. In case, actual building cost as incurred by landlord is adopted and if work is


done departmentally with labor contract, add 10% to 15% of such cost towards
contractor's profit which is saved by the landlord. Similarly, if second hand
materials are used in the construction, adopt lesser rate of construction than
normal cost for relevant year.

9. In case suit premises have unenclosed open balcony area, cost of construction
of such area should be adopted at 50% of the rate adopted for remaining area.
Similarly, if on ground floor, unenclosed open area is provided for suit premises,
adopt 1/4rate than normal rate or work out actual cost of such outlay detailed
quantity method.

10. It should be remembered that cost of construction is estimated always on the


basis of total built up floor area of the building including staircase and common
amenity areas like entrance hall and common corridor area but fair rent is
always determined on net carpet area of the premises which is exclusive of all
common amenity areas.

Page | 26
VARIOUS PURPOSE OF

VALUATION I 06
• Valuation of Properties
• Valuation Standards
• Indian Accounting Standards
• Valuer as an Expert Witness in Court
• Valuers’ Functions & Responsibilities

Page | 1
IOV-Registered Valuers Foundation

(This page is blank)


Page | 2
VARIOUS PURPOSE OF
VALUATION I 06
1. VALUATION OF PROPERTIES
As already discussed value is function of time, place and purpose, function purpose plays
major role in deciding value of an asset. Below are few purposes enlisted for
understanding of members.( Please add :partition suits, Insolvency purpose, Financial
Reporting for Company Matters, Share Valuation under section purposes of Valuation)

As already discussed value is function of time, place and purpose, function purpose plays
major role in deciding value of an asset. Below are few purposes enlisted for
understanding of members

Loan or Mortgage

Valuation is required when loans are taken against the property as security. The property
can be prime security or Collateral security.

Buying or Selling Property

Valuers advice is necessary, when it is required to buy or sell a property. A Valuer studies
the comparable in the vicinity and give advice.

Property Tax

Valuation is required to assess the property tax of a property. The basis of the taxes is
the valuation of the property.

Cost of construction

For determining investment during filling of income tax return or during assessment /
scrutiny, ITO would like to ascertain whether you have not used Black-money or
unaccountable income during construction and are not showing less cost of the house

Page | 3
IOV-Registered Valuers Foundation

than actual incurred.

Capital gain tax

To determine the fair market value as on 01/04/1981 or 01/04/2001 so as to arrive at the


Capital Gains a valuation report is required.

Wealth tax

It is worked out as per Wealth Tax Rules. Presently there is no wealth Tax.

Rent Function

Valuation is required to determine the rent of a property. Annual Rent can be established
by comparable or certain percentage of value. This percentage varies from type of
property like residential, commercial or Industrial and location to location.

Compulsory acquisition

Whenever a property is acquired by law; compensation is paid to the owner. To


determine the amount of compensation, valuation of the property is required.

Valuation for Insurance

A valuation report is obtained in order to insure a property correctly. Insurance value is a


combination of the Replacement Values and provisional Expenses. Insurance is required
for insuring the house against damages like Fire, Earthquake, Flood, Riots etc.

Investment

The property gives two types of return, namely capital appreciation and rent. An investor
buys the property for the better returns. He needs valuation.

Family Partition

This is most commonly required so as to make fair and just partition amongst members /
co-partners of Hindu Undivided Family – HUF.

Auction bids

To fix either ‘offset’ value i.e. minimum value people have to bid during action or if you
want to participate in the auction bid, how much one should bid a valuation report is
helpful.

Stamp duty

A stamp value fixed at circle rate is required for registration of property. Stamp duty is
fixed on the basis of such value or consideration paid, whichever is higher .

Page | 4
L&B/Chapter-6/Various Purpose of Valuation

Court fee

To pay for stamp during court cases, valuation is required

Betterment charges

It is levied by Town Planning authority and valuation is required if you want to challenge
it.

Probate

It is the name of instrument awarded by court allowing transfer of property based on the
will of the deceased to their heirs.

Insolvency

The Insolvency and Bankruptcy Code, 2016, the resolution plan is provide for repayment
of debts of operational creditors in such manner as may be specified by the Board which
shall not be less that the amount to be paid to the operational creditors in the event of
liquidation of the corporate debtor. In such a case of insolvency, liquidation value is
required from a registered valuer.

Financial Reporting for Company matters

To manage strategic accounting and financial reporting work that is core to


communicating company performance. financial reporting valuations is required to
report the globalization of business activities and increasing audit and regulatory
scrutiny,

Share Valuation

A valuation of shares to account for share-based payments. you need to value


financial assets (e.g. unquoted investments) and/or liabilities. you need advice on the
impairment testing of goodwill or other assets.

LAND ACQUISITION
GIS can effectively be utilized for all valuation works provided proper database is available.
For government work, valuation for land acquisition is of prime importance. By
application of GIS, land acquisition valuation can be done quickly and accurately. Land
and building method of valuation is generally applied for land acquisition purpose. In
most such cases, the lands are not fully developed by structures standing thereon. In these
cases lands and buildings are valued separately. Added together this gives the value of the
property. By application of GIS a base map can be prepared showing dag number or
municipal number, location and other details of the property. From a computer database,

Page | 5
IOV-Registered Valuers Foundation

ownership details of dag or premises can easily be ascertained. To ascertain the market
value of the land, land prices in adjoining identical areas on the date of notification is
necessary, which can be done by superimposing land sale data obtained from digitized and
computerized, sale transactions information from the Registry Office. This would give a clear
picture of the land value of the particular plot in question. The exercise can be completed within
a very short period provided a database is readily available.

Under normal circumstances compilation of such documents would have taken considerable
time as all the information would have to be collected from various offices like Land
Acquisition Office, Registry Office etc. which is difficult and time consuming.

Even putting up land acquisition proposal has becomes more easy with the application of
GIS and Information Technology. Mouza maps with dag numbers can be superimposed on
satellite imageries. This gives a clear indication of dag nos. which are vacant and dag nos.
which are covered with structures and parts of settlements.

For example, in New Town, Kolkata the said principle has been adopted. It very much simplified
putting up land acquisition proposals. Here the settlement areas were kept separate from
town area for acquisition. Satellite imageries helped immensely in finding out settlements
and fixing of boundary of the township. The maps so prepared were checked in the field
with total station and GPS within a very short period. This helped in verification of land
acquisition proposals.

MUNICIPAL RATING
Valuation for municipal rating is totally different from valuation for land acquisition or
income tax, wealth tax, estate duty purpose or other purposes. Generally municipal annual
value (MAV) is the guiding factor. For valuation of land for municipal rating a certain
percentage of the price of land is taken as the annual value. Outgoing like taxes, etc. are
deducted to find out the net annual value. A certain percentage is again deducted as per the
act to find the Municipal Annual Value.

In case of land with structure, annual valuation is generally determined on the basis of
return from the property. Prevailing market rent is the major guiding factor. On the basis of
the market rent, gross annual return from the property is determined. From this,
outgoings are deducted to find out municipal annual value after again deducting as per
the local act. The principle is followed for residential as well as commercial buildings. For
buildings like Cinema, Theatre, Multiplex etc. the procedure is to find out the gross return
from the sale of tickets, which forms the basis of valuation.

Whatever be the methodology of valuation, determination of tax, can be simplified if the

Page | 6
L&B/Chapter-6/Various Purpose of Valuation

necessary database is available.

VALUATION FOR STAMP DUTY


Here also the government uses a unique formula by which area wise value of land and
structure is taken to assess the stamp duty for registration of property. This cannot be a
rational method to be followed for such purpose. If computer database of property sale
values prevailing in adjoining areas are available a valuer can easily determine value of the
property for stamp duty purpose. The role of valuer for such valuation should be
acknowledged and accepted by the government to make the system rational.

VALUATION UNDER DIRECT TAX LAWS


Similarly for calculating values under Direct Tax Laws, a database is necessary. Basic
sections of the act can also be used by putting this database of the Act in a computer and
valuation for taxation can be calculated.

MORTGAGE FINANCE
Property particularly in the lower range of value is usually acquired by either individuals
or family trusts or smaller institutional investors. In most cases such a purchaser
generally proceeds only if part of the capital can be borrowed. The capital is usually
borrowed by way of a mortgage to be secured over the property. It would therefore be
logical that values of investment property purchased by such investors are influenced by
conditions prevailing in the mortgage market.

In today's deregulated economy, the financial services sector has seen considerable growth
and dynamisms. The borrower has had opened up a diversity of sources of funds and
range of financial services.

The traditional sources of funds range from loans made by individuals, private
finance companies, mortgage companies, building societies, banks, insurance
companies, provident funds and such other institutions either privately or government
supported. Individuals are, in general, the major potential source of funds for those
organizations who seek either depositors or policy holders or contributors to their
particular schemes. These funds are then again loaned to others by the lending
institutions. Because of competition in the financial marketplace, the type of borrowing
package available varies considerably. The interest rates likewise vary. However, in
general, these will tend to settle within a comparatively narrow range for the same type
of risk associated with the lending. Interest rates constantly change according to supply

Page | 7
IOV-Registered Valuers Foundation

of money, government policy etc. in response to perceived macroeconomic conditions.

As expected, not all borrowers will obtain mortgage funds on the same basis. Interest
rates as well as the terms and conditions of the loan will vary. Some lenders might not be
willing to lend on a particular type of property. There may also be some other relevant
considerations. It therefore transpires that each mortgage transaction in respect of an
individual property is unique in itself.

The financial breakup of an investment property purchased with borrowed capital


comprises two parts: the purchaser's own capital, known as equity capital, and the
borrowed capital known as mortgage capital. At the time the property is purchased, the
equity capital is simply the difference between the purchase price and the mortgage
capital. Subsequently, the original ratio of equity capital to purchase price, and mortgage
capital to purchase price will change. This is caused by changes in the value of the
property itself and changes in the amount of mortgage capital. The latter changes either
as a result of subsequent repayments of principal, thus reducing the mortgage or
additional borrowings, thus increasing the mortgage. Essentially, equity capital is,
therefore, the difference between the market value of the property at any time and the
outstanding balance of any mortgage loans owed at that same time.

Since the purchase of a mortgage financed property necessitates a commitment to repay


interest and principal under the terms and conditions of the mortgage, it is obvious that this
commitment must be measured against the expectations of the investor and the
characteristics of the property itself.

Page | 8
L&B/Chapter-6/Various Purpose of Valuation

Most often, the investor will view the property as an independent investment. At the very
minimum, it should provide sufficient income to discharge the loan and any other outgoings
not otherwise recoverable from the lessee or lessees, and to meet any tax burden.
Whether the investor requires a surplus beyond this to provide a return on the equity is
another matter. Many investors choose to ignore such a return in order that they can invest in
what is expected to be an appreciating asset which, in future, will provide such a return
when income subsequently increases.

The service of the debt incurred by the mortgage is known as debt service and may
represent payment of interest alone, or both interest and principal. In the latter case, the
loan is being progressively reduced and the investor's equity is increasing progressively. In
the former case, the loan remains at the same level throughout.

It is, therefore, obvious that if an investor establishes the amount of mortgage amount that can
be borrowed on a property together with its cost of borrowing and then sets the specific rate of
return required on equity capital, these in turn will establish the value of the particular
property. This may or may not be the market value of the property.

In reality the mix of capital will vary between purchasers. Some purchasers might be able to
acquire the investment without borrowing. In such circumstances the investor's rate of return
on equity would be equal to the yield on the investment. The rates are one and the same
because the equity represents the full amount of the purchase price. Other purchasers will
borrow, but the return on equity will not be equal to the yield unless the mortgage finance rate
is at that same rate. 15.2 BASIC FINANCE WEIGHTED CAPITALIZATION

The basic finance weighted capitalization equation is:

R= (M x 0 + (E x y) (15.1)

where,R= The overall capitalization ratem, M= The mortgage ratio i.e., loan to value

The annual debt service factor for the mortgage, E=The equity ratio i.e., equity to value,

Y= The rate of return or yield on equity

Although the overall capitalization rate that is derived by the above equation has weighted the
financial components of the investment, it may or may not have an implicit mathematical
treatment of income. As in basic capitalization, it can be viewed as capitalizing a long
term constant income in perpetuity. Alternatively it may be assumed to represent a rate
which can assume other prospects for the income. Whichever view is adopted, it is to be
remembered that it is the investor's required rate of return or yield. This yield in turn is tied to
the prospects of the property.

Page | 9
IOV-Registered Valuers Foundation

EXAMPLE 1

A purchaser is offered a property with a net income of Rs. 52,000 per annum. The purchaser
assumes that a first mortgage can be raised at 60% of the purchase price. The mortgage will
be at an interest rate of 15% per annum. The purchaser will fund the balance of the purchase
price and requires a 10% return on equity. What is the value of the property?

Ans: Rs. 4,00,000

OTHER PURPOSES

ASSET VALUATION UNDER SARFAESI ACT


17.0 Valuation of secured assets, the procedure and practice under SARFAESI Act 2002

The procedures for valuation of movable secured assets and immovable secured assets
are different and they are dealt by separate rules under Security Interest (Enforcement)
Rules 2002. Rule 5 of S.I.(E)Rules deals with valuation of movable secured assets. After
taking possession under sub-rule (1) of rule 4 and in any case before sale, the authorized
officer shall obtain the “estimated value” of the movable secured assets and thereafter, if
considered necessary, fix in consultation with the secured creditor, the reserve price of
the assets to be sold in realization of the dues of the secured creditor. Oxford Dictionary
meaning of ‘estimate’ is ‘a judgment that one makes without having exact details or
figures about size, amount, cost etc’. The rule is silent about involvement of approved
valuer for valuation of movable secured assets. Value means intrinsic worth or cost or
price for sale of a thing or property (Union of India and Ors. Vs. Bombay Tyre
International Ltd. &Ors. AIR 1984 SC 420= (1984)1 SCC 467).

Solution

R = Mf + Ey (15.1)

= (.60 x .15) + (.40 x .10)


= 0.13
V = 52,000 /0.13 Rs. 4,00,000

Wealth Tax Act 1957 (and Wealth Tax Rules 1957) is the only law which deals with
valuation of properties (both movables and immovable). The valuation report by a
registered valuer in respect of any asset specified in column (1) of the table below shall
be in the form specified in the corresponding entry in column (2) thereof and shall be

Page | 10
L&B/Chapter-6/Various Purpose of Valuation

verified in the manner indicated in such Form.

Stocks, shares, debentures, securities, shares in partnership firms and business assets
including goodwill but excluding those referred to in any other item in this table

………….Form No. 6

Machinery and plant…………….Form No 7

Jewelry Form No.8

Works of art………. Form No.9

Life interest, reversions and interest in expectancy…Form No.10

Certificate of quoted shares/debentures of a company…Form No11

The estimate of the value of property is a material fact to enable the purchaser to know
its value.

Rule 2(1) (d) of Security Interest (Enforcement) Rules defines "approved valuer."
"Approved valuer" means a person registered as a valuer under section 34AB of the
Wealth-tax Act, 1957, and approved by the board of directors or board of trustees of the
secured creditor, as the case may be;

SEC.34AB REGISTRATION OF VALUERS


1. The Chief Commissioner or Director General shall maintain ‘Register of Valuers’ in
which the names and addresses of the valuer shall be entered.

Valuers has to apply to the Chief Commissioner or the Director General for
registration along with a declaration (see sub-sec.3 of sec 34AB of Wealth Tax
Act)

Valuation Report to be in the prescribed format prescribed under Rule 8D of


Wealth Tax Rules 1957).

“The report of valuation by a registered valuer in respect of any asset specified in column
(1) of the table below shall be in the form specified in the corresponding entry in column
(2) thereof and shall be verified in the manner indicated in such Form

Table
1 2
Immovable property (other than agricultural lands, plantations, Form O-1
forests, mines and quarries
Agricultural lands (other than coffee, tea, rubber and cardamom Form O-2
plantations)

Page | 11
IOV-Registered Valuers Foundation

Coffee, tea, rubber and cardamom plantations Form O-3

Forests Form O-4

Mines and Quarries Form O-5

1. Sec.34AC deals with restriction on practice as registered Valuers.

2. Sec.34AD deals with removal & restoration of registered Valuers.

According to Sec.37 of SARFAESI Act, the Act is in addition to any other law for the time
being in force. The provisions of Wealth Tax Act 1957 and the Wealth Tax Rules 1957
which are not inconsistent to the provisions of SARFAESI Act can be followed. The valuer
should not only be registered under Sec. 34-AB of Wealth Tax Act 1957 but also be
approved by bank’s Board. Otherwise his report cannot be considered as valid document
in the scrutiny of the action of the secured creditor by the tribunal under Sec. 17(3) of
the Act. The estimate of the value of property is a material fact to enable the purchaser
to know its value. Therefore it must be verified accurately and reasonably. The valuation
is question of fact. It should be fixed on relevant material. The concept of Reserve Price
is not synonymous with valuation of the property. These two terms operate in different
fields. An invitation to offer is not an offer. It is an attempt to ascertain whether an offer
can be obtained with a margin. Valuation Report of the borrower also can be invited.
Improper valuation may mislead big businessmen from participating in the bid. (Swastic
Agency and Ors. Vs. State Bank of India &Ors.:AIR 2009 Orissa 147).The difference
between the ‘valuation’ and ‘reserve price’ is that the fixation of upset price may be an
indication of the probable price which the property may fetch whereas the valuation is
dependent on relevant material (Duncan Industries Ltd. Vs. State of U.P. &Ors. AIR
2000 SC 355). Reserve Price is the price with which the public auction starts and below
which the bids are not permitted (State of U.P. Vs. Shiv Charan Sharma &Ors. : AIR
1981 SC 1722).Accuracy and reasonableness are two essential things in valuation of
secured asset, If the valuation is not accurate and reasonable, the same cannot be
accepted. (Bhupender Singh Vs.State Bank of Patiala: AIR 2008 Punjab & Haryana
148). In the cited case it was noted that, the house at the time of sanctioning the loan
was higher than the valuation recorded at the time of sale. In one case valuation is
recorded but the valuer has not taken into consideration the new additions to the
property as well as the tenants occupying it. Property occupied by tenant will be higher
in value than the self occupation for the purpose of levy of property tax by municipal
authorities. In another case valuation report is stated to be caused on a particular date.
But the house was over crowded by marriage party and relatives of the borrower on that

Page | 12
L&B/Chapter-6/Various Purpose of Valuation

particular day. Hence taking measurements is practically impossible. This kind of


valuation is known as symbolic valuation. Symbolic valuation is not known to law. In case
of movable machinery or hypothecated goods there is scope to contend that the value is
depreciated and hence approved valuer is not involved under the rule Rule 5. But this
may lead to preparation of wrong estimate(value). Hence it would be appropriate to
amend the Rule 5 involving approved valuer who can prepare estimate (value) keeping in
mind the prevailing market value for the movables.

VALUATION UNDER LAND ACQUISITION ACT 2013


(Valuation with reference to THE RIGHT TO FAIR COMPENSATION AND
TRANSPARENCY IN LAND ACQUISITION, REHABILITATION AND RESETTLEMENT
ACT 2013 – (herein after called as “RTFCT LARR Act 2013).

(Key words: I: Land Act Brief and history, Retrospective operation of LA ACT 1894, II:
Public Purpose and Consent of land owners, III: Time limit for AWARD, IV:, Special
provisions to safeguard food security V: Urgency Clause, VI & VII deals with minimum
compensation payable in the Principal Act and VIII: deals with amendments passed in
Lok Sabha on 10/3/2015).

(Quote) The land acquisition process is carried out under the provisions of the Land
Acquisition Act, 1894 which came into force w.e.f. 2 February, 1899. This Act has been
amended from time to time (in pre-independence and post-independence). So far, the
Act has been amended 17 times. Since 1960's large scale acquisition is also being done
for companies and private sector. Various sections of the Act have also been amended
from time to time by the State Governments to meet their specific requirements.
(Unquote) Source: 31st Lok Sabha Report May, 2012/Vaisakha, 1934 (Saka).

Land acquisition refers to the process by which government forcibly acquires


private property for public purpose without the concurrent of the land owner. The
land owner is not a willing seller, therefore, compensation and the way in which
compensation were payable, is to be fair and reasonable. RTFCAT LARR Act 2013
(The LARR Act) provides for land acquisition as well as rehabilitation and
resettlement (R & R) and replaces the Land Acquisition Act 1894.

Retrospective operation: Where no award under the 1894 LA Act has been made,
the new Act shall apply with regard to compensation in the following
circumstances: where an award has been made but the affected individuals have
not accepted compensation or have not yet given up possession, and the
proceedings have been pending for 5 years or more and where a majority of
individuals in an affected area have not received compensation, then the new law

Page | 13
IOV-Registered Valuers Foundation

shall apply.

Public purpose & Consent Clauses:

When the appropriate When the appropriate Where private companies


government acquires land for government acquires land for purchase through private
public purposes u/s 2(1) public purposes for PPP projects, negotiation or appropriate
Private companies government acquires for private
LA, compensation, R&R shall apply. companies a part of area. S: 2 (3).
S: 2 (1). LA, compensation, R&R shall apply. R & R only shall be applicable
S: 2(2) (a) &(b).
.
(a) A private company purchases
Public Purpose Activities: S: 3(za) land,equal to or more than such
read with S: 2(1). limits in rural areas or urban areas,
(a) for strategic purposes, (b) for (a) For public private partnership as may be prescribed by the
infrastructure projects, (c) projects, where the ownership of appropriate Government through
project for project affected the land continues to vest with the private negotiations with the owner
families, (d) project for housing Government, for public purpose as of the land in accordance with the
for such income groups to be defined u/s 2(1). provisions of section 46, (b) a private
specified by the appropriate company requests the appropriate
government, (e) project for (b) For private companies for public Government for acquisition of a part
planned development or the purpose as defined u/s 2 (1). of an area so prescribed for a public
improvement of village sites or purpose, provided that where a
any site in the urban areas or private company requests the
provision of land for residential appropriate Government for partial
purposes for the weaker sections acquisition of land for public
in rural and urban areas, and (f) purpose, then, the rehabilitation and
project for residential purposes resettlement entitlements under the
to the poor or landless or Io Second Schedule shall be applicable
persons residing in areas for the entire area which includes
affected by natural calamities, or the land purchased by the private
to persons displaced or affected company and acquired by the
by reason of the implementation Government for the project as a
of any scheme undertaken by the whole.
Government, any local authority
or a corporation owned or
controlled by the State.
No consent of people is required Consent of 70% for PPP / 80% for Not available (N/A)
Private Companies of affected
people is required. . It includes
consent for amount of
compensation vides section 26 (1)
(c).
R & R: Second and Third R&R: Second and Third Schedules R&R: Second Schedule for private
Schedules shall be applicable. shall be applicable. purchase.
Affected families: S: 3 (c) I to vi. Affected families S;3 ( c ) Affected families: N/A
I and v.
Appropriate Government = specified person: State governments, and the government of Puduchery, Central
government in case of Union Territories except Puduchery and where land acquisition involving more than

Page | 14
L&B/Chapter-6/Various Purpose of Valuation

one state government.


Any person other than appropriate government is “any person other than a specified person”.
Social Impact Assessment (SIA) study is mandatory SIA Study is not known.

Time Limit for award assuming no extension

Possession u/s 38 (1)


Appraisal (EIA) u/s 7

Notification u/s 11

Declaration u/s 19

Individuals u/s 21
Notification u/s 4
Assessment (SIA)

PN (Preliminary

Notification to

compensation
extension) for
(Assuming no
Social Impact

Expert Group

Award u/s 25

Total time
(4) & 7(5)
(2)

(4)
6 months 2 months 12 months 12 months 12 months 12 months 3 to 18 59 months.
months (See note
(see below below)
Note)

Note: After payment within 3 months for compensation; within 6 months for monetary
part of R & R and within 18 months for infrastructure entitlement of Schedule II & III

Section: 10. Food Security (Special Provision to safeguard food security): (a) The
collector has to make sure that no other unutilized land is available before he moves to
acquire farm land. (b) Restrictions/threshold on limit of irrigated multi-crop land and
net sown area per district or state available for acquisition is left to the discretion of state
governments. (c) If acquired, the state government has to cultivate an equivalent area of
land elsewhere as agricultural land. If they cannot do this, then they must deposit an
amount equivalent to its value in an account to be used for the purposes of enhancing
food security.

Section. 40. Urgency Clause in the Principal LARR ACT 2013;

S (40) (1) Urgency clause can be invoked for a public purpose for the Defense of lndia or
National Security or for any emergencies arising out of natural calamities or any other
emergency after expiration of 30 days from publication of S. 21 Notification (Notice to
persons interested) with prior approval Parliament; though no compensation is paid.

Before taking possession of any land, eighty percent of compensation is payable to the
persons interested entitled thereto.

The appropriate Government also can waive from application of any one or many
provisions from chapter II to VI. I.e. Determination of social impact and public purpose,
Irrigated land, Notification and acquisition, R&R Awards and procedures and, if it does so
direct, a declaration may be made under section 19 (Publication ol' declaration and

Page | 15
IOV-Registered Valuers Foundation

summary of Rehabilitation [and Resettlement) in respect of the land at any time after the
date of the publication of the preliminary notification under section 11 (1).

An additional compensation of seventy-five per cent. of the total compensation as


determined under section 27. Shall be paid by the Collector in respect of land and
property for acquisition of which proceedings have been initiated under sub-section (/)
of this section:

Provided that no additional compensation will be required to be paid in case the project
is one that affects the sovereignty and integrity of lndia, the security and strategic
interests of the State or relations with foreign States.

AMENDMENT TO THE PRINCIPAL ACT 2013: An amendment to the Principal Act 2013
was passed in the Lok Sabha on 10/3/2015 which is summarized as below: (Source: ^
TheRFCTLARR_AmendmentBIll_Passed by Lok_Sabha (1)pdf).

Substitution of certain expression throughout the ACT RTFCT LARR 2013: (30 of
2013): For the words “PRIVATE COMPANY” wherever they occur, the words “PRIVATE
ENTITY” shall be substituted”.

New Insertion in Section 3 (ii) (yy) as: “PRIVATE ENTITY” means any entity other than
Government Entity or Undertakings and includes PROPRIETORYSHIP, PARTNERSHIP
COMPANY, CORPORATION, NON-PROFIT MAKING ORGANISATION or OTHER ENTITY
under any law for the time being in force.

New Insertion Section 10 (A) Food Security: 10A (1). Exemption from Social Impact
& Special Provision to safeguard food security: The appropriate Government may, in
the public interest, by notification, exempt any of the following projects from the
application of the provisions of Chapter II and Chapter III of this Act, namely:

1. Such projects vital to national security or defense of India and every part thereof,
including preparation for defense or defense production;

2. Rural infrastructure including electrification;

3. Affordable housing and housing for the poor people;

4. industrial corridors set up by the appropriate Government and its undertakings


(in which case the land shall be acquired up to one kilometer on both sides of
designated railway line or roads for such industrial corridor); and

5. Infrastructure projects include projects under public-private partnership where


the ownership of land continues to vest with the Government.": Provided that the
appropriate Government shall, before the issue of notification, ensure the extent

Page | 16
L&B/Chapter-6/Various Purpose of Valuation

of land for the proposed acquisition keeping in view the bare minimum land
required for such project.

6. The appropriate Government shall undertake a survey of its wasteland including


arid land and maintain a record containing details of such land, in such manner
as may be prescribed by the appropriate Government.

Section 24 deals with Land acquisition process under Act No. I of 1894 shall be deemed
to have lapsed in certain cases & Remedy in the Principle Act; The amendment is the
delay due to court/tribunal proceedings shall not be counted.

Amendment to Section 31: In the principal Act, in section 31, in sub-section (2), in clause
(h), after the words ‘affected families’, the words ‘including compulsory employment to
at least one member of such affected family of a farm laborer' shall be inserted.

In the author’s view, in addition to employment to one member in the family, the
Government to ensure the rest of family members to continue in agricultural
sectors with necessary support.

Section 67A (New Insertion) Hearing to be held by Authority in district or districts to


decide grievances: ‘ The Authority shall, after receiving reference under section 64 and
after giving notice of such reference to all parties concerned, hold the hearing in the
district where the land acquisition takes place for settlement of the objections raised in
the reference.’

Section 87 deals with Government Servants Accountability and the amendment protects
Government Servants thus: Where an offence under this Act has been committed by any
person who is or was employed in the Central Government or the State Government, as
the case may be, at the time of commission of such alleged offence, no court shall take
cognizance of such offence except with the previous sanction of the appropriate
Government, in the manner provided in section 197 of the Code of Criminal Procedure,
1973.".

17.2 8 7 : Section 101: Return of unutilized land after 5 years is amended as “A period for
setting up of any project or five years whichever is later.

17.2 8 8 : S(105) and the Fourth Schedule (1) Subject sub section 3 (1), the provisions of
this Act shall not apply to the enactments relating to land acquisition specified in the
Fourth Schedule.(2) Subject to sub-section (2) of section 106. the Central Government
may, by notification. omit or add to any of the enactments specified in the Fourth
Schedule.; THE FOURTH SCHEDULE (See section 105). LIST OF ENACTMENTS

Page | 17
IOV-Registered Valuers Foundation

REGULATING LAND ACQUISITION AND REHABILITATION AND RESETTLEMENT (13 Laws)


were exempted from R & R Provisions is mended as The Bill brings the compensation,
rehabilitation, and resettlement provisions of these 13 laws in consonance with the LARR
Act, 2013.

17.2 8 9 : (S (113) (1) Power to remove difficulties: (i) for the words "the provisions of this
Part", the words "the provision of this Act" shall be substituted; (ii) in the proviso, for the
words "a period of two years", the words "a period of five years" shall be substituted.

Collective legislative decision of upper houses is awaited.

2. STUDY OF VALUATIONSTANDARDS AS PER


PROVISIONS OF COMPANIES ACT-2013
Until Mar 2018, the Valuers were recognized only under wealth tax act under section
34AB. There was no formal training or specialized education to Valuers but the whole
process was based on experience only. With growing need of specialized training and
education, the profession was brought in the preview of law with duty along with
responsibility along with punishments. This was done under Companies Act 2013.

The provisions about valuation by registered valuer are contained in Section 247 of
chapter XVII of company’s act, 2013. Section 247 has been made effective from 18 th
October 2017. Also, MCA have notified company’s (registered valuers and valuations)
rules 2017. The rules are basically about who could become valuers and the process of
registration and de-registration of these valuers with IBBI.

Section 247(1) provides that where a valuation is required to be made, it shall be valued
by qualified experienced registered valuer.

Section 247(2) provides that valuer should make impartial, true and fair valuation with
due diligence.

Section 247(3) provides that the valuer shall be punishable with a fine of Rs. 25,000 to 1
lakh. If valuer intends to defraud the company or its members then he shall be
punishable with imprisonment up to 1 year and fine of Rs. 1 lakh to Rs. 5 lakh.

Section 247(4) provides to refund the fees and pay damages to the company for the loss
arising out of incorrect or misleading statements.

As per companies’ rules 2017, a valuer of land and building should be graduate engineer
or architect or equivalent with 5 years’ experience or a member of professional institute
(AMIE) established under the act. The authorities (IBBI) shall conduct the examination

Page | 18
L&B/Chapter-6/Various Purpose of Valuation

after 50 hours training through registered valuers organization (RVO) to test knowledge,
skills, values and ethics based on the syllabus. After passing the examination and after
registering with IBBI and after taking certificate of practice a valuer will be allowed to do
the practice.

A valuer shall maintain the record for at least 3 years, comply with the code of conduct,
He/she will make valuations as per internationally accepted valuation standards or
standards adopted by RVO.

A valuation report shall contain


1. Background information of asset
2. Purpose of valuation and appointing authority
3. Identity of valuer and experts involved
4. Disclosure of interest or conflict
5. Date of appointment, valuation date and report date
6. Inspections and investigations
7. Sources of information relied upon
8. Valuation standards followed
9. Restrictions on use of report
10. Major factors in valuation
11. Conclusions
12. Caveats, Limitations and disclaimers

REGISTERED VALUER ORGANIZATION (RVO)


1. Registered under section 25 of companies’ act 1956 or section 8 of companies’
act 2013.
2. Conducts educational courses in valuation
3. Grants membership and certificate of practice after training.
4. Enforces a code of conduct
5. Provides continuous education
6. Monitors and reviews the functioning including the quality of service of Valuers
7. Have a mechanism to address grievances and conduct disciplinary proceedings.

CODE OF CONDUCT
1. Integrity and fairness – Honesty, no misrepresentation, keep public interest
foremost
2. Professional competence and do care – due diligence, updating
3. Independence and disclosure of interest – act with objectivity without bias,
conflict of interest, coercion

Page | 19
IOV-Registered Valuers Foundation

4. Confidentiality- not to divulge confidential information


5. Information management - Maintain written contemporaneous records, be
available for inspection and investigations, and maintain 3 years record.
6. Gift and hospitality – shall not accept or offer
7. Remuneration and cost – In a transparent manner
8. Occupation, employability and restrictions –
9. Refrain from accepting too many assignments, not to conduct the business,
discrediting the profession.
10. No success fee should be charge Z

MONITORING OF MEMBERS
a. To monitor professional activities and conduct of members
b. Members shall submit information about ongoing and concluded engagements
at least twice a year.
c. Monitoring committee shall review information and records submitted by
members
d. Monitoring policy –

i.Frequency of monitoring
ii.Format for collection of information
iii.Obligations of members to comply
iv. Use, analyze and store the information
v. Performance evaluation
vi. Any other matter
vii. To maintain privacy of members
viii.To maintain confidentiality of information received except required by
law
ix. To be non-discriminatory

Page | 20
L&B/Chapter-6/Various Purpose of Valuation

REFERENCE TO VALUATION BY REGISTERED VALUERS, UNDER


COMPANY’S ACT 2013-

SR. No Topic Provision


1 Further issue of share capital Section 52(1)(c)
2 Audit committee Section 177(4)(vi) - Valuation of
undertaking and assets
3 Restriction on non-cash Section 192- Particulars of
transactions involving arrangements and value of assets
directors involved
4 Power to compromise or Section 230(2)(c)(v)&(3)- valuation of
make arrangements with the shares and the property and all
creditors and members assets, tangible and intangible,
movable and immovable, of the
company by a registered valuer.
5 To mergers and Section 230(2)(c) & (d) and (3)
amalgamations of companies
6 Purchase of minority share Section 236(2)
holding
7 Submission of report by Section 281(1)(a) – within 60 days a
company liquidator valuation report containing nature and
details of assets is required
8 Return of allotment Rule 12 of companies rules 2014- in
case of securities allotted as fully or
partly paid up for consideration other
than cash
9 Issue of sweat equity shares Rule 8 of companies rule 2014 –
explanatory statement shall contain
principal terms and conditions on
which sweat equity shares are to be
issued including basis of valuation
10 Issue of shares on preferential Rule 13 of companies rule 2014 - the
basis price of shares shall be determined on
the basis of valuation report
11 Provision of money by Rule 16 of companies rule 2014
company for purchase of its
own shares by its employees
or by trustees for the benefit
of employees
Exclusions from deposits Rule 2(1)(c)(ix) of companies rule 2014
12 – the bonds and debentures should be
secured by the charge of assets and
shall not exceed market value of such
assets.

Page | 21
IOV-Registered Valuers Foundation

REFERENCE TO REGISTERED VALUERS UNDER IBC 2016 –

SR. No Topic Provision

RP shall appoint two registered Valuers under


regulation 27 within 7 days of his appointment to
determine the liquidation value of corporate debtors in
IBC of India
1 accordance with regulations 35. If two opinions are
regulations, 2016
significantly different he may appoint third. The average
of closest two shall be considered as a liquidation value.
IM (Reg. 36) shall contain assets and liabilities

Valuation of assets intended to be sold (Reg. 35) -


IBC of India Liquidator shall appoint two registered valuers under
2
regulations, 2016 Reg. 34(2) to determine the realizable value. AM (Reg.
34(2)) shall contain assets in accordance with Reg. 35.

Section 59(3) – Majority directors of the company, if


Voluntary
formed and opinion that company will be able to pay its
3 liquidation of
debts in full fromthe proceeds of the assets to be sold
corporate persons
in the voluntary liquidation.

IBBI Regulations
4 Reg. 38(1)(c) – To prepare final statement by liquidator
2017

3. STUDY OF INDIAN ACCOUNTING


STANDARDS (IND AS 16) AS APPLICABLE
TO VALUATION
IND AS 16 deals with Accounting & depreciation of property, plant and equipment which
are covered by existing Accounting Standard 10 (Accounting for Fixed Assets) & AS 6
(Depreciation Accounting).

MEANING OF FIXED ASSETS

Fixed assets are assets which are held for use in production of goods, rendering of
services, administrative use, and rental purpose. It does not include assets held for sale.
[Discarded Assets, held for sale are excluded].

Page | 22
L&B/Chapter-6/Various Purpose of Valuation

COMPONENT BASED ACCOUNTING

IND AS 16 requires Component Based Accounting.

It means components of Assets having SUBSTANTIAL VALUE and different useful life
should be recorded as separate asset.

Schedule II of companies act, 2013 also mandates use of component based accounting.
ACCOUNTING FOR REAL ESTATE DEVELOPERS

Existing AS 10 specifically excludes accounting for real estate developers from its scope,
whereas Ind AS 16 does not exclude such real estate developers from its scope.

RECOGNITION PRINCIPLE:

Fixed Asset should be recognized if it is:-

a) Probable that future economic benefit will flow to entity AND

b) Cost can be measured

Existing AS 10 does not give any recognition principle for recognition of a fixed asset.

As per the Existing AS 10, any item which satisfies the definition of a fixed asset should
be recognized as a fixed asset.

INITIAL RECOGNITION

(Acquired, Self-constructed Asset)

At [Purchase price + Import duties + Non-refundable taxes]

ADD: Directly attributable expenses [Installation + test run exp.]

ADD: Expected exp. For dismantling such asset due to legal obligation

Note: in case of exchange of assets, value of asset acquired should be: Fair value of
Asset given up ± cash transfers

Note: Major inspection costs incurred for making assets use worthy should be
capitalized, when incurred.

Note: With regard to self-constructed assets, Ind AS 16, specifically states that the cost
of abnormal amounts of wasted material, labour, or other resources incurred in the
construction of an asset is not included in the cost of the assets. Fixed Asset can be

Page | 23
IOV-Registered Valuers Foundation

subsequently measured at cost or revalued amount.

Note: if revaluation policy is adopted, it should be based on fair value as per IND as 113.

Note: revaluation should be done at regular frequency

Note: revaluation profit should be transferred to other comprehensive items (OCI)


DEPRECIATION

i) Depreciation is a systematic allocation of assets other than land (freehold), over useful
life of asset.

ii) Depreciation is charged to profit and loss account only. It is not to be adjusted against
revaluation reserve.

iii) Systematic means any method which reflects consumption pattern of asset future
economic benefit do not reflect true consumption pattern, unless justified.

iv) Method of depreciation can be changed such change should be prospective and
considered as change in accounting estimates.

v) Useful life and residual value should be reviewed at least once in a year.

Disclosures

i. Measurement basis used for determining carrying amount

ii. Depreciation Methods used

iii. Useful lives or depreciation rates used

iv. Gross carrying amount and accumulated depreciation at beginning and end of year

v. Reconciliation of carrying amount at the beginning and end of year

Disclosures related to Revalued Assets

i. Effective date of revaluation

ii. Whether an independent Valuer was involved

iii. Methods and significant assumptions applied in estimating fair values of items

iv. Extent to which fair values of items were determined directly by reference to
observable prices in active market

v. Revaluation Surplus, indicating change for the period and any restriction on
distribution of balance to shareholders

Page | 24
L&B/Chapter-6/Various Purpose of Valuation

4. VALUER AS AN EXPERT WITNESS IN COURT


INTRODUCTION

Expert Witness: Definition - A person whose level of specialized knowledge or skill in a


particular field qualifies them to present their opinion about the facts of a case during
legal proceedings.

Opinions of experts comes under Section 45 of Indian Evidence Act, 1872.It is a general
rule that no importance should be given to the opinion of a third person. However, if the
case demands the necessity of a specialized knowledge, which, in general, the judge
does not have, exception is given.

PRE-REQUISITES FOR EXPERT EVIDENCE

1. Subject-matter of the case requires opinion of expert

2. Witness called must be an expert in that technical field

3. Expert witness must be a truthful person

4. Expert should have academic qualification besides experience and special


knowledge in the field

EXPERT WITNESS: REGULATIONS


1. It must be proved that the person giving the evidence is an expert.
2. If the Court comes to the conclusion that the person who has given evidence is
not an expert; his opinion has to be discarded.
3. Expert must be cross-examined in court. Mere paper submission of his report will
not be suffice.
4. Expert opinion is a corroborative evidence and should not be the sole evidence
to be relied upon.

HOW AN EXPERT’S TESTIMONY DIFFERS FROM THAT OF AN ORDINARY WITNESS?

• An expert’s evidence is not confined to what actually took place, but covers his
opinions on facts
• An expert can refer to and rely upon experiments conducted by him in the
absence of the other party.

• An expert may quote passages from well-known text books on the subject and
may refer to them to refresh his memory.

• An expert may state facts relating to other cases in pari material similar to the

Page | 25
IOV-Registered Valuers Foundation

case under investigation.

TYPES OF EXPERT WITNESS


1. Testifying experts
2. Educating witness
3. Reporting witness
4. Non-testifying experts
LEGAL ASPECTS
1. Written Submission & Documentary Evidence -” REPORT”
2. Summons
3. Court proceedings

REPORT PREPARATION: The Guiding principle for the Report is the Purpose for which
the Valuation is undertaken.

Purpose
• PRIMARY LOAN
INDIVID UAL REQUEST
• COLLATERAL SECURITY
• INCREASING LOAN
INSTITUTION REQUEST • ARBITRATION

COURT DIRECTIVE • ADJUDICIATION

Basics
1. Identification of Property
2. Location
3. How to get there
4. Boundaries

Site Inspection
1. Surrounding Area development
2. Shape and topography of Land
3. Roads & communication facilities- nearest Bus facility, Railway Station,
distance from centre of Town
4. Water & Sewage facility
5. Commercial potential- basis for arriving

Measure
1. Measurements of the property
2. Difference – land and document
3. What did you do?

Page | 26
L&B/Chapter-6/Various Purpose of Valuation

Documents Required

• PARENT TITLE DEED • AS CONSTRUCTED PLAN

• PRESENT TITLE DEED • COMMENCEMENT


CERTIFICATE
• CONVERSION
CERTIFICATE • COMPLETION
CERTIFICATE
• RTC
• OC
• ENCUMBERANCE
CERTIFICATE • LOAN AVAILED

• TAX RECEIPT • PRESENT STATUS

• KHATA • RENT/LEASE
AGREEMENT
• SANCTIONED PLAN

NOTE :

1. Had you requested for any more documents from Bank/Borrower?

2. Mention documents produced & not produced

3. Any other issue which could affect the Valuation


Check & Verify

1. Actual possession during visit

2. Online : Ownership, Land Records ,Guideline value, Tax

3. RTC of Property

4. Conversion Certificate

5. Classification of Land Use as prescribed by concerned Authority- does it


conform?

6. Has it been notified for Acquisition by any Government body?

Market Value : The Market value is the net important deduction arrived at after the
entire exercise of study and scrutiny. Usually it is the part of the report that the borrower,
banker, lawyer and judge look at with maximum interest.

The market value assessed is affected by numerous factors and considerations such as :

Page | 27
IOV-Registered Valuers Foundation

1. Land Details

2. Building Details

3. Method of Valuation adopted


i. Land and Building separately
ii. Land &Building together
iii. Composite rate in case of Flats

Usage/ purpose

Cost of development of Infrastructure

Amenities & Services

Large Acreage- Provision for Roads, Civic Amenity Sites, Parks

Commercial/ Industrial – road, rail, air, sea access

SUMMONS

A Summons is An Order to Appear before a Judge or Magistrate, or the Writ containing


such an Order. A summons is so important because it provides all parties involved in a
lawsuit with the basic information necessary to participate and remain informed.

1. A judicial summons is served on a person involved in a legal proceeding.


2. Legal action may be in progress against the person, or the person's presence as
witness may be required

3. The summons announces a date by which the person or persons summoned


must either appear in court, or respond in writing to the court or the opposing
party or parties
4. The summons contains the names of the plaintiff and defendant, Title and File
number of the case and The Court and its address

COURT PROCEEDINGS

Oath taking :Under Section 7 of the Indian Oaths Acts, 1873,before a witness is called
upon to give evidence he should be asked to stand before the Presiding Officer of the
Court and take oath.

Identification:

1. Name ( in full)

2. Father’s name

3. Age

4. Occupation/ Qualification

5. Post

Page | 28
L&B/Chapter-6/Various Purpose of Valuation

6. Years of holding post and date whence

7. Present post

Written Submission & Documentary Evidence

1. Written submissions and Documents produced

2. To be confirmed as issued by you


3. Signature and seal to be confirmed

Cross Examination

By Definition is the formal interrogation of a witness called by the other party in a court
of law to challenge or extend testimony already given. The purpose is to elicit favorable
facts from the witness, or to impeach the credibility of the testifying witness to lessen the
weight of unfavorable testimony.

The cross-examiner – Lawyer

Must be effective at getting the witness to reveal the truth


Reveal confusion as to the facts such as time, dates, people, places, wording etc

More often than not, a cross-examiner will also attempt to undermine the credibility of a
witness

Lawyer Typically say’s….

1. You are not Qualified

2. You have submitted report based on Bank’s or Prosecution Agency’s dictates

3. You have not seen the particular property/Valued Asset

4. You have not visited the concerned offices and verified genuineness of
Documents

5. In case you have made any Comments on Legality of Ownership- whether you
are legally Qualified to make Observations on Title

6. In depth on reports- Whether you visited- what date-who accompanied you

7. Answers to specifics of various items in your report

Witness

1. Only Face and Address the Judge

2. No paper can be held by you or referred to during Court proceedings- hence


thorough study to be made before entering Witness Box

Page | 29
IOV-Registered Valuers Foundation

3. The less the witness says, and the slower the witness speaks, the more control
they can maintain under the pressure of a crafty opponent.
4. The key for a witness is to understand the facts that they believe to be the case
and not add additional thoughts to those facts

5. Sticking to the brief known facts is key for the witness, making it difficult for the
cross-examiner to make the witness appear confused, biased or deceitful.

5. VALUATION AND JUDGMENT


These standards set out principles for undertaking valuations. Applying these principles
to specific situations will require the exercise of judgment. That judgment must be
applied properly and should not be used to overstate or understate the valuation result.
The proper exercise of judgment should always have regard to the stated objectives of
the standards applicable to the valuation.

Independence and Objectivity

Many valuations are undertaken for purposes where either the commissioning party or a
third party will expect or need the valuation to be free from bias. The process of
valuation involves making judgments as to the weight to be given to different factual
data or assumptions in arriving at a conclusion. For a valuation to be credible, it is
important that those judgments can be seen to have been made in an environment that
promotes transparency and minimizes the influence of any subjective factors influencing
the process.

Many states have laws or regulations as to who may value particular classes of assets for
various purposes. Many professional bodies and valuation providers have ethical codes
that require the identification and disclosure of potential conflicts of interest. The
purpose of these standards is to set internationally recognized procedures and
definitions for the preparation and reporting of valuation opinions. They are not
concerned with the relationship between those commissioning valuations and those
undertaking them, and matters relating to the conduct and ethical behavior of Valuers is
for professional bodies or other bodies that have a regulatory role over Valuers.

While conduct rules for Valuers are outside the scope of these standards, it is
nevertheless a fundamental expectation that appropriate controls and procedures are in
place to ensure the appropriate degree of independence and objectivity in the valuation
process so that the results can be seen to be free from bias. Where the purpose of the
valuation requires the Valuer to have a specific status, or disclosures confirming the
Valuer’s status to be made, the requirements are set out in the appropriate standard

Page | 30
L&B/Chapter-6/Various Purpose of Valuation

COMPETENCE
Because valuation requires the exercise of skill and judgment, it is a fundamental
expectation that valuations are prepared by an individual or firm having the appropriate
technical skills, experience and knowledge of the subject of the valuation, the market in
which it trades and the purpose of the valuation. For complex or large multi asset
valuations it is acceptable for the Valuer to seek assistance from specialists in certain
aspects of the overall assignment, providing this is disclosed in the scope of work.

VALUERS FUNCTIONS AND RESPONSIBILITIES


Valuer's Obligation in Determination of Value or Estimated Cost

1. To act within the remit of the written scope of work including the objectives of
the exercise.

2. To determine a value that is germane and appropriate (l.e. apposite) and aligned
to the assigned scope of work. The Valuer must determine the appropriate
approach and methodology for data collection for the particular assignment and
also employ them to arrive at the opposite value in light of the assigned task.
Additional needed opinion of specialized professionals for legal opinion or
appropriate tests/ opinion on structural! Other aspects may be sought after
advising the client on their necessity in determining the opposite value.

3. In his report the Valuer may describe his assessment of the suitability of the
particular method for arriving at a monetary value or cost estimate as
determined by him to a particular scope of work to preclude any potential
attempts whether inadvertently or premeditated, to use the report for other
unintended purposes.
4. Valuer much use all required methods and approaches necessary to produce a
credible opinion and back it with an analysis of the evidence available to the
Valuer and his logic support review opinions with relevant evidence and logic;

5. Valuer must strive to avoid reaching false results due to choose of inappropriate
method or gross inaccuracy stemming from application of an otherwise
appropriate method wrongly or with inaccurate computation or some such

6. Any description of the property must include a description of identification


aspects of the property, a statement of the legal rights and restrictions
comprised in the ownership, physical condition, and the characteristics of the
property which enhance or detract from its value. Zoning restrictions, building
codes, easements, leases, etc., are essential elements of the description.
7. The Valuer must be free to select the process considered most appropriate to
reach the result of the valuation exercise as derived from the scope of work and
after considering all the factors that have a bearing on the value and be
presented in a cogent manner. A member Valuer must describe in sufficient

Page | 31
IOV-Registered Valuers Foundation

detail, in each assignment, the nature and meaning of the specific value that is
being determined.
8. A Valuer must invest adequate time for inspection and investigation to determine
opinion.

Valuer's Additional Obligation to His Client

1. The relationship between client and Valuer is not one of principal and agent. The
Valuer is an independent opinion provider. Having acted as a Valuer and
obtained information and access to the property in that capacity, he cannot act
as a negotiator or a party's advocate in the same property or transactions related
to the same property

2. Valuer is bound to declare that there is no conflict of interest in present or any


prospective in future in the said property.
3. To make the client aware when there is any requirement to the fact of the
engagement as Valuer being kept confidential -

i. In any future assignment pertaining to that property within next four


years from any other client, prior to appointment, a full disclosure of the
fact and level of his involvement as Valuer will be made

ii. That details that identify the property or its owner will not be included or
listed or otherwise published in any statement of competency required
from the Valuer by any Professional Association/ public authority as
proof of professional experience without prior consent of the person
concern. however, when required to do so under due process of law or
as proof required in any disciplinary proceeding of any member the
same may be divulged

Responsibility to third party: The Valuation Report is primarily for the use of the
client for the specific purpose it was obtained. However if the stated purpose of
the valuation as detailed in the appointment already included use by a third
party, then this third party also has a right to rely on the validity and objectivity
of the Valuer’s reports and stands in the same capacity as the client who
engaged the Valuer. Members of the IOV Registered Valuers Foundation
recognize their responsibility to those parties, other than the client, who draw
their authority from the client to make use of their reports and rely on the
opinion expressed in the report.

Page | 32
L&B/Chapter-6/Various Purpose of Valuation

VALUER'S OBLIGATION TO ATTAIN, UPDATE, DULY DECLARE


Qualification and Competency

1. IOV Registered Valuers Foundation provides opportunity for Continuous


Professional Development and constant updating of knowledge of its members
through hosting of Annual Valuers Congress, seminars/ national seminars,
training courses by independently its branches/ headquarter or in association
with other agencies. It also provides a system for professional certification
through its exams, material for self-study through publication of a technical
magazine "Indian Valuer" for keeping abreast of case laws, methods, changes in
provisions, rules and regulation governing valuation. A practicing member and a
Fellow member is obliged to attend a combination of such events or appear in
exams in every block of three years to comply with his obligation to constantly
maintain and upgrade his competency in his field.

2. A Valuer may not normally accept an appointment requiring him to value a


property of a type he is not duly qualified to value. If valuation after designated
category is done the following must be satisfied:

i. he fully acquaints the client including in writing the limitations of his


qualifications and provision of any law or rules on this matters and client
gives his written consent Or
ii. he associates himself with another Valuer or Valuers who possess the
required qualifications

A Valuer cannot claim or imply that he has professional qualifications and


experience which he does not possess in reality or to state or conceal his
qualifications in a manner that can lead to erroneous conclusion about his
qualifications.
In this section qualification shall include professional degree, empanelment with
any specific statutory authority and/or membership of and specific category of
membership of IOV Registered Valuers Foundation and other relevant
professional bodies.

A Valuer relies upon documents prepared by engineers, lawyers, architects,


accountants, government officials or a authenticated 0r registered by
government officials, government authorities. The quality of his conclusion may
depend on the quality of these documents. It is sufficient if the Valuer records
the nature and full identifying details of the documents, whether original was
available for verification, who provided them to him if the additional other tasks
were not assigned to validate their authenticity. If the Valuer becomes aware of
special information regarding validity of the documents he should declare to the

Page | 33
IOV-Registered Valuers Foundation

client. However, the Valuer should assure himself that it is within his capacity to
interpret these documents or seek appropriate professional advice.
To aid IOV Registered Valuers Foundation in upholding and propagating sound
and fair professional practices

VALUER'S SPECIAL OBLIGATION IN LEGAL PROCEEDINGS


INVOLVING PROPERTY OR PROPERTY OWNER
1. The Valuer must waive any prior confidentiality undertaking to client to comply
with legal obligation to do so by any order of an authority or law enforcing body
supported by any provisions of statute, ordinance, or court or regulation.

2. In a legal or quasi-legal proceeding including before tax authorities, the valuer is


an officer assisting the statutory authorities/court. He is thus obliged to present
an unbiased opinion of the value of the property and allied details backed by full
disclosure of the truthful and unbiased analysis of facts/basis. In such a
proceeding the Valuer cannot over emphasize points that aid is client or
underplay points adverse to his client for he is a Valuer and not an advocate of
the client's case.

3. An expert opinion provided in course of tax proceedings or before a court of law


or other judicial or quasi-judicial forums, must be preceded by a best judgment
valuation by the Valuer and backed by a formal written valuation report being
prepared.

4. Valuer in any such legal proceeding is providing an opinion on the value. The
legal rights of the ownership of an interest in real property are matters of legal
scrutiny, not valuation opinion, and that the Valuer discharges his obligations in
this regard by stating the sources of these data for legal rights.

5. When two or more potential client’s chance to seek a Valuer’s services with
respect to the same property or any action including legal cantered on the same
property, the Valuer may not properly serve more than one party without due
declaration and formal consent from each such party.

6. When testifying in the court or before a quasi-judicial body or any statutory


authority it is unprofessional and unethical for a member to conceal, belittle, or
add irrelevant information or otherwise deflect attention from or conversely
high-light or play-up any aspect so as to interfere in the proper appreciation of
facts, data, or opinions by the authority which, if fully stated or appreciated,
might change the decision of the deciding authority and be against the interest
of the client.

Page | 34
L&B/Chapter-6/Various Purpose of Valuation

VALUER'S OBLIGATION TO THE PROFESSION, MEMBERS AND TO


THE IOV REGISTERED VALUERS FOUNDATION

1. A member of the IOV Registered Valuers Foundation recognize the distinct


Professional Character of Valuation Practice which is based on synthesis of
technical and applied knowledge drawn from diverse fields like economics,
engineering, architecture, commerce, accountancy, law (including case law,
common law and customary provisions etc.), agriculture, plantation, forestry etc.
but is not subsumed wholly in anyone or more of these fields nor is an adjunct of
any of these or others. Members must not belittle the profession of valuation and
property appraisal in any manner.

2. The Valuer has an obligation to protect the professional reputation


of all Valuers (whether members of the IOV Registered Valuers Foundation or
not) who subscribe to and practice in accordance with or as per standards
comparable to the Standards of Professional and Business Practices and Code of
Ethics of the IOV Registered Valuers Foundation.

3. Members are duty bound to extend respect and professional courtesies to fellow
members and also to members of national and international associations with
which IOV Registered Valuers Foundation has professional association.

4. The IOV Registered Valuers Foundation declares that it is unethical for a Valuer
to injure, or attempt to injure the professional reputation or prospects of any
fellow member by false or malicious statements, or innuendo or subtle
blandishments or selective publication of facts to color conclusions drawn there
from which is at variance from conclusion that would be drawn if all facts were
known and such comparable actions.

5. By virtue of gaining membership it shall be presumed members understand and


undertake to exercise caution and abstain from publicizing defamatory matters
and salacious gossip in print, audio, social media, digital media etc. Such
requirement will not impair right for fair criticism backed by full disclosure and
due notice to the IOV Registered Valuers Foundation, subject member, and fair
opportunity accorded to each to counter them in the same forum and medium.
This duty shall not impair right of legal action and recourses permissible under
law.

6. Members may use authorized designation, acronym to inform, public and client
of affiliation with the IOV Registered Valuers Foundation.
7. It is clarified external liaison with other stakeholders and counterpart agencies,
advocacy on policy aspects are in the domain of the Board of the IOV Registered
Valuers Foundation. Members may make their representation to IOV Registered
Valuers Foundation or any of its Board members for a duly considered position.
Direct communication on such matters with counterparts without prior consent
of the Honorary General Secretary will be unethical. It requires due deliberation
to present a considered stand of IOV Registered Valuers Foundation. It is

Page | 35
IOV-Registered Valuers Foundation

unethical for a member oppose the considered stand taken by the Institution' of
Valuers in direct communication to stakeholder agencies and government while
holding himself out as office bearer of IOV Registered Valuers Foundation or its
branches. In general, it is unethical for a member elects to make representations
directly on any such matter while holding out as or not doing enough to prevent
the impression that they are speaking for IOV Registered Valuers Foundation or
as the office bearer of any of its branch/ Board. When directly conveying their
views the submission should be clarified as being "personal opinion" and/ or
submitted in "individual capacity".

However, it is clarified invitation to officials of counterpart agencies or soliciting financial


support for events/ meeting shall not be hit by this provision.

VALUER'S OBLIGATION TO SOCIETY AND IN PROFESSIONAL OR


DISCIPLINARY ACTIONS
1. Members owe an obligation towards society - to avoid adoption and
perpetuation of fraud, dishonesty, false statements or moral turpitude – in
personal and professional matters.
2. Seeking and continuing membership of IOV Registered Valuers Foundation is a
commitment to adhere to highest standards of personal and professional
integrity
3. When called upon, a member consents to join or aid any individual entrusted
with or any team constituted by the Board of the IOV Registered Valuers
Foundation for fact finding, evidence gathering, resolve or otherwise deal with
any action warranting disciplinary proceedings. In such situation all
communication on the process will be as authorised by the committee.

4. Where appropriate authority of IOV Registered Valuers Foundation decide to


hold internal proceedings against members in camera, to not divulge details of
such proceedings unless authorised to do so.

5. Any member undertakes to keep IOV Registered Valuers Foundation informed


about action initiated by any client or government agency against him/her -
whether banks, financial institutions, or others. If the larger issue relevant to the
profession of valuation is involved, IOV Registered Valuers Foundation may elect
to implead itself as a party to defend the same.

6. In the event of a complaint against the conduct of a member of IOV Registered


Valuers Foundation any person or client may write to COO, IOV Registered
Valuers Foundation identifying the member at least with name, address and
complained action with full supporting annexed to the complaint. The IOV
Registered Valuers Foundation shall call upon further details and clarifications as
needed at different stage, verify the complaint and if found valid initiate
proceedings.

Page | 36
L&B/Chapter-6/Various Purpose of Valuation

AGGREGATION
1. The value of an individual asset is often dependent upon its association with
other related assets. Examples include:

i. offsetting assets and liabilities in a portfolio of financial instrument


ii. a portfolio of properties that complement each other by providing a
prospective buyer with either a critical mass or a presence in strategic
locations
iii. a group of machines in a production line, or the software required to
operate machines recipes and patents that support a brand.

Where a valuation is required of assets that are held in conjunction with other
complementary or related assets it is important to clearly define whether it is the
group or portfolio that is to be valued or the individual assets. If the latter, it is
also important to establish whether each asset is assumed to be valued as part
of the whole group or portfolio, as an individual item but assuming that the
other assets are available or as an individual item but assuming that the other
assets are not available.

Page | 37
IOV-Registered Valuers Foundation

Page | 38 (This page is blank)


IMPORTANT CASE LAWS ON
PRINCIPLES OF VALUATION

OF REAL ESTATE I 07
1. K.P. Varghese Vs. Income Tax Officer and Another, (1981) 131
ITR 597 (SC)
2. Gold Coast Selection Trust Ltd. Vs. Humphrey (1948) 2 All-ER
379 and (1949) 17 ITR 19
3. R.C. Cooper Vs. Union of India, (1970) AIR SC 564
4. Hays Will Trust Vs. Hays and Others (1971) 1WLR 758
5. V. C. Ramachandran Vs. CWT (1979) 126 ITR 157 Karnataka HC
6. Subhkaran Chowdhary Vs. I.A.C. (Acq), (1979) 118 ITR 777
Kolkata HC (Special Value/ FMV)
7. Wenger & Co. Vs. DVO (1978) 115 ITR 648 Delhi HC
(Combination of Methods)
8. Sorab D. Talati Vs. Joseph Michem, Appeal No. 101 0f 1949 in
R.A. Application No. 805 of 1948
9. CWT Vs. P.N. Sikand (1977) 107 ITR 922 (SC)
10. SLAO (Eluru) Vs. Jasti Rohini (1995) 1SCC 717 SC
11. Shubh Ram and Others Vs. State of Haryana (2010) 1SCC 444
12. Jawajee Nagnathan Vs. Revenue Divisional Officer (1994) SCC
(4) 595 (SC)
13. Chimanlal Hargovinddas Vs. SLAO, AIR SC 1652
14. Duncan Industries Ltd. Vs. State of U.P. and Other AIR 2000 SC
355
15. CWT Vs. Purshottam N. Amersey and Anr., (1969) 71 ITR 180
(Bom)
16. Dr. K.R. Dhairawan and Others Vs. J.H. Thakur and Others AIR
1958 SC 789

Page | 1
CASE LAWS I 07
1. K.P. VARGHESE VS ITO (1981) 131 ITR 597
(SC)
In the case of K. P. Varghese’, Supreme Court held, “It is well known fact borne out by
practical experience that the determination of fair market value of a capital asset is
generally a matter of estimate based to some extent on guess work and despite the
utmost bona fides, the estimate of the fair market value is bound to vary from individual
to individual. The postulate underlying Sub sec. (2) is that the difference between one
honest valuation and another may range up to 15 % and that constitutes the class of
margin cases which are taken out of the preview of sub-sec (2) in order to avoid hardship
to the assessee.” Here, the court considered 15 % variation between two estimates as
normal.

2. GOLD COAST TRUST LTD. VS HUMPHRAY


(1949) 17 ITR 19
In case of Gold Coast Selection Trust Ltd.', justice Viscount Simon stated, “If the asset is
difficult to value but none the less of a money value, the best valuation possible must be
made. Valuation is not an exact science. Mathematical certainty is not demanded, nor
indeed is it possible. It is for Commissioners to express in terms of the money value,
attributed by them to the asset. their estimate, and this is a conclusion of fact to be
drawn from the evidence before them”. Rustam C Cooper vs Union of India AIR 1970 SC
564

In Bank Nationalization Case 2, the Supreme Court has explained the word property as,
“In its normal connotation ‘property’ means the “highest right a man can have to
anything, being that right which one has to lands or tenements, goods or chattels which

Page | 2
L&B/Chapter-7/Case Laws

does not depend on another’s courtesy: it includes ownership, estates and interests in
corporeal things, and also rights such as trademarks, copyrights, patents and even rights
in personam capable of transfer or transmission, such as debts, and signifies a beneficial
right to or a thing considered as having a money ‘ value, especially with reference to
transfer or succession, and to their capacity of being injured”. Thus the court has
included all types of property viz. tangible.

3. HAYS WILL TRUST VS HAYS AND OTHERS


(1971) 1 WLR 758
In case of Hayes Will Trusts*, Ungoed-Thomas J. of Chancery Division held : “They are
directed to the sale being in such manner as would obtain the best possible price in the
market. It does not mean that the price to be fixed by valuation is the highest possible
price that might be obtained. It has been established time and again in these courts, as it
was in our case, that there is a range of price, in some circumstances wide, which
competent Valuers would recognize as the price which ‘property would fetch if sold in
the open market. “The most likely price, in the absence of consultation between the
Valuers representing conflicting interests, would presumably be the mean price.”

4. V.C. RAMCHANDRAN VS CWT (1979) 126


ITR 157 KARNATAKA HC
In case of V. C. Ramachandran*®"', the Karnataka High Court stated, “In our opinion the
principle applicable for the interpretation of taxing statutes cannot be imported and
applied to the valuation of property in a given case, which constitutes a question of fact.
If there are more than one valuation of the same property, the one which is reasonable
and nearer to the correct market value, having due regard to all the relevant facts and
circumstances of the case alone should be accepted. Therefore, in such a case if the
higher valuation between several valuations appears to be a reasonable one, the same
has got to be accepted. We are unable to accede to the submission that in cases where
value of the property is ascertained under more than one method, the lowest should be
accepted.”

Page | 3
IOV-Registered Valuers Foundation

5. SUBH KARAN CHOUDHURY VS IAC (1979)


118 ITR 111 KOLKATTA HC (SPECIAL VALUE
/ FMV)
In the case of Subhkaran Chowdhury®, Calcutta the High Court held, “Valuation of fully
tenanted property should be made on the basis of capitalisation of rental method.”

6. WENGER & CO. VS DVO (1978) 115 ITR 648


DELHI HC (COMBINATION OF METHODS)
In the case of Wenger & co.’, Delhi the High Court, held, “District Valuation officer
adopted two methods to value the property. For owner occupied portion he calculated
the value on the basis of what were the rates prevalent for sale of commercial flats in
Connaught place extension area. For the tenanted portion he capitalised the rental value.
It is well-known fact that giving possession of buildings, though previously rented out,
fetches better market price. It cannot be assumed that the hypothetical purchaser would
let out the self-occupied portions which he buys from the hypothetical seller or would let
out such portions in the condition in which he buys them. The method adopted by
District V. O. and his approach is not only acceptable but also in accordance with the
principles of evaluation. “

7. SORAB TALATI VS JOSHEPH MICHEM


APPEAL 101 OF 1949 - VOL.- 2 OF SOC -
PAGE 162 (BOMBAY) (INVEST THEORY OF
RENT)
In 1949, Small Causes Court of Bombay, for the first time, laid down principles of Rent
Fixation (Standard Rent) in the case of SorabTalati’. In this case, the Court approved of
Investment Theory in preference to Comparable Rent Theory to fix Standard Rent of the
rent controlled premises. In this case, the court considered return or yield from Gilt
Edged Security as the basis for determining fair return to the landlord on his investments
in land and buildings. Considering and comparing alternative forms of sound
investments viz. Govt. Security and immovable property, the Court upheld following
returns as fair returns to the landlord on his capital investment in land and buildings.

Page | 4
L&B/Chapter-7/Case Laws

1.50% return i.e. more than the average yield on long term government security was
approved as fair return on land investment and 2.50 % return more than the average
yield on long term government security was approved as fair return on investment in
buildings. For leasehold properties, 1% extra yield on both types of investment was
considered fair, to account for extra risk of investing capital, in leasehold properties.
Obviously, these norms and principles continued to be followed for several years for all
types of rented properties and even for other purposes.

8. CWT VS PN. SIKAND (1977) 107 ITR 922 SC


In case of P. N. Sikand*’, the Supreme Court held, “It is clear on the application of this test
that in present case, 50% of the unearned increase in the value of the land 10.

Would be diverted to the lessor before it reaches the hands of the assessee as part of the
price. The assessee holds the leasehold interest on condition that if he assigns it, 50 % of the
unearned increase in the value of the land will be payable to the lessor.” Court further stated,
“it must be held that in determining the value of the leasehold interest of the assessee in the
land for the purpose of assessment to wealth tax, the price which the leasehold interest
would fetch in the open market were it not encumbered or affected by the burden or
restriction contained in clause (13) of the lease deed, would have to be reduced by 50%. of
the unearned increase in the value of the land on the basis of the hypothetical sale on the
valuation date.” SLAO (Eluru) vs Jasti Rohini (1995) 1 SCC 717 SC

In case of Sp.LandAcq. Officer Elura’*, the Supreme Court held, “Value fetched by sale of small
extent land cannot be adopted for large extent land. Loss of land for road and park, expenses
for development should be deducted”.

9. SHUBH RAM AND OTHERS VS STATE OF


HARYANA (2010) 1 SCC 444
In the case of Subhram**, the Supreme Court discussed this entire process in following
words.

“The value of one Sq.Yds. of undeveloped land is not the same as one Sq.Yd. Of
developed residential plot. If there is large tract of agricultural or undeveloped land,
obviously the entire extent cannot be sold as residential plots. If such land has to be sold
as residential plots, it is first necessary to make a layout of plots in such land. This would
mean that a provision will have to be made for roads Jo provide access to each plot in
the layout. In hypothetical layout method of determination of market value, as a first
step, the areas that will be used for roads, drains, parks/playgrounds and community
areas, will have to be excluded from the total extent of the acquired land. But merely

Page | 5
IOV-Registered Valuers Foundation

deducting the areas required for roads, drains, parks will not convert a large tract of
undeveloped land into developed ‘residential layout. For that, considerable financial
outlay has to be made.

The land will have to be converted from agricultural to non agricultural residential use by
paying necessary fees to the revenue authorities. Then the roads will have to be
asphalted or concreted. Drains will have to be dug and lined with R.C.C. or stone, for
drainage of rain water. Electricity, water and sewage lines will have to be laid. Deposits
will have to be made to the authorities dealing with electricity, water and sewage
removal. The development will also involve the service of surveyors, engineers and
developers. All these involve considerable expenditure. Further, as there 12.

Will be time gap between the expenditure for development and the actual sale of plots,
the cost of development will also have an element of interest on investment. The
developer who undertakes the development and invests monies for development would
also expect a reasonable profit when the plots are sold. All these expenditure and factors
are standardized into 33% ‘deduction towards expenses of development.”

10. JAWAJI NAGNATHAN VS REV. DIV.


OFFICER (1994) SCC - 4 PAGE 595 SC
In case of JawajeeNagnatham, the Court held, “It is therefore, clear that the Basic
Valuation Register prepared and maintained for the purpose of collecting stamp duty
has no statutory base or force. It cannot form a foundation to determine the market
value mentioned there under in instrument brought for registration. Evidence of bona
fide sales between willing prudent vendor and prudent vendee of the land acquired or
situated near about that land possessing same or similar advantageous features would
furnish basis to determine market value.”

11. CHIMANLAL HARGOVINDDAS VS SLAO -


PUNE, AIR 1988 SC 1652
The following factors must be etched on the mental screen:
A reference under section 18 of the Land Acquisition Act is not an appeal against the
award and the Court cannot take into account the material relied upon by the Land
Acquisition officer in his Award unless the same material is produced and proved before
the Court.
So also the Award of the Land Acquisition officer is not to be treated as a judgment of

Page | 6
L&B/Chapter-7/Case Laws

the trial Court open or exposed to challenge before the Court hearing the Reference. It is
merely an offer made by the Land Acquisition officer and the material utilised by him for
making his valuation cannot be utilised by the Court unless produced and proved before
it. It is not the function of the Court to suit in appeal against the Award, approve or
disapprove its reasoning, or correct its error or affirm, modify or reverse the conclusion
reached by the Land Acquisition officer, as if it were an appellate court.
The Court has to treat the reference as an original proceeding before it and determine
the market value afresh on the basis of the material produced before it. (4)

The claimant is in the position of a plaintiff who has to show that the price offered for his
land in the award is inadequate on the basis of the materials produced in the Court. Of
course the materials placed and proved by the other side can also be taken into account
for this purpose. (5) The market value of land under acquisition has to be determined as
on the crucial date of publication of the notification under sec. 4 of the Land Acquisition
Act (dates of Notifications under secs. 6 and 9 are irrelevant).

The determination has to be made standing on the date line of valuation (date of
publication of notification under sec. 4) as if the valuer is a hypothetical purchaser willing
to purchase land from the open market and is prepared to pay a reasonable price as on
that day. It has also to be assumed that the vendor is willing to sell the land at a
reasonable price.

In doing so by the instances method, the Court has to correlate the market value
reflected in the most comparable instance which provides the index of market value.
Only genuine instances have to be taken into account. (Some times instances are rigged
up in anticipation of Acquisition of land).

Even post notification instances can be taken into account (1) if they are very proximate,
(2) genuine and (3) the acquisition itself has not motivated the purchaser to pay a higher
price on account of the resultant improvement in development prospects.
The most comparable instances out of the genuine instances have to be identified on the
following considerations:

1. proximity from time angle,


2. Proximity from situation angle.

Having identified the instances which provide the index of market value the price
reflected therein may be taken as the norm and the market value of the land under
acquisition may be deduced by making suitable adjustments for the plus and minus
factors vis-a-vis land under acquisition by placing the two in juxtaposition.

A balance-sheet of plus and minus factors may be drawn for this purpose and the
relevant factors may be evaluated in terms of price variation as a prudent purchaser
would do. (13)

Page | 7
IOV-Registered Valuers Foundation

The market value of the land under acquisition has there after to be deduced by loading
the price reflected in the instance taken as norm for plus factors and unloading it for
minus factors (14) The exercise indicated in clauses (11) to (13) has to be undertaken in a
common sense manner as a prudent man of the world of business would do. We may
illustrate some such illustrative (not exhaustive) factors:

Positive Factors Negative factors

1 Smallness of size Largeness of area


2 Proximity to a road Situation in the interior at a distance from
the road
3 Frontage on a road Narrow strip of land with very small
frontage compared to depth
4 Nearness to developed area Lower level requiring the depresses portion
to be filled up
5 Regular shape Remoteness from developed locality

6 Level vis-a-vis land under acquisition Some special disadvantage factors which
would deter a purchaser.
7 Special value for an owner of an
adjoin property to whom it may have
some very special advantage

The evaluation of these factors of course depends on the facts of each case. There
cannot be any hard and fast or rigid rule. Common sense is the best and most reliable
guide. For instance, take the factor regarding the size. A building plot of land say 500 to
1000 sq. yds cannot be compared with a large tract or block of land of say 1000 sq. yds
or more. Firstly while a smaller plot is within the reach of many, a large block of land will
have to be developed by preparing a lay out, carving out roads, leaving open space,
plotting out smaller plots, waiting for purchasers (meanwhile the invested money will be
blocked up) and the hazards of an entrepreneur. The factor (16)

can be discounted by making a deduction by way of an allowance at an appropriate rate


ranging approx. between 20% to 50% to account for land required to be set apart for
carving out lands and plotting out small plots. The discounting will to some extent also
depend on whether it is a rural area or urban area, whether building activity is picking
up, and whether waiting period during which the capital of the entrepreneur would be
looked up, will be longer or shorter and the attendant hazards.

Every case must be dealt with on its own facts pattern bearing in mind all these factors
as a prudent purchaser of land in which position the Judge must place himself.
These are general guidelines to be applied with understanding informed with common
sense.

Page | 8
L&B/Chapter-7/Case Laws

12. DUNCANS INDUSTRIES LTD. VS. STATE


OF U.P. AIR 2000 SC 355
On the scope of judicial interference in the stamp duty determined by the
authorities under the Stamp Act;

Subject Civil

Court Allahabad High Court

Decided On Jul-07-1997

Case Number Civil Misc. Writ Petn. No. 9170 of 1995

Judge S.R. Singh, J.

Reported in AIR1998All72

Acts Stamp Act, 1899 - Sections 2(10) and 47A; Uttar


Pradesh Stamp Rules

Appellant Duncans Industries Limited

Respondent State of U.P. and others.

Appellant Advocate P.K. Mukerji and ;Shanti Bhushan, Advs.

Respondent Advocate Bharat Ji Agrawal, Adv. and ;Standing Counsel

Disposition Petition dismissed

Cases Referred Official Liquidator v. Sri Krishna Deo

The petitioner has approached this Court by means of the present petition, assailing the
legality and propriety of the two orders -- one dated 20-2-1995 passed in proceeding
under Section 47A(2) of the Stamp Act, 1899, by Additional District Magistrate (Finance
and Revenue), Kanpur Nagar, levying stamp duty of Rs. 37,01,26,832-50/- and slapping a
penalty of Rs.30,53,167-50/- and the other dated 4-4-1995 passed by the Chief
Controlling Revenue Authority in Stamp Revision No. 36,of 1995-96 Kanpur Nagar under
Section 56 of the Stamp Act, preferred against the aforesaid order of the Additional Distt.
Magistrate (Finance and Revenue), Kanpur Nagar. The revision has been allowed partly
vide impugned order dated 4-4-1995 of the Chief Controlling Revenue Authority, setting
aside the order passed by the Addl. District Magistrate (Finance and Revenue), Kanpur

Page | 9
IOV-Registered Valuers Foundation

Nagar in so far as imposition of penalty was concerned and slightly modifying the order
under revision as regards the stamp duty holding that the total stamp duty payable on
the deed of conveyance dated 9-6-1994 would be Rs. 36,68,08,887-50/- at an estimated
total value of Rs. 2,53,14,61,115/-.

Stamp Duty on Immovable Property in a Slump Sale

Although individual values cannot be assigned to the various assets for purposes of the
transaction in a slump sale, appropriate values have to be considered for purposes of
stamp duties. Under the Indian Stamp Act, 1899, stamp duty is payable in relation to
transfer of immovable properties. Generally, anything embedded in, or attached to, the
earth (such as land or buildings) is considered immovable property and any transfer of
the same can attract significant stamp duties. So, in any business transfer arrangement
that seeks to transfer plant and machinery together with the land, and such plant and
machinery is embedded in, or attached to, the earth, the same will be treated as
immovable property and its transfer can be stamped accordingly.
While land/buildings are considered immovable property, whether machinery that has
been installed becomes immovable property depends on the degree and permanency of
the attachment, and the purpose of installing and attaching the machinery. For instance,
the Supreme Court has held that a fertilizer plant, sold as part of a slump sale along with
land and building, is immovable property as it was always intended that the plant
remains permanently affixed to the land and building being transferred. The question
whether a machinery which is embedded in the earth is movable property or an
immovable property, depends upon the facts and circumstances of each case. Primarily,
the court will have to take into consideration the intention of the parties when it decided
to embed the machinery whether such embedment was intended to be temporary or
permanent. A careful perusal of the agreement of sale and the conveyance deed along
with the attendant circumstances and taking into consideration the nature of
machineries involved clearly shows that the machineries which have been embedded in
the earth to constitute a fertiliser plant in the instant case, are definitely embedded
permanently with a view to utilise the same as a fertiliser plant. The description of the
machines as seen in the Schedule attached to the deed of conveyance also shows
without any doubt that they were set up permanently in the land in question with a view
to operate a fertilizer plant and the same was not embedded to dismantle and remove
the same for the purpose of sale as machinery at any point of time. The facts as could be
found also show that the purpose for which these machines were embedded was to use
the plant as a factory for the manufacture of fertilizer at various stages of its production.
Hence, the contention that these machines should be treated as movables cannot be
accepted. Nor can it be said that the plant and machinery could have been transferred by

Page | 10
L&B/Chapter-7/Case Laws

delivery of possession on any date prior to the date of conveyance of the title to the
land. However, this finding was specific to the facts of that case (Duncan Industries Ltd. v.
State of U. P., AIR 2000 SC 355).

Further, the Supreme Court also rejected the contention of the seller that the plant and
machinery was transferred by delivery. The Court held that unless the machinery was
physically removed from the factory and delivered to the buyer at some other location, it
would not be considered to be a sale of goods, which are transferrable by delivery.
Therefore, it would be considered that the sale deed for the immovable property should
have also been stamped with the value of the plant and machinery.

Keyword :- Duncan Industries – Stamp duty on Slum Sale Value/ Defining immovable
Property in case of transfer of property that consists of Land, Building, Plant and
machinery although machinery is as movable for levying stamp duty.

13. COMMISSIONER OF WEALTH-TAX,


...VSPURSHOTTAM N. AMERSEY AND
ANR. ON 16 MARCH, 1968.
Equivalent citations: 1969 71 ITR 180 Bom

Subject Direct Taxation


Court Mumbai High Court
Decided On Mar-16-1968
Case Number Wealth-tax Reference No. 3 of 1963
Judge Kotwal, C.J. and ;V.S. Desai, J.
Reported in [1969]71ITR180(Bom)
Acts Wealth Tax Act, 1957 - Sections 2, 3, 7, 7(1), 7(11), 21, 21(2) and
21(4)
Appellant Commissioner of Wealth-tax, Bombay City Ii
Respondent Purshottam N. Amersey and anr.
Appellant Advocate G.N. Joshi, Adv.
Respondent Advocate R.J. Kolah, Adv.
Judgment:
Kotval, C.J.
1. The only question referred for our decision is :

Page | 11
IOV-Registered Valuers Foundation

'Whether, on the facts and in the circumstances of the case, and having regard to the terms
of annexure 'A', the Tribunal was justified in holding that the interest of the assesse under the
trust had no value ?'

2. This question has been framed in connection with the assessments to wealth-tax of two
individuals who are brothers, Purshottam N. Amersey and Manoranjan N. Amersey, but at the
hearing before the Tribunal and before us it was agreed that the facts and circumstances in
the cases of both the assesse are the same and that the documents which fall to be
considered in each case are also similar. Therefore, in the statement of the case reference has
been made to the facts of only one case, that of the assesse, Purshottam, and we shall also
for the purposes of the decision of this reference refer to the facts and circumstances of that
case.

3. PurshottamAmersey and his brother were partners along with others in


Messrs.AmerseyDamodar of Bombay. In that firm, Purshottam had to the credit of his
account in the year 1949 a sum of Rs. 4,50,000 lying in deposit with the firm. On 8th
September, 1949, he declared a trust of this amount. The amount was not made over to
anyone but it appears that by a mere book entry the amount standing to the credit of
Purshottam was held to the credit of the trust fund. Under the provisions of that trust, to
which we shall presently advert, both the assesse were entitled to certain benefits which in
the assessment year 1960-61 the authorities under the Wealth-tax Act sought to bring to tax
in the hands of these two persons as individuals. Both these persons are otherwise, it
appears, wealthy persons and own considerable property which has been taken into account
towards the computation of their net wealth. So far as their beneficial interest arising out of
the trust deed is concerned, having regard to the provisions of section 21(2) of the Wealth-
tax Act, the interest can at the opinion of the department be taxed as being the net wealth of
the beneficiaries under the trust or it can be taxed in the hands of the trustees subject to the
conditions prescribed in that section. The assesses have no objection to the amount of net
wealth being taxed in the hands of the trustees, but the department finds it advantageous to
add it to the net wealth of the assesse as the beneficiaries under the trust fund and assess the
assesses along with their other considerable wealth. That has given rise to the question which
has been referred.

4. Now, the terms of the indentures of trust dated 8th September, 1949, are in both the cases
identical and so we will refer to the terms of one of them. The trust deed executed by
Purshottam recites that he as the settlor was appointing three persons as the trustees, the
three persons being himself, his father, Nandlal, and his mother, Bai Manjulabai. The subject
of the trust is the amount of Rs. 4,50,000 lying in deposit with the firm of
MessrsAmerseyDamodar of Bombay. The first clause merely sets out that the trust is being
declared out of natural love and affection which the settlor bears towards the members of his
family and diverse other good causes and consideration. The second clause lays down the

Page | 12
L&B/Chapter-7/Case Laws

terms and conditions of the trust and is important for the purposes of the question referred.
That clause provides as follows :

'2. The trustees shall hold and stand possessed of the trust fund upon trust :

(a) To recover the interest, dividends, profits and income of the trust fund and to pay out of
the same the charges for collection and all other outgoings, if any;

(b) To apply the balance of such interest, dividends, profits and income of the trust fund
(hereinafter for brevity's sake referred to as 'the net income') for the support, maintenance
and advancement in life and otherwise for the benefit of the settlor and his wife (provided
such wife is born before the date of these presents) in such manner as to enable the settlor
to live as far as possible with the same comforts and to enjoy life in the same manner as he is
accustomed to do And in case of any surplus income at the end of any year to accumulate
the same for a period of eighteen years from the date hereof and to add the same to the
corpus of the trust fund And after the expiration of the said period of eighteen years in case
of any surplus income at the end of any year to hand over the same to the settlor Provided
further that in applying the net income as aforesaid the trustees shall not be entitled to take
into account any other income from any other source that the settlor may be receiving at the
time AND IT IS HEREBY EXPRESSLY AGREED AND DECLARED that the trustees shall not be
liable or accountable to any one for any act bona fide done by them or for any payment bona
fide made by them in pursuance of the provisions of this clause and in particular they shall
not be accountable or responsible f or the amounts expended or applied by them or the
manner in which or the purpose for which the same shall be applied and all moneys so
expended or applied by the trustees in their absolute discretion shall not be questioned by
any party in any court of law or otherwise howsoever;

(c) If a child or children is born to the settlor, then on the death of the settlor to divide the
corpus of the trust fund amongst all the children of the settlor, if more than one in the
proportion of four shares for every male child to one share for every female child;

(d) If no child is born to the settlor, then on the death of the settlor, leaving a wife him
surviving and provided such wife is born before the date of these presents, to pay the net
income to such wife until her death or remarriage whichever event shall first happen Provided
further that in the event of such wife being born after the date of these presents, to pay to
the wives an amount equivalent to twenty-five per cent. of the corpus of the trust fund at the
time of the death of the settlor;

(e) Subject to the provisions of sub-clause (d) hereof, on the death of the settlor without a
child being born to him, the trustees shall hold and stand possessed of the corpus of the
trust fund upon trust to divide the same amongst the heirs of the settlor according to the law
of intestate succession amongst Hindus.'

Page | 13
IOV-Registered Valuers Foundation

5. The rest of the clauses except the two mentioned below are hardly material for the
decision of the question before us. In clause 4 it is provided that the trustees shall be at
liberty to keep the trust fund as a deposit at interest in the firm of Messr .AmerseyDamodar -
a firm in which the assesses, as we have said, were themselves partners. Next as regards the
vacancy in the office of trustee, clause 8 provides that the surviving or continuing trustee or
trustees for the time being or they refusing the retiring trustees or failing them the heirs,
executors and administrators of the last surviving or continuing trustee should appoint a new
trustee or trustees 'but so as not to be more than five nor less than two...' trustees upon
'every or any such appointment'.

6. It will be noticed that sub-clause (b) of clause 2 of the trust deed provides that the trustees
shall 'apply' the balance of the trust funds after the charges for collection and other
outgoings 'for the support, maintenance and advancement in life and otherwise for the
benefit of the settlor and his wife (Provided such wife is born before the date of these
presents) in such manner as to enable the settlor to live as far as possible with the same
comforts and to enjoy life in the same manner as he is accustomed to do....' The words within
the brackets, it was explained, were inserted in this sub-clause in order to guard against the
trust being voidable on the ground of infringing the rule against perpetuity. The provision
giving interest to the assesse is a very general provision that the trust fund shall be applied
for the support, maintenance and advancement in life of the settlor and his wife. On that
date, 8th September,1949, the settlor was not married. The assesse, ManoranjanAmersey, was
married on the 18th December, 1950, and the assesse, Purshottam, was married on the 22nd
April, 1958, that is to say, about seven and a half years after the trust deed. The words in sub-
clause (b) of clause 2 may, therefore, have been inserted as explained in order to indicate that
the wife to be was, on the date of the settlement, not (sic) in existence. However, we say this
only by way of an explanation and it does not affect the decision on the question before us.

7. Before we proceed to state the respective contentions of the parties, it is necessary at this
stage to clear a small point is whether sub-clause (b) of clause 2 of the trust deed makes the
wife an immediate beneficiary under the trust deed or whether so long as the settlor was
alive the wife had no beneficial interest under the trust deed. No doubt in clause 2(b) the
words used are '..... for the benefit of the settlor and his wife.....', but it seems to us that
though these words are put in, in order to refer generally to the parties to the trust, the
subsequent provisions make it clear beyond any doubt that, so long as the settlor was alive, it
was for his support, maintenance and advancement in life that the balance of the trust funds
was to be applied. Firstly, the words 'for the benefit of the settlor and his wife' are
immediately succeeded by the words 'in such manner as to enable the settlor to live as far as
possible with the same comforts and to enjoy life in the same manner as he is accustomed to
do', which would indicate that the main provision laying down the conditions upon which the
trust fund was to be applied was with reference only to the assesse and not to his wife.

Page | 14
L&B/Chapter-7/Case Laws

Similarly, the later provisions in sub-clause (b) of clause 2 says that any other income from
any other source that the settlor may be receiving trust funds as aforesaid., suggesting that
he was to enjoy the beneficial interest himself at some stage. But any doubt in this matter as
to whether the wife a s all a beneficiary is dispelled by a consideration of sub-clause (d) of
clause 2. In the event that no child is born to the settlor and his wife, then on the death of the
settlor, the provision is 'to pay the net income to such wife until her death or re-marriage
whichever event shall first happen'. The provision to pay the net income to the wife comes in
only at this stage, which clearly suggests that the earlier reference to the wife in sub-clause
(b) of clause 2 was only a general reference to the overall purpose of the trust deed.

8. Now, when the wealth-tax authorities sought to bring to tax the interest of the assesse
under the provisions of the trust deed, it was contended before the Wealth-tax Officer that
the assesse's life interest was not definite and ascertainable and, therefore, the trustees alone
should be taxed directly in respect of the entire estate in their hands under section 21(4) of
the Wealth-tax Act. That it appears was the principal contention before the Wealth-tax Officer
and that contention was negatived by the Wealth-tax Officer.

9. When the assesse took the matter to the Appellate Assistant Commissioner another
contention was also raised. It is stated in the first paragraph of the Appellate Assistant
Commissioner's order : 'The first contention raised in this appeal in that the assesse is
incapable of being assessed in respect of his life interest in the trust created by the deed
dated September 8, 1949'. The principal contention which was urged before the Wealth-tax
Officer was also raised before the Appellate Assistant Commissioner as can be seen from
paragraph 2 of the Appellate Assistant Commissioner's order :

'It is contended that the assesse has no right to any portion of the income of the said trust,
that his share therein is indeterminate and unknown, that the assesse has really no life
interest at all in the said trust and finally that no purchaser would be willing to pay any
amount for the so-called life interest.'

10. The Appellate Assistant Commissioner held so far as the second contention before him
was concerned that it could not be said that the assesse's interest was absolutely without
value. As to the other question, whether the assesse's share was indeterminate or unknown,
the Appellate Assistant Commissioner held that the assesse's interpretation of the clause
regarding the extent of his interest in the trust would be only germane to the question of
valuation and it could not affect the question whether the assesse is not possessed of the
asset at all. On this view the Appellate Assistant Commissioner dismissed the appeal, though
he made a change in the language, with which we a re not here concerned.

11. The assesse carried the matter by way of appeal to the Income-tax Appellate Tribunal and
the Tribunal has allowed the assesse's appeal. Before the Tribunal, the principal contention

Page | 15
IOV-Registered Valuers Foundation

was that the value of any asset has to be estimated so as to arrive at its price 'if sold in the
market on the valuation date'. The Tribunal considered the circumstances and the terms of
the deed of trust and came to the final conclusion that : 'In our opinion the value of the said
interest is nil'. The reasons which prevailed with the Tribunal were as stated in paragraph 6 of
its order that having regard to the nature of the benefit given to the assesse under the trust
deed, the interest available to the assesse will have no value in the market whatever because
the benefit received is of a peculiar kind. The Tribunal stressed that under the trust deed no
cash payment to the assesse as such was contemplated but that the provision was to apply
the trust fund for the support, maintenance and advancement in life of the assesse. Secondly,
they pointed out that upon the terms of the trust deed the interest of the assesses was a
personal interest and 'is restricted in its enjoyment to them personally and such interest is
not capable of being transferred to third parties'. Therefore, it was property which was
incapable of being transferred to any third parties and no question arose of its fetching any
price in the market. The Tribunal also gave an alternative finding that, even assuming that the
above interpretation of the document was not correct, nobody would be wiling to come
forward to purchase an asset of this kind, because even a willing purchaser, who was desirous
of taking an assignment of the respective interests of the assesses under the trust deed,
would not be in a position to compel the trustees to make any payment to them. Thus, the
content or quantum of the interest so far as third parties were concerned, would be of a
problematic character and, therefore, the Tribunal held that the interest of the assesse was
such 'as would lead the asset being of no value o r of insignificant value'. It will be noticed
that here (paragraph 7) the Tribunal came to the conclusion that the asset was of an
insignificant value whereas in the earlier paragraph and in its final conclusion it has held that
'the value of the said interest is nil'.

12. In the arguments before us counsel on behalf of the department has challenged the
interpretation placed upon the terms of the deed of trust and the finding that the value of
the interest of the assesse was nil. At any rate, counsel has urged that those considerations
which had prevailed with the Tribunal in coming to the conclusion, that the value of the s aid
interest is nil, are hardly germane to the question which really arises, namely, whether in the
circumstances and upon the terms of the trust deed the interest of the assesse had no value.
He has urged that the question whether the property was not capable of being transferred or
that the benefit reserved under the deed was of a peculiar kind is irrelevant upon a
consideration of the provisions of the Wealth-tax Act in determining whether this was the net
wealth of the assesse under section 3 of the Act, Mr. Joshi has urged that the simple question
which the authorities had to decide was, in the first place, whether this was net wealth in the
hands of the assesse and for that purpose they had to determine whether it constituted an
asset within the definition of that word in section 2(e) read with the definition of 'net wealth'
in section 2(m) and that the question as to the mode of valuation has been unnecessarily
mixed up with the question whether it was 'net wealth' in the hands of the assesse. He urged

Page | 16
L&B/Chapter-7/Case Laws

that the question which the Tribunal has considered arises only upon the provisions of
section 7(1) which deals with the question as to how the value of the assets is to be
determined and even there the Tribunal has misconstrued the provisions of that sub-section.

13. In reply Mr.Kolah on behalf of the assesse has urged that, having regard to the provisions
of section 3, which must be read in the light of section 7(1), the Tribunal was right in the
conclusion which it reached that the assesse's beneficial interest in the trust fund was of no
value. Counsel for both the parties have referred to certain decisions to which we will
presently advert. Mr.Kolah also, raised the other contention which, as we have said, was the
principal contention which the assesse raised before the Wealth-tax Officer to begin with,
namely, that having regard to the provisions of section 21(4) of the Act, the share of the
assesse under the trust deed was indeterminate and unknown and that, therefore, the net
wealth could be taxed in the hands of the trustees alone having regard to the provisions of
section 21(2). Mr. Joshi has objected to the assesse being allowed to raise this question upon
the present reference and considerable arguments took place as to whether that question
ought to be considered by us and whether the assesse can at the stage of reference be
allowed to raise it.

14. Section 3, which is the charging section in the Wealth-tax Act, provides that, subject to
the other provisions contained in the Act, there shall be charged a tax in respect of the net
wealth on the corresponding valuation date of every individual. We are not concerned here
with the other provisions of this sub-section. The expression 'net wealth' in the sub-section is
defined in section 2(m) to mean 'the amount by which the aggregate value computed in
accordance with the provisions of this Act of all the assets, wherever located, belonging to
the assesse on the valuation date ....' We are not here concerned with the inclusive clause of
this definition. The net wealth, therefore, consists of all the assets wherever located belonging
to the assesse on the valuation date. The word 'assets' is also defined in section 2(e) to
include 'property of every description, movable or immovable. 'Then there are certain specific
exclusions none of which can be applied here. It is clear from these provisions laying down
the charge of the tax that every kind of property, movable or immovable, of an assesse has to
be taken into account and its aggregate value computed in accordance with the provisions of
the Act. The first question, therefore, which has to be determined in any assessment under
the Wealth-tax Act is what constitutes the net wealth of the assesse.

15. Section 7 deals with the question of how the value of the asset is to be determined and
sub-section (1) provides as follows :

'Subject to any rules made in this behalf, the value of any asset, other than cash, for the
purposes of this Act, shall be estimated to be the price which in the opinion of the Wealth-tax
Officer it would fetch if sold in the open market on the valuation date.'

Page | 17
IOV-Registered Valuers Foundation

16. The Tribunal has stressed the concluding words of sub-section (1) of of section 7, 'if sold
in the open market on the valuation date' and in view of those words it has proceeded to
consider whether the interest of the assesse under the trust could have any value in the open
market on the valuation date and has come to the conclusion that it had little or no value.
Therefore, the Tribunal allowed the appeal and held that the interest was not an asset under
the Wealth-tax Act. Now, it seems to us that virtually the order of the Tribunal amounts to
this that, having taken into account the provisions of section 7(1) which merely deals with
how the value of assets has to be determined, it has ultimately held that the interest of the
assess was not an asset at all. We shall presently show that it reached this conclusion because
of a misconstruction of section 7(1) and because of an incorrect view as to what its provisions
implied.

17. The definition of 'assets' in section 2(e) read with the definition of 'net wealth' in section
2(m) are extremely comprehensive provisions and all assets are include in 'net wealth' by the
very definition and in the definition of 'asset' property of every description, movable or
immovable, is included. Therefore, when section 3 imposes the charge of wealth-tax upon the
net wealth, it necessarily includes any and every description of the property of the assesse,
movable or immovable, barring of course the exceptions stated in section 2(e) or any other
provisions of the statute. What is more, it is clear upon the wording of section 3 that the
moment a right, interest or other property falls within the definition of 'assets' the Act
assumes that it must have some value. that is by section 3 merely states that there shall be
charged a tax in respect of the net wealth. Since 'net wealth' includes all a sets wherever
located belonging to the assesse and since 'assets' includes the property of every description,
movable or immovable, it is clear that everything of value which an assesse possesses is
brought into the ambit of the charging section and is assumed to have some value, however
little. Therefore, in the first place, it is not enough to say, as the Tribunal has stated in its
order at certain places, that the interest of the assesse in the present case was 'of insignificant
value'. Even if it is of insignificant value, it will clearly be chargeable under section 3 for the
proper amount of wealth-tax.

18. Section 7 merely deals with the made in which the value of the assets has to determined
and we agree with Mr.Kolah that the charging section because of its opening words 'Subject
to the other provisions contained in this Act' must be held to be subject to section 7(1); but
we do not think that section 7(1) could be utilised in the manner in which it has been used in
this case to nullify the provisions of section 3 itself. What has been done in the present case is
that, utilising the provisions of the Act which only provides a machinery for determining the
value of the asset, a conclusion has been reached that the asset in this case has no value
whatever, in other words, that it is not an asset at all. The fallacy of this reasoning lies in this
that it was not clearly realised that the purpose of section 7 was not to indicate what is not an
asset but merely to indicate how it has to be valued if it is an asset.

Page | 18
L&B/Chapter-7/Case Laws

19. In our opinion, the Tribunal should have first of all considered whether the interest of the
assesse amounted to his 'net wealth' within the meaning of section 3 read along with the
definition and that would entail a finding from the Tribunal whether or not the interest was
an asset. If it was an asset it would be his net wealth. Once it is held that the interest of the
assesse was his asset, the Tribunal could not have come to the conclusion on the basis of
section 7(1) that it was not his asset, which is virtually what the Tribunal has done. It was,
therefore, rightly argued by Mr. Joshi on behalf of the Commissioner that the only question
that arose upon the findings of the Tribunal was one of valuation and the Tribunal could not
have come to the conclusion that 'the value of the said interest is nil' as it has done in the
concluding portion of paragraph 8 of its order.

20. Even in the conclusion to which the Tribunal reached that the interest of the assesse in
the trust property had no value whatever and upon the reasoning which it followed, which in
our opinion was incorrect, there is another error of construction of section 7(11). The
principle which sub-section 1 of section 7 lays down is that the value of any asset is to be the
estimated price which the asset 'would fetch if sold in the open market on the valuation date'.
From its order it appears that the Tribunal construed this clause of sub-section (1) of section
7. It proceeded to consider whether the interest of the assesse in fact had any value in the
open market and having considered several of the terms of the trust deed, on which we will
presently have something to say, it held that 'this is a property which is incapable of being
transferred to any third parties as the interest of the respective beneficiaries is restricted to
them personally'. Then it proceeded further and held that even assuming that this
interpretation of section 7(1) was incorrect, in their opinion in f act nobody would be willing
to come forward to purchase an asset of this kind. They showed certain circumstances and
concluded that 'in our opinion, it is such as would lead the asset being of no value or of
insignificant value'. Later on, in paragraph 8, however, they came to the conclusion that the
value of the interest was nil.

21. Now, it was rightly argued on behalf of the department that, in the first place, these
findings of the Tribunal are themselves conflicting and that the Tribunal was in two minds as
to whether the asset had some little or no value at all. At one stage, it said that it was of
insignificant value and at another stage that it had no value, but apart from that it seems to
us that the cardinal error which the Tribunal made was to hold that under the words of
section 7(1) '.....it would fetch if sold in the open market.....' it must have regard to the actual
facts and the actual circumstances and decide whether the asset could be sold or not in the
open market. In that respect it seems to us that the Tribunal was clearly wrong on a point of
construction. When the statute uses the words 'if sold in the open market', it does not
contemplate any actual sale or the actual state of the market, but only enjoins that it should
be assumed that there is an open market and the property can be sold in such a market and
on that basis directs that the value should be found out. It is a hypothetical case which is

Page | 19
IOV-Registered Valuers Foundation

contemplated by those words of the sub-section. The tax Officer must assume that there is
an open market in which the asset can be sold and proceed to value it on that basis. The use
of the words 'if sold' creates a fictional position which the tax officer has to assume.

22. The Wealth-tax Act is a comparatively new taxation statute and there is little to guide the
court in the construction of section 7, but much assistance can be derived from the
interpretation of similar provisions in other statutes, particularly the English statutes which
are in pari material. Reliance was placed upon two decisions in England, which, in our
opinion, are apposite in the construction of section 7(1). One such section was section 7, sub-
section (5) of the British Finance Act, 1894, which also provided for the valuation of property
as follows :

'The principal value of any property shall be estimated to be the price which, in the opinion of
the Commissioners, such property would fetch if sold in the open market at the time of the
death of the deceased.'

23. The concluding words 'if sold in the open market' are identical with the words used in
section 7(1) of the Indian statute.

24. In Commissioner of Inland Revenue v. Crossman [1937] A. C. 26; 2 E. D. C. 537. the House
of Lords was called upon to construe those very words. In that case the deceased was entitled
to a number of ordinary rigid restrictions upon the alienation and transfer of those shares in
the company and an argument similar to the one which prevailed with the Tribunal was
urged that, in view of the restrictions on alienation and transfer of the shares, no one would
in fact purchase those shares nor could they be put in the open market. In dealing with the
point Viscount Hailsham L. C. pointed out (see page 39) 2 E. D. C. 537, 550,551,552. that :

'In order to reach the right conclusion upon the construction to be placed upon the sub-
section and its application to the facts of the present case, it seems to me essential to
determine what is the property which has to be valued'

and then he proceeds to state what that property was in the following words :

'In my view, the property which, passed at the death of the deceased consisted of the shares
in the company, and this is not the less try because the terms of the articles limited the rights
of the deceased shareholder or of his executors to deal with the shares, and gave certain
privileges and rights of pre-emption on his death. If I am right so far, it follows that the
Commissioners have to estimate the price which the shares would fetch if sold in the open
market at the time of the death of the deceased.'

25. Then the House of Lords referred to the decision of Lord Blanesburgh in an Irish case,
Attorney-General for Ireland v. Jameson [1905] 2 I. R. 218., where Lord Blanesburgh had taken

Page | 20
L&B/Chapter-7/Case Laws

a view taken in the present case by the Tribunal and commenting on that view Lord Hailsham
observed :

'My Lords, it seems to me that this construction involves treating the provisions of section 7,
sub-section (5), as if their true effect were to make the existence of an open market a
condition of liability instead of merely to prescribe the open market price as the measure of
value' and in a later passage at page 49 2 E. D. C. 537, 551, 552. :

'But the purpose of section 7, sub-section (5), is not to define the property in respect of which
estate duty is to be levied, but merely to afford a method of ascertaining its value. If the view
entertained by the Court of Appeal were correct, it would follow that any property which
could not be sold in the open market would escape estate duty altogether. That seems to me
quite an unnecessary and unnatural construction to place upon the language of the statute....
I think that full justice is done to the meaning of the sub-section if the property to be valued
is determined by the earlier sections and section 7 is treated as being merely a statutory
direction as to the method by which the value is to be ascertained. In order to comply with
that statutory direction, it is necessary to make the assumptions which the statute directs.
This is not to ignore the limitations attached to the share.'

26. In re Cassel : Public Trustee v. Moutbatten [1927] 2 Ch. 275; 2 E. D. C. 345. is another
casein which the provisions of section 7(5) and the other provisions of the British Finance Act,
1894, fell to be construed. In that case the testator bequeathed 'Book House and contents'
and the stables held therewith which he had taken on lease expiring in 1995, to trustees upon
trust to allow 'C' to have the use and enjoyment thereof for life, and after her death upon the
like trust for the benefit of Lady 'L' for life. 'C', the first tenant for life, died and was succeeded
a s tenant for life by Lady 'L'. The question was how the estate duty on C's death in respect of
the benefit of the annual expenditure on Brook House, should be borne as between Lady 'L'
and the testator's residuary estate. In that connection the provisions of section 7(5) of the
British Finance Act, 1894, fell to be construed. An argument was advanced before Russell J., as
he then was, that the case would not be governed by section 7(5) and, therefore, the property
could not be valued. This point, the learned judge answered at page 281 :

'Next, how is the principal value of that property to be ascertained The machinery provided
by the Act is section 7, sub-section (5), supplemented to some extent by section 7, sub-
section (8). That machinery does not exactly fit the present case, because from the personal
nature of division, the benefit of it could not be sold at all; an outsider purchaser would not
obtain the delivery of the goods. Nevertheless, the machinery must be made to fit, and I think
it can be made to fit.'

27. The provisions of the Wealth-tax Act, in our opinion, indicate a similar construction.
Though no doubt, section 3, which is the charging section, begins with the words 'Subject to

Page | 21
IOV-Registered Valuers Foundation

the other provisions contained in this Act' and because of those words would subject to
section 7(1), a consideration of the definition of 'net wealth' in section 2(m) and 'asset' in
section 2(e) clearly shows that property of every description, movable immovable, would be
included in the definition of 'net wealth' and the net wealth is chargeable to wealth-tax under
section 3. Therefore, the first question which must be decided under the Act is whether the
interest of the assesse, as in the present case, was his net wealth or, in other words, as asset
of his. If it was, it would be chargeable to wealth-tax and though the provisions as to
valuation may not directly apply they must be made to apply.

28. But it seems to us that it is not necessary to go so far as that in the present case. A
consideration of the terms of the trust deed and of the interest granted to the assesse will
clearly show that the conclusion which the Tribunal reached, that the value of the said
interest is nil, is erroneous. We have already said that the mere fact that the property was not
capable of being transferred is not a consideration which ought to have prevailed as shown
by the authorities to which we have referred above. The error which the Tribunal committed
in that respect was to have regard to the actual position in the actual market whereas upon
the statute what they should have considered is, assuming a hypothetical market, what would
be the price if the interest was sold.

29. Now, turning to the provisions of the trust deed, we have already said that under the
provisions of clause 2(b) so long as the assesse was alive we do not think that the wife had
any interest in the trust fund or its income. We have indicated that reading clause 2(b) in the
context of sub-clause (d) of clause 2, it is clear that the beneficial interest was in the first
instance to go to the assesse for his 'support, maintenance and advancement in life' and
otherwise for his benefit. The object of the trust was to provide support, maintenance and
advancement in life of the assesse and the subsequent conditions were all in favour of the
assesse. It has been provided that the support and maintenance which he was to receive was
such as to enable him to live as far as possible with the same comforts and to enjoy life in the
same manner as he was accustomed to do. It was further provided that the trustees shall not
be liable or accountable to anyone for any act bona fide done by them in pursuance of the
provisions of clause 2(b) of the trust deed. The provisions are sweeping in their nature and
objects of the trust are stated in the most generous terms in favour of the assesse. Since the
assesse was a trustee along with his parents, it is clear that he could at any time have
obtained whatever funds were available out of the income of the trust property. There is also
no doubt that the assesse as the beneficiary under the trust deed could legally claim that the
amount should be paid to him.

30. It was urged by Mr.Kolah that the condition of the trust was that he should receive the
income of the trust fund only for the support, maintenance and advancement in life and in
such a manner as to enable him to live as far as possible with the same comforts and to enjoy
life in the same manner as he was accustomed to do. Therefore, Mr.Kolah urged that he was

Page | 22
L&B/Chapter-7/Case Laws

not the sole judge, but that his parents, who were also the trustees, could have judged for
themselves whether he required the moneys according to his standard of life and they would
be entitled to refuse to pay him if he lived extravagantly or spent in excess of the
requirements of his maintenance or support.

31. We do not regard the provisions of the trust deed as indicating that the assesse could
ever be denied payment of the trust funds. We have already said that in stating the purpose
and object of the trust such general words have been used that it would be impossible
having regard to those words to say that the assesse would not be entitled to any moneys
out of the trust funds. The moment that conclusion is reached, the interest would be an
interest having some value and if it has a value it would be chargeable in his hands. It was
also urged that no one would come forward having regard to these terms to purchase such
an interest. We have already said that the question whether an actual purchaser would
purchase such an interest is irrelevant having regard to the provisions of section 7(1), because
that section contemplates a hypothetical market and a sale in such a market, but we do not
think also that in fact the interest of the assesse was such that nobody would ever care to
purchase it. It was an interest which was a substantial interest and which was easily available
to the assesse on the terms of the trust deed. He was himself a trustee and the other trustees
were merely his parents. On the provisions of the trust deed it is clear that the assesse could
compel payment from the trustees of the income from the trust funds to the extent it was
available. Therefore, it could not be said that the value of the interest was nil. As to its being
'insignificant', however insignificant it could be valued. In this matter we think that the
Tribunal has clearly misconstrued the trust deed.

32. Thus, in our opinion, the Tribunal went wrong both in its interpretation of section 7(1)
read with the provisions of the charging section 3 as also in its construction of the trust deed
in the present case. Its findings are, therefore, clearly induced by an error of law. We are
unable to accept the conclusions that the value of the interest of the assesse was either
insignificant or nil. It was a valuable interest which has to be valued having regard to the
provisions of section 7(1) and the other provisions of the Act. The question referred to us,
therefore, must be answered in the negative.

33. So far it will be noticed that we have considered the question arising upon the basis of
section 3 with section 7(1) of the Act, but Mr.Kolah, on behalf of the assesse, raised a further
question relying upon the provisions of section 21(4) of the Act. Section 21 makes provision
for assessment in special cases and one of the cases contemplated in sub-assessment in
special cases and one of the case of a trust the wealth-tax shall be levied upon and
recoverable from the person on whose behalf the assets are held and the provisions of the
Act were to apply accordingly, but then sub-section (2) gave a further option to the
department. It made the following provision :

Page | 23
IOV-Registered Valuers Foundation

'Nothing contained in sub-section (1) shall prevent either the direct assessment of the person
on whose behalf or for whose benefit the assets above referred to are held, or the recovery
from such person of the tax payable in respect of such assets.'

34. By this sub-section, therefore, an option was given to the department to assess either the
trustees or the beneficiary in the case of a trust. Obviously such an option was given in the
interest of the revenue. The department can at its choice add the amount of the net wealth to
the assessment of the trustees or to the assessment of the beneficiary whichever was
beneficial to the revenue and charge the wealth-tax on that basis, but sub-section (4) of
section 21 provides :

'Notwithstanding anything contained in this section, where the shares of the persons on
whose behalf or for whose benefit any such assets are held are indeterminate or unknown,
the wealth-tax may be levied upon and recovered from the court of wards, administrator-
general, official trustee, receiver, manager or other person aforesaid as if the persons on
whose behalf or for whose benefit the assets are held were an individual.... for the purposes
of this Act.'

35. The effect of sub-section (4), therefore, is that sub-section (2) would not be available to
the department where the shares of the persons on whose behalf any assets are held are
indeterminate or unknown, that it is to say, if the shares of the beneficiary in the case of the
trust are indeterminate or unknown. Relying upon this provision Mr.Kolah urged that in this
case the shares of the assesse and his wife are indeterminate and unknown and, therefore,
the department must necessarily go only against the trustees and cannot add the amount of
the interest to the net wealth of the beneficiary. In this respect he pointed to the provisions
of section 41(1), first proviso, of the Indian Income-tax Act, and the construction put upon
that proviso in several cases; particularly he relied upon Commissioner of Income-tax v.
PuthiyaPonmanichintakamWakf: [1962]44ITR172(SC) ., Commissioner of Income-tax v.
ManilalDhanji : [1962]44ITR876(SC) and Commissioner of Income-tax v. Lady
RatanbaiMathuradas : [1968]67ITR504(Bom) . to urge that it must be held that the interest of
the assesse and his wife in the present case was indeterminate and unknown.

36. Mr. Joshi on behalf of the department took strong exception to this point being allowed
to be raised by Mr.Kolah or it being determined at all, for he urged that it has not been
raised, not argued before the Tribunal and the assesse did not ask for a question to be
referred nor is it to be found referred to in the statement of the case. That the point was
urged before the wealth-tax Officer is clear from the quotation in paragraph 5 of the
statement of the case itself from the order of the Wealth-tax Officer. The Wealth-tax Officer
dealing with this question said :

Page | 24
L&B/Chapter-7/Case Laws

'Therefore I hold that life interest of the assesse who was the settlor as well as one of the
trustees extended to the full amount of the income. Therefore section 21[4] is not applicable
to the assets in the hands of the trustees, except to the extent of the remainder after
deduction of [sic] the value of life interest.'

37. In the appeal before the Appellate Assistant Commissioner also this question was raised.
He has stated in paragraph 2 of his order :

'It is contended that the assesse has to right to any portion of then income of the said trust,
that his share therein is indeterminate and unknown, that the assesse has really no life
interest at all in the said trust and finally that no purchaser would be willing to pay any
amount for the so-called life interest.'

38. Therefore, the point that the assesse's share therein was indeterminate and unknown was
clearly raised in the arguments on behalf of the assesse and that could only have reference to
the provisions of section 21(4). Of course, the Appellate Assistant Commissioner brushed
aside the question because in his view that contention of the assesse could only be
considered for the purpose of evaluating the asset and not because the assesse was not
possessed of the asset at all. He held in paragraph 5 :

'Be that as it may, it cannot in my view be gain said that the assesse is possessed of an asset
constituted of his life interest in the trust, and at its highest the assesses contention can be
considered for the purpose of evaluating the asset but not that the assesse is not possessed
of the asset at all.'

39. Upon the view that the Appellate Assistant Commissioner thus took - a view which in our
opinion was not correct - he did not further go into the question because he felt that it was
only a question which had to be considered in evaluating the asset.

40. When the matter came before the Tribunal the principal contention raised on behalf of
the assesse was that, assuming that it was an asset, it could not be brought within the words
'if sold in the open market' in that section. The Tribunal, as we have shown, accepted this
contention and since it accepted this contention there was no question of considering the
question arising under section 21(4), but the Tribunal has in paragraph 8 of its order held :

'Having considered the terms of the agreement and the contentions of the parties, we are of
the view that no amount is liable to be taken in these two assessments as representing the
value of the interest of the respective assesses under the two trusts.'

41. In other words, the Tribunal has stated that upon the view which they were taking the
question under section 21(2), as to whether the amount is liable to be taken in the
assessments made on the beneficiaries or on the trustees, did not arise for decision. It seems

Page | 25
IOV-Registered Valuers Foundation

to us that this passage in the order of the Tribunal can only have reference to sub-section (2)
of section 21 and, if so, it could only arise upon the argument that the shares of the assesse
and his wife were indeterminate and unknown within the meaning of those words in sub-
section (4). In that view it does appear that the question must have been adumbrated before
the Tribunal although, upon the view which they took, it became unnecessary to decide it and
the Tribunal have said so. We were also referred by Mr.Kolah to the memorandum of appeal
on behalf of the assesse to the Tribunal and in ground No. 1 that question has in terms been
raised. Though this document is not included in the paper-book it was certainly a part of the
record of the Tribunal and so we have referred to it. The view which the Tribunal took upon
this question in paragraph 8 of its order is referred to also in the statement of the case in
paragraph 9.

42. In this view and in the circumstances, we do not think that we can shut out the assesse
from raising the question. In fact, for a considerable time we felt that we would ourselves
have to determine that question and after reframing the question referred. The question is
really part and parcel of a question of valuation of the assesse's interest. The principal
question before the authorities was whether the assesse's interest in the trust fund should be
valued for the purpose of wealth-tax and that entailed deciding the two questions, whether it
was an asset, in the first place, and, therefore, constituted his net wealth and, in the second
place, how it was to be valued. The two, it seems to us, were merely answers to one and the
same question. It has been held recently by the Supreme Court that an aspect of a question is
itself a distinct question for the purpose of section 66(1) of the Act : see BhanjiBagawandas v.
Commissioner of Income-tax [1968] 67 I. T. R. 80. As to the argument on behalf of the
department that the assesse has not asked for a reference and should have had the question
framed and referred, we do not see how it is possible for the assesse to do so. Upon the
findings given by the Tribunal that the interest of the assesse did not constitute his asset at
all, no further question arose to be decide in favour of the assesse. He had thus wholly
succeeded before the Tribunal and we cannot see how the assesse in those circumstances
could have asked that a reference be made upon another aspect of the same question. We
do not think, therefore, that in the present case the assesse could or ought to have asked for
this question to be referred. Mr. Joshi on behalf of the department relied upon a decision of
this court in Girdhardas& Co. Ltd. v. Commissioner of Income-tax : [1957]31ITR82(Bom) . That
case turned upon its own facts and, even so, all that was laid down in that decision was that,
where a party which has lost the case before the Tribunal applies for a reference and a
reference is determined upon, the party which has won may apply for reference of other
questions of law which arise from the order of the Tribunal. The decision is not an authority
for the proposition, however, that every party must in every case apply for a reference of
other questions which arise from the order of the Tribunal or that if the successful party does
not do so, it cannot raise it thereafter. The principles which are to be observed in references
were generally laid down by the Supreme Court in Commissioner of Income-tax v. Scindia

Page | 26
L&B/Chapter-7/Case Laws

Steam Navigation Co. Ltd. : [1961]42ITR589(SC) . and that case also stresses that where the
question itself was in issue, there is no further limitation imposed by the section that the
reference should be limited to those aspects of the question which had been argued before
the Tribunal. This decision also stresses that it will be an over-refinement of the position to
hold that each aspect of a question is itself a distinct question for the purpose of section
66(1) of the Act.

43. In view of what we have said were of the opinion that the assesse is entitled to raise the
question that his share and that of his wife in the fund were indeterminate and unknown and
that, therefore, having regard to the provisions of section 21(4), the department can only
assess the trustees and not the assesse. We would have proceeded to determine the question
ourselves, but we do find that, though the question was raised before the Tribunal, the
Tribunal has not considered it or given any finding on it, because it held that the assesse's
share under the trust deed was not an asset at all. We, therefore, think it fair both to the
department and to the assesse that this question should be pronounced upon by the
Tribunal and a clear-cut finding given. With these observations we would dispose of the
question referred to us by answering it in the negative. The assesse will pay the costs of this
reference to the Commissioner.

14. DR. K. A. DHAIRYAWAN AND OTHERS


VS J. R. THAKUR AND OTHERS ON 28
APRIL, 1958
Equivalent citations: 1958 AIR 789, 1959 SCR 799

Subject Bombay Rents, Hotel and Lodging House Control Act,


1947

Court Mumbai High Court

Decided On Apr-28-1958

Case Number Suit No. 2325 of 1948

Judge IMAM, SYED JAFFER

Acts Bombay Rents, Hotel and Lodging House Control Act,


1947 – Section 13, 108.

Appellant Dr. K A Dhairyawan& Others

Respondent J R Thakur & Others

Page | 27
IOV-Registered Valuers Foundation

Case:

1. The lessors granted a lease of a parcel of land to the lessees for 21 years at a rent
of Rs.50 per month. Under the terms of the lease the lessees were to construct a
double storeyed building on the land at a cost of not less than Rs. 10,000. The
construction had to be to the satisfaction of the lessors' engineers, and the
building had to be insured for at least Rs. 12,000 in the joint names of the lessors
and the lessees with an insurance firm approved by the lessors.

2. In case of damage or destruction the building was to be repaired out of the


money received from the insurance company. On the termination of the lease
either at the end of 21 years or earlier, the lessees were to surrender and yield up
the demised premises including the building with its fixtures and appurtenances
to the lessors without any compensation for the same.

3. After the expiry of the 21 years the lessors filed a suit for a declaration that they
were entitled to the building, and were entitled to claim possession of the same
and to recover the rents and profits thereof.

4. The lessees pleaded that they were also lessees of the building and were
protected from eviction there from by the provisions of the Bombay Rents, Hotel
and Lodging House Control Act, 1947 and that the covenant for delivery of
possession of the building could not be enforced as the lease in respect of the
land could not be terminated on account of the protection given by the Act.

5. Held that upon a proper construction of the lease there was demise only of the
land and not of the building and consequently the provisions of the Act did not
apply to the contract for delivery of possession of the building.
Theownership in the building was with the lessees and in whichthe lessors
had no right while the lease subsisted. There was noabsolute rule of law in India
that whatever was affixed or built on the soil became part of it, and was subject
to the same rights of property as tile soil itself.

6. Narayan Das Khettry v. Jatindra Nath Roy Chowdhury, (1926) 54 I.A. 218 and
VallabhdasNarranji v. Development Officer, Bandra (1928) 56 I.A. 259 followed.

7. Held, further, that the provisions of the Act did not provide for a continuation of
the lease beyond the specified period stated therein. The Act merely gave to the
lessee who continued in possession even after the expiry of tile period of the
lease the status of a statutory tenant and protected him from eviction.

8. The appeal, accordingly, is allowed with costs throughout and the decision of the

Page | 28
L&B/Chapter-7/Case Laws

High Court in appeal is set aside. The appellants are entitled to a declaration that
the building constructed on the land demised under the lease, Ext. A, belongs to
the Mankeshwar Temple Trust and the -aid trust is entitled to recover all the
rents and profits from the, same and the respondents have no right, title and
interest therein since the expiration of the said lease. The first respondent is
directed to render an account of the rents received by him from the tenants of
the building from 23-5-48 and to pay to the appellants the amount found due,
after accounting, with interest at 6% per annum from 23-5-48 until payment.
There will be an order of injunction restraining the respondents, their agents and
servants from interfering with the collection of rents and profits by the appellants
from the tenants of the aforesaid building. Appeal allowed.

Page | 29
PRINCIPLES OF
INSURANCE & LOSS
ASSESSMENT I 08
1. Insurance
2. Fire Policies
3. Elements of Insurance
4. Preparation of claim
5. Obligations and Rights of Insurer and Insured

Page | 1
IOV-Registered Valuers Foundation

(This page is blank)


Page | 2
PRINCIPLES OF INSURANCE
AND LOSS ASSESSMENT I 08
1. INSURANCE
Insurance is a means of protection from financial loss. It is a form of risk management,
primarily used to hedge against the risk of a contingent or uncertain loss.An entity which
provides insurance is known as an insurer, insurance company, insurance carrier or underwriter.
A person or entity who buys insurance is known as an insured or as a policyholder. The
insurance transaction involves the insured assuming a guaranteed and known relatively small
loss in the form of payment to the insurer in exchange for the insurer's promise to compensate
the insured in the event of a covered loss. The loss may or may not be financial, but it must be
reducible to financial terms, and usually involves something in which the insured has
an insurable interest established by ownership, possession, or preexisting relationship.

FUNCTIONS AND BENEFITS OF INSURANCE

Insurance has many functions and benefits, some of which we may describe as primary and
others as ancillary or secondary, as follows:

1. Primary functions/benefits:
2. Insurance is essentially a risk transfer mechanism, removing, for a premium, the
potential financial loss from the individual and placing it upon the insurer. The
primary benefit is seen in the financial compensation made available to insured
victims of the various insured events. On the commercial side, this enables
businesses to survive major fires, liabilities, etc. From a personal point of view, the
money is of great help in times of tragedy (life insurance) or other times of need.
3. Ancillary functions/benefits:
4. Insurance contributes to society directly or indirectly in many different ways. These
will include:
i. Employment: the insurance industry is a significant factor in the local
workforce.

Page | 3
IOV-Registered Valuers Foundation

ii. Financial services: since the relative decline in manufacturing in Hong


Kong, financial services have assumed a much greater role in the local
economy, insurance being a major element in the financial services sector.
iii. Loss prevention and loss reduction (collectively referred to as ‘loss
control’): the practice of insurance includes various surveys and
inspections related to risk management.
iv. Savings/investments: life insurance, particularly, offers a convenient and
effective way of providing for the future. With the introduction of the
Mandatory Provident Fund Schemes in 2000, the value of insurance
products in providing for the welfare of people in old age or family
tragedy is very evident.
v. Economic growth/development: it will be obvious that few people would
venture their capital on costly projects without the protection of insurance
(in most cases, bank financing will just not be available without insurance
cover). Thus, developments of every kind, from erection of bridges to
building construction and a host of other projects, are encouraged and
made possible partly because insurance is available.

INSURANCE CONTRACTS
Through a contract of insurance, the insurer agrees to make good any loss on the insured
property or loss of life (as the case may be) that may occur in course of time in
consideration for a small premium to be paid by the insured. Apart from the above
essentials of a valid contract, insurance contracts are subject to additional principles.

The principles of insurance are as follows,

1. Principle of Utmost good faith


2. Principle of Insurable interest

3. Principle of Indemnity
4. Principle of Subrogation

5. Principle of Contribution

6. Principle of Proximate cause

7. Principle of Loss of Minimization

The distinctive features are based on the basic principles. Though all contracts share
fundamental concepts and basic elements, insurance contracts typically possess a number
of characteristics not widely found in other types of contractual agreements. The most
common features of insurance are as follows,

1. Aleatory

2. Adhesion

Page | 4
L&B/Chapter-8/Case Laws

3. Utmost Good Faith

4. Executory

5. Unilateral

6. Conditional

7. Personal contract
8. Warranties and Representations

9. Misrepresentations and Concealments

10. Fraud

11. Impersonation (False pretenses)

12. Parol (or Oral) evidence rule

INSURABLE INTEREST

An insurable interest is a stake in the value of an entity or event for which an insurance policy
is purchased to mitigate risk of loss. Insurable interest is a basic requirement for the issuance
of an insurance policy, making it legal and valid and protecting against intentionally harmful
acts. Entities not subject to financial loss from an event do not have an insurable interest and
cannot purchase an insurance policy to cover that event.

Legal liability will also create an insurable interest. Bailment commonly creates a legal liability
for the Bailee, and, thus, creates an insurable interest in the property for the Bailee. For
instance, an auto repair shop can have insurance on the vehicles on its lot for possible damage
or theft, even though the shop does not own them.

DUTIES OF INSURER
1. Pay Benefits
2. Risk Assessment

3. Reserves for Policy

4. Privacy Protection

RIGHTS OF THE INSURER


1. Right to avoid the policy

2. Right of entry and control over the property.

3. Right of reinstatement.
4. Right to subrogation.

Page | 5
IOV-Registered Valuers Foundation

5. Right of contribution.

6. Right to salvage.

DUTIES OF INSURED
1. Application honesty

2. Keep down exposure to risk

3. Pay premiums

4. Keep current information

RIGHTS OF THE INSURED


Insured persons have the right to,

1. Cancel a life insurance policy within 15 days from the date of receipt of the policy
document. If you disagree to any of the terms or conditions in the policy

2. Return the policy stating the reasons for objection


3. Insured will be entitled to a refund of the premium paid

4. A proportionate risk premium for the period on cover and the expenses incurred
by the insurer on medical examination and stamp duty charges will be deducted

5. If it is a unit linked insurance policy (ULIP) in addition, the insurer can repurchase
the units at the price on the cancellation date.

THE SPECIFIC RIGHTS OF POLICYHOLDERS/ INSURED ARE AS


FOLLOWS
1. Right to a financially sound and viable insurance company.

2. Right to access insurance companies’ official financial information.

3. Right to be informed of the license status of insurance companies, intermediaries


and soliciting agents.

4. Right to be offered a duly approved insurance product.

5. Right to be informed of the benefits, exclusions and other provisions under the
policy.

6. Right to receive the policy.

7. Right to confidentiality of information.

8. Right to efficient service from insurance companies, intermediaries, and soliciting


agents.
9. Right to prompt and fair settlement of claims.

Page | 6
L&B/Chapter-8/Case Laws

10. Right to seek assistance from the Insurance Commission.

2. FIRE POLICIES
There are a number of fire insurance policies to suit different interests. A number of
factors are considered before deciding about the kinds of policies to be taken. Factors
considered are as below,

1. The type of risk involved.

2. The nature of the property to be insured.

3. The contents of the property.

4. Occupancy hazards.

5. Exposure hazards.

6. The time element.

TYPES OF FIRE POLICIES


The following kinds of policies are generally issued for fire insurance,

1. Valued Policy:

These policies are generally issued for those goods or property whose value cannot be
determined after their loss or damage. These goods may include works of art, jewelry,
paintings, etc.

2. Specific Policy:

Under this policy the risk is insured for a specific sum. In case of loss of property, the
insurer will pay the loss if it is less than the specified amount. The actual value of property
is not taken into consideration.

3. Average Policy:

If the ‘average clause’ is applicable to a policy, it is called Average Policy. Average clause is
added to penalize the insured for taking up a policy for a lesser sum than the value of the
property. The compensation payable is proportionately reduced if the value of the policy
is less than the value of the property.

4. Floating Policy:

A floating policy is taken up to cover the risk of goods lying at different places. The goods
should belong to the same person and one policy will cover the risk of all these goods.

Page | 7
IOV-Registered Valuers Foundation

This policy is useful to those businessmen who are engaged in import and export of
goods and the goods lie in warehouses at different places. The premium charged is
generally the average of the premium that would have been paid, if specific policies would
have been taken for all these goods. Average clause always applies to these policies.

5. Comprehensive Policy:

A policy may be taken up to cover up all types of risks, including fire. A policy may be
issued to cover risk like fire, explosion, lightening, burglary, riots, labour disturbances etc.
This is called a comprehensive policy or all-risk policy.

6. Consequential Loss Policy:

Fire may dislocate work in the factory. Production may go down while the fixed expenses
continue at the same rate. A policy may be taken up to cover up consequential loss or loss
of profits. The loss of profits is calculated on the basis of loss of sales. A separate policy
may be taken up for standing charges also.

7. Replacement Policy:

The underwriter provides compensation on the basis of market price of the property. The
amount of compensation is calculated after taking into account the amount of
depreciation.

REINSTATEMENT
If an insured person fails to pay the premium due to various circumstances and as a result
the insurance policy gets terminated, then the insurance coverage can be renewed. This
process of putting the insurance policy back after a lapse is known as reinstatement.

POLICIES FOR OTHER PERILS


The insurance provides protection to the properties against damages caused due to perils
like natural calamities, machinery breakdown, theft/ burglary, etc. as covered under the
insurance policy

The fire insurance policies do not cover perils mentioned below:

1. Spontaneous combustion

2. Burning of property by order of any Public Authority

3. Property undergoing any heating or drying process

4. Explosion of boilers (other than domestic boilers)

5. Total or partial cessation of work

Page | 8
L&B/Chapter-8/Case Laws

6. Permanent or temporary dispossession by order of Government

7. Normal cracking or settlement or bedding down of new structures

8. War or warlike operations, Nuclear perils

9. Pollution or contamination

10. Overrunning, excessive pressure, short circuiting, etc.

Perils which are not covered unless add-on covers are bought for the specific risk:

1. Terrorism

2. Earthquake

3. Burglary, Housebreaking, theft, etc.

The additional premium must be paid for add-on covers.

THE BENEFICIARY& RESTRICTIVE CLAUSES IN INSURANCE


Beneficiary Clause:

This is customary to most policies and allows you to name the recipient of your death
benefit. Generally the recipient is either an individual or your estate. It is a wise decision to
make an individual the beneficiary to insure prompt payment of the death benefit.

Restrictive Clause:

The restrictive clause is important in the healthcare setting because it partially determines
the extent and limitations of coverage.

The best way to define a restrictive clause in the healthcare insurance setting is to start by
defining what is known as an ‘insuring clause’. It is the clause in an insurance policy setting
forth the kind and degree of coverage granted by the insurer.

3. ELEMENTS OF INSURANCE
VALUE AT RISK (VAR)
It is a measure of the risk of loss for investments. It estimates how much a set of
investments might lose (with a given probability), given normal market conditions, in a set
time period such as a day. VaR is typically used by firms and regulators in the financial
industry to gauge the amount of assets needed to cover possible losses.

In case of Insurance, one can look at not only the risky business lines, but also the risks by
type of risk: interest rate risk, mortality, morbidity, lapses, credit risk, currency risk, equity
risk, event risk. He can look atthe amount of risk that each type of risk contributes to the

Page | 9
IOV-Registered Valuers Foundation

total risk of the portfolio, and the amount of risk that each business line contributes.

SOME IMPORTANT TERMINOLOGIES

1. Sum Insured:

The sum insured is the maximum amount the insurance company will pay in the event
of a claim.

2. Over Insurance:

Over insurance can be defined as the situation where an insured has bought so much
coverage that it exceeds the actual cash value (or the replacement cost) of the risk or
property insured.

3. Under Insurance:

It is the state of an individual having some form of insurance that does not offer
complete financial protection to health or asset insured.

4. Condition of average:
It is the insurance term used when calculating a payout against a claim where the
policy undervalues the sum insured. In the event of partial loss, the amount paid
against a claim will be in the same proportion as the value of the underinsurance. The
formula used is,
5. Payout= Claim x Sum Insured/ Current value

6. Insurance Inflation Protection:


Insurance inflation protection is an insurance policy feature in which the value of
benefits increases by a pre-defined percentage at specific time periods. Insurance
inflation protection is designed to allow policyholders to make sure that the benefits
they receive can keep up with the inflation rate.
7. Depreciation in Insurance:

Depreciation is calculated by evaluating an item’s Replacement Cost Value (RCV) and


its life expectancy. RCV represents the current cost of repairing the item or replacing it
with a similar one, while life expectancy is the item’s average expected lifespan.

8. Obsolescence in insurance:

It is significant decline in the competitiveness, usefulness, or value of an article or


property. Obsolescence occurs generally due to the availability of alternatives that
perform better or are cheaper or both, or due to changes in user preferences,
requirements, or styles. It is distinct from fall in value (depreciation) due to physical
deterioration or normal wear and tear. Obsolescence is a major factor in operating
risk. Insurance companies take obsolescence into account to reduce the amount of
claim to be paid on damaged or destroyed property.
9. Betterment Insurance:

Page | 10
L&B/Chapter-8/Case Laws

Betterment insurance refers to insurance coverage that a tenant in a rental property


carries to cover any additions or modifications to the leased space that are made to fit
his or her, or a business's, particular needs. The property owner typically holds an
insurance policy to cover the structure; the tenant procures a policy to cover any
improvements made to the property.

4. PREPARATION OF CLAIM
GENERALLY FOLLOWED STEPS WHILE PREPARATION OF CLAIMS:

Settling insurance claims is just one aspect of the claims management process. The time it
takes to process a claim involves several stages beginning with a person filing a claim. The
stages that follow determine if a claim has merit as well as how much the insurance
company will pay. Insurance customers expect a company to settle claims quickly and to
their satisfaction. Because high customer satisfaction levels can give a company a
competitive edge, reducing the time it takes to settle insurance claims is one way to
decrease the number of customer complaints and improve service. The use of claims
management system software that speeds the process and minimizes costs offers a
practical solution.
1. Analyze the Coverage
2. Review the provisions and exclusions of the policy under claim analyze the policy
to understand its relation to the losses occurred.
3. Get Assistance for Immediate Needs
4. Request that the insurance company release an advance of funds.
5. Organize and prepare a complete list of additional living expenses in order to get
reimbursed for them. Organize and prepare a business continuity plan (for the
business owner) to facilitate continued operations in a timely manner.
6. Prepare a Damage Estimate
7. Provide a detailed review of all damaged property, documenting damage to all
areas.
8. Provide line item estimates, calculate quantities, unit cost and total costs to cover
the loss, take help of professional, in case of need.
9. Prepare and submit all of the above to the insurance company using industry-
standard forms; thereby, enhancing a prompt and accurate response to the claim.
10. Negotiate The Settlement
11. Attend all meetings with adjuster(s)
12. Manage any insurance claim problems or challenges.
13. Accept settlement offers after proper review.
14. Settle the Claim Successfully
15. Resolve claims as quickly as possible with the insurance company.

Page | 11
IOV-Registered Valuers Foundation

(This page is blank)


Page | 12
REPORT WRITING I 09
• Report -QUALITY, STRUCTURE, STYLE
• The report writing for the various purposes of valuation
• Contents of a report

Page | 1
IOV-Registered Valuers Foundation

(This page is blank)


Page | 2
REPORT WRITING I 09
1. REPORT -QUALITY, STRUCTURE, STYLE
A valuation report originates with the instruction given by client, in response to which a
valuation assignment is undertaken by a valuer. The term of reference given by the client
defines nature, scope and limitations.
Writing a report requires technical as well as communication skill and attitude.

Skill is nothing but it is experts ability to carry out a task with a pre-determined result
within given time.

Attitude is settled way of thinking or point of view. It is emotional and mental entity that
inheres in character or a person.
IVS 2017

IOV Registered Valuer’s Foundation has opted for IVS 2017 as valuation standard, hence
all the reports of a Registered Valuer should comply with the standard.

SR. No Topic Provision


IVS Framework When a statement is made that valuation has been
carried out in accordance with IVS, it is implicit that
valuation has been prepared incompliance with
standards issued by IVSC. The terms like valuer,
objectivity, competence, and departures are
discussed.
General Standards –

IVS 101 Scope of work A scope of work describes the terms of valuation
engagement such as asset, purpose and
responsibilities of parties involved, bases of value,
valuation date, limitations, source of information,
assumptions, restrictions on use and identifying that
the valuation was performed in accordance with IVS.
IVS 102 Investigations and Investigations made during the course of valuation
compliance must be appropriate for the purpose of valuation.
Sufficient evidence must be assembled so that the

Page | 3
IOV-Registered Valuers Foundation

valuation is properly supported. The information


collected must be credible.
IVS 103 Reporting It Is essential that valuation report communicates the
user a clear understanding of valuation. It provides
useful information, scope, purpose, intended views
along with assumptions, limitations that directly
affect the valuation. The report should cover scope,
approach, method, inputs, assumptions, conclusion
and date of report which may differ from valuation
date.
IVS 104 Bases of value The bases of value describe the fundamental
premises on which the reported values will be based.
The bases of value may influence or dictate selection
of method, inputs and assumptions and ultimately
opinion of value. The defined bases of value are
Market value, market rent, equitable value,
investment value, synergistic value, liquidation value
along with other value definitions and terms like
HABU.
IVS 105 Valuation approaches Three approaches namely – Market approach,
and methods Income approach and Cost approach are discussed.
They are all based on economic principles of price
equilibrium, anticipation of benefits and substitution.
Market approach covers comparable transaction
method, rule of thumb, discounts, and premiums.
Income approach provides an indication of value by
converting future cash flow. Topic includes DCF,
Gordon growth model, salvage value and discount
rate. In cost approach replacement cost method,
reproduction cost method, summation method and
other factors like depreciation and obsolescence are
discussed.

Salient Features of a good report –

A professional appearance, logical organization of the work and absence of spelling and
grammatical errors are the salient features of a good report. The following factors are the
major criteria for a well written report.

9. Organization - A well-organized report is logical within each section and in fact


throughout the entire report. The sections of the report should interrelate and not
contradict each other. Demonstrative or suggestive reports are given in this lesson.
10. Grammar and Spelling – Simplicity of language is the key. All technical terms need to
be defined for the benefit of the layman. Typographical errors and spellings and
punctuation marks should be checked out thoroughly.

Page | 4
L&B/Chapter-9/Report Writing

11. Calculations/Mathematics – A report could lose its credibility if there is any


calculation or mathematical error. Rounding off of figures should have consistency.
12. Appearance of the report – The report should be professionally typed and bound
properly, If photographs or drawings are enclosed, these should be clear and
presented in a way that adds to the professional status of the report.

2. THE REPORT WRITING FOR THE VARIOUS


PURPOSES OF VALUATION
16. Sell / Purchase – Every seller like to have a best possible price to the assets and
buyer, the lowest. The in-between process is negotiations. A valuers report, which as a
back-up of instances of sales in the market or return over investment may be bridging
between buyer and seller.

17. Mortgage – Mortgage is defined in the transfer of property act as a transfer of an


interest in specific immovable property for the purpose of securing the payment of
money advanced or to be advanced by way of loan. The method of valuation of
mortgage property should be according to normal principles of valuation. The
mortgage property should be valued on the basis of its existing use only. However,
certain possibilities of enhancement in value should be taken into account. The
bankers generally demand fair market value along with realizable value and auction
value.

18. Taxation – A report by a registered Valuer, under wealth-tax act 1957 under section
34 AB, is necessary in the cases of taxation. Earlier, the report was required for the
purpose of wealth tax. Since now, there is no more wealth tax, no valuation report for
wealth tax is required. However, in the cases of Income tax matters, the valuation is
required under various sections. The report is to be submitted in form O-1. For the
purpose of wealth tax, the method was prescribed in the act. But for income tax
purposes the fair market value is applicable in most of the cases. For the capital gain
tax purpose, the valuation is required as on 1/4/2001. Below is the list of sections of
Income Tax where the valuation is required.

1 Income from house property Section 22

2 Definition of Fair market value Section 2 (22B)

3 Capital Asset in fringe areas Section 2 (14)(b) (iii) (b)

4 Capital Gains Section 45

5 Mode of Computation of capital gains Section 48

6 Cost of acquisition in certain cases like HUF, Section 49 (1)


Gift, WILL, Succession, Trust

Page | 5
IOV-Registered Valuers Foundation

7 Valuation where consideration received is not Section 50 D


ascertainable e.g. goodwill
8 Reg. full value consideration for transfer of Section 50 C
assets
9 Reg. full value consideration for transfer of Section 43 CA
assets

10 To curtail gift usually used to convert black Section 56 (2) (vii)


money into white

19. Insurance – Insurance policies enable property owner to reduce or eliminate certain
measurable risk of financial loss. In the case of loss, one of the best ways to estimate
the replacement cost is by making a detailed estimate at current rates and then
deducting for depreciation. While writing the report Valuer should take into account
various perils of insurance.

20. Liquidation – One of the key task of an IRP/RP is to appoint two registered Valuers,
who intern will determined liquidation value of the corporate debtor. A Valuer should
do take care of physical verification of inventory and assets.

3. CONTENTS OF A REPORT
The report writing process involves following steps

21. Identification of purpose of report - Writing is always easier when you are clear about
purpose of the report. It’s like even you are travelling in a dark, if you know the path,
you can reach the destination. That is why; one should have clarity about the purpose
of report.
22. Defining scope and limitations

23. Adopting methodology

24. Data collection

25. Data analysis

26. Conclusion and opinion

27. Disclosures

There are certain questions which a valuer should ask himself.

28. What is to be achieved through this report?

Who is reader of the report? This is important from the point of view that, if report is
for bank, and banker is going to read it then the language and content of the report
should be such that it can be easily understood by banker. If it is for the court
purposes, then the language of writing should be legal, and report should contain
supportive judgment of Court cases. If it is for income tax, then the methods known to

Page | 6
L&B/Chapter-9/Report Writing

the tax authorities will impress them. Hence the reader of the report is important.

29. Collection of data is important aspect of report. One has to collect data in such a
manner that it will help you in evaluation. e.g. for construction cost, one should collect
data like strata of foundation, type of foundation and super structure, use and loads
on building, future provisions, specifications, water tanks, lifts, their type, pavement,
compounding fencing… each item is important.
30. Accountability – A Valuer is responsible for the findings in his report.

Keeping above points in mind, a report can be written by a Valuer. The essential content
of the report are:

31. Instruction from client –about date, purpose etc

32. Date as on which valuation is required – There are 4 important dates (1) Date of
appointment (2) Date of site visit (3) Date of valuation (4) Date of reporting.

33. Location – The location of the property, should be explained by way of postal/ legal
address and also by way of Google images showing latitude and longitude. The
location should be explained by a major landmark in the vicinity.
34. Ownership History – This is a important aspect of reporting, as lot of information can
be collected through it, like number of owners with names and age, shares of each,
type of ownership i.e. free hold, leasehold etc., rights of each in a property, possession
status, type of use etc.

35. Inspection, Investigation, Data Collection and analysis – The data of transactions
in the vicinity is collected, presented and analyzed. Every instance available may not
be comparable, hence by using appropriate method it should be made comparable.
The important points of comparison are time of a transaction under comparison,
location, age, size, shape, specifications along with social, legal, physical and
economical aspects.

36. Type of construction – While evaluating cost of construction, the type of


construction matters much. Whether the building is load bearing or RCC or Shed or a
temporary structure will matter a much. Along with that number of stories, amenities
and specifications, balance life of the property, status of maintenance are all
important factors.
37. Valuation Method – The determinants of the method are type of property, use of
property and data available for comparison.

38. Value estimation – Value is not the exact price tag but an estimate of likely exchange
of money against property. This is based on the method adopted and the reliability of
data collected. There can be two or more valuations of a same property but one which
is most appropriate and reasonable in view of present and future use and income
generation can be considered as value.
39. Assumptions – In the process of valuation, it is not that all the things have backup of
a data. Sometimes that you accept something as truth even though you don’t have
proof for it. All such assumptions should be mentioned in the report.

Page | 7
IOV-Registered Valuers Foundation

40. Major factors considered during valuation

41. Limitations – Limitations are conditions that put limit on something. E.g. The
report for one purpose should not be construed with another.

42. Caveats – Caveat is a sort of limiting condition or a warning or a proviso of a


specific stipulation. It is a statement that certain action may not be taken without
informing the person who give the notice.

43. Conclusion – It is the last statement of the report which states the value assigned
with specific details.

Page | 8
CASE STUDIES I 10
• Case Studies

Page | 1
IOV-Registered Valuers Foundation

(This page is blank)


Page | 2
CASE STUDIES I 10
CASE STUDIES
CASE STUDY NO. 1

Flat number 2, 4 ,6 and 8 , all admeasuring 100 sqm, at Park avenue are recently built
exactly similar flats in same building. The sales data is as below

No. 2 sold last month – Rs. 72,00,000. Vacant possession

No. 6 – let out for Rs. 50,000 per month net

No. 8 – sold year back – Rs. 64,00,000. Vacant possession

No. 4 - Vacant . Available for sale / rent

Q1 – What is current Y.P?

44. 12

45. 8.33%

46. 50,000

47. 4. 7.2

Ans: 72,00,000/6,00,000 = 12

Q2 – what is rise in rent considering same Y.P.

48. 12.5%

49. 12

50. 8%

Page | 3
IOV-Registered Valuers Foundation

51. 5,33,333

Ans: Net annual rent, year ago

52. 64,00,000 / 12 = 5,33,333

53. Rise in Rent = (6,00,000 – 5,33,333) / 5,33,333 = 12.5%

Q3 – What gross rent flat no. 4 should fetch, if outgoings are 15%?

54. Rs. 58,823 per month

55. Rs. 57,500 per month

56. Rs. 64,00,000

57. Rs. 72,00,000

ANs: Gross rent – out goings = Net rent

58. 50,000 /.85 = 58,823

59. i.e. 58823-15% = 50000

Q4 – What will be current expected net rent if ROI is dropped to 3%?

60. Rs.50,000 per month

61. Rs. 1,83,600

62. Rs. 1,53,000 per annum

63. Rs. 180 per sqm per month

Ans: 72,00,000 x 3% / 12 = Rs. 18,000 per month

18000 / 100 sqm = Rs. 180 per sqm per month

CASE STUDY – 2

A petrol pump sales averagely 2KL lits MS and 5 KL HS daily in 2017 year. Company pays
annual rent at the rate of 6% of land value. Land area is 4,000 sqm and rate is Rs.1,000 per
sqm.

The petrol & diesel is sold at Rs. 80 & Rs. 50 per lit and bought at Rs. 75 & Rs. 45 per lit.
The staff salary is Rs.1,00,000 per month. The half yearly property tax is Rs. 50,000 and

Page | 4
L&B/Chapter-10/Case Studies

electricity bill is averagely Rs.5,000 per month

Q 1 – What is Annual Land rent?

64. 2,40,000

65. 40,00,000

66. 1,000

67. 20,000

Ans: 1. 2,40,000 = 400 x 1000 x 6%

Q-2 What is total annual income?

68. Rs. 149890000/-

Rs. 14,96,50,000/-

Rs. 58,640,000/-

Rs. 91,490,000/-

Ans: 1. Rs. 149890000/-

Q3. What is total profit?

69. Rs. 1,17,05,000/-

70. Rs. 1,17,55,000/-

71. Rs. 1,17,15,000/-

72. Rs. 1,16,55,000/-

Ans: 1,16,55,000/-

Income 149,890,000

Expenditure 138,235,000

Profit 11,655,000

Q 6. What is business Value if rate of capitalization is 12%?

73. Rs. 9,32,40,000/-

74. Rs.14,56,87,500/-

75. Rs. 9,71,25,000/-

Page | 5
IOV-Registered Valuers Foundation

76. Rs. 9,37,20,000/-

Ans: Rs. 9,32,40,000/-

Profit = 11,655,000

Cap =12.5% & YP is 8

Value = Profit x YP = 93,240,000

CASE STUDY 3

77. Mr. Sanjay Purchased a land admeasuring 500 sqm in 1964 for Rs. 10,000/-

78. In 1990, he constructed a B’low admeasuring 250 sqm spending Rs. 6 Lakhs.

79. He sold the land and building on 15th Aug 2018 for Rs. 1.50 Crores
80. The case is referred for capital gains tax

Q1 What is date of Valuation?

81. 15th august 2018

82. 16th august 2018

83. 1st April 1981

84. 1st April 2001

Ans: 1st April 2001.

Q-2 If market rate of land in 2001 is Rs. 2000 per sqm what is land value?

85. Rs. 10,000/-

86. Rs. 10,00,000/-

87. Rs. 1,00,000/-

88. Rs. 3,57,000/-

Ans: 10,00,000/-

Page | 6
L&B/Chapter-10/Case Studies

Q3. What is cost of building in 2001 if cost of similar construction is Rs. 5000 per sqm and
depreciation is 20%.

89. Rs. 12,50,000/-

90. Rs. 6,00,000/-

91. Rs. 10,00,000/-

92. Rs. 2,50,000/-

Ans: 10,00,000 = 250x5000x80%

Q 4. What is Valuation as on 1/4/2001?

93. Rs. 200000000/-

94. Rs. 2,00000/-

95. Rs. 2000000/-

96. Rs. 200,000/-

Ans: 20,00,000/-

CASE STUDY - 4

A residential building of G +2 storied with rental income ground, 1 st& 2nd floor is rs.
3600/- month, Rs. 4000/- p.m. & Rs.4500/- p.m. respectively. Property tax Rs. 20,000.00.for
6 months N.A. Tax 800/- per year. Building is 40 years old Condition.

Area of plot is 800 sqm

Prevailing market rate of land in this locality is Rs. 4000/- sqm

Rent Act is not applicable, repairs 10% of G.R. expected rate of return is 9%

Total life of building is considered as 60 years

Q 1 : What is the gross annual yield of the property.

97. Rs. 52,600.00

98. Rs. 82,620.00

99. Rs. 1,45,200.00

Page | 7
IOV-Registered Valuers Foundation

100. Rs. 1,50,000.00

Ans: Gross annual yield = (3600 +400 + 4500) x 12 = 1,45,200.00

Q 2 : What is the net yield from the property

101. Rs. 52,600.00

102. Rs. 1,45,000.00

103. Rs. 89,880.00

104. Rs. 95,000.00

Ans: Net Yield = Gross yield – Out goings

Gross yield = Rs. 1,45,200.00

Page | 8

You might also like